Você está na página 1de 432

j.

Miller Dias de Araújo

Os Segredos da Álgebra

para IME ITA Olimpíadas

Editora Vestseller
FORTALEZA - CE
1a Edição Julho/2018
Apresentação
Escrever um livro não é só expor nele a matéria e resolver as questões
selecionadas.
É, antes, relacionar conteúdos, conjugando didaticamente idéias e métodos
eficazes ao longo da obra. Trata-se de um trabalho árduo, pois são vários os
aspectos (muitas vezes sobrepostos) a abordar, e somente quem é autor sabe
a dificuldade de levar tudo isso a um bom termo.
Por outro lado, a escrita deste livro ofereceu-me novas experiências e
oportunidades de amadurecimento tanto como professor quanto como autor.
Além disso, o fato de haver poucos, talvez pouquíssimos, livros hoje no
mercado que detalham os assuntos de que trato aqui fez com que minha
motivação fosse ainda maior.
Nos oito primeiros capítulos, exponho toda a teoria, de forma detalhada
e demonstrada. Trato do assunto e, após fazê-lo, trago exemplos resolvidos
para facilitar a compreensão. Na sequência, há uma batería de exercícios para
o leitor praticar os assuntos e fixar o que aprendeu.
Você verá que a maioria das questões tem mais de uma resolução, e
algumas delas foram repetidas em capítulos diferentes. Isso se dá para que
você possa, se quiser, resolvê-las com o assunto daquele novo capitulo,
aumente sua visão geral dos conteúdos e afie ainda mais seu raciocínio para
utilizar em exercícios futuros.
Nos dois últimos capítulos, concentrei os gabaritos, as sugestões e as
resoluções, para que o leitor possa conferir suas próprias respostas e,
possivelmente, aprender idéias novas também a partir das resoluções.

Miller Dias De Araújo


Prefácio
No século XVIII, o matemático francês Jean Le Rond D'Alembert
afirmou "A álgebra é generosa: frequentemente ela dá mais do que se lhe
pediu". D'Alembert, que possui importantes descobertas na álgebra, como o
Teorema Fundamental da Álgebra, não poderia ter sido mais feliz em sua
célebre frase. A álgebra é a ferramenta mestra para o desenvolvimento de
todas as ciências naturais. O que seria da física sem a álgebra? O que seria
das outras áreas da matemática sem a álgebra?
No nosso pais, de forma geral, as pessoas menosprezam a importância
da matemática em suas vidas. Mal sabem essas pessoas o quanto elas seriam
mais felizes se soubessem, por exemplo, calcular o consumo mensal de cada
aparelho eletrônico de sua residência ou calcular o consumo de combustível de
seu automóvel em faixas diferentes de velocidade. Para tanto, bastaria que
tivessem uma base mais sólida de álgebra e aritmética. E essa base mais sólida
em matemática aprende-se com bons professores e com bons livros.
No âmbito dos concursos militares e olimpíadas cientificas, uma base
sólida em matemática não é uma opção, é uma obrigação. Há um detalhe
importante sobre o ensino dos primeiros conceitos de álgebra: eles são
abordados no ensino fundamental, momento em que não há espaço para
aprofundamentos, fazendo com que os livros apresentem a álgebra de maneira
superficial. Nos vestibulares tradicionais (ENEM e vestibulares de
universidades federais, estaduais e particulares), não há uma exigência muito
grande quanto à álgebra. Porém, nos concursos militares e olimpíadas de
matemática, a álgebra é exigida em um nível muito alto, sendo necessário um
estudo mais aprofundado.
De modo a auxiliar nesse aprofundamento, a obra "Os Segredos da
Álgebra Para IME/ITA/OLIMPÍADAS", do eminente autor Miller Dias De Araújo,
é perfeita. Todos os tópicos da álgebra fundamental estão presentes, incluindo
potenciação, radiciação, produtos notáveis e fatoraçâo. Em cada capitulo, a
teoria algébrica é apresentada de forma detalhada, seguida de muitos
problemas resolvidos. Várias identidades algébricas são demonstradas,
algumas pouco conhecidas. Certamente, a cereja do bolo são os exercícios
propostos, em grande quantidade e de excelente qualidade, a maioria retirada
de concursos militares e olimpíadas de matemática de todo o mundo. Você vai
se deparar com exercícios do Colégio Naval, ITA, IME e também de olimpíadas
de diversos países. Deve-se destacar a organização dos capítulos, nos quais
os conceitos mais fundamentais são apresentados primeiro, para somente
depois constarem os tópicos mais avançados, facilitando o entendimento do
leitor. O último capitulo do livro é reservado para as soluções e dicas dos
exercicios propostos.
A presente obra é um verdadeiro tesouro, indispensável para qualquer
pessoa que deseja formar uma base sólida em álgebra. Recomendo ao leitor
que resolva esse livro de capa a capa, prestando atenção nas nuances de cada
passagem algébrica. Existe muita poesia, arte e magia nessa obra prima do
Miller Dias. Parabéns ao grande mestre!
Tenho que admitir que, durante a leitura do livro, me perguntei o quanto
minha vida de concurseiro militar teria sido mais fácil se eu tivesse, à época,
um livro de tão alto nivel para ajudar minha base algébrica. Certamente esta
obra de Miller Dias De Araújo será uma importante ferramenta para todos
formarem uma base mais sólida em álgebra.

Marcelo Rufino de Oliveira


Agradecimentos

A Deus, pelo dom da vida.

Aos meus pais, que sempre me apoiaram e me incentivaram em todas as


minhas caminhadas.

A meus amigos, em especial Thais Pereira Soares e Johnatan Bruno, pelo


apoio, motivação e incentivo; Meus professores, que ensinaram-me com muita
dedicação.

Ao professor Renato Brito, pelas criticas e sugestões.

Ao professor Marcelo Rufino pela escrita do prefácio.

Ao professor Cláudio Neves pelas revisões.

Ao professor Luiz Vieira dos Santos, pelos ensinamentos, pela paciência e por
ter-me iniciado no caminho da Matemática IME ITA.
Sumário
Capítulo 01: Potenciação
1.1) Definição 11
1.2) Produto de Potências de Mesma Base 11
1.3) Divisão de Potências de Mesma Base. 12
1.4) Potência Elevada à Potência 13
1.5) Produto Elevado á Mesma Potência 13
1.6) Potência Elevada á Potência, n vezes .. 17
1.7) Potência de Ordem Superior 17
1.8) Potência com Expoente em PG 21
1.9) Potência com Expoente Negativo 26
1.10) Divisão Composta 26
1.11) Potência com Expoente Fracionário... 27

Capítulo 02: Radiciação


2.1) Definição 31
2.2) Produto de Radicais de Mesmo índice 32
2.3) Divisão de Radicais de Mesmo índice 33
2.4) Raiz de uma Raiz 36
2.5) Produto de Radicais de índices Diferentes. 43
2.6) Raiz de Fração Composta 45
2.7) Séries Finitas de Radicais 52
2.8) Séries Infinitas de Radicais 61
2.9) Divisão Composta Infinita 81
2.10) Radicais em Cadeia Infinita 84
2.11) Operações com Radicais 90

Capítulo 03: Racionalização


3.1) Quocientes Notáveis 93
3.2) Fator Racionalizante 94
3.3) Radicais Duplos......................................... 105
3.4) Tópicos Avançados em Radicais Duplos. 124

Capítulo 04: Expressões Algébricas


4.1) Tipos de Expressões Algébricas 125
4.2) Valor Numérico 126
4.3) Operações com Expressões Algébricas. 127
Capítulo 05: Produtos Notáveis
5.1) Quadrado da Soma de Dois Termos 130
5.2) Quadrado da Diferença Entre Dois Termos.. 131
5.3) Identidade de Legendre para a Soma 132
5.4) Identidade de Legendre para a Diferença 133
5.5) Identidade de Lagrange para a Soma 133
5.6) Identidade de Lagrange para a Diferença.... 134
5.7) Produto da Soma pela Diferença................... 140
5.8) Identidades de Stevin 142
5.9) Cubo da Soma de Dois Termos 149
5.10) Cubo da Diferença de Dois Termos 149
5.11) Identidade de Cauchy 152
5.12) Quarta Potência da Soma e da Diferença.. 157
5.13) Identidades de Legendre 158
5.14) Quinta Potência da Soma 160
5.15) Quinta Potência da Diferença 161
5.16) Identidades para Termos Recíprocos 165
5.17) Quadrado da Soma de Três Termos 173
5 18) Identidade de Lagrange Para Três Termos.. 175
5.19) Produto Dois a Dois Elevado ao Quadrado. 176
5.20) Identidades de Argand 181
5.21) Quadrado da Soma de Quatro Termos 182
5.22) Cubo da Soma de Três Termos 183
5.23) Identidade de Gauss...................................... 185
5.24) Soma de Quatro Termos Elevado ao Cubo. 186
5.25) Quarta Potência de Três Termos 187
5.26) Identidades de Stevin para Três Termos 189
5.27) Identidade de Sophie-Germain 191
5.28) Identidade de Chrystal 194
5.29) Identidades Condicionais 199
5.30) Tópicos Avançados em Produtos Notáveis. 211

Capítulo 06: Fatoração


6.1) Critérios de Fatoração 213
6.2) Agrupamento ou "evidência” 213
6.3) Quocientes Notáveis 214
6.4) Completando o Produto Notável 216
6.5) Cruzadinha Simples 226
6.6) Teorema do Fator ou das Raizes Racionais 230
6.7) Fatorando Polinõmios do 3o Grau 231
6.8) Cruzadinha Dupla 234
6.9) Cruzadinha Dupla Especial 238
6.10) Fatorando Polinõmios do 5o Grau 243
6.11) Cruzadinha Tripla........................... 246
6.12) Tópicos Avançados em Fatoração. 256

Capitulo 07: Polinômios Simétricos


7.1) Forma de um Polinõmio Simétrico........... 259
7.2) Propriedades dos Polinômios Simétricos. 260
7.3) Fatoração por Polinõmio Simétrico.......... 261
7.4) Polinõmio Alternado................................... 263
7.5) Propriedades dos Polinômios Alternados 264
7 6) Fatoração por Polinõmio Alternado.......... 265

Capítulo 08: Somas de Newton


8.1) Somas de Newton para Dois Termos............. 272
8.2) A Notação Sigma.............................................. 274
8.3) Somas de Newton para Três Termos............ 276
8.4) Generalização para um Polinõmio de Grau n 280

Capítulo 09: Respostas e Sugestões

Capítulo 10: Resoluções

Bibliografia
Os Segredos da Álgebra para IME/ITA/OLIMPÍADAS 11

Capítulo 01 - Potenciação

Introdução
A determinação da potência de um número é feita pela multiplicação de fatores
iguais. O expoente possui um papel fundamental na potenciação, pois ele é
quem define quantas vezes a base será multiplicada por ela mesma. Vejamos
como trabalhar com essa ferramenta importantíssima na resolução de
problemas.

1.1) Definição:
Dado um número real "a" qualquer, tomemos n como um número natural. O
produto de n fatores “a" é igual á enésima potência de “a”.
"n" vezes
a a - ...a = an ; VneNeaeR.

Consequências da definição:
|a° = l|; VaeR-{0,®). |0a = 0|; Va sR-|0, ra}.
a) c)

b)
a1 = a|; Va e R . |la = l|; Va e R - {«>}.
d)

Exemplos de Aplicação 01
a) 2001° =1. d) 01785 = 0.
b) 1o = 1. e) 01 = 0 .
(4O96202)1 =4096'.202 f) 750 234 _ 1
C)

1.2) Propriedades das Potências:


A seguir, veremos as propriedades mais importantes das potências. Com elas,
iremos resolver vários problemas usando o minimo de cálculo algébrico. Vamos
lá!

P1. Produto de Potências de Mesma Base:


Num produto de potências de mesma base, repete-se a base e somam-se os
expoentes.
|am ■ an = am~n]
12 1 Potenciação

Demonstração:
"m"vezes "n" vezes ”m + n" vezes
aim an =a-a-...-a a a-... a a'im-an = a a a a-... a

im • a" = a.m
1 ; Vm, n e N e a e R .

Exemplos Resolvidos 01
a) 53 -515 =53 + 15 => 53 -51s=518.
g36 + 25
b) 8136 ■8125 => 8136 ■81'l25=8161.

m2” ■ m7p = m‘2p + 7p => m2p ■ m7p=m,59P


C)

P2. Divisão de Potências de Mesma Base:


Numa divisão de potências de mesma base, repete-se a base e subtraem-se
os expoentes.

—=am“n

an

Demonstração:
"m"vezes "m - n" vezes
am 1 am
— = a ■ a •... • a--------------- —= a■a a a ■a
an aa...-a an
"n" vezes

am
— = am-n ; Vm, n e N, a e R e an * 0.
an

Exemplos Resolvidos 02
q30 o30
a) 2- = 3,30-12
: ó <18
31Z 312
O2016 92016
f_ 21008
b) - ----- = 2: ,2016-1008
21008 21008

v100 X100
*___ __ *100-72 _____ - y .28
C)
x72 72
X
Os Segredos da Álgebra para IME/ITA/OLIMPÍADAS 13

P3. Potência Elevada a Potência:


Numa potência de uma potência, repete-se a base e multiplicam-se os
expoentes.
(am)n = amn

Demonstração:
"n" vezes
(am)n=a' am ; Vm, n e N e a g R.

Consequência:
"n" vezes "n" vezes "n"vezes
.m
•am •...am •am ■am -am ....-a"' ■am ■...arn
"p“ vezes

((an,)nJ’ = (amn = am'np

Exemplos Resolvidos 03
(725)3 = 7•25-3
25.3 (725)3 = 775
a)
:
(p2q)m=p2qm
b) (p2<’f=p2qm
:
/ rr. \3n
c) [(p2q) ] =P: 2q m■3n 6qmn

P4. Produto Elevado à Mesma Potência:


Num produto elevado á potência, eleva-se cada base ao expoente.

(ab)n =anbn

Demonstração:
_______ "n"vezes_______ "n"vezes "n"vezes
(ab)n = (a b) (a b)-... (a b) (a-b)n = aa-...abb-...b

(a b)n = an • bn ; Vn e N e a, b e R .
14 1 Potenciação

Consequências:

01) Produto de vários termos, elevado a potência:


O produto de vários termos elevado a potência é igual ao produto de cada termo
elevado a potência.

(abc...)n=anbncn...

Demonstração:
"n"vezes
(a b c...)n = (a b c...) (a b c...)-... (a b c...)
"n" vezes "n" vezes "n" vezes
<=> (abc...)n = aa-...abb-...bcc-...c

(a-b c...)n = an bn cn... .

Exemplos Resolvidos 04
a) : •x
(p • m)2"1'x = (p)2m x ■ (m).2m (P ■ m)'I
2m - x
=P
: •x
2m ■ m2m x

b) (a ■ b • c)25 =(a)25 • (b)25 • (b)25 , u \2S ■c25


(abc) = a25 ■b25

02) Se as potências tiverem expoentes:


Generalizando a consequência anterior para produto de potências, tem-se:

(axby-cz...)n=a'!nx ,bny.cnz

Demonstração:
________________ "n"vezes_______________
,n
(a* b* -cz...)n =(ax -by -cz...)-(ax by cz-(ax -by -c2...)
■n“ vezes "n" vezes "n“ vezes
<=> (ax by cz...) = axax-...axbyby...bycz-cz-...cz

(ax by-cz= a'inx ■ bny • cnz


Os Segredos da Álgebra para IME/ITA/OLIMPÍADAS 15

Exemplos Resolvidos 05

a) (p' )2m=p)2m.(rn5n)2m =>(p4q- iSnj2m =p8<i m .m10n m


m‘

b) (ax ■by-c2)3 = (ax)3 ■ (by j3 ■ (bz j3 (ax by cz)3 =ai:3x-b3y


c32

Problemas Propostos

Questão 1.1 (AHSME-1952)


2n+4 -2•(2n)
, quando simplificado, é:
2 (2n+3)

b) -2n+1 c) 1-2"

Questão 1.2
k2d
Determine ^(2aj Jt2c

Questão 1.3

Determine ^22a2 -32b2 -72c2)

Questão 1.4 (AHSME-1971)


O número 2~(2k+1) -2-(2k~1) + 2"2k é igual a:

a) 2“2k b) 2“<2k-1) c) -2-<2k+1> d) 0 e)2

Questão 1.5

104 1516 3311 -7717 -8413


Simplifique
520,1430,3q40 .^28
16 1 Potenciação

Questão 1.6

5n' 2n~1
Simplifique
51’ 21-n '

Questão 1.7

X°k+2 -xnk+1 + xnk


Simplifique
,nk-2 - xnk-1 + xnk
x'

Questão 1.8
k
x vezes xnvezes xkvezes
Simplifique xn + xnk + ... + x"k -xn + xn+... + xn xk + xk+... + xk.
.k

Questão 1.9

bavezes aavezes
Reduza ab + ab +. .. + ab-bb+bb+... + b£b.

Questão 1.10

bavezes _ abvezes
1 -baba-...-ba.
Reduza ab ab -...-alb
Os Segredos da Álgebra para IME/ITA/OLIMPÍADAS 17

P5. Potência Elevada a Potência, Elevada a Potência “n” Vezes:


Numa potência elevada a potência, elevada a potência n vezes, o expoente é
dado pelo produto de potência de mesma base.
"n” vezes

= aim ; Vm, neN e ae R

Demonstração:
"n” vezes "n" vezes
-.m "n"vezes
g m • m .m = am"

Exemplos de Aplicação 02
"100"vezes "n" vezes
/ ,\3 ”7?
a) H(23)3]' = 23,100 d) = n5"

"x"vezes
73 "2017"vezes
= 33* ■ x36
b)
e) í = 3',362017
"a" vezes
a
C) = aa3

P6. Potência de Ordem Superior:


Numa potência de ordem superior, repete-se a base e eleva-se o expoente a
potência.

"p" vezes

i"P
= am ; Vm, n, peNeaeR
18 1 Potenciação

Demonstração:
"p"vezes "p"vezes
'( • Y1

a
H']" = am n
"p"vezes
n ■■■ n
alv
HJ = amnP

Consequências:

1) Se as potências forem distintas:


Se as potências forem distintas, repete-se a base e multiplicam-se os
expoentes distintos.

= amn p’r5...
;Vae R.

Exemplos de Aplicação 03
“n" vezes 5q>|
.\n z n x3P
c) (2 ) _ 22n-3p-5q

a)
[H] J = 22
2nn
.
"m" vezes

= aaXt>y'cZ
d)
am
b) a = am

2) Se os expoentes forem distintos:


Se as potências forem distintas e seus respectivos expoentes forem iguais,
podemos usar a propriedade do produto elevado a potência.

r" Y'
(am" J = a(mpr...)n
; Va e R .
Os Segredos da Álgebra para IME/ITA/OLIMPÍADAS 19

Demonstração:

vnT' r" Y"

(a-f = amnpnr"... = a(m p-r...)"

Exemplos Resolvidos 06

7n 7n
5n)

a)
(32nf = 3(2 3 5 7)"

7
= 3210"

>(4z)n 1 (42)" '

b)
í n x"Y2y)n = n(x-2y4z)n = n(8x-y Z)n

3) Se os expoentes forem distintos e suas bases iguais:


Se as potências forem distintas e suas respectivas bases forem iguais,
podemos usar a propriedade do produto de potência de mesma base.

ms'l
z \mq n + q + s + ...
^am )
=a ; Va e R.

Demonstração:

ms ms
/ \mq z n n + q-s^...
(a ) _ amn-mq- ms...
=a ■
20 1 Potenciação

Exemplos Resolvidos 07

,60 \ 260 '

(5q2 930 930 \


9100

(52i°y
*30 + 60
a) = 52

xxY xx'l
xX + X + X v3x
b) =x =X

4) Se os expoente forem iguais e as bases iguais:


Se os expoentes forem iguais e suas respectivas bases também são iguais,
podemos usar o produto de potências de mesma base.

"p"vezes

mn'|
mnY
am" = amnP

Demonstração:
"p" vezes "p" vezes

ynn” "p" vezes mn Y"


z xmn , n 'l
(■ ) =a
mn- mn- mn...
(am j = aHP
7
"p"vezes

mn Y"
z _y
(a ) omnp
= a
Os Segredos da Álgebra para IME/ITA/OLIMPÍADAS 21

Exemplos Resolvidos 08
"x"vezes "x"vezes
xx'l xx>|
X2
= x*XX = XX
a)

"x+1”vezes “x+1" vezes


yzY yz)

b) =x
yZ (X + 1)
(41 = x^Z

P7. Potência de mesma base e expoentes em PG:


Numa potência com expoentes em PG. repete-se a base, e o expoente
expressa-se como soma de uma PG.
"n" parênteses
f rn"-l'|
,m,LL xm
> V
a... a a(a(a)mj =a

Demonstração:

"n" parênteses "n" parênteses

m mY m
(a)m2
a... a a(a(a)mj a... ai am

”n” parênteses
\m
am2 (a)m3

"n" parênteses
f mn-l'|
m ---------
I m-1
a... a a(a(a)' = ai ' '
22 1 Potenciação

Generalização:
Numa potência com expoentes em PG de potências, repete-se a base, e o
expoente expressa-se como soma de uma PG.

"p" parênteses

m p+1-m
( ( m \rn V.m \ n ■
m-1
an... an an an(an) =a

Demonstração:

"p" parênteses "p" parênteses


m m
an... an^an(an)mj,m = an..|an(an(an
•a

"p" parênteses "p" parênteses


m
• anm2 • anrr|3
an... an an-a'nm • anm2

"p" parênteses

n • p + m + m2 + m3j |
= an...

"p" parênteses

an..|an^an(an)mjm'
í
n• m•
=a , L

"p" parênteses
.. >.m
an... a"^an(an)mj
=a
Os Segredos da Álgebra para IME/ITA/OLIMPlADAS 23

Exemplo Resolvido 09

"10" parênteses
7^2
3-(3(3)2)
Determine

Resolução: Podemos escrever:

"10" parênteses "10" parênteses


2 7 2- (2--1)
^3...(3(3)2]' j =3 =>[3...(3(3)2)' J =3

"10" parênteses "10" parênteses



Í3...(3(3)2)-''f =32<1023) i3...^3(3)2| =3,2046
:

Exemplo Resolvido 10
"x" parênteses
x
x '■
x(x(x)x)X:
Calcule x... x

Resolução: Podemos escrever:


"x" parênteses
xX ,X+1

= x ík
X

X... x(x/ ' =X

Exemplo Resolvido 11

"2016" parênteses
,.x10

Determine 29... 29Í29(29) j


24 1 Potenciação

Resolução: Podemos escrever:


"2016" parênteses
.•\1O 1O2°16 + 1-1O
E= 29 ...Í29 ^29 (29 .10 A 9.
10-1
=> E = 2

102017 10
9
9 E_ 2(102017-10)
=> E = 2

Problemas Propostos

Questão 1.11 (Harvard-MIT-2012)

Se 44“ 128/ ,2n


V22 , encontre o valor de n.

Questão 1.12
"2016"vezes
35 '

Determine

Questão 1.13
"x" vezes
Determine xx • xx •... ■ xx .

Questão 1.14
"x"vezes
x
^xx -(xx • • (xx)) j
Determine xx •
Os Segredos da Álgebra para IME/ITA/OLIMPÍADAS 25

Questão 1.15
"50" vezes
Determine a3b4 a3b4 •...•a3b4 .

Questão 1.16
"100"vezes
Determine a3b4c5 a3b4c5 •... a3b4c5 .

Questão 1.17____________________________________
Determine x ■ (x2■ (x3 )3 ■ (x4 . • (xn.

n(n + 1)(2n +1)


Sugestão: Use 12 +22 +32 + ... + n2 =
6

Questão 1.18_____________________________
Determine x2 (x2)3-(x3)4 .(x4)5 •...■(x")"*1

Sugestão: Use 1-2 + 2-3 + 3-4 + ... + n (n + 1) = n(n +1)(n + 2)

Questão 1.19
k2 3 4 .n
Determine x-^x2) j -^x3) j -^(x4j j '■■■'^(xn) j

2
Sugestão: Use 13 +23 +33 + ... + n3 = n(n +1)
2

Q u e stã o 1.20
.4 n+2
Determine

Sugestão: Use 1 • 2• 3 + 2■ 3• 4 +... + n (n +1)■ (n + 2) = n(n + 1)(n + 2)(n + 3)


4
26 1 Potenciação

Agora vamos ver as bases fracionárias e os expoentes negativos. Vamos lá!

1.3) Potência com Expoente Negativo:


Numa potência com expoente negativo, inverte-se a base, e coloca-se o
expoente positivo.
1 m 1
a~m => a~m
am

Consequências:

a) a
11
a
2
a a

b)
■or- b1
a

c) f => a b^
am

Exemplos Resolvidos 12

a) 2' 1=í-f =-
2
c)
7
4
4
7

b)
2 d)
125
3 64

1.4) Divisão Composta:


Numa divisão composta, podemos alternar os expoentes de cada termo da
divisão composta.
a = a+1-b‘1 ■c+1
; Va, b, c... e R - {0}
Os Segredos da Álgebra para IME/ITA/OLIMPÍADAS 27

Exemplos Resolvidos 13

2 = 22 3-2 = 4 2 4
a) - = 2+1 -3 • 2+1 • 3
32 9

■a+1 -b' a _a3


b) = a+1-b' ■ a+1 = a3 • b“2
1 b2'

b b

a a
b b
A a ), Aa J,

Vimos as bases fracionárias. Agora veremos os expoentes fracionários. Sigam-


me!

1.5) Potência com Expoente Fracionário:


Numa potência com expoente fracionário, o denominador é o Índice do radical,
e o numerador é o expoente do radicando.

a)

b)

c)

f 1Y
f1'fp
Vp f ijs.l
— Vm <a=almJ
d)

e)
28 1 Potenciação

Exemplos Resolvidos 14

a) 16'
ü = Vi6 ieü = 4 .

w«(?) => 16 3U f220

= JU”) 25 16

C) :7í = x>
x’

1 a'

d) J a
ía'
1
e)

Problemas Propostos

Questão 1.21 Questão 1.22

2a^ Y 2a %
Simplifique Simplifique
aX.b%' k b^3
à /

Questão 1.23 (AHSME-1954)


1 4
O valor de — a° + - 64 2 -(32) 5 é:
16

a) 1 —
16
b) 1—
16
c)1
d)^
8
e)±
16
Os Segredos da Álgebra para IME/ITA/OLIMPÍADAS 29

Questão 1.24 (AHSME-1971)

1' 1 ' i ' 2


Se S = 1 + 2’32 1 + 2’18 1+2 4 1+2 2
1+2 8 , então S é igual a:

_ 1_
1 1-2" 32
a) 1. b) 1-2 32 c) 1-2 32
2

4-^1
1
2

Questão 1.25

Reduzindo a expressão (-a2)3 (-a , obtemos:

3)ã
b) 2
c) 9

e) a9

Questão 1.26
m m
m2-1
m+1 m-1 ' m
m m +m m

Simplifique _1_
2 m
m mm +1
30 1 Potenciação

Questão 1.27

an2

Determine
íbn3J

Questão 1.28 Questão 1.29

Sabendo que n é par, determine: Sabendo que n é par, determine:


an an

•2n

a14n a"3
a-4

Aa-(-1>n'n2
Os Segredos da Álgebra para IME/ITA/OLIMPÍADAS 31

Capítulo 02 - Radiciação
Introdução
A radiciação nada mais é que a operação inversa à potenciação, ou seja, ela é
utilizada para representar, de maneira diferente, uma potência com expoente
fracionário. Vamos conhecer essa ferramenta que nos ajudará a resolver vários
problemas.

2.1) Definição:
Dado um número real não negativo a e um número natural m>1, chama-se
raiz m-ésima de a o número real não negativo b, tal que bm = a .

|t]/ã = b bm =~ã]

Exemplos de Aplicação:
Exemplo 01: 3/27 = 3 <=> 33 = 27 .

Exemplo 02: v/2401 = 7 <=> 74 = 2401 .

Exemplo 03: ^15625 = 5 <=> 56 =15625.

2.2) Como Expoente Fracionário:


A raiz m-ésima de um número a poder ser definida como sendo uma potência
de a, com expoente sendo o inverso de m, assim:

^ = a' lãl
Consequência:
Se tiver expoente, esse fica como numerador.

'Va" Jã.
Exemplo Resolvido 15: Efetue 1^ã.

Resolução: Podemos escrever: 1^ã = a


32 2 Radiciação

Resolução: Podemos escrever: W2 J?) 112 =121.

Exemplo Resolvido 17: Mostre que x/32 = 2 .

fÉ)
5^5 = 2'5' = 2=2.
Resolução: Podemos escrever: x/32

2.3) Propriedades da Radiciação:


As propriedades da radiciação facilitam os cálculos de expressões numéricas
e equações que envolvem raizes.

P1. Produto de Raizes de Mesmo índice:


Num produto de raizes de mesmo índice, repete-se o radical e efetua-se a
multiplicação.

Consequências:
a) |^.^b.^.... = ^a.b.c....|.

Exemplo Resolvido 18: Efetue x/ã ■ x/b • x/c ■ ^/d .

Resolução: x/ã • x/b ■ x/c • Vd = x/abcd .

Exemplo Resolvido 19: Efetue x/64 • x/8Í .

Resolução: Podemos escrever:


í/64 ■ í/81 = 3/64-81 = xM4 - 34 = x/lã4 =12.

Exemplo Resolvido 20: Mostre que ^•Vã3-^ = c4x/ãFT3.


Resolução: Podemos escrever.
x/ã2 ■ Vb3 ■ x/c4" = \]a2 -b3 c4 = c •
w.
Os Segredos da Álgebra para IME/ITA/OLIMPÍADAS 33

P2. Divisão de Raizes de Mesmo Índice:


Numa divisão de raizes de mesmo Índice, repete-se o radical e efetua-se a
divisão.

■q/b Nb

Consequência:
Numa divisão de raizes elevada a potência, repete-se o radical e conserva-se
a potência.

an
——. - ml-----
rpLp \ bp

6^20
Exemplo Resolvido 21: Efetue .

^20 /2 0
Resolução: Podemos escrever: —=- = 6/—
v5 v 5 ^5

a2
Exemplo Resolvido 22: Mostre que —=?- = —

Resolução: Podemos escrever:

m/„2m
va í_2m
a
— ---- = m-----
m/Ç™ V bm

1P/
Exemplo Resolvido 23: Mostre que — = 2-1^9.
1^210

Resolução: Podemos escrever:

1^22<32 '22o-32
E= = 10 _ 1^210 -32 = 2-
10/^ 21O

loç^FTã
E = = 2-1^9.
10/210
34 2 Radiciação

Problemas Propostos

Questão 2.1 (CN-1964)

. 8/r
Simplifique ■

Questão 2.2 (CN-2000)

Sabendo que = 1996®, ,/y =19944 e z4 = 19998 , com (x > 0, y > 0 e


2
z > 0). o valor de (xyz)s é:
a) 1999®
b) 19996
2
c) 1999®
d) 1999’6
e) 1999"9

Questão 2.3 (AHSME-1956)


r i . ~i4 r r- 4
Simplificando , temos:

a) a16
b) a12
c) a8
d) a4
e) a2

Questão 2.4 (AHSME-1998)

Se N > 1, então ?/N, vale:


1 1 2 13
a) N27 b) N9 c) N3 d) N27 e) N
Os Segredos da Álgebra para IME/ITA/OLIMPÍADAS 35

Questão 2.5 (Harvard-MIT-1998)__________________________


Dado que r e s são inteiros positivos e primos relativos, tal que

r 2(V2+VÍÕ)
, encontre r e s.
s 5p3 + Vs)

Questão 2.6
lya—b ] b
Simplifique a expressão —.
7b-a +2b-a

Questão 2.7

íaa+bbb+ba-b-aa
Simplifique a expressão —.
a'i2b ■ ba + b2a ■ ab

Questão 2.8

Simplifique a expressão

Questão 2.9

a aa
(aa*1
a(aa)a
Simplifique a expressão a aa
36 2 Radiciação

P3. Raiz de uma Raiz:


Para a raiz de uma raiz, repete-se o radicando e multiplicam-se os índices.

Vi
Consequências:

a) xjpJnxHi =m np Vi .
"n"vezes "n" vezes
"n"vezes
m■m•m m
b) -Vi =-Vi
"m" vezes "m" vezes
" m " vezes ,—— —

C)

"n" vezes "n"vezes


”n"vezes

d)
2"\j 2"^ 2"Vi 2m 2m ■ 2m .... 2m J2mJ2m7^/i =^ví.
dí7 c". bm -cn dp -■■■y-
e)

f) j<bn^=bm bn bP-Vi.-. =^^-ví .

J ^\/ã — • x/p '


g)

'Vn ...y-
h)

i) -Vi -Vi.

j) V
Os Segredos da Álgebra para IME/ITA/OLIMPÍADAS 37

J ’ipd "y/~
k)

^mb ■ nc • pd -...j—

^mb • mc • md ■... >—


x/a

Exemplo Resolvido 24: Efetue 7T^.


Resolução: Podemos escrever:

=> =

Exemplo Resolvido 25: Efetue

Resolução: Podemos escrever:

"2016"vezes

Exemplo Resolvido 26: Mostre que

Resolução: Podemos escrever:


vezes „2016.. ve2es -2016-yezes

V?W=3—
"2010"vezes

Exemplo Resolvido 27: Mostre que


38 2 Radiciação

Resolução: Podemos escrever:


" 2010 " vezes "2010" vezes
2010"vezes ------- •--------
-*2&jã=2—-
“10"vezes

... 210l
Exemplo Resolvido 28: Efetue \

Resolução: Podemos escrever:


”10" vezes
"10" vezes
e=21°.2’°.2^.2^ e =
E =

E = ^V2 E^t^00)

Exemplo Resolvido 29: Efetue 3Í .


Resolução: Podemos escrever:

E = 3VW- EE == ^-^V2 E = 9-^ 32/Ê = ^V2.

(m + 2)(1-m)
2
Exemplo Resolvido 30: Mostre que
Resolução: Podemos escrever:

32 -33 -34 .... r q2 + 3 + 4+...+m _


e =37J 'a ------------- Va ----------- Vã
1
f(2 + m)(m-1) (m + 2)(m-1) (m + 2)(1-m)
2 2 2
E = Í------------ : = a3
Vã = a3

6»252 -315 530 r-


Exemplo Resolvido 31: Mostre que —-------------- -Ja .
Os Segredos da Álgebra para IME/ITA/OLIMPÍADAS 39

Resolução: Podemos escrever:


_ S^.6^Íã6^°.6^y-
E=

6^2l2 315 420-530 r 6^212 31S 24° 530 6^2S2 315 530^
E =---------------------- Va = =
Exemplo Resolvido 32: Efetue

Resolução: Podemos escrever:

E=
= E.£^72
= e.2^2 = E.£3js= e.s-®í , E-^72.

Exemplo Resolvido 33: Efetue

Resolução: Podemos escrever:

Vx-Sjx6^-...^
E y/X — ------------ Vx

E=
JsHsMg) Jx

2°/a1° b15 -c16


Exemplo Resolvido 34: Mostre que ------------------- Vx .

Resolução: Podemos escrever:


= e_7S■^3 ^)- _ c_2^° 2^?.2^-
E

.16

^2y
Exemplo Resolvido 35: Mostre que '
40 2 Radiciação

Resolução: Podemos escrever:

E =
^x2^ X^2YJ ?33y
=> E =------------------------- ' X*

(33)y x (x2.2-33)y J(54x2ly



=> E =---------- => E = - Çy => e = -------- X*

Exemplo Resolvido 36: Mostre que


& &
«
I1?)
Resolução: Podemos escrever:

x“
Vx* ■ xy • xy x* ^xy <- y + y + y
E=

E = xx x
x.xHr) .■. F xm
E = xx

Problemas Propostos

Questão 2.10

3/4/5)
Simplifique ?a 4/

Questão 2.11

Simplifique n+3
|an2+5n
jan2
n/a3n2
Os Segredos da Álgebra para IME/ITA/OLIMPÍADAS 41

Questão 2.12

Simplifique

Questão 2.13
J ^16^4
Simplifique
V2 ^/4 -1^2 '

Questão 2.14
J64n +162n
Simplifique
V 8n+32n

Questão 2.15

Simplifique —\l'^=b=

Questão 2.16

=w
y
foç ÍVx b
Determine xy, se —=

Questão 2.17

Simplifique KG 75a
, para a * 0 .

Questão 2.18

Simplifique «2
r-
42 2 Radiciação

Questão 2.19

5~5 1 -5J5
5
Simplifique 5V5 ■«’ .[-WM

Questão 2.20
..,.2-9^2
|^9 l
í3J
9^2
3^'2 n-
Simplifique H3 92

Questão 2.21

Se a = bc , então , vale:

ci«H
a) c
b) 1
c) c“1

d) Vc
e) c2
Os Segredos da Álgebra para IME/ITA/OLIMPÍADAS 43

P4. Produto de Radicais de índices Diferentes:


Num produto de radicais de indices diferentes, repete-se o radicando, tira-se o
MMC dos índices, o resultado divide-se pelo Índice de cada radicando e
multiplica-se pelo expoente do radicando.

^•0/5 = -- "77“^

Consequências:
m^.n/^=^aP)n.(bq)m = ^(apn)(bqm) .
a)

b)

ap ■ 'Vb = ^(ap)m ■b ap ■ mJb = ^(am p)


c) ■b .

d)
b Vbm
a_
----- = m
e)
bp
w bp ' 1^
Vb

^an m a"
f) -------= m--------- .
~bP~
M bp Vbm p

Exemplo Resolvido 37: Efetue \/2 • í/3 .


Resolução: Podemos escrever:

E = 3V2-4V3 => E = ~TFT? E = 1^16 27 ^2 -^3 = 1?/432.

Exemplo Resolvido 38: Efetue 577.4^.


Resolução: Podemos escrever:

E = 57a7.4Ví? =, E = ^(a2)4.(b3)5 E = Mj(a2'4)-(b3'5)


.15
44 2 Radiciaçào

Exemplo Resolvido 39: Mostre que 5 ■ 3/ÍÕ = 3/1250 .


Resolução: Podemos escrever:
10 => 5 ■ 3/lÕ = 3/125-10
5.3/w = 3^ ■■10 5 • W = 3/1250 .

Exemplo Resolvido 40: Mostre que 72 ■ 13/TÕ = 1\/lO-720

Resolução: Podemos escrever:


E = 72-13/iÕ=>E = 1^72)10
■10 =>E = 1^71o2)-1O E = 1^1O-720

2016/(22014
Exemplo Resolvido 41: Mostre que
= 2016^
2
Resolução: Podemos escrever:
2016/22014 '22014 = 2°1^J E=2016^.
E= => E = 2016 — —■ E
2 22016

3/^
Exemplo Resolvido 42: Efetue — .

Resolução: Podemos escrever:


%3 3/3 _ 3| 3
3P 3P V33p

Problemas Propostos

Questão 2.22
^a""-2 Jami3
Simplifique

Questão 2.23
Simplifique 3^“ ^-3/81.
Os Segredos da Álgebra para IME/ITA/OLIMPÍADAS 45

P5. Raiz de Fração Composta 01:


A raiz de uma fração composta é igual a cada termo da fração composta
elevado a expoentes alternados.
Lll Lll
=
■^b

Consequências:
w
a)
"l/ã = a'MmM í' m lin', -cíK + m_• nL_)
•p .
mnJE
m n-p
lc

Í+-L-I
b) = mn;:.cl m-npj

C) J?/ã_
°A/b

= aK).bí'mP2j.c(+mq3)
d)
^VbP
46 2 Radiciação

e)
—— "Vã a' ■ a •... = a

^Jã

1_ 1 1 l
f)
an "Vã" =a •a a
n ■
.m J

wl
J
rr _an_

Exemplo Resolvido 43: Mostre que

ièr
Resolução: Podemos escrever:

cr
tu
Exemplo Resolvido 44: Mostre que
I a = a(3M’12hJ60U’120)

Resolução: Podemos escrever:

& .,®„(-a.c(i) j-í)


3-^/d
Os Segredos da Álgebra para IME/ITA/OLIMPÍADAS 47

í-rAÍ' I I
3-____ = 3 m3 > . 2^ m4 '
Exemplo Resolvido 45: Mostre que

Resolução: Podemos escrever:

E'O=” </3

nlísJ </2

m2+1
m4
/. E = 3'

í x2-x+1 I

Exemplo Resolvido 46: Mostre que = xv l x3 '

Resolução: Podemos escrever:


1_ 1 1
Vx => E = x.Gü.xHJJx3) • x' => E = x'
* X2 X3

4/í
í X2-X+ll
X3 X3 X3 c X3
=> E = x E = xv 1

P6. Raiz de Fração Composta 02:


A fração composta das raízes é igual ao radicando elevado a expoente
fracionários e alternados com os denominadores sendo o produto dos Índices
de acordo com as raizes que os comportam.

>
1 )
m-n-pj •cV
48 2 Radiciação

Consequências:

w
m ■ m • m /—
------ -ya

a) Vt-’ J+-)
m "GíL = a(4m3)J’m2).c(’m).
b)
m • n ■ p r-
------ Va
^te
C)
H1

= te Jd^) a(-^).aH)
m ■ n • p »—
-- ■ s/a

m c ^te _ Jm-n p -r.i..,.')


•te
H- m•n m J

^m n-p) J m-n) J+ m)

d)
V7 =a •a

f. m-m •
_ a(m n-p S . t
m-n m

m m r-
—-ya ,(* m m-m) a( m m) J+ m)

e) •te = a' ■ a

m • m ■ my

j’í. "te =a(à‘;H+' J.


Os Segredos da Álgebra para IME/ITA/OLIMPÍADAS 49

m • m m r~jr

__n___
m ■m•m
0 =a •a
m ■ m • m í~—

J® _ a
a
nU 1
m2 m

|i J3^2 ( 1 1 ( 1 >
Exemplo Resolvido 47: Mostre que 1\| I 3 3600 J 3U 120 J

Resolução: Podemos escrever:


10-12-30^

= 2(3500] 3( 120] 5(10)


—73
175

1 3125
Exemplo Resolvido 48: Efetue 725

Resolução: Podemos escrever:


5Í5^ —7^5-
f 250 "l f 125^ (25'|
V 3125 73125 => E = 2^ 53 ' ■ 3^ 52 h 7^ 5 *
E=Í 725
<2501
=> E =

<125’1 <251
5/725

=> E = 2<125M~25J A5 J E = 22 • 3~5 • 75 .

Exemplo Resolvido 49:

Resolução: Podemos escrever:


. =- 4 ■ 9 16 25,ya4025
(2^4025

E 1
4

a
a
a
=> E =
49-16Vã
___
50 2 Radiciação

=> E = a^ 4025 ) ( f 4°25__ L_ + _L_1]


4-9-16-25 J J .-4.9-16) J49) J-4) J 4-9-1625 4-916 4-9 4j
•d => E = a
( 4025 25 400 3600 ] <4025-25+ 400-3600]
I 4-916-25 “ 4-916-25 ’ 4-9-16 25 4-916-25) =>E=al 4-9-16-25 J
=> E a
f 800 1 ( 2 ]
E = a'4’9'16'25A=> E = a'-4'9' => E = a W E = 1?/i.
f l2®! Í25'|
2-3^ 32 7^
E

Exemplo Resolvido 50: Mostre que Hí


J---- X---- _ x\
X
X3-X2 + X-1
X4

Resolução: Podemos escrever:


x - x x - Xy~

E = x' ■ X •x

( 1 1 1 í f 1 X X2 X3 í x3-x2+x-1
r?’x4‘>4\4. ) «4
E = x‘ E = x' .-. E = x'

Problemas Propostos

Questão 2.24 Questão 2.25


a3b
Reduza Simplifique
3 S
rw
Questão 2.26

Simplifique
Os Segredos da Álgebra para IME/ITA/OLIMPÍADAS 51

Questão 2.27

Simplifique ^2 3/x4 77

Questão 2.28 (*)

Determine o valor de n, tal que

a) 102 b) 101 c) 100 d) 99 e) 98

Questão 2.29 (*)


n radicais

2
N x3!
X21
Sendo n um número impar, simplifique
x1!
n(n+1)
a)1 b) x 2 c) xn d) x’n e) x
a(qn-l) rq[(n-1)-qn -nqn-1+lj
(*) Observação: Use S nas questões
q-1 (q-1)'2

2.28 e 2.29.
52 2 Radiciação

A partir de agora veremos várias séries finitas. Atenção aos raciocínios das
demonstrações, pois dão uma visão além do alcance. Vejamos!

2.4) Séries Finitas de Radicais:


Nesta seção, veremos as séries finitas de radicais. São séries que facilitam
muitos cálculos e são ferramentas rápidas e eficazes na hora da prova.

S1. Multiplicação de Raizes na Forma 2m.


Esta primeira série aborda produto de radicais cujas potências aumentam em
PG, o resultado é a raiz de índice da potência 2™, eo radicando é a soma de
uma PG.
"m" vezes

'ada 'aja = a2"1"1

Demonstração:
"m" vezes "m" vezes “m" vezes

E= => E = 'a- aa2a4-...


"m" vezes
"m“ vezes
2m-1
2-2-2-...2^a1+2+4...+2m
E= => E= ’ => E= a 2-1
”m

'a1 ía2m-1

Generalização:
"m" vezes

Demonstração:
"m" vezes "m" vezes "m" vezes
an7an^ 'an -a',2n => E = 'an-a2n •a4n
E= => E = \an
"m" vezes
“m" vezes
<2n(l+2+4...+2m)
> E=
Os Segredos da Álgebra para IME/ITA/OLIMPÍADAS 53

”m” vezes

=> E =
"10"vezes
‘ ~------ - r 1O23'|
Exemplo Resolvido 51: Mostre que iJ3\/3i/3\/3 =3^1024'

Resolução: Podemos escrever:


"10“vezes Í210-i] 1024-n
e = JaVâTa
=> E = ^ ^E = 3W => E = 3
. 1024 )

"10" vezes
■ . ' = í10231
V3V373V3 =3^1024<
‘‘25" vezes z 1

Exemplo Resolvido 52: Mostre que >/l6\/l6\/l6>/Í6 = 2^ 223 I

Resolução: Podemos escrever:


"25"vezes f225-l) (225-l)
=>E = ^
E= V16Vl6x/Í6 => E = 16l 22 25 > => E = (24j, 225 I
M25"vezes
22.Í3?L1] 22.Í32ízl
=> E — 2. I 225 J => E = 2, l 22 223 >/i6a/i6VÍ6 = Z 223 <

S2. Multiplicação de Raízes na Forma m".


Esta série é uma generalização da série anterior, o resultado segue o mesmo
raciocínio.
"n” vezes

Demonstração:
"n"vezes "n”vezes
"n" vezes
|m/m| i ■a1”2
=> E a_ —
a m => E = V Va-a m

"n"vezes
"n" vezes

■••mm ml m - mm •... m
+ m + m2 + ... => E =---------------- Va1’ m + m2 -r. mn
=> E =
54 2 Radiciação

n£ í—
m -1 1|
mn-1
mn- (m -1) L'- 1
mn (m -1)
E= V ou E = a' ou E = aL

Generalização:
"n" vezes
n•
I m -1
^an ^an ^17 = V ai k 7

Demonstração:
"n”vezes "n" vezes
E= >n ^an m/gn m m 'an -anm
=o E = •
"n"vezes
"n" vezes

m mi
=> E = 'ananm ■ a™2
‘n“ vezes
mn-1
m m m ...-rn| n(umTm2 n-
m -1
E= a

n H'1)
mn (m -1)
n n
m -1 mn (m -1)
ou E = a ou E = a

"5" vezes

Exemplo Resolvido 53: Efetue

Resolução: Podemos escrever:


”5"vezes
E = ^5 . f J4 55J
í s5~11

"16" vezes

Exemplo Resolvido 54: Efetue

Resolução: Podemos escrever:


"16"Yezes 10is | fio16-il 1016/

E= 1
■•■10/7
1?/71^ => E = ~v7u°-1J => E = ~W 9 J
Os Segredos da Álgebra para IME/ITA/OLIMPÍADAS 55

S3. Divisão de Raizes na Forma 2m.


Esta série é com divisão, em vez de produto, segue o mesmo raciocínio das
séries anteriores.
"m" vezes

Va -5-7a * Vã = a 3
Demonstração:
"m"vezes "m" vezes "m" vezes

E = '\ja + \]a^y/ã a-r-a2 => E = W+a4...


"m" vezes
"m" vezes
E= 222- '27a1-2 + 4--H-2)m

"m" vezes 1-(-2)m


(-2)-1 3
=> E= ' a 7a + a

”m"vezes

; se m for par.

"m"vezes

xja -r >/a -r >/ã


se m for ímpar.

Generalização:

”m”vezes
n• 1 - (-2)m
3
a

Demonstração:
"m" vezes "m" vezes

E = ’\|anZ an-?-van => E = 'an-r-a2n


56 2 Radiciação

"m" vezes
"m" vezes

=> E =

2m n
É 22-2..,2n-(1-2.4-„. + (-2)mj
=> E = a

"m" vezes
n.ílsHT]
\jan . \jan a = \ aI ' 3 }

"m" vezes

Jan^Van- ; se m for par.


"m" vezes

an + se m for ímpar.

"8" vezes

Exemplo Resolvido 55: >J3^Z3 +73

Resolução: Podemos escrever:

"8"vezes 1-(-2)8
(1-281 í 1 - 28 1
__ === £ 3
2!
3' 3 ' E = 3' 3 28 !
E = 'tf3* 73-73 =>E= 3 =>E =

"9" vezes
Exemplo Resolvido 56: '^5 4- í—]
Võ = 5^3 2

Resolução: Podemos escrever:


"9" vezes 29/ 1-(-2)9
?! I
E= V5-75-V5 =>E = => E = 5 3 E

S4. Divisão de Raízes na Forma mn.


Essa série é uma generalização da série anterior, o resultado segue o mesmo
raciocínio.

”n"vezes
m +1
a
Os Segredos da Álgebra para IME/ITA/OLIMPÍADAS 57

Demonstração:
"n" vezes "n"vezes
"n"vezes

E= T/ 'Vã =>E = 4 mm 'a4-am


=>E =
"n" vezes
"n" vezes

=> E =

í (-m)n- 1 "n" vezes 1-(-m)n


nf
=> E= ' I l-m) -1
ak
/—m= —
a
m t-1

"n" vezes

; se n for par
n vezes

m/
= se n for ímpar

Generalização:

"n" vezes

Demonstração:
"n"vezes "n"vezes

-rí/an + 'y] m/
E= an => E an^anm

"n"vezes
"n" vezes

E=
”n" vezes
m m m ••• mn • - m + m2 - ...+(-m)nj n•
(-m) -1
E= a

"n"vezes 1-(-m)n
= m? n■

i^an^,>yãF m/
a
m +1
58 2 Radiciação

; se n for par.
"n"vezes

se n for ímpar.

"2n" vezes

Exemplo Resolvido 57: Mostre que ■107a,107^ = abi io2nJ

Resolução: Podemos escrever:


"2n“vezes 1-(-10)2n 102"í p-102n'|
102n
E= •■io7T^^a -r1 Syã => E = \a
10+1
=>E = -- \Vakl 11 '
"2n"vezes (1-102n 1
1_ 1~10<
E = a102"' 11 101'a + 1^ = abl-102nJ

"2n-1" vezes / 1 + 62"-1


7■62n-’
Exemplo Resolvido 58: Mostre que

Resolução: Podemos escrever:


W2n-1;vezes
62n~1
1+ 62n’1
62n~1 14-6';2n-1 j
6+1 7
E= ■6O 6Vx =
1 + 6;2n-1j "2n-1" vezes
I 1 ‘
6/ = xl7-62"-1 I
Os Segredos da Álgebra para IME/ITA/OLIMPÍADAS 59

Problemas Propostos

Questão 2.30

Efetue E =

Questão 2.31
Qual o valor de E = -^3® 7

Questão 2.32
Qual o valor de E^1^.'^.1^.....’0^ ?

Questão 2.33

Qual o valor de

Questão 2.34

Efetue 6J7 5
Questão 2.35
"n" radicais

X X X... x Vx
Se x * 0 e n é um número par, determine
X + Vx-r-.-.v/x
"n“ radicais
2(2" -1) 3(1- 2 4(1-2-")
a) x 3 b) x 2“ c) x 3
2(1+ 2‘n )
d) x” 3 e) x1 + 2°

Questão 2.36

Qual o valor de
"n" radicais
60 2 Radiei ação

Questão 2.37

"n“ radicais

Questão 2.38

"n" radicais

Questão 2.39

Qual o valor de
"n" radicais

íuestão 2.40

x X2 */ X3 x/ x4 x/ xx
Simplifique VX vx Vx VX

Questão 2.41

Qual o valor de
radicais

Questão 2.42

Determine o valor de

a) x|n-2»+2'

b) x("~1)+2'
Dica: Soma dos termos de uma P.A.
c) xn+2'n a(qn-l) rq[(n-1)-qn-n-qn’1+l]
d) x(n+1>+2' S q“1 + (q-1): ,2
Os Segredos da Álgebra para IME/ITA/OLIMPÍADAS 61

A partir de agora, veremos as séries infinitas. Atenção aos raciocínios que


aumentam a visão do leitor. Venha comigo!

2.5) Séries Infinitas:


Nesta seção, veremos as séries infinitas de radicais, cujos raciocinios fornecem
uma boa maturidade para quem os estuda. São séries que facilitam muitos
cálculos e são ferramentas rápidas e eficazes na hora da prova, vejamos

S1. Radicais em Soma:


Aqui veremos as séries em soma, acompanhe cada caso com atenção.
a) Soma de Radicais Simples:
/ - 1 + Vi + 4a
Va-í-...cD =------------------
2

Demonstração:
xja- Va + => x22 = a + Va + Va"^ ...a> => x2 = a + x
=> x
X=
•\22 — 4.1.(—a) => A=1 + 4a;
=> X2- x-a = 0; A = (-1)

-(-1) ± >?1 + 4a 1 + Vi -r4a


x= 2J => x
=> x=- 2

I I ' E 1 + Vi + 4a
a + va +... co =--------------- .
2

b) Soma de Radicais com Termo Fora da Raiz:


^a + bVa + bVa + ...oo^b + 7^

Demonstração:
x = Va + b Va + bVã + ...a> => x2 = a +bVa + b>/a+ ...00 => x2=a + bx

=> x2 - bx - a = 0 ; => A = (-b)2 - 4.1 .(-a) /. A = b2+4a;

-(- b)± Vb2 + 4a b + Vb2 +4a


X ~ 2.1 => x - ----------------
2
I I < b +Vb2 + 4a
va + b>/a + bva + ...oo =-------- - --------
62 2 Radiciação

c) Soma de Radicais com Termos em Produto:


b + x/b2 + 4ab
x/ab + bx/ab a-bx/ab+ ...00 =
2
Demonstração:
x = x/ab + bx/ab a-bx/ab a-...oo => x2 = ab a- bx/ab + bx/ab a- ...«>'

=> x2=ab + bx => x2-bx-ab = 0;


=> A = (-b)2 - 4.1.(-ab) => A = b2+4ab

-(- b)± x/b2 + 4ab b + x/b2 + 4ab b + x/b2 + 4ab


X “ 2J ~ 2 ' X“ 2

b + x/b2 + 4ab
.-. x/ab + b x/ab + bx/ab+ ...■» = ; VabeR+.
2

d) Soma com Termos em Produto e um Termo Fora da Raiz:


I , / . r b + Vb2 + 4ac
bx/ac +
Vac + b\/ac + bx/ac
bvac + ...oo =--------- ----------
+ ...oo

Demonstração:
x = x/ac + bx/ac + bx/ac + ...co => x2 = ac + bx/ac + bx/ac a ...oo
=> x2 = ac + bx => x2-bx-ac = 0;
=> A = (- b)2 - 4.1.(- ac) => A = b2+4ac

-(- b) ± x/b2 + 4ac b ± x/b2 + 4ac b + x/b2 + 4ac


.-. x = —----- ------------------- => x = - ---------------- -- .'. x =--------------------
2.1 2 2
b + x/b2 + 4ac
.-. x/ac a- b x/ac + bx/ac + ...oo = ; Vac e R+ .
2

e) Soma com Produto de Termos Consecutivos:


^a(a + 1) +Ja(aa1) + Ja(a +1) + ...oo
= a+1

Demonstração:
x = Ja(a + 1) + Ja(a a-1)a- Ja(a + 1) + .
..CO
Os Segredos da Álgebra para IME/ITA/OLIMPÍADAS 63

=> x2 = a (a +1) + ^a(a + 1) + 7a(a + 1) + ...CO

=> x2=a(a + 1) + x => x2-x-a(a + 1) = 0;


=> A = (-1)2-4.1.[-a(a + 1)] => A = 1 + 4a2 + 4a A = (1 + 2a)2;

(~1)±^/(1 + 2a)2 _ 1 +1 + 2a
2 +2a
X= => X = => x =-------- .-. x = 1 + a
2.1 ~ 2 2
^a(a + 1)+ Ja(a +1) + ^a(a + 1) + ...co
= a + 1 ; Va(a +1) e R+ .

f) Soma com Produto de Três Termos Consecutivos:


1 + ^/l + 4a(a + 1)(a + 2)
^a(a+ 1)(a + 2) +^/ã(a + 1)(a+ 2) + ...co =
2

Demonstração:
= Ja (a +1) (a + 2) + ^a(a + 1)(a + 2) + ^(a + 1)(a + 2) + ...co
X

> x2 = a(a + 1)(a + 2) + ^a(a + 1)(a + 2)+ ^ã(a + 1)(a + 2) + ...CO

=> x2 = a(a+ 1)(a + 2) + x => x2 - x - a(a + 1)(a + 2) = 0 ;

=> A = (-1)2-4.1.[-a(a + 1)(a + 2)J A = 1 + 4a(a + 1)(a + 2) ;

-(-1)± Vl + 4a(a + 1)(a + 2) 1+ ^1 + 4a(a + 1)(a + 2)


=> X =
X= 2.1 2
I-------------------------- , 1 +Jl + 4a(a + 1)(a + 2)
.-. ^a(a + 1)(a + 2) + 7a(a + 1)(a + 2) + ...co = —i--------- -— ----------- ;

g) Soma de Termos Consecutivos e com Termo Fora da Raiz.

(a - 2) + -ha2 +4
J a (a +1) + (a - 2) ^a(a + 1) + (a-2)^ã(a + 1) + =
2

Demonstração:
x = ^a(a + 1) + (a-2)^(ã+1) + (a-2)>/a(a + 1) + .
..oo
64 2 Radiciação

=> x2 = a (a + 1) + (a - 2)^a(a +1) + (a-2)^/a(a +1) +

=■ x2 = a(a + 1) + (a - 2)x => x2 - (a - 2)x - a(a +1) = 0 ;

=> A = [-(a - 2)]2 - 4.1 ,[-a(a + 1)] => A = (a-2)2 + 4a(a +1)

=> A = a2 - /1-á" + 4 + 4a2 + pá A = 5a2 + 4 ;


[-(a - 2)]± -Jèa2 + 4 (a - 2) + 75a2 + 4
X = => X =
25 X= 2
(a - 2) + 75a2 + 4
Ja(a +1) + (a - 2) ^a(a +1) + ...oo =
2 r
Va(a +1), (a - 2) e R+ .

h) Soma interessante:
•Ja(2a + 1) + (a + 1) ^a(2a +1) + (a + 1)^a(2a +1) +.. = 2a + 1

Demonstração:
= ^a(2a +1) + (a + 1)^a(2a +1) + (a + 1)^a(2a +
x 1)+ ...«?

=> x2 = a (2a +1) + (a + 1)^a(2a + 1) + (a + 1)^a(2:a + 1) +...00

=> x2 = a(2a +1) +(a + 1)x => x2 - (a + 1)x - a (2a +1) = 0 ;

=> A = [-(a + 1)]2 -4.1.[-a(2a + 1)] => A = a2 + 2a +1 + 8a2 + 4a

=> A = 9a2+6a + 1 A = (3a + 1)2;

[-(a + 1)]±J(3a + 1)2 3 + *1 + 3a +1


x= .-. [x = 2a + l|.
=> X =---------------------
2.1 2

i) Soma com Inverso do Produto Consecutivo:


Ja(a-1) + Ja(a-1) + Ja(a-1) +...» = a ; Va(a-1)e .

Demonstração:
x = Ja(a -1) + ^a(a-1) + 7ã(a-1) + ...00
Os Segredos da Álgebra para IME/ITA/OLIMPÍADAS 65

=> x2 = a(a-1) + y/a(a-1)+y/ã(a-1) + ...c°

=> xz=a(a-1) + x => x2 - x-a(a-1) = 0;


=> A = (-1)2 -4.1 ,[-a(a-1)] => A = 1 + 4a2-4a A = (2a-1)2;

(-1)± ^/(2a-1)2 1 + 2a-1 2a


x= ---------- i------------- => x =------------- => x = — x=a.
2.1 2 2
■. ^a(a-1)+ ^a(a-1)+ ^a(a-1) + ...x = a ; Va (a -1) e r'_ .

j) Soma com Termos em PA:

a+b

Demonstração:

a + b = J(a + b)2 => a + b = 7a2 +2ab + b2 => a + b = ya2+b(2a + b)

*2a + b = ^(2a + b)2 => 2a +b = ^[a + (a + b)]2

=> 2a + b = y]a2 + 2a(a + b) + (a + b)2

=> 2a + b = ^a2 + (a + b)(2a +a+ b) 2a + b = \|a2 +(a + b)(3a + b)

* * 3a + b = ^(3a + b)2 => 3a + b = 7[a + (2a + b)]2

=> 3a + b = yja2 + 2a(2a + b) + (2a + b)'.2

=> 3a + b = ^a2 + (2a + b)(2a + 2a + b)

=> 3a + b = ja2 + (2a + b)(4a + b) • ■ •

Logo, temos:
a + b = ^a2 + b(2a + b) => a + b = ^a2 + b^a2 +(a + b)(3a + b)

=> a + b = ^|a2+bjã2+(a + b) Jã2 + (2a + b)(4a + b)


66 2 Radiciação

a+b .

Exemplo Resolvido 59: Efetue 72 + 72 + ...oo .


Resolução: Podemos escrever:
E = 72+ 72 + ...co =. E = 1 + ^2+4'2 1 + 7T+8 1 + 79
=> E = => E —
2 2
E = l^ .■. x/2 + \/2 + ...co = 2 .
2

Exemplo Resolvido 60: Qual o valor de ^4+ 3^4+ 374 + .. ?

Resolução: Podemos escrever:

E = 74 + 374 + ..Z => E =


3 + 7^+ 4-4 => E =
2
3 + 725 3 +5 8 r- A
=> E = => E =------- => E = - => E = 4 .
2 2 2

Exemplo Resolvido 61: Qual o valor de 7l2 + 4712 + ...co = 6 ?

Resolução: Podemos escrever:


E = 7l2 + 47Í2 + ...°o => E = 73 ■ 4 + 473^4 +...00
4 + 742 + 4-3-4 4 + 716 + 48 4+764
=> E = 2 => E = => E =
2 2
4+8 r- 12
7l2 + 47Í2 + ...co = 6.
=> t =-------- => E = —
2 2

Exemplo Resolvido 62: Efetue 73-8 + 573 8 + ...

Resolução: Podemos escrever:


I------------ , 5 + 7õ2 + 4-3-8 5+725 + 96
E = 73-8 + 573 ■ 8 +...oo => E =-------------
2 2
5 + 7Í2Í 5 + 11 16 .-. 73-8 + 57^8 + ...co = 8.
=> E = => E =-------- => E = —
2 2 2
Os Segredos da Álgebra para IME/ITA/OLIMPÍADAS 67

Exemplo Resolvido 63: Qual o valor de 742 + 742 + 742 +.. ?

Resolução: Podemos escrever:

E = 742 + 742 + 742+777 => E = Í6- 7 + |6- 7 + 6- 7 +...co


6+1 6+1 7 6+1

E = 6 +1 742 + 742 + 742 + .

Exemplo Resolvido 64: Efetue 72■2•3 + 72 ■ 2 ■ 3 +.. .00 .

Resolução: Podemos escrever:


E = 72 • 2 ■ 3 + 72 • 2 ■ 3 + ...■»

=> E = 72(1 + 1)(1 + 2) + 72(1 + 1)(1 + 2) + ...a

1+71 + 4 2 2-3 E 1+71 + 48 1 + 749


=> E = => E = => E =
2 " 2 2
1+7 8
=> E =------ => E = — => 72-2-3 + ^-2-3 + ...co = 4.
2 2

Exemplo Resolvido 65: Efetue 772 + 6-^72 + 6772 + 6...00 .


Resolução: Podemos escrever:

E = 772 + 6772 + 6772 + 6...co => E = 78•9 + 678-9 + 678-9 + 6...cc


(8-2) + 75-82 4
=> E = ^8(8 +1) + (8 - 2)78(8+ 1) + ...m => E =
2
6 + 7320 + 4 E 6 + 7324 E = 6±18 . E = 12
=> E = => E =
2 " 2 2

Exemplo Resolvido 66: Efetue 7210 + 117210 +117. .

Resolução: Podemos escrever:


E = ^210 + 11^210 +1 iV...» = ^10(21)+ 11^/10(2-10+i) + ii777í

E = ^10(2-10 + 1) + (10 + 1) 710(2 10 + 1) + ...co = 2-10 + 1 E = 21


68 2 Radiciação

Exemplo Resolvido 67: Qual o valor de 712-11 + 712-11 + ...00 ?


Resolução: Podemos escrever:
7l2-11 + 7Í2-11 + ...CO = ^12 (12-1) +712 (12-1) + . ..co
=• 7l2-11 + 7Í2-11 + ...oo =12.

Exemplo Resolvido 68: Qual o valor de yjQ + 7^9+ 10^9 + 137...


?
Resolução: Podemos escrever:
E = \]9 + 7^9+ 10^9 +1377?»

=■ E = j32+7^32+(3 + 7)^2 + (2-3 + 7)77»


=» E = 3 + 7

.-. 79 + 7Va +1 o>/9Ti37777^ =10.

Problemas Propostos

Questão 2,43 (AHSME-1954/Stanford-2010)

Se x =

a) x = 1 b) 0 < x < 1
c) 1 < x < 2 d) infinito
e) x > 2 , mas finito

Questão 2.44 (Harvard-MIT-2000)

Qual o valor de

Questão 2.45 (Harvard-MIT-2000)

Qual o valor de
Os Segredos da Álgebra para IME/ITA/OLIMPÍADAS 69

Questão 2.46

Qual o valor de

Questão 2.47

Qual o valor de

Questão 2.48

Qual o valor de ?

Questão 2.49

Qual o valor de

Questão 2.50

Qual o valor de

Questão 2.51

Qual o valor de
I o / 2 (x + n)^a(x + 2n) + (n + a)2 + (x + 2n)... ?
lax + (n + a) + x Ja(x + n) + (n + a)

Questão 2.52
^60 + ^60 + ^60+77 ?
Qual o valor de
70 2 Radiciaçâo

S2. Radicais em Diferença:


a) Diferença de Radicais Simples:
/------ / -1 +Jl + 4a
a - Va -...» =------------------
2
Demonstração:
x= a - \la - -7a -7..co => x2 = a - Va - Va - ...<» => x2=a-x

=> x2 + x - a = 0 ; => A = 12-4.1.(-a) => A = 1 + 4a;

-1 ± V1 + 4a -1 + Vl + 4a
x =----------------- => x =-----------------
2.1 2
I i -1 + Vl + 4a
a-Va-...co =----------------- .
2

b) Diferença de Termos Consecutivos:


Ja(a + 1)- Ja(a + 1)- Jã(a +1) - ...= =a ; Va(a + 1)e .

Demonstração:
x = ^a(a +1) - N/a(a + 1)-/a(a + 1)-...cor

=> x2 = a (a +1) - Ja (a + 1) - ^a(a + 1)-...oo

=> x2=a(a + 1)-x => x2 + x - a(a +1) = 0 ;


=> a = 12-4.1.[-a(a + 1)] => A = 1 + 4a2+4a A = (1 + 2a)2;

-1± 7(1 + 2a)2 -1 + 1 +2a 2a


X = => x =--------------- => x = — .'. x = a .
2.1 2 2
.-. ^a(a +1) - Ja(a +1)- ^a(a +1) -.
..oo = a ; Va(a + 1) e R+ .

c) Diferença do Inverso de Termos Consecutivos:

a = a-1 ; Va(a-1)eR*+ .
Os Segredos da Álgebra para IME/ITA/OLIMPÍADAS 71

Demonstração:

x= (a -1) - ^a(a-1)~7a(a-1)->í7Í

=> x2 = a (a -1) - ^a(a-1)-7á(a-1)-^..oc

=> x2=a(a-1)-x => x2 + x-a(a-1) = 0 ;

=> A = 12-4.1 ,[-a(a-1)J => A = 1 + 4a2-4a A = (2a-1)2;

7(23-1)2 -1■ + -2a-1' -2a-2-


x= => x =--------------- => x =--------- /. x = a -1.
2.1 2 2
.-. 7a(a-1)-7a(a -1)- ,/a(a -1)-5/...'jo =a-1 ; Va(a-1)e R'+ .

d) Diferença interessante:

= a -1

Demonstração:

x = ^2a(a -1) - (a +1) ^2a(a-1) -(a + 1)^2a(a-1)-(a + 1) x/...co

=> x2 = 2a(a -1)- (a +1)^2a(a -1)- (a + 1)^2a(a -1) -(a +1) V...«j

=> x2 = 2a(a-1)-(a + 1)x => x2 + (a + 1)x - 2a(a -1) = 0 ;

=> A = (a+ 1)2-4.1.[-2a(a -1)] => A = a2 + 2a +1 +8a2 - 8a

(a + 1)±7(3a-1)z
=> A = 9a2-6a + 1 .-. A = (3a-1)2; => x =
2.1
-a-1 + 3a-1 2a-2
=> X =----------------------- => X =--------- - x = a-1.
2 2
.-. 72a(a-1)-(a-|-1)V2a(a-1)-(a-1)>^°° = a-1 ; Va(a-1) e K*. .
72 2 Radiciação

Exemplo Resolvido 69: Qual o valor de 720-720-...co ?


Resolução: Podemos escrever:
-1 + 71 + 80
720-720-7/T = -1 + 71 + 4-20 => 720-720-...00 =
2 2
-1 + 781 = -1 + 9
=> 720-720-7?^ = => 720-720-/..co =--------
2 2
= 8
=> 720 - 720-.'..00 = — 720 - 720 - .Tot = 4 .
2

Exemplo Resolvido 70: Efetue 7s■ 6- V5-6-75-6-...co .

Resolução: Podemos escrever:


75-6-75^6-...□□ = 75 • (5 +1) - 75 (5 + 1)-75 (5 + 1)-...Z
.-. 7õ ■ 6-75-6-75^6-.= 5 .

Exemplo Resolvido 71: Qual o valor de 7l00 99- 7l00-99-7...co ?

Resolução: Podemos escrever:


7l 00-99-^100-99 - V...CO = ^100(100 -1) - 7100(100-1) - >/...oo

=> Jl00-99~7l 00-99-7777 =100-1

=> V100 ■ 99 - 7l 00-99-7777 = 99 .

Exemplo Resolvido 72: Efetue 7l 2 -4>/l2-47...cõ .

Resolução: Podemos escrever:

7i2-4V?2 -4^...00 = 72-3-2-4V2-3-2-4V?TZ =>


7l 2 - 4/Í2 - = ^2 ■ 3 • (3 -1) - (3 +1) 72 • 3 • (3 -1) - (3? '...CO

=> y12 — 47l 2 — 4-7770 =3-1 => 7l2-47l2 - 47...00 =2.


Os Segredos da Álgebra para IME/ITA/OLIMPÍADAS 73

S3. Radicais Alternados


a) Radicais Alternados Simples:

1 + 71 + 4a _ .
--------------- ; 0 < a < 1
2
7 a - Va + Ví-...<» =
-1 +V4a-3
a>1
2

Demonstração: Nesta demonstração usaremos uma técnica de fatoração que


será mostrada no capitulo sobre fatoração.

x = Va - Va + Va
Vã-— ...oo => x2 = a - 7a + x fa + x = a - x2

=> a + x = (a -x2)|2 => a + x = a2 -2ax2 + x4

=> x4 -2ax2 + a2-x-a = 0

Somando e subtraindo x3, x2 e ax, temos:


x4 -|2ax21 + a2-x-a = 0

=> x4 - ax2 - ax2 + a2 - x - a + x3 - x3 +, x2 - „x2 ++ ax


ax - ax = 0

=> x2(x2 -a-xj + x(x2 - x-a)-a(x2 -a-xj + ^x2 -x-a) = 0

=> (x2-a-x)(x2 + x-a + l) = 0 x2- x-a = 0 ou x2 + x- a + 1 = 0

, .,2 . . ■
x2-x-a = 0 => A = (-1)4 - 4.1 .(-a)- A = 1 + 4a;
x = ~(-1)±Vl + 4a
2.1
X2 rX-a + 1 = 0 => A = 12-4.1.(-a-í-1) A = 4a-3;
-1± 74a-3 —1 + 74a — 3
x =------- X =--------------------- a > 1.
2.1 2

b) Radicais Alternados de Termos Consecutivos:

^a (a +1) - Ja(a + 1) = a ; Va > 0 .


74 2 Radiciação

Demonstração:
x = Ja(a +1) - ^a(a + 1) + ^(a + 1) — x/. ..co

=> x2 = a(a + 1)-^a(a + 1) + x => ^a(a + 1) = a(a + 1)- x2

= (a(a + 1)-x2]2
=> a(a + 1)

=> a (a + 1) + x = [a (a +1)]2 -2a (a + 1)xz + x4

=> x4 -2a(a + 1)x2-x-a(a + 1) + [a(a + 1)]2 =0

Fazendo k = a(a + 1), temos uma equação do 2° grau em função de k (*):

x4 -2a(a + 1)x2 - x-a(a+ 1) + [a(a+ 1)]2 =0

=> x4 -2kx2 -x -k + k2=0 => k2-(2x2 +l)k + (x4-x) = O

=> A = [-(2x2+1^]2 -4.1.(x4 -x)


=> A = 4x4 +4x2 + 1-4x4 +4x

=> A = 4x2+4x + 1 => A = (2x + 1)2

_-[-(2x2+l)]±7(2x + 1)2
2x2 +1±2x + 1
=> k =
2/í 2
2x2+1 + 2x + 1 2x2 +1-(2x + 1)
=> kt=--------------------- - ou k1 “ 2

k-, = x2 + x + 1 ou k
I 1 = x2 +

(*) Observação: É muito interessante essa técnica de encontrar as raizes de


uma equação, em função do parâmetro.
Assim, temos:
x4-2kx2-x-k + k2 = 0 => 1-[k-(x2 + x + l)].[k - (x2 + x)J = 0
=> [a(a +1) - (x2 + x + l)]-[a(a + 1)-(x2 + x)] = 0

a(a + 1)-(x2 + x + 1I: = 0 ou


ou a(a + 1)-(x2 + x) = 0

x2 + x +1 - a(a +1) = 0 (não serve) ou x2 + x - a (a + 1) = 0

x2 + x-a(a + 1) = 0 => x2 + x-a-a2=0 => x2-a2 + x-a = 0

=> (x + a)(x-a) + x-a = 0 => (x-a)(x + a + 1) = 0


Os Segredos da Álgebra para IME/ITA/OLIMPÍADAS 75

=> x - a = 0 |x = a| ou x + a + 1 = 0 => [7 = —a —1| (não serve)

Logo:

= a ; Va > 0.

3 3 '3 3
Exemplo Resolvido 73: Mostre que ---- ...co = — .
4 4 4 2

3
Resolução: Note que 0 < - < 1, então:
4

1 + Í1 + 4 • 3
'3 4
E= E=— =s E =
4 2

1+74 => E = l^ '3 2 I3 3


=o E = ---- ...co = —
2 2 4 4 4 2

Exemplo Resolvido 74: Mostre que \/3-a/3 + x/3-...co = 1.

Resolução: Note que 3 > 1, então:

E = 73->/3 + V3-...<o => E = ~1 + ^'3~3_


=> E = => E =
2

=> E = => E = Z1±3 => E = 2 73 - 73 + = 1.


2 2 2

Exemplo Resolvido 75: Efetue 78-9- ^8 ■ 9 + V8~9 - 77?õõ .

Resolução: Podemos escrever:


^8 • 9 - ^8 ■ 9 + ^8 ■ 9 - = ^8(8 + 1)- ^8(8 +1) + ^8(8 + 1) - 77^

■\J8-9- 8.
76 2 Radiciação

Problemas Propostos

Questão 2.53

'21
Qual o valor de 11-
16

Questão 2.54 (Rússia/IMO-Longlist-1969)


Prove que, para a > b2 , ocorre a identidade
/ K / K I-----K /-------- I 3b2 b
>Ja-b\la + bJa-b>/a + ... = Ja--------
V 4 2

Questão 2.55

Qual o valor de

Questão 2.56

Qual o valor de

Questão 2.57

Qual o valor de 2450-x/2450 —

Questão 2.58

Qual o valor de 77^


Questão 2.59

33 '33
Qual o valor de
64 64
Os Segredos da Álgebra para IME/ITA/OLIMPÍADAS 77

S4. Radicais em Produto:

a 'ava...cc = a

Demonstração:
x = 7a7ax/ãT^» => x2 = ax x = 0 ou x = a
=a .

a) Generalização do Produto Anterior, para Raiz de índice m:

V^a.-.co = ——Vã ; Vm e R - {0,1} e a > 0 .

Demonstração:

x = => xm = ax => — = a => xm-1=a


X
m
x='—vã ^a...w = ——Vã ; Vm e R - {0,1} e a > 0 .

Outra forma:
1
1 Am 1
= am“1
a a(a...co)m ; Vm gX-{0,1} e a>0.

Demonstração:
2
2' m
2 m
— = a => xm-1 = a
x = a a(a...co)m => xm = ax =>
X

2
1 1 Am 1
x = am"1 , J-'lm
a a(a...co)m = am-1 ; Vm e R - {0,1} e a > 0 .

b) Generalização para n-ésima Potência de índice m.

m/
T/an Vm gR - {0,1] e a > 0 .
78 2 Radiciação

Demonstração:
x=mJan 'V^an...oo í2 = an
=» x = ^'an -x xm = an ■ x
X
:m-1 =an x = ~>/ã".
=> x’

TJan m/an n\/ Vm e 111 - {0,1} e a > 0 .

Outra forma:
1
n
2 'im
a a(a...co)m = am"1 ; Vm e R - {0,1} e a > 0 .

Demonstração:
1
2> m
2' m 2
X = an an(an...»)m => x = (an •x)"’ xm = an • x

n
íZ = an => Xm"1 = an x = a'm-1
x
1
2) m n
2 = am“1
a a(a...co)m ; Vm e R - {0,1} e a > 0 .

Exemplo Resolvido 76: Efetue 00^1 00 Vi00.. .°o .

Resolução: É imediato que J100J1 00>/100...co = 100 .

Exemplo Resolvido 77: Efetue ^2^2^27^ = \/2 .

Resolução: Podemos escrever:


^2^77=—^ ^2^2^2...» = $2 .
Os Segredos da Álgebra para IME/ITA/OLIMPÍADAS 79

f i I9
Exemplo Resollvido 78: Mostre que 81 8(8...00)9

Resolução: Podemos escrever:


1
11
(
8 8(8...oo)g
1)9 * = 8<í9—-1-I1 =• í f
8 8(8...00)9
|9
9 í 11
= 8v8J

S5. Radicais em Divisão


a) Radicais Simples:

Demonstração:
X = 7a -r Jãã-...» => x2=a-s-x => x2 = — => x3=a
x
x = x/ã \[t

b) Generalização da Raiz Anterior para Raiz m-ésima:


r\la + r7ã+ l\Za-s-..S = Jã ; Vm e R - {0,1} e a > 0 .

Demonstração:
x = 2a-rVaTrVa-...» xm’1
m +1 /—
=> xm = a -i- x => = a /. x =----- x/a

m/
...» = —--Jã ; Vm e R - {0,1} e a>0.

Outra forma:
1
1
1 Im
a -5- a-(a-...co;')m = amt1 ; tfm e X - {0,1} e a > 0 .
80 2 Radiciação

Demonstração:
1
Um
1 Im a
x= a -r a*(a-s-...oo•)m xm = a -e- x => xm
x
k
1
xm + 1 = a x = am + 1
1
1
_ glT) +1 ; tfm e IR -{0,1} e a > 0.
a -e- a + (a + ...»Jm

c) Generalização Para a n-ésima Potência e Raiz de índice m.

^.^a" a-^a" a-...00 = —; Vm e R - {0,1} e a > 0 .

Demonstração:
* = Ían+n£^an=> x = rx/ana-x
=> x = xm = an -r x
xm + 1 = an x = ~■7a"
xm • x = an

’an+ \lan+...°o = —-jã" ; Vm e R - {0,1} e a > 0 .

Outra forma:

1 'j m n
1 'Im
a-(a*. . .cojm = am + 1 ; Vm e R- {0,1} e a > 0 .

Demonstração:
1
1 'm

X =
an -J an *(an -e-...»)™
m
=> x = (an :)(a
Os Segredos da Álgebra para IME/ITA/OLIMPÍADAS 81

n
=> xm=an+x => xm = — xm*1 =an x = am + 1
X
1
1 'Im
n
_ gOl + 1
an + an (an ; Vm g R - {0,1} e a > 0 .

d) Divisão Composta 01:


a
= m^

i a

'ífl
m

Demonstração:

a xm = ? = xm.1 „a
x= X

m
m

I ■0T a
ce

- ="W

I ífi"fll
a
82 2 Radiciação

e) Divisão Composta 02:

&

Demonstração:

p.
mm m

m-m /—
------ Va
-^1
m mm J
=a(íi-^+^+
= a

p '■f-
■p-
1
m+1 N a
m m a _3(m + V
rj

Exemplo Resolvido 79: Efetue 7274 727 4727 4-/..00 .

Resolução: Podemos escrever:

727 4^27 4-7277 ...CO = 727 => 727-x/27-4 727 -5-,..00 = 3 .

Exemplo Resolvido 80: Efetue ^10244 7l0244-...« .

Resolução: Podemos escrever:

024 4-71024 4-...co = —71024 7l024 4TÍÕ24 4...» = 71024

=> 5/1024471024 4-...» = 7?° x/l024 4 TÍ024 4...» =4.

_1_ A
( — 110 1° =11lnj.
m
Exemplo Resolvido 81: Mostre que 11 + 11 + (11 + ...co)io
Os Segredos da Álgebra para IME/ITA/OLIMPÍADAS 83

Resolução: Podemos escrever:


_i
2' 10 1
( 1 110
11-r 11-r(l1-r...<Z))l0 = ^10 + 1

1 A
( 1 V0 ” = 11^
11-5- 11-=-(1l4-. ..00)10
(-1

Exemplo Resolvido 82: Mostre que


64
= .

2016

Resolução: Podemos escrever:

64 2016/ZT 64 = 2016^6
- ------ V64

2016 2016
2016

I I
2016
64

2016 2016
2016

I I
84 2 Radiciação

6^128
V 128
Exemplo Resolvido 83: Qual o valor de 128 ?
CO

Resolução: Podemos escrever:


Ü^Í28
V 128
3 128 ' = 64.

S6. Radicais em Cadeia


a) Potência em cadeia infinita 01:

Se xx' = a , então =&.


Demonstração:

xx xx
X =a xa = a X = a^.

b) Potência em cadeia infinita 02:

=x
Demonstração:

= a => (Vx) = a => í/x = => x = a =x

c) Potência em cadeia infinita 03:

=a
Demonstração:

_i_
■X 1
= X => = x => \/ã = x a£ = x^X
Os Segredos da Álgebra para IME/ITA/OLIMPÍADAS 85

X X
=> ab’1 = xb’a.
= xb3 => ab 1

Por comparação, temos:

X
x
b' =b a -1 = - - => x = a =a
a -------------------
Observação: Se x*a , não poderiamos usar o resultado.

d) Potência em cadeia infinita 04:

X
x
Se x = a, então x =a a

Demonstração:

X
'x = a => x7* =
£
X a => x = a X = aa

e) Potência em cadeia infinita 05:

Se = a , então -a a

Demonstração:

_1_
x = a => xn^ =
a => x =

f) Potência em cadeia infinita 06:

= xx
Demonstração:
86 2 Radiciação

Fazendo Vã = y => a = yx, temos :


=s> xy = a => xy = yx => x = y (Por comparação)

=> a = yx => a = xx

Observação: Para x = 2 ex = 4, temos duas soluções, visto que:

24 = 42 e 42 = 24.
Logo:

2’"2
|x = 2| => 2’2 =a => 2^=a

Fazendo Vã = y => a = y2, temos :


=> 2y=a => 2y = y2 => y = 2 ou y = 4 (Por comparação)

=> a = 22 |a = 4| ou a = 24 |a = 16|.

x = 4| => 4 = a => 4^ = a.

Fazendo Vã = y => a = y4, temos:


=> 4y=a => 4y=y4 => y = 2 ouy = 4 (Por comparação)

=> a = 42 |a = 16| ou a = 44 |a = 256|.

Exemplo Resolvido 84: Se x = 5 , qual o valor de x?


Resolução: Podemos escrever:

X xx = 5 => x5 = 5 => x = ^5.

Vê67®
Exemplo Resolvido 85: Qual o valor de ^6 ?
Os Segredos da Álgebra para IME/ITA/OLIMPÍADAS 87

' 6/—
Resolução: E imediato que 76 =6.

■4J?'
4r-^4
Exemplo Resolvido 86: Qual o valor de v7 ?

■tff
Resolução: E imediato que T7 =7 .

Problemas Propostos

Questão 2.60

7
.00
Qual o valor de E = — —?

Questão 2.61

26
x9+26J x9-26J x9 -5-,..00
Qual o valor de E = — ?

Questão 2.62 Questão 2.63


32
Simplifique ÍW
729
Simplifique 729
oc
4J
88 2 Radiciação

Questão 2.64

Qual o valor de x, sabendo que x,xx = 2017?

Questão 2.65

Qual o valor de x, sabendo que x:xx = 27?

Questão 2.66

Qual o valor de 1Ç/TÕ

Questão 2.67
24^243 24^?
Qual o valor de 24?/243 ?

Questão 2.68

■X (w) .
Se Mm = e M 5V ' , determine x.
a)1 b) 3 c) 5 d) 7 e) 9

Questão 2.69

.2^

Qual o valor de 2J2 ?

Questão 2.70

7^(5^)
Qual o valor de (5V3 j ?

Questão 2.71

Qual o valor de x, sabendo que x = 625?


Os Segredos da Álgebra para IME/ITA/OLIMPÍADAS 89

Questão 2.72

Qual o valor de x, sabendo que x 1024?

Questão 2.73

Qual o valor de x, sabendo que x

Questão 2.74

Qual o valor de x, sabendo que x =32?

Questão 2.75

7 ' 77’7
Qual o valor de 7 ' ?

Questão 2.76

Qual o valor de 13
90 2 Radiciação

Desse ponto em diante, veremos as operações com radicais. Para efetuar soma
e subtração, usaremos a estratégia de “colocar em evidência" e então
efetuaremos a operação. Para efetuar multiplicação e divisão, usaremos as
propriedades de multiplicação e divisão vistas anteriormente.
Vejamos a seguir como operar com radicais!

2.6) Operações com Radicais: ______________


Nesta seção, iremos dividir em duas partes: a parte "a" será a adição e a
subtração, e a parte "b" será a multiplicação e a divisão. Para efetuar as
operações com radicais da parte “a”, devemos verificar se os radicais são
semelhantes, isto é, se os radicais têm o mesmo Índice.

a) Adição e Subtração:
Podemos reduzir os radicais em uma soma ou diferença, desde que sejam
radicais semelhantes.

Exemplo Resolvido 87: Efetue 3VÍ + 7VÍ .


Resolução: Podemos escrever:
3V4+7W = (3 + 7)VÍ => 3VÍ + 7VÍ = 1 oVÍ .

Exemplo Resolvido 88: Efetue 2oVã +8Vã-l4Vã .


Resolução: Podemos escrever:
20Vã + 8Vã-14^ = (20 + 8-14)^ã 20^ã + 8x/ã-14\/ã = 14^ã.

Exemplo Resolvido 89: Efetue mvÇz+p^ + q^.


Resolução: Podemos escrever:
+ p\Zã^ + q\íã* = (m + p + q)1^^.

Podemos também manipular os radicais de modo a ter termos semelhantes,


vejamos alguns exemplos.

Exemplo Resolvido 90: Efetue Vã + Vãã .


Resolução: Podemos escrever:
Vã + Vãã = Ví~ã + V16-2 => Vã + Vãã = Ví•Vã + VTã■ Vã
=> Vã + V3ã = 2-Vã + 4-Vã => Vã + Vãã = (2 + 4)Vã
=> Vã + Vãã = 6Vã.
Os Segredos da Álgebra para IME/ITA/OLIMPÍADAS 91

Exemplo 91: Mostre que 2^ã-3/27a4 + 3/125a = (7 -3a) 3/ã.

Resolução: Podemos escrever:


23/ã - ^27a4 + 3/125a = 23/ã - 3/27 a3 a + 3/125 a

=> 2^ã-v,27a4 +3/l25a = 23^-3/27 -3/ã3■ 34ã + 3/Í25 ■ 3/ã

=> 2%ã - y/27aA + 3/125a = 23/ã - 3 ■ a ■ 3/ã + 5 ■ 3/ã

=> 2^/ã - ^27a4 + 25a = (2 - 3a + 5) 3/ã

23/ã - ^27aA + 3/125a = (7 - 3a)3/ã .

b) Multiplicação e Divisão:
Para a multiplicação e divisão, devemos considerar dois casos: se os radicais
tiverem o mesmo Índice e se os radicais tiverem índices diferentes.

Caso 01: Se os radicais possuem o mesmo índice.


Se os radicais possuem o mesmo indice, você vai utilizar a propriedade do
produto de radicais de mesmo índice.

Exemplo Resolvido 92: Efetue 2-42-sJÃÁ .


Resolução: Podemos escrever:
272 -5>/TÍ =10^2-11 => 2^2 -5Vil =10^22 .

Exemplo Resolvido 93: Mostre que 3^-4^ ■ • 2\/d = 24 -3/abc2d .


Resolução: Podemos escrever:
3$ã ■ 4l//b • tfc? ■ 2Q/d = 3-42 \/a b c2 d

=> 3^ ■ 4^ ■ ^/c2” • 2\/d = 24-^bc^d .

Exemplo Resolvido 94: Efetute m .

Resolução: —= m.nE
p!Vb P Vb
92 2 Radiciação

10 Võ
Exemplo Resolvido 95: Efetue
4^3 '
Resolução: Podemos escrever:
10^5 10 Jõ 10^5 5 3Í5
4^3 ” 4 V3 43/3 "2 V3 '

Caso 02: Se os radicais tiverem indices diferentes, devemos efetuar os


seguintes passos:
Passo 01: Tira-se o mmc dos indices. o mmc será o novo Índice.
Passo 02: Efetua-se a divisão do mmc por cada Índice anterior, e o resultado
multiplica-se pelo expoente do radicando.
Passo 03: Efetua-se a operação como no caso anterior.

Exemplo Resolvido 96: Efetue Vã • Vb ■ V8 .

Resolução: Efetuando os três passos acima, temos:


/ / o \fnn
E = rVã-VbV8 E = mn^/anP mn^bmP mnP/gmn => E = mWan bm (23)

E= ■ bm 23mn

Exemplo Resolvido 97: Efetue 2Vã ■ Vl8a2 ■ V25a .

Resolução: Podemos escrever:

E = 2 -Vã -\/l8a2 -VÜã => E = 2-1\/Ç4 • 1^(l8a2)3 • 1^(25a)2

1x/ãT4 ■ 1<7l 83 a6 •1^252 -a2 => E = 2-1^a4 -(2 ■ 32)3 a6 -252 a2


E = 2-

2 -1^a4+6+2 ■ 23 ■ 36 (52)2 => E = 2■ 1Va12 23 -36 -54


=> E

.-. E = 2a ■ 1\/23 ■ 36 ■ 54 .
Os Segredos da Álgebra para IME/ITA/OLIMPÍADAS 93

Capítulo 03 - Racionalização

Introdução
Racionalizar uma fração consiste em eliminar o radical (ou os radicais) que
estiverem no denominador, para tal tarefa usamos o chamado fator de
racionalização.

3.1) Quocientes Notáveis


Os quocientes notáveis são expressões que vêm diretamente dos produtos
notáveis, ferramenta que você verá mais à frente. Serão muito usados na
racionalização, nesta seção veremos os mais utilizados e a generalização, mais
à frente veremos mais dessa ferramenta fortíssima!
a2-b2
a) —------- = a + b .
a-b
a3-b3
b) = a2 + ab + b2.
a-b
a3 + b3 2 u u2
c) - ------- = a - ab + b .
a+b
a4-b4
d) = (a + b)(a2+b2].
a-b
as-b5 = a4 + a3b + a2b2 + ab3 + b4.
e)
a-b
a5+b5
f) = a4 -a3b + a2b2 -ab3 +b4.
a+b
—~bn = an-1 + an-2b + an-3b2 + + bn-1
g)
a-b
^ = a--a'
h) 2b + a' 3b2-...-bn-1, para n par.
a+b
an-bn , 2bn
i) ---------- = a ,n-1 - a 2b + an-3b2-,.. + bn --------- , para n impar .
a+b a+b
an+bn ,
j) ---------- = a in"1 - an~2b + a' 3b2 -,.. + bn"1, para n ímpar.
a+b
an+bn
k) = an-1-a' 2b + an~3b2 -,..-bn 1 + , para n par .
a+b a +b
94 3 Racionalização

an--bn
I) = an~1+a' 2b + a1 ’2b2 +... + bn~1 + ——
a -b a -b

3.2) Fator Racionalizante


O fator racionalizante é o fator que elimina o radical quando efetuamos a
multiplicação pela fração. Vejamos alguns casos em que usamos o fator
racionalizante e outros bem interessantes.

Caso 01: Quando o denominador é da forma VÃ :


Note que nesse caso ao multiplicarmos o denominador por VÃ , fica:

N N VÃ N N -VÃ N N VÃ
Và “VÃ'Và VÓ^)2 /à A

Assim, VÃ é o fator racionalizante.


75
Exemplo Resolvido 98: Mostre que =
72 2
Resolução: Podemos escrever:
1 1 1 72 1 _V2
72 ~ 72 72 72 “ 72 2

Exemplo Resolvido 99: Mostre que -X= = 7-VÍÕ


Viõ 10
Resolução: Podemos escrever:
7 = 7 VTÕ 7 = 7-VÍÕ 7 7-VÍÕ
Viõ “ Viõ Viõ Viõ " (VTõ)2 Viõ 10

3 /3 3
Exemplo Resolvido 100: Mostre que -V= =
VÍ2 2
Resolução: Podemos escrever:
373 _ 3>/3 VÍ2 3y/3 _ 373 ■ 12 3y/3 _ 3736
VÍ2 “ VÍ2 VÍ2 VÍ2 “ (VÍ2)2 VÍ2 “ 12

373 3-6 3Õ3 18 373 3


=> vn = r
=> ■■■:------------ = -------------------- =>

VÍ2 12 VÍ2 ” 12
Os Segredos da Álgebra para IME/ITA/OLIMPÍADAS 95

Caso 02: Quando o denominador é da forma Va™ :


Note que nesse caso ao . fica:

N N Va"~™ n n Va"-™
VÙ VÙ Va"-™ VÙ VÙ • Va"-™
N nVa"~™ N n ■ Va"-™
VÙ VÙ •An"™ VÙ ry^m+n-m

N N ■ Va"~™ N
VÙ VÃ" VÙ A

Assim, Va" ™ é o fator racionalizante.

Exemplo Resolvido 101: Mostre que


3
=
3-W
10
Resolução: Podemos escrever:

3 3 i<y2io-3 3 3V27 3 3 1\^~


10^3"
1V? 1 Va1 0-3 10^5“ ~ ■1V27
3 3-'1VÃ7 3
10/ 1^3 10

3^
Exemplo Resolvido 102: Mostre que =
V23
Resolução: Podemos escrever:
VÕ ^3 VF^3 3/3 3j3-t/2 V3_ 1VF~23
V? V? 'V?"3 V? V?^ V23 -2
VÕ = 1fe-33-23 ^3 = 1^63
4^5" 4^4 4^3 2

Caso 03: Quando o denominador é um quociente notável:


Neste caso você multiplica tudo pelo fator do quociente notável.
96 3 Racionalização

12
Exemplo Resolvido 103: Mostre que = V5-1.
3 + 375
Resolução: Podemos escrever:

12 12 3-375 12 12(3-375)
3+377~ 3 + 377 3-377 3+377~ 32 -(375)2

12 36-3675 12 36 (1-75)
3 + 375 - 9-45 3 + 375 ^36

=> 3 +12375 -1í1-^) = 12


3 + 375
= 75-1.

Exemplo Resolvido 104: Mostre que -4^ = 72.(77 - 73)


77 + 77 v 1
Resolução: Podemos escrever:

477 477 77-77 472 472 (77-73)


77 + 77 77 + 77 77-77
4^2 ^ 472.(77 - 73)
47 + 43

4^2
ww
472 (77-77)
77+73" 7-3 47 + 43 " 4

Exemplo Resolvido 105: Mostre que . - = 7^ + 2770 ,


77-2TÍÕ
Resolução: Podemos escrever:
3 3 77 + 2VÍÕ 3 3 ■ ^7 + 2770
V7-2VTÕ I7 - 27ÍÕ 5/7 + 2770 7? - 2770 ^72 -(27ÍÕ)2

3 3 ■ ^7 + 2770 3 3-77 + 2TÍÕ


^7-2-JÍÕ 749 - 40 •^7-2440 75
3 3-77 + 2770 3
= V7 + 2VÍÕ.
77-2770 3 77-2770
Os Segredos da Álgebra para IME/ITA/OLIMPÍADAS 97

1 yÃã + Vu+VÃ
Exemplo Resolvido 106: Mostre que
57-55 5
Resolução: Podemos escrever:

1 1
(5^)2 + 57-5/2 + (5/^)2
57-3/2 “5/7-5/5 (5^)2 + 5^5^ + (5^)2

1 5/49 + 5/Í4 + 5/4 1 5/49 + 5/14 + 5/4


57-5/5" (57)3_(3/2)3 57-55 7-2

1 549 + 5Í4 + 5/4


5/7-55 5

_____________1_____________
Exemplo 107: Mostre que = 5/3-55.
55Í + 5/54 + 5/36 + 554 + 5/Í6
Resolução: Podemos escrever:
____________ 1____________ =____________
5/81 + 5/54 + 5/36 + 5/24 + 5/Í6 5/3^ + ^2-33

1 1 5/5-5/5
5/81 + 5/54 + 536 + 5/24 + 5/Í6 5/81 + 5/54 + 536 + 5/24 + 5/Í6 55-55

1_________ 55-55
5/81 + 5/54 +5/36 + 5/24 + 5'16 ^)5_(5/5)5

1_________ 55-5/5
5/81 + 5/54+^36 + 5/24 + 5/16 3-2
=0 ___________ 1___________ = 5/3-5/2
5/81 + 5/54 + 5^6+5/24 + 5/16
98 3 Racionalização

Caso 04: Formas interessantes que seguem a mesma linha de raciocínio.


N
1) Forma -----7=----- 7= ■
x/A + x/B+x/C
N _ N x/Ã + Vb-x/C
VÃ + x/B + x/c " x/Ã + x/B + x/c x/Ã + x/B-x/C
N N-(x/Ã + x/B-x/C)

VÃ + Vb + x/c-^^2-^)2
N-(x/Ã + x/B-x/Õ)
N
x/Ã + x/B + x/C- A + 2x/ÃB+B-C
N N-(x/Ã + x/B-x/C) A + B-C-2x/ÃB
x/Ã + x/B + x/C - A + B-C + 2x/ÃB A + B-C-2x/ÃB
(x/Ã + x/B-x/c)-(a + B-C-2x/ÃB)
N N-
x/Ã+x/B + x/Õ (A + B-C)2 -(2x/ÃB)2

N N(x/Ã+x/B-x/c)-(A + B-C-2x/ÃB)
x/Ã + Vb + x/c (A + B-C)2 -4AB

N
2) Forma
x/Ã + x/B-x/C '
N N x/Ã + x/B + x/c
x/Ã + x/B-x/c x/Ã + x/B+x/Õ x/Ã + x/b + x/C
N N-(x/Ã + x/B + x/c)

VÃ + VÈ-Vc ’(vã + Vb)2-(Vc)2


N-(x/Ã + x/B + x/c)
N
x/Ã+x/B-x/C ' A + 2x/ÃB+B-C

N-(x/Ã + VB + x/c) a + B-C-2x/ÃB


N
x/Ã + x/B-x/C ” A + B-C + 2x/ÃB A + B-C-2x/ÃB

N_______ N ■ (x/Ã + VÊ + x/c) ■ (A + B - C - 2x/ÃB)


x/Ã + x/B -JÕ ~ 2
(A + B-C)2 -(2x/ÃB)'
Os Segredos da Álgebra para IME/ITA/OLIMPÍADAS 99

N n.(VÃ + 7b + 7c)-(a+b-c-2'/ãb)
VÃ + >?B-VC (A + B-C)2-4AB

N
3) Forma
x/Ã-x/b + x/Õ '
N N x/Ã-x/B-x/Õ
JÂ - Tb + Vc ” VÃ - 7b + Vc TÃ-Tb-Vc
N n(x/ã-7b-Vc)
VÃ-s/b + Tc (7ã-7b)2-(7c)2
N n(7ã-7b-7c)
x/Ã-x/b + x/Õ A-2x/ÃB+B-C
__ N n (7Ã-7b-7c) A-B-C + 2x/ÃB
x/Ã-x/b + x/Õ ” A + B-C-2x/ÃB A + B-C + 2x/ÃB
N n (VÃ-7b-x/c) (A + B-C+2x/ÃB)
x/Ã-Vb + x/C (A + B-C)2-(2x/Ãb)2

N N(x/Ã-x/B-x/c)(A-rB-C + 2x/ÃB)
x/Ã-x/B + x/Õ (A + B-C)2-4AB

N
4) Forma
x/Ã-x/B-x/C '
N N x/Ã-x/b + TÕ
x/Ã-x/B-x/Õ x/Ã-x/B-x/Õ VÃ-x/b + x/c
N N-(x/Ã-x/B + x/Õ)

VÃ-x/b-Vc "(tã-x/b)2-(x/c)2

N N-(x/Ã-x/B+x/Õ)
x/Ã-x/B-x/Õ" A-2x/ÃB+B-C
N N-(x/Ã-x/B + x/c) a + B-C + 2x/ÃB
x/Ã-x/B-x/Õ A + B - C - 2x/ÃB A + B-C+ 2x/ÃB
100 3 Racionalização

(VÃ - VÊ + Tc) • (A + B - C + 2-VÃB )


(A + B-C)2 -(2x/ÃB)2

N(7a- 7b + ,/c) (A + B-C + 2VÃB)


(A + B-C)2 -4AB

1 7Í2+VÍ8-VãÕ
Exemplo Resolvido 108: Mostre que
V2 + V3 +Vê 12
Resolução: Podemos escrever:

1
(V2+Vã-V5)-(2 + 3-5-2Vr3)
E=
Tã + Vã + Vê (2 + 3-5)2-4-2-3

(72 +Vã- Vê) -(-2 Vê) (Vã + V3-Vê).(V6)


“ -24
=> E =
12
1 V12 + VÍ8 - Vãõ
Vã + Vã + Vê” 12

5
Exemplo Resolvido 109: Mostre que = 5 Vii-15 + 7x/5-2-J55 .
1 + 7Ü-75
Resolução: Podemos escrever:
5 5-(l + VÜ + V5)-(1 + 11-5-2VmÍ)
E=
l+s/íi-x/5 (1 + 11-5)2-4-1-11

5 ■ (1 + VTi + Vê) • (7 - 2VH)


(7)2-44

X • (7 - 2Vil + 7VÍ1 - 2 ■ 11 + 7V5 - 2V55)

5
= 5VH-15 + 7V5-2x/55.
1+ Vil-75
Os Segredos da Álgebra para IME/ITA/OLIMPlADAS 101

1
Exemplo Resolvido 110: Efetue
73 - 72 + Tiõ'
Resolução: Podemos escrever:
■j (v^-72-7ÍÕ)-(3 + 2-10 + 273T2)
73 - 72 + 7ÍÕ (3 + 2-10)2 -4-3-2
1 (73 - 72 - 7ÍÕ) -(-5 + 276)
77 — 77 + 770 (-5)2-24
1
= -573+3-272+572-2 277 + 57ÍÕ-2• 27Í5
77 - 77 + 77Õ
1 = 277 - 977 + 57TÕ - 4777.
j3-j2 + y/v5

77 (377-742 -277)
Exemplo Resolvido 111: Mostre que
77-77-77 12

Resolução: Podemos escrever:


77-(74-T7 + 77)-(4 + 3-7 + 2T4”7)
77-77-77 (4 + 3-7)2 -4-4 3

^2 77 (2-77+ 77) (2743)


77 - 77 - 77 ” -48
^2 (272-T6 + 7Í4) (473)
77 - 77 - 77 ~ -48
72 (76-7Í4-272)-73
74 - 73 - 77” 12
7^ (372 - 742 - 276)
74-73-77” 12
102 3 Racionalização

Problemas Propostos

Questão 3.1 Questão 3.2


Q 1
Racionalize —==. Racionalize
2 43 4/342

Questão 3.3 (CN-1976) ______________


AVÃ-3V3
Simplifique a expressão
VÃ-x/3
a) A-9 + An/3 b) A + 3 + V3Ã c) A-3VÃ
d) 3-A + V3 e) 9 +VÃ

Questão 3.4 (CN-1999-Modificada)


2 2
Racionalize
45-43 42'

Questão 3.5

Qual o valor de 45-43 + 42 ?

Questão 3.6

Racionalize
43
2-?8 + 3^5 - 7>/2 '

Questão 3.7

Racionalize
42+45
472 - 445 + 4V2 '

Questão 3.8
12
Racionalize
442+345 + 4? '

Questão 3.9
T- ” i i i “
Raciona ize —= + — + -—= + —— .
42 42 64ã 9

Os Segredos da Álgebra para IME/ITA/OLIMPÍADAS 103

Questão 3.10
1 1
Qual o valor de ?
Í/2-V2 Ü2 + 42

Questão 3.11

72 f2 + 73
Racionalize
J2 ^2 + 73

Questão 3.12
4
Racionalize
^16 + 875

Questão 3.13

V2 + 7l4-i-476
Racionalize
78 + 75-276

Questão 3.14
5If3>/27 ~^2y/8
Racionalize —
V27-x/Í8

Questão 3.15 (Moscou 1982)


__________ 2__________
Simplifique a expressão
^4 - 3^/5 + 275 - ^125

Questão 3.16
„ , , J x/2+K
Qual o valor de —?
^2-1

Questão 3.17

2a - 3b - ,'ab
Racionalize
2y/ã-37b
104 3 Racionalização

Questão 3.18 (CN-1983)

'2 + 73 '2-73
Efetuando , obtém-se:
2-73 V2 + 73
a) 4 b) 73 c) 72 d)? e) 1
3
Questão 3.19 (CN-1991)

O valor de
7^8 + 772-1-7^8-772-1 é:
778 - 772 +1
a) 1 b) 72 c) 2 d) 272 e) 372

Questão 3.20 (CN-1994)


1
O número é igual a:
7275 + 3
a) 772+1 b) '72+2 c) 772-1 d) 72-72 e)

Questão 3.21 (CN-1997)


3(72 + 73 + 75 + 2) 1
O valor de _______________________ g‘

2^(72 + 73 + 75 +1)2 -ij 72 + 73 + 75 '

73 + 472 - 7Í5 273+372-730 273 + 372 + 4730


a) ------ 12------ b) ------- Ta-------
b) c)
24
73 + 72 + 75 273 - 372 - 730
d) e)
12 24
Questão 3.22 (CN-2012)

______ 3 + 76______ A2
Sabendo que A = , qual o valor de ?
573-2712-732 + 750
a) 57F b) VÕ® c) 73® d) e)

Questão 3.23
1
Racionalize
72-73 ’
Os Segredos da Álgebra para IME/ITA/OLIMPÍADAS 105

3.3) Radicais Duplos


Radicais duplos são radicais em soma ou diferença de termos que podem ser
transformados em algum produto notável.
Veremos alguns radicais duplos:
01) Radicais da forma: Va ± VB .

Queremos transformar Va ± -Jb em uma soma ou diferença de radicais. Então,


vamos desenvolver isso:

Demonstração 01:
Seja Va + Vb = Vm + -Jn e VA - Vb = Vm - Vn . temos:
Quando somamos:
Va + Vb + -Ja - VÊ = Vm + Vn + Vm - Vn

=> Va + -7b + Va-Vb = 2x/m. (eq1)


Quando subtraímos:
Va + Vb - Va - Vb = Vm + Vn -(Vm - Vn)

=> Va + VB - Va - VB = Vm + Vn - \/rn + Vn
Va + 7b - Va-Vb = 2Vn. (eq2)
Queremos m e n, então, elevando (eq1) e (eq2) ao quadrado, temos:
(Va + Vb + Va-Vb)2 = (2Vm)2

=> A + Vb + 2p(A + Vb)(a- Vb) A - VB = 4m

=> 2A + 2- A22 -b) = 4m => 4m = 2A + 2 ■ A2 -B)

2A + 2- A2 -B) A A2 -B)
=> m =----------- m=—
4 |__ 2
/ / I \2 2
l Va+ Vb-Va-Vb =(2Vn)
=> A + Vb - 2 ■ ^(a-Vb)(a-VB ) + A - Vb = 4n

=> 2A-2- A2 -b) = 4n => 4n = 2A-2- a2-b)


106 3 Racionalização

2
2A-2.^A2 -B) . A->2-B)
=> n =------------ — n =------- 2-------------
4 2
Assim, temos:

x/a + x/b

Geralmente, chamamos C = x/A2 -B , dai essa expressão fica:

x/a ± x/b = fA + C + lA-C


2 V 2

Demonstração 02:
Uma outra forma é enxergar como trinómio quadrado perfeito, a saber:
x/a ± x/B = I a ± 2 Fb~ => >/a + x/B = x/m + n + 2x/mn
Vm+n Vmn

=> x/a ± x/B = x/m ± 2x/m ■ x/n + n => x/a±Vb = x/m ± x/n)2

x/a ± x/B = |x/m ± x/n |.

Demonstração 03: Qual o valor de x/a + x/b + x/A-x/B ?

Seja x/a + x/b = x , x/a-x/b = y e E = x + y>0, podemos escrever (*):

x2 + 2xy + y2 = (x + y)2 => x2 + y2 = (x + y)2 - 2xy

=> (x/a + x/b^2 + pA-Vej2 = E2 - 2 ■ Va + x/B • x/A-x/B

=> A + x/B+A-x7b = E2 - 2 ■ ^A2 -(x/b)2 => E2 = 2A + 2 • x/a2 -B

=> E2 = 2Ía + x/a2 -b) E A + x/a2 -B >0.

(*) Observação: Esses produtos notáveis serão vistos no capítulo 5, com todos
os detalhes!
Os Segredos da Álgebra para IME/ITA/OLIMPÍADAS 107

Demonstração 04: Qual o valor de Va + -Jb + Va - 5/B ?

_____ n_____ ,_____ P_____


m
Seja x = 5/A + 5/B + Va - 5/B => x + (-Va +Vb) + (-VA-5/B ) = 0 . então

dos produtos notáveis condicionais, temos que (*):


m + n + p = 0 => m2 + n2 - p2 =-2(mn-r-mp + np)
=> m2 + n2 + p2 = -2[m(n + p) + np]

=> x2 + (-Va + Vb )22 + (-Va - 7b )22 = -2^x(-x) + (-Va + 5/ã)(-Va - 7b )j

=> x2 + A + 5/B + A - 5/ã = -2 -x2 + ^A2 -(5/ã)2 I

=> x2 +2A = 2x2 -2VA2 -B => x2 +2A = 2x2 -2Va2 -B

=> x2 = 2A + 2■ Va2-B x = ^2(a + Va2 -b) >0.

(*) Observação: Esses produtos notáveis serão vistos no capitulo 5, com todos
os detalhes!

Exemplo Resolvido 112: Determine \/3-2\l2 .

Resolução 01: Podemos escrever:


5/3-25/2 = V3-^2-2Z :=> 73-25/2 = 5/3-5/8

A=3 e B— C—
= 8, C- = 5/
Vaa 2 - B => C = V32 -8 =• |C = 1

Logo : 5/3 - 5/8 = '3 + 1 - J = 5/3-Vã = 5/2 -1.


2

Resolução 02: Podemos escrever;

V3-2V2 = Vi - 2^2 + 2 => 73-2^2 = -2-1.5/2+(^)2

=• V3-2J2 = ^(l - 72)2 => ^3-272 = |1 - 721, como 1-5/2 <0

=> 73-25/2 =-(1-5/2) V3-2V2 = 5/2-1.


108 3 Racionalização

Exemplo Resolvido 113: Determine 75 + 724 .


Resolução 01: Podemos escrever:
75 + 724 => A = 5 e B = 24, C = 7õ2 -24 => C = 725 - 24 .'. |C = 1|.

Logo : 7a~ 7b = - ÍÃH2 7i7^7= /5Tl+JE1


2 ' V 2 V 2
75 + 724 =73 + 72.

Resolução 02: Podemos escrever:


75 + 724 =73 + 276+2 => 75 + 724 = ^(73)2 + 2 ■ 73 ■ 72 + (Ti?)2

=> 75 + 724 = ^(73 + 72^ => 75 + 724 = |73 + 72|, como

73 + 72 > 0 => 75 + 724 = 73 + 72.

Exemplo Resolvido 114: Determine 728-1073 .


Resolução 01: Podemos escrever:

728-1073 = 728-73-102 => 728-1073 = 728 - 73ÕÕ


=> A = 28 e B = 300, C = 7282 -300 => C = 7784-300

=> C = 7484 .-. |C = 22|. Logo : 7a-7b =

=> 728-73ÕÕ = ^28 + 22-^28~22 => 728 - 73ÕÕ


2
=> 728-73ÕÕ = 725 - 73 => 728 - 7300 = 5-73.
Resolução 02: Podemos escrever:
728-1073 = 725 - 2-5-73 + 3
=> 728-1073 =^52 - 2-5-73 + (73)2

=> 728-1073 = ^(5 - 73)22 => 728-1073 = |õ- Ts|, como

5 - 73 > 0 => 728-1073 = 5 - 73.


Os Segredos da Álgebra para IME/ITA/OLIMPÍADAS 109

Exemplo Resolvido 115: Qual o valor de 73 + Vã + 73 - Vã ?

Resolução 01: Seja 73 + Vã = x , 73 - Vã = y e E = x + y >0. então (*):

x2+2xy + y2 = (x + y)2 => x2 + y2 = (x + y)2-2xy (*)

=> (73 +Vã)2+p3-Vã)2 =E2-2-V3 + Vã -V3-Vã

=> 3 + Vã + 3-Vã = E2-2-^32-(Vã)2 =>


E2 = 6 + 2 ■ V9-8

=> E2 = 6 + 2 x/í E2=6 + 2 => E2=8 E = Vã = 2V2>0.

(*) Observação: Esses produtos notáveis serão vistos no capitulo 5, com todos
os detalhes!

Resolução 02: Chamando a expressão toda de x, podemos escrever:


x = 73 +Vã + 73-Vã => x- 73 +Vã - 73-Vã = O . Então dos produtos
notáveis condicionais, temos que (*):
m+n+p=0 => m2 + n2 + p2 = -2(mn + mp + np) (*)
=> m2 + n2 + p2 = -2[m(n + p) + np]

=> x2 +(-x/3 +Vã)2 +(-73-Vã)2 = -2[x(—x) + (-■J3 + Vã) ('—73 — Vã))

=> X2 + 3 + Vã + 3 - Vã = -2 -X2 + ^32 - (Vã)2

=> x2 +6 = 2x2 -279-8 => x2=6 + 2n/Í => x2 - 8 x = Vã = 2 Vã > 0.

(*) Observação: Esses produtos notáveis serão vistos no capitulo 5. com todos
os detalhes!

02) Radicais da forma: V á±Vb.

Queremos transformar v A ± Vb em uma soma ou diferença de radicais. Então,


vamos desenvolver isso:
Demonstração 01: Seja \/a +VÈ = m + Vn e ^A-Vb =m- Vã.
Quando somamos, temos:

x/a + Vã + 7A — Vb = m + Vn + m -7ã 3A + Vã + 37a - Vb = 2m. (eq1)


110 3 Racionalização

Quando subtraímos, temos:


Ta t x-'B - Ta - Tb = m + x/n - (m- x/n) =>

x/a - X^B - Ta~\/B = m + x/n-m + x/n x/A + x/B - x/a - x/B = 2 x/n. (eq2)

Queremos m e n, então, elevando (eq1) e (eq2) ao cubo, temos:


^x/a + Tb -r x/a - Vb )3 = (2m)3 =>

a+ xfê + 3 ■ J(a + Tb)2 (A - x/B) + 3 ■ J(A + x/b)(A-Tb)2 + A-Tb =8m3

=> 2A + 3 ^(a + Tb)(a-Tb)^^a + Tb+^a-Tb) = 8m3

2A + 3-
• ^(A + x/b)(a - TB)}(2m) = 8m3 =>

^3^A2 - (Tb)2 ] - 2A = 0 4m3 - 3m-^A2 - B j - A =


8m3 - 6m ■ 0.

Como A e B são dados, temos que o valor pedido é racional.

Continuando o desenvolvimento, note que, se subtrairmos, não chegaremos ao


resultado direto, então vamos encontrar n, em função de m, multiplicando as
duas equações:
= mi2‘ -n
(?/^7jB).(3k^/B) = (m + x/n)(m- x/n) => ^A2 - (Tb)

Ma2~b) •
n = m'

Demonstração 02: Podemos pensar da seguinte forma:


(Aí-JBj3 = A3±3A2Tb + 3A(Tb)2±(Tb)3

=> (A ± x^B )3 = A3 ± 3 ■ A2 ■ x/B + 3 ■ AB + Bx/B

3|(a±Tb)3 = ^(a2 + 3b)-a±(3a2+b)Tb .

Demonstração 03: Qual o valor de ÇA + -,'B + TA - x/B ?

Seja x/a + Tb = x, ^A-x/b = y e E = x + y>0, podemos escrever (*):


Os Segredos da Álgebra para IME/ITA/OLIMPÍADAS 111

(x + y)3 = x3 +y3 +3xy(x + y) (*)

E3 = + 3E • 7a + Tb • ^A-TÊ3

=> e3=a + Tb + a- Tb + 3E-3/a2- (Tb)2

=> E3 = 2A + 3E • 'a2-B E3 - 3E -Ta2 - B - 2A = 0. (’*)

(*) Observação: Esses produtos notáveis serão vistos no capítulo 5, com todos
os detalhes!!!!
(**) Observação: Você aprenderá a resolver equações de terceiro grau no
capítulo sobre Fatoração!

Demonstração 04: Qual o valor de ^A + x/B + ^A-Tb ?

seja x = ^a + Tb+^a-Tb => x + (-^a + Tb ) + (-^a - Tb j = 0, temos dos

produtos notáveis condicionais (*):


m + n + p = 0 => m3 + n3 + p3 = 3mnp (*)

x3+(_3^7^j3 + (_3^jgj3 = 3 X (-^/Ã7^b) (-^Tb)

=■ x3-(a + Tb)-(a-Tb) = 3x-^a2 -(Tb)2

=> x3 - a - Tb - a + Tb = 3x • x/a2 - b

x3 -3x■ x/a2-B-2A = 0. (**).

(*) Observação: Esses produtos notáveis serão vistos no capitulo 5, com todos
os detalhes!!!!
(") Observação: Você aprenderá a resolver equações de terceiro grau no
capítulo sobre Fatoração!

Exemplo Resolvido 116: Determine \/l0 + 6x/3 .

Resolução 01: Podemos montara seguinte equação:


^/l 0 + 673 = 0 + T3-62 = [ 0 + Tl~Õ81
112 3 Racionalização

4m3 - 3m ■ pA2 -BJ-A = 0 => 4m3-3m- ^102 -108^-10 = 0

=> 4m'.35-3m í^T00-108)-10 = 0 => 4m3-3m (^8)-10 = 0

=> 4m3 -3m• (-2)-10 = 0 => 4m3 + 6m-10 = 0.


Usando o teorema do fator (*), note que a única raiz real dessa equação
4m3 + 6m-10 = 0 é1,ou seja m = 1. Assim, substituindo na outra expressão
temos:
n = m2 -w(io2 -108) => n = 12 — ^(-8) => n = 1-(-2) => n = 3.

Logo: ^A + Vb = m + Vn => ^10 + ViÕ8 =1 + 5/3.

(*) Observação: Você aprenderá o teorema do fator no capitulo de Fatoração.

Resolução 02: Podemos escrever:


^10 + 6^3 =3/(a + ^)3 => (a2+3B)a + (3A2 + b)>/B=10 + 6>/3

Por comparação, temos:


B = 3 => 3A2+B = 6 => 3A2+3 = 6 => 3A2 = 6-3 => 3A2 =3
=> A2 =1 => A = ±1. (eq1) e (a2+3B)a = 10 => (a2+3 3)a = 10

=>A3 + 9A = 10. (eq2)

Note que A = -1 não satisfaz (eq2). Portanto A = 1.

Logo: ^10 + 6v/3 = ^(1 + ^10 + 673 = 1 + , / 3 .

Exemplo Resolvido 117: Determine ç7 + 5^2 .


Resolução 01: Podemos montar a seguinte equação:
^7 + 5-/2 = ^7 + >/2-52 =|^7 + '/5Õ

4m3 -3m-^A2 -bJ - A = 0 => 4m3 - 3m • ^72-5oJ-7 =O


=> 4m.33 -3m )^/49 ^5Õ)-7 = 0 => 4m3-3m (^i)-7 = 0

=> 4m3 -3m (-1)-7 = 0 => 4m3 + 3m-7 = 0.


Os Segredos da Álgebra para IME/ITA/OLIMPÍADAS 113

Usando o teorema do fator (*), note que a raiz real dessa equação
4m3 + 3m-7 = 0 é1,ou seja m = 1. Assim, substituindo na outra expressão
temos:
n = m2 - ^72 - 50) => n = 12 -^(-1) => n = 1-(-1) => n = 2 .

Logo: 3/A + 7b =m + 7n ^7 + TõÕ =1+75.

(*) Observação: Você aprenderá o teorema do fator no capítulo de Fatoração.

Resolução 02: Podemos escrever:


^7 + 5/5 =^(A+ ^'3 2 +B)7b
Tb)' (A2+3B:)a + (3A2 =7 + 575.

Por comparação, temos:


B = 2 => 3A2+B = 5 => 3A2 + 2 = 5 => 3A2 = 5-2 => 3A2 =3
=> A2 =1 => A = ±1.
±1. (eq1) e (a2 + 3B)a = 7 => (A2 +3-2)A = 7
(eq!) e

=>A3+6A = 7. (eq2)

Note que A = -1 não satisfaz (eq2). Portanto A = 1.

Logo: ^7 + 5^2 = ^(l + T5)3 => 7?+ 575 = 1 + 75 .

Exemplo Resolvido 118: Determine ^45 - 2975 .


Resolução 01: Podemos montar a seguinte equação:
745-29/2 = ^45 - \ZT292 1^45^7/1682 |
-3m-
4m3-3m- (/a2-Bj-A = 0 => 4mi3J- 3m-^452 -1682j-45 = 0

=> 4m.33 - 3m -(/2Í1025-1682)-45 = 0 => 4m3 - 3m ■ (7343) - 45 = 0

=• 4m3 -3m-7-45 = 0 => 4m3 -21m-45 = 0.

Usando o teorema do fator (*), note que a raiz real dessa equação
4m3 - 21m -45 = 0 é 3, ou seja m = 3 .
114 3 Racionalização

Assim, substituindo na outra expressão temos:


n = m2-3^452 -1682) => n = 32-7343 => n = 9-7 => n = 2.

Logo: 3/A-7b = m-7n ^45-7Í682 =3-72.

(*) Observação: Você aprenderá o teorema do fator no capítulo de Fatoração.

Resolução 02: Podemos escrever:


745-2972 =^(A-7b)3 => (A2 + 3B)A - (3A2 + B)7B = 45 - 2972 .

Por comparação, temos:


B = 2 => 3A2+B = 29 => 3A2 + 2 = 29 => 3A2=29-2 => 3A2 = 27

A2 = 9 A = ±3. (eq1) e (A2 + 3B)A = 45 => (a2+3-6)a = 45


=> A3 + 6A = 45. (eq2)

Note que A = -3 não satisfaz (eq2). Portanto A = 3.

Logo: ^45 - 2972 = ^3-72 )3 => ^45-2972 =3-72 .

Exemplo Resolvido 119: Qual o valor de ^10 + 7108 + 0 - 7108 ?

Resolução 01: Seja 710 + 7108 = x , \/l0-7w8 = y e E = x + y > 0, temos

dos produtos notáveis (*):


(x +y)3 = x3+y3 + 3xy(x +y) =•
3 _________ _____ ___
E3 =(7iO-7w8 )3 +^/lO-7TÕ8 j'I + 3E ■ 0 + 7TÕ8 ■ 7l 0 - 7iÕ8

=> E3 =10Tx/ÍÕ8+10-7ÍÕ8+3E-^102-(7TÕ8)2 =>

E3 = 20 + 3E • 7l00 -108 => E3 = 20 + 3E ■ 7=8 => E3 =20 + 3E (-2)

=> E3=20-6E E3+6E-20 = 0. (**)

Note que, pelo teorema do fator, 2 é raiz dessa equação de terceiro grau.
Logo: E = 2.

(*) Observação: Esses produtos notáveis serão vistos no capitulo 5, com todos
os detalhes!
Os Segredos da Álgebra para IME/ITA/OLIMPÍADAS 115

(**) Observação: Você aprenderá a resolver equações de terceiro grau no


capitulo sobre Fatoração!

Resolução 02:
n p______
m •----- --------- '-------
Seja x = ^10 +VTÕ8 + 0 — 7TÕ8 => x + (-3x/l0 + /^) + (-^ o - Tiõã; = o,
então dos produtos notáveis condicionais, temos que (*):
m + n + p = 0 => m3 + n3 + p3 = 3mnp =>
x3 + (-^10 + 7ÍÕ8 )3 + VTÕsj3 = 3x ■ (-^10 + ^ÍÕ8^-^/lÕ^

=> x3-(10 +VÍÕ8)-(l0-yfÕ8) = 3x-^102-(v/iÕ8)2

=, x3-10-VÍÕ8-10 + 7TÕ8 = 3x ^100-108


=> x3 - 20 = 3x • \/-8 => x3-20 = 3x (-2) x3 + 6x - 20 = 0. (")

Note que, pelo teorema do fator, 2 é raiz dessa equação de terceiro grau.
Logo: x = 2.

(*) Observação: Esses produtos notáveis serão vistos no capitulo 5, com todos
os detalhes!

(**) Observação: Você aprenderá a resolver equações de terceiro grau no


capitulo sobre Fatoração!

03) Radicais da forma \/A ± vB .


Para radicais com índice 4, basta fazer radical duplo de índice 2, duas vezes.
Vejamos alguns exemplos:

Exemplo Resolvido 120: Mostre que v161 + 72>/5 -2 + ^5 .

Resolução: Podemos escrever:

a/161 + 7275 = ^81 + 2-9-4>/5 + 80 x/l61 + 72>/5 =^(9 + 4V5)Z

como 9 + 4>/5 >0, x/l61 + 72>?5 = ^9 + 4>Í5 .


116 3 Racionalização

79 + 475 = 74 + 2 ■ 272 + 5 => ^9 + 4^5 = 7 2 + 75 f

como 2 + 75 > 0 , 7161 + 7275 = 2 + Tõ .

Exemplo Resolvido 121: Mostre que ^193-132^2 = 3-\Í2 .

Resolução: Podemos escrever:


7l93-132x/2 = t/121-2-11-672 + 72
=. 7l 93-13272 = ^(l1-672)2 => 7l 93 -13272 = ^|l 1 - 6v^|

como 11 - 672 > 0 , 7l 93-13272 = 7l 1-672


,2
7l 1-672 = 79-2-372 + 2 => 7l 1-672 = 7(3- x/2)'

=> 7l1-6x/2 = |3- x/21 como 3- 72 > 0


=. 7l93-132x^ = 3 - 72 .
Até agora, vimos radicais para cuja resolução usamos as formas (a + b)2 ,

(a + b)3 e (a+ b)4 . Vamos ver agora aqueles em que usamos as formas

(a + b + c)2 e (a + b + c)3. Vamos lá:

Como (a + b + c)2=a2+b2+c2 + 2ab + 2ac + 2bc , podemos fazer

a = x/m, b = x^n, c = 7p . dai fica:

(x/m + x/n + 7p) = m +n + p +2x/nin+27mp + 27np

(x/m -rx/n + Tp)2 = m + n + p + x/4mn + 74mp + -JÃnp

Então temos:
04) Radicais da forma: 7a + x/B + ~Jc + x/Õ .

7A + x/B + x/c + x/d = ^m + n + p + 2x/mn + 2jmp + 2^/np

=> 7A + x/B + x/C + x/D=7m + n + p+ 74mn + ^4mp + 74np


Os Segredos da Álgebra para IME/ITA/OLIMPÍADAS 117

=> Va + Vb + Vc + VÕ = Vm + Vn + Vp )

Va + Vb + Vc + Vd = Vm + Vn + Vp -

Onde: A = mtn + p, B = 4mn , C = 4mp , D = 4np.

Exemplo Resolvido 122: Efetue 7"l 0 + 2Vã + 2^10 + 2>/Í5 .

Resolução: Podemos escrever:


V1O + 2V6 + 2VÍÕ + 2VÍ5 = 72 + 3 + 5 +272-Vã+ 272 -Vã + 2Vã-Vã .

Note que m = 2, n = 3 , p = 5. Logo:

7l0 +2-7(5+2-VÍÕ + 2VÍ5 = ^(75 + Vã + Vã)2

VlO + 2V6 + 2ViÕ + 2VÍ5 = Vã + Vã + Vã.

Exemplo Resolvido 123: Efetue \J37 +12 Vã + 6\/Í4 + 4721 .

Resolução: Podemos escrever:


E = V37 + 12V6 + 6VÍ4 +4721
=> E = ^37 + 2-Í2]-3-V2|-p3]+2 -3-V2|-V7 + 2 -|2-V3V 77

=> E = J37 + 2-V2 32 |-|V3-22] + 2-72-32 |-77+ 2 p3^|-x

=> E = Jl8 + 12 + 7 + 2-VÍ8| rÃ2] + 2-VÍ8|-V7 + 2 |VÍ2|-77 .

Note que m = 18, n = 12, p = 7. Logo:


E = V37 + 12V6 + 6VÍ4+4721 => E = 7(VÍ8 + VÍ2 + V7)2

=> E = VÍ8 + VÍ2 + 77 E = 372 + 273 + 77.


118 3 Racionalização

Problemas Propostos

Questão 3.24

Efetue 73-78
Questão 3.25
Efetue x/ô - 4-Í2 .

Questão 3.26
Efetue 7l61 + 72V5 .

Questão 3.27

Efetue

Questão 3.28
Efetue x/l7--12x/2 .

Questão 3.29
Efetue 732-10^7.

Questão 3.30

Efetue 7)6-677 .

Questão 3.31
Qual o valor de Vl0 -4>/6 ?

Questão 3.32
Qual o valor de Js + A^/s ?

Questão 3.33
Qual o valor de 7>3 + x/48 ?
Os Segredos da Álgebra para IME/ITA/OLIMPÍADAS 119

Questão 3.34
Qual o valor de 729-1275 - 729+1275 ?

Questão 3.35
Se a igualdade 7x + 2 + 272x = 7l 1 + 378 é satisfeita, determine o valor de x.

Questão 3.36

Determine o valor de M em 73 - 78 + 75 - 724 = M


73+r
Questão 3.37
Determine o valor de >J39-'\2-j3 +^4 + 2^3 .

Questão 3.38 (CN-1984)

A expressão 73 + 2 7s72 - 73 - 2 7272 é igual a:


a) 1 b) 2 c) 3 d) 4 e) 5

Questão 3.39
Qual o valor de 743-1277 -743 + 1277 ?

Questão 3.40
Qual o valor de 753-2077 - 753 + 2077 ?

Questão 3.41
Qual o valor de 757-4077 - 757 + 4077 ?

Questão 3.42
Qual o valor de 71?2-9673 ?

Questão 3.43
Qual o valor de 732+1077 + 732 -1O77 ?
120 3 Racionalização

Questão 3.44 (Princeton-2006)


Simplifique 7? + 475 + 7? -475 .

Questão 3.45 (AMC-2011)


Qual dos valores abaixo é igual a 79 - 675 + 79 + 675 ?

. 775
a) 375 b) 275 c) — d) 375 e) 6

Questão 3.46 (AHSME-1970)


O número 73 + 275 - 73-275 é igual a:

a) 2 b) 275 c) 475 d) 7Õ e) 275

Questão 3.47 (IMO-Longlist-1988 - Modificada)

(11 + 675) ■ 7ii-675 - (11 - 675) • 7i 1 + 675


Calcule o valor de x em x = ------- ' --- v — ' --------
775 + 2 + 775 -2) + 775 +11
Questão 3.48 (AHSME-1976)

Se N =
775 + 2 + 775 -2 - 73-275 , então N é igual a:
775 + 1
a)1 b) 275-1 e) NDA

Questão 3.49
Qual o valor de 72 + 75 ?

Questão 3.50 (CN-1982)


O valor de 7l0 + 675 é:

a) 1 + 77 b) 1 + 76 c) 1 + 75 d) 1 + 75 e) 1 + 72

Questão 3.51 (CN-2011)


O número real ^26-1575 é igual a:

a) 5 - 75 b) 77 - 475 c) 3-72
d) 713-375 e)2
Os Segredos da Álgebra para IME/ITA/OLIMPlADAS 121

Questão 3.52 (Stanford-2008)


'17V7-r45
Simplifique ?/
4

Questão 3.53 (IME-02/03)


Demostre que ^20+1472 + ^20-14^2 é um número inteiro múltiplo de 4.

Questão 3.54
Mostre que 3l 26 -1573 + 726 +1573 é um número inteiro.

Questão 3.55 (Turquia-2007-Modificada)


Determine o valor de 3/2 + 75 + 3/2 - 7õ .

Questão 3.56 (AHSME-1980)


A soma 3^5 + 27^3 + ^5-27l3^. é igual a:

. , 3/65 1 + ^13
a)l b) — C) ~2~
C)
d) 3/2 e)1

Questão 3.57 (Turquia-2009-Modificada)


Determinando o valor de x = 3/l 1 + 7337 + \/l 1-7337 , quanto vale x3 +18x ?

Questão 3.58
Qual o valor de ^5^2+7 - ^5>/2-7 ?

Questão 3.59 (IMO-Longlist-1973)

O número vV5 + 2 + V\/5 - 2 , é racional ou irracional?

Questão 3.60 (Suécia-2001)


1 1
Mostre que (Võ2 + 5^3 + (752 -5^3 é irracional.
122 3 Racionalização

Questão 3.61 (Malásia-2010)

Mostre que existem inteiros m e n, tais que — = x/VõÕ +7 -^V5Õ-7 .


n

Questão 3.62 (lrã-1989)


1

Mostre que
^n + Vn2 + l)3 +(n- ^M3 é um número inteiro positivo se
mím2 + 3
n =----------- para algum m inteiro positivo.
2

Questão 3.63 (IME-90/91)


. . J,L 125 j 125 .
Mostre que o numero + J9 + - fl-3 + J9 + e racional.

Questão 3.64 (CN-2004-Modificada)


Simplifique \J49 + 2üVê .

Questão 3.65

Qual o valor de ^8 + V4Õ + -J2Õ + -Jõ ?

Questão 3.66 (Kosovo-2013)

Prove que Vl0 + V24 + 740 +760 = 72 + Vê + Vê .

Questão 3.67 (AIME-2006)


O número V104Vê + 468VÍÕ + 144VÍ5 + 2006 pode ser escrito como
aV2+bV3+cVõ , onde a, b e c são inteiros positivos. Determine abc.

Questão 3.68
Qual o valor de \^8 + 4V48 ?

Questão 3.69
Qual o valor de ^28 - 4^48 ?
Os Segredos da Álgebra para IME/ITA/OLIMPÍADAS 123

Questão 3.70
Qual o valor de \/l 7 + 12T2 ?

Questão 3.71 (AIME-1990)


3 3
Determine (52 + 6743)?-(52-6743)? .

Questão 3.72 (CN-2003)

Se a = 74 - '10 + 275 e b = 74 + 7l0 + 2>/5 , então a + b é igual a:


a) 7TÕ b) 4 c) 272 d) 75+1 e) 73 + 2
124 3 Racionalização

3,4) Tópicos Avançados


Nesse tópico avançado veremos como se comporta um radical duplo com
Índice 3, em cuja resolução usaremos (a + b + c)3 . Vamos lá:

Desenvolvendo (a + b + c)3, temos:

(a + b + c)3 = a3 + b3 + c3 + 3ab2 + 3ac2 + 3a2b + 3a2c + 3b2c + 3bc2 + 6abc ,

podemos fazer a = \/m, b = \Zn, c = ^p , daí fica:

(a + b + c)3 = a3 + b3 + c3 + 3ab2 + 3ac2 + 3a2b + 3a2c + 3b2c + 3bc2 + 6abc

(3m + ?/n + ^p) =m+n+p+3 ímn2 + 3 xjmp2 + 3 3'm2n + 3 \)m2p +

+ 3 3/n2p + 3 ^np2 + 6 ^mnp

+ x/n + = m + n + p + x/27mn2 + ^27mp2 + \/27m2n +

+ ^27m2p + ^27n2p -r ^27^7 + 3/216mnp

Então, tirando a raiz cúbica, temos;

Radicais da forma: ^A + \/b + ^C + ^D + \/Ê + ^F + ?/g + .

^a+^'b + ^+^d+?/ê+?/f + ^g + ?/h =

\/m + n + p + ^27mn2 + ^27mp2 + \/27m2n + ^27m2p + ^27n2p + ^27np2 + ^216mnp

=» 3Va + 37b-^c + 37d + ^e+?/f + ^g + 37h=^ + 3VÍ + ^)3

3ylA^3jB+3Jc + j3D + l/Ê + ^ + 3jGÜ/H=3^ + 3^^

Onde:
A = m + n + p, B = 27mn2 , C = 27mp2 , D = 27m2n. E = 27m2p,
F = 27n2p, G = 27np2, H = 216mnp.
Os Segredos da Álgebra para IME/ITA/OLIMPÍADAS 125

Capitulo 04 - Expressões Algébricas

Introdução
São expressões matemáticas que apresentam letras e podem conter números.
As letras nas expressões são chamadas de variáveis, o que significa que o
valor de cada letra pode ser substituído por um valor numérico. Atualmente as
letras associadas a números constituem a base da álgebra e contribuem de
forma eficiente na resolução de várias situações matemáticas. Vamos conhecer
os diversos tipos de expressões que nos ajudarão a resolver inúmeros
problemas!

4.1) Tipos de Expressões Algébricas


a) Monômio
É a expressão algébrica na qual as operações entre os símbolos são somente
multiplicação ou divisão.
Exemplo: M = |x3y6z4w11.

2
Coeficiente: - .
------------------ 7
Parte Literal: x3y6z4w11.

Note que o grau de um monômio é a soma dos expoentes da parte literal. No


exemplo, temos: Gr(M) = 3 + 6 + 4 + 11 = 24.

b) Binômio
É a soma ou diferença entre DOIS monõmios.

Exemplo: B = ya8b5c4 - 5m11n3 .

11
Coeficientes: •— e 5.
------------------- 2
Partes Literais: a8b5c4 e m11n3.
Note que o grau de um binômio é a maior soma dos expoentes das partes
literais. No exemplo, temos:
Gr(B) = 8 + 5 + 4 = 17 ou Gr(B) = 11 + 3 = 14 Gr(B) = 17.

c) Trinômio
É a soma ou diferença entre TRÊS monõmios.
126 4 Expressões Algébricas

Exemplo: T = -3a6b + 2x7y3z + 1.


Coeficientes: -3, 2 e 1.
Partes Literais: a6b e x7y3z .
Note que o grau de um trinõmio é a maior soma dos expoentes das partes
literais. No exemplo, temos:
Gr(T) = 6 + 1 = 7 ou Gr(T) = 7 + 3 + 1 = 11 .-. Gr(T) = 11.

d) Polinômio
É a soma ou diferença entre QUATRO OU MAIS monômios.

Exemplo: P = x8y8-2x7y7 + x4y4 +xy-10.


Coeficientes: 1, -2 1 e 10.
Partes Literais: x8y8, x7y7, x4y,44 e xy.
Note que o grau de um polinômio é a maior soma dos expoentes das partes
iterais. No exemplo, temos:
Gr(P) = 8 + 8 = 16 ou Gr(P) = 7 + 7 = 14 ou Gr (P) = 4 + 4 = 8
ou Gr(P) = 1 + 1 = 2 Gr(P) = 16.

Observação: Podemos chamar monômios, binômios ou trinômios de


polinômios, sem perda de generalidade.

4.2) Valor Numérico


O valor numérico de uma expressão algébrica é o resultado da operação que
efetuamos, quando substituímos os valores das variáveis.

Exemplo Resolvido 124: Determine o valor numérico da expressão abaixo


para x = 2.
2x4 — x3 — 4x2 + 5x — 11.

Resolução: Basta substituirmos o valor de x, então:


2x4 - X3 - 4x2 + 5x -11 = 2 ■ 24 - 23 - 4 ■ 22 + 5 ■ 2 -11
=> 2x4 - X3 - 4x2+5X -11 = 32-8-16+ 10 -11
=> 2x4 - x3 - 4x2 + 5x -11 = 7.
Os Segredos da Álgebra para IME/ITA/OLIMPÍADAS 127

4.3) Operações com as Expressões Algébricas


a) Adição de Polinômios
A adição de polinômios é efetuada pela soma dos termos semelhantes (termos
com a mesma parte literal). Conserva-se a parte literal e somam-se os
coeficientes.
Exemplo Resolvido 125: Efetue x3y2 +5xy + 3x3y2 .
Resolução: Note que temos duas partes literais. Conservamos a parte literal e
somamos os coeficientes, o resto se repete.
1x3y2 + 5xy + 3x3y2 = 4x3y2 + 5xy.

Exemplo Resolvido 126: Efetue 3x4y + 3xy3 +2z2 +5x4y + 3xy3 -9z2 +1.
Resolução: Conservamos a parte literal e somamos os coeficientes, o resto se
repete.
3x4y + 3xy3 + 2z2 + 5x4y + 3xy3 + 9z2 +1 = 8x4y + 6xy3 +11 z2 +1.

b) Subtração de Polinômios
A subtração de polinômios é efetuada pela soma dos termos semelhantes
(termos com a mesma parte literal). Conserva-se a parte literal e subtraem-se
os coeficientes.

Exemplo Resolvido 127: Efetue -3xy2 + 24xy2 -32x2y-4x2y .


Resolução: Note que temos duas partes literais. Conservamos a parte literal e
somamos os coeficientes, o resto se repete.
-3xy2 + 24xy2 - 32x2y - 4x2y = 21 xy2 - 36x2y .

Exemplo Resolvido 128: Efetue xyz-2xyz + 2ab -5ab - mn + 7mn .


Resolução: Conservamos a parte literal e somamos os coeficientes, o resto se
repete.
xyz-2xyz + 2ab-5ab-mn + 7mn = 6mn-3ab- xyz.

c) Multiplicação de Polinômios
Para multiplicar dois polinômios, cada termo de um polinômio deve multiplicar
cada termo do outro polinômio, propriedade que chamamos de "distributiva".
128 4 Expressões Algébricas

Lembrando que multiplicamos os coeficientes e as partes literais de dada termo.


No final, se obtivermos termos semelhantes, somamos ou subtraímos conforme
a operação aparecer.

Vejamos os exemplos:

Exemplo Resolvido 129: Efetue (~3xy)(+4x5y2 j.

Resolução: Note que temos duas partes literais. Conservamos a parte literal e
somamos os coeficientes, o resto se repete.
(-3xy](+4x5y2) = -12x1+5y1+2 => (-3xy)(+4x5y2j = -12xsy3 .

Exemplo Resolvido 130: Efetue (- 7x3)(x4y2z-2xyz3).

Resolução: Conservamos a parte literal e somamos os coeficientes, o resto se


repete.
(-7x3)(x4y2z-2xyz3) = - 7x3+4y2z + 7 - 2x3+1yz3

(-7x3)(x4y2z-2xyz3) = -7x7y2z + 14x4yz3.

Exemplo Resolvindo 131: Efetue (ab2 + 2abc2 )(-a2c - b4c + 4ab i.

Resolução: Note que temos duas partes literais. Conservamos a parte literal e
somamos os coeficientes, o resto se repete.
(ab2 + 2abc2 )(a2c - b4c) = ab2 (-a2c - b4c) + 2abc2 (-a2c - b4c)

(ab2 + 2abc2 )(a2c - b4c j = a1+2b2c + ab2+4c - 2a1+2bc2+1 - 2ab1+4c2+1

(ab2 + 2abc2 j(a2c - b4c) = a3b2c + ab6c - 2a3bc3 - 2absc3.

Exemplo Resolvido 132: Efetue (xy-2yz + 2xz)(x2y2 + x2z2 -y2z2).

Resolução: Conservamos a parte literal e somamos os coeficientes, o resto se


repete.
(xy-1)(x3y’3: + x2y2 - xy + 6) = x4y4 + x^y^ - x2y2 + 6xy - Z ,3

-x2y2 +xy-6
(xy -l)(x3y3 + x2y2 -xy + 6) = x4y4 -2x2y2 + 7xy -6.
Os Segredos da Álgebra para IME/ITA/OLIMPÍADAS 129

d) Divisão de Polinõmios
Na divisão de monômios, efetuamos a divisão dos coeficientes (quando
possível) e efetuamos a divisão da parte literal.

Observação: Aqui, irei apenas citar a divisão de monômios, para não fugir dos
objetivos deste livro.
(+24x5y2)
Exemplo Resolvido 133: Efetue
(-3xy)
Resolução: Note que temos duas partes literais. Conservamos a parte literal e
somamos os coeficientes, o resto se repete.
(+24xsy2) (+24x5y2)
+24 / , =-8x4y■
= —3 '(X y
(-3xy) (~3xy)

Exemplo Resolvido 134: Efetue ^x3^ + ^-^x4y2zy

Resolução: Conservamos a parte literal e somamos os coeficientes, o resto se


repete.
2
„ . . XX
Coeficiente: +-—-X
2
X X 3
3
x8 x8~4 x8 x4
Parte Literal:
x4y2z y2z x4y2z y2z

Problemas Propostos

Questão 4.1 (CN-1952)


Efetue a multiplicação (x2 - 5x + 9^(x + 3) .

Questão 4.2 (CN-1952)


Simplifique a expressão 7l6x\ --(x-5y)■ 7^7 •
130 5 Produtos Notáveis

Capítulo 05 - Produtos Notáveis

Introdução
No cálculo algébrico, existem várias expressões algébricas (ou polinõmios) cujo
uso é bastante frequente em fatorações e simplificações, essas expressões são
chamadas de produtos notáveis. Neste capitulo, vamos estudar essas
ferramentas importantíssimas e muito eficazes nas simplificações de
expressões algébricas.

5.1) Quadrado da Soma de Dois Termos


O quadrado da soma de dois termos é igual ao quadrado do primeiro termo,
mais o dobro do produtos dos dois termos, mais o quadrado do segundo termo.
b)2 = a2 + 2ab + b2
(a

Demonstração:
(a + b)2 = (a + b)(a + b) (a + b)2 = a2 + ab + ab + b2

.'. (a + b)2 = a2 + 2ab + b2

Exemplo Resolvido 135: Efetue (2x + 3)2.

Resolução: Podemos escrever:


(2x + 3)2 =(2x + 3)(2x + 3) <=> (2x + 3)2 = (2x)2 + 2x ■ 3+ 3 ■ 2x + 32

<=> (2x + 3)Z = 4x2 +6x + 6x + 9 (2x + 3)2 = 4x2 + 12x + 9.

Exemplo Resolvido 136: Efetue (x + 2)2 .

Resolução: Podemos escrever:


(x + 2)2 = (x + 2)(x + 2) o (x + 2)2 = x2+ x-2 + 2x + 22

<=> (x + 2)2 = x2+2x + 2x + 4 .-. (x+ 2)2 = x2 + 4x +4.


Os Segredos da Álgebra para IME/ITA/OLIMPÍADAS 131

Exemplo Resolvido 137: Efetue (4: 1)2-

Resolução: Podemos escrever:


(4x i-1)2 = (4x +1)■ (4x +1) <=> (4x + 1)2 =(4x)2+4x-1 + 1-4x + 12

o (4x + 1)2 = 16x2 +4x + 4x + 1 .-. (4x + 1)2 =16x2+8x + 1.

5.2) Quadrado da Diferença entre Dois Termos


O quadrado da soma de dois termos é igual ao quadrado do primeiro termo,
menos o dobro do produtos dos dois termos, mais o quadrado do segundo
termo.
(a - b)2 = a2 - 2ab + b2

Demonstração:
(a - b)2 = (a - b) (a - b) <= (a - b)2 = a2 - ab - ab + b2

.-. (a - b)2 = a2 - 2ab + b2 .

Exemplo Resolvido 138: Efetue (3x-2)2.

Resolução: Podemos escrever:


(3x-2)2 =(3x-2)(3x-2) o (3x-2)2 = (3x)2-3x-2-2-3x + 22

<=> (3x - 2)2 = 9x2 - 6x - 6x +4 .-. (3x-2)2 = 9x2-12x + 4.

Exemplo Resolvido 139: Efetue (3a3 - b2 j .

Resolução: Podemos escrever:


E = (3a3-b2)2 => E = (3a3-b2)-(3a3-b2) =>

E = (3a3 )2 - 3a3 • b2 - b2 • 3a3 + (b2 )2 =• E = 9a6 - 3a3b2 - 3a3b2 + b4

.-. (3a3 - b2 )2 = 9a6 - 6a3b2 + b4.

Exemplo Resolvido 140: Efetue (am 2")2.


132 5 Produtos Notáveis

Resolução: Podemos escrever:


= (am-2n)2 => E = (am-2n) (am-2n) =>
E

= (am)2-am-2n-2n-am+(2n)2=> E = a2m -2 am -2n +22n


E

(am-2n)2 =a2m-2n+1am+22n.
am -2n = a'

5.3) Identidade de Legendre para a Soma


A soma dos quadrados da soma e da diferença entre dois termos é igual ao
dobro da soma dos quadrados de cada termo.
(a + b)2 + (a - b)2 = 2(a2 + b2)

Demonstração:
(a + b)2 + (a - b)2 = a2 + 2ab + b2 a2 - 2ab + b2

=> (a + b)2 +(a-b)2 = 2a2 +2b2 (a + b)'.2 (a-b)2 = 2(a2 + b2) .

Exemplo Resolvido 141: Mostre que (7a+ b2) + (7a-b2) =2^49a2+b4).

Resolução: Podemos escrever:


= (7a + b2 )2 + (7a - b2 )2 => E = 2^(7a)2 + (b2 )2 I E = 2(49a2 +b4).
E

Exemplo Resolvido 142: Mostre que (l + x3) + (l-x3) = 2(l + x6).

Resolução: Podemos escrever:


E = (l + x3)2 +(l- x3)2 => E = 2[(1)2+(x3)2] E = 2(l + x6).

Exemplo Resolvido 143: Efetue (9 pm + 4! +(9 pm - 4) = 2(81 p2m + 16).

Resolução: Podemos escrever:


+ 4)2 + (9 pm - 4)2 => E = 2^(9pm)2+42
E = (9 pm

E = 2(81p2m+16).
Os Segredos da Álgebra para IME/ITA/OLIMPÍADAS 133

5.4) Identidade de Legendre para a Diferença


A diferença entre os quadrados da soma e da diferença entre dois termos é
igual a quatro vezes o produto desses dois termos.
(a + b)2 - (a - b)2 = 4ab

Demonstração:
b)2 - (a - b)2 = a2 + 2ab + b2 - (a2 - 2ab + b2) <
(a
» (a b)2 - (a - b)2 = a2 + 2ab + b2 - a2 + 2ab - b2

(a + b)2 - (a - b)2 = 4ab .

Exemplo Resolvido 144: Mostre que (x4 + 1) -(x4-l) = 4x4 .

Resolução: Podemos escrever:


(x4+l)2-(x4-l)2 =4-x4-1 /. (x4+l)2-(x4-l)2 =4x4 .

Exemplo Resolvido 145: (ab + c)2 -(ab-c)2 = 4abc .

Resolução: Podemos escrever:


(ab + c)2 -(ab-c)2 = 4ab c (ab + c)2 -(ab-c)2 = 4abc .

Exemplo Resolvido 146: Calcule (a2b3 + cmdn i -(a2b3-cmdn) .

Resolução: Podemos escrever:

(a2b3+c:mdn)2-(a2b3-cm'd
: n )2 = 4(a2b3 )-(cmdn )

(a2b3 + cmdn)2 -(a2b3 — c':mdn)2 =4a2b3cmdn .

5.5) Identidade de Lagrange para a Soma:


Podemos generalizar a soma de dois quadrados, e o resultado será:
(ax + by)2 + (ay - bx)2 = (a2 + b2j(x2 + y2)
134 5 Produtos Notáveis

Demonstração:
E = (ax + by)2 + (ay -bx).2‘
=> E = (ax)2 + 2 - ax by + (by)2 + (ay)2 - 2 ay-bx + (bx)2
=0 E = a2x2 + b2y2 + a2y2 + b2x2 => E = a2(x2+ y2) + b2(y2+ x2)
(ax + by)2 + (ay - bx)2 = (a2 + b2)(x2 + y2) .

Exemplo Resolvido 147: (2x + 3y)2+(2y-3x)2 =13(x2+y:

Resolução: Podemos escrever:

E = (2x + 3y)2+(2y-3x)2 => E = ^22 + 32j(x2+y2) E = 13(x2+y2).

Exemplo Resolvido 148: Determine ^5mx + 3n) +(5m-3nx) .

Resolução: Podemos escrever:


(5mx + 3n)2 +(5m -3nx)2 = |^5m)2 +(bn)2^j(x2 +12)

(5mx + 3n)2 + (5m - 3nx)2 = (52m + 32n)(x2 +1).

\2 z .
Exemplo Resolvido 149: Calcule í ■ p3 + X- - q4 I (84 b .3?
v c J vq "?p
Resolução: Podemos escrever:
2 2

O-
/ u \22
\ /
f U \2
b
(r-P3 *?■
■■■ (a b a
’4]JV ’4 ?P
b 3]
J 49+ c4
,6
pW).
C'

5.6) Identidade de Lagrange para a Diferença:


Podemos generalizar a diferença de dois quadrados, e o resultado será:
(ax + by)2 - (ay + bx)2 = (a2 - b2j(x2- y2)
Os Segredos da Álgebra para IME/ITA/OLIMPÍADAS 135

Demonstração:
E = (ax + by)2 - (ay + bx)2
=o E = (ax)2 + 2 ax by + (by)2-[(ay)2+2 ■ay bx + (bx)2)

=• E = a2x2 + 2•ax•by + b2y2 - a2y2 - 2 ay■bx - b2x2


=> E = a2x2 + b2y2 - a2y2 - b2x2 => E = a2(x2 -y2)-b2(x2 -y2)
(ax + by)2 - (ay + bx)2 = (a2 - b2)(x2- y2) .

Exemplo Resolvido 150: (4b + 3c)2-(3c-4c)2 = 7(b2-c2)

Resolução: Podemos escrever:


E = (4b + 3c)2 - (3c - 4c)2 => E = í42 - 32 ](b2 - c2)
E = 7(b2-c2).
116 9 / '

Exemplo Resolvido 151: Determine (ab2Vc + 3yVx )2-(ab2Vx +3y7c)2 .

Resolução: Podemos escrever:


E = (ab2Vc+3yVx) - (ab2Vx + 3yx/c)2

=> E = ^ab2j2-(3y)2J(Vcj'2 (Ví)2] E = (a2b4-9y2)(c-x).

Exemplo Resolvido 152: Determine


(a2009b2010+c2011d2012j2_((a2009d2012 +b2010c2011)2

Resolução: Podemos escrever:


_(a2009b2010 +c2011d2012j2 ,2009^2012 _b2010c2011 j2
E a2UÜ8d‘

=> E = [(a2009)2 -(c2011)2J(b',2010 j2 d'|2012 j2"j

E = (a4018-c4022)(b4020 -d4024 )•
136 5 Produtos Notáveis

Problemas Propostos

Questão 5.1 (Noruega-1999)


7772 - 662
Compute
777 + 66

Questão 5.2 (Noruega-1999)


Se xy = 6 e x2y + xy2 = 63 , determine x2 + y22 .

Questão 5.3 (Noruega-1998)


Sejam a > b números reais, tais que a2 + b2 = 31 e ab = 3 . Então, quanto vale
a-b?

Questão 5.4 (Harvard-MIT-2012)


>ejam a e b números complexos tais que 2a + 3b = 10 e 4a2 + 9b2 = 20 ,
determine o valor de ab.

Questão 5.5 (AHSME-1958)


7Se” Z 1 í~
xy = b e — + — = a , então (x + y)2 é igual a:
x y
a) (a + 2b)2 b)a2+b2

c)b(ab + 2) d) ab(b + 2)

Questão 5.6 (Noruega-1998)


Seja p o maior fator primo de 9991. Então, a soma dos algarismos de p vale?

Questão 5.7 (Putnam-2001-Modificada)


Simplifique x4-(2n-4)x2+(n-2)2.

Questão 5.8

Se a igualdade Vx + 2 + 2\/2x = y11 + 3^6 é satisfeita, determine o valor de x.


Os Segredos da Álgebra para IME/ITA/OLIMPÍADAS 137

Questão 5.9 (AHSME-1955)


Se r e s são as très raízes da equação x2 -px + q = 0 , então r2 + s2 , é

a)p2+2q b)p2-2q c) p2 + q2

d) p2 -q2 e) p2

Questão 5.10 (Singapura-2014)


Se a e p são as raízes da equação 3x2 + x -1 = 0 , onde a > p. determine o

a p
valor de — + — .
P a

Questão 5.11 (AHSME-1951)

Seres são as raizes da equação ax2 + bx + c = 0 , determine -7 + -5-:


r2 s2
b2 - 4ac b2 -4ac
a) b2 -4ac b) c)
2a
b2 -2ac b2 -2ac
d) e)

Questão 5.12 (Hungria)


Sejam a, b, c e d números reais tais que a2+b2=1 e c2+d2=1. Se


ac + bd = —, determine o valor positivo de ad - bc .

Questão 5.13 (Harvard/MIT-2008)


Suponha que a. b, c. d são números reais satisfazendo a>b>c>d>0,
a2 + d2 = 1, b2 + c2 = 1 e ac + bd = V . Determine ab - cd.
3

Questão 5.14 (Eõtvõs-1933)


Sejam a, b. c e d números reais tais que a2 + b2 = c2 + d2 = 1. Se ac + bd 0.
determine o valor de ab - cd.
138 5 Produtos Notáveis

Questão 5.15 (india-1998-Modificada)


Mostre que (a2 + 3b2)(c2 +3d2j = (ac + 3bd)2 + 3(ad-bc)2 .

Questão 5.16 (Hong Kong-2002)


Sejam x,, x2, e y2 números reais, satisfazendo as equações x2 + 5x2 =10,

x2y, xiY2 = 5 e x^ + 5x2y2 = -/105 . Qual o valor de Xfy, + 5x2y2 = 7105 ?

Questão 5.17 (Eslovênia-2010/ Kosovo-2013)


a+b a-b
Sejam a e b números reais, tais que |a| * |b| e -------+------- = 6 .
a-b a+b
a3 + b3 a3 - b3
Determine —----- - + —----- - .
a3-b3 a3 + b3

Questão 5.18 (Júnior Balkan-1997)


x2 + y2 + x2 ,2
-y x8 + y',8 x8-y,8l
Dados x e y reais com = k, determine
x2-y2 x2+y',2 7^?J + x8 + y8
em termos de k.
Os Segredos da Álgebra para IME/ITA/OLIMPÍADAS 139

A soma de dois quadrados pode ser escrita de duas maneiras: a primeira é em


forma de quadrado da soma, e a segunda é em forma de quadrado da
diferença.

5.7) Soma de Dois Quadrados em Forma de Soma:


A primeira escrita é em forma de quadrado da soma.
a2 + b2 = (a + b)2 - 2ab

Demonstração:
a2 + 2ab + b2 = (a + b)2 a2 + b2 = (a b)2 - 2ab .

Exemplo Resolvido 153: Reescreva 49 + x2 como quadrado da soma.

Resolução: Podemos escrever:


49 + x2 = 72 + x2 => 49 + x2 = (7 + x),22 - 2 • 7 • x

49 + x2 =(7 + x)2-14x.

Exemplo Resolvido 154: Reescreva 16x2+121 como quadrado da soma.

Resolução: Podemos escrever:


16x2 + 121 = (4x)2 +112 => 16x2 +121 = (4x +11)2 - 2 ■ 4x-11

16x2 +121 = (4x +11)2 - 88x .

5.8) Soma de Dois Quadrados em Forma de Diferença:


A segunda escrita é em forma de quadrado da diferença.
a2 + b2 = (a - b)2 + 2ab

Demonstração:
a2 - 2ab + b2 = (a - b)2 .'. a2 + b2 = (a - b)2 + 2ab .

c8
Exemplo Resolvido 155: Reescreva 4a4b2+ — como quadrado da diferença.
140 5 Produtos Notáveis

Resolução: Podemos escrever:


2
E =4a4b2 g =E.(2A)\[Ç]
c4 .-. E = Í2a2b-^-
=> E =Í2a2b- — I + 2-2azb — C4 I 4a2bc4
+----------- ■
5 5 5 5

Exemplo Resolvido 156: Reescreva 9x2p+25y4q como quadrado da


diferença.

Resolução: Podemos escrever:


E = 9x2p+25y4q => E = (3xp)2 + (5y2q)2 =>

= (3xp-5y2q)2+2-3xp 5y2q E = (3xp-5y2q)2 + 30xp • y2q


E

5.9) Produto da Soma pela Diferença:_______________________________ ___


O produto da soma de dois termos pela diferença desses mesmos dois termos
é igual ao quadrado do primeiro termo menos o quadrado do segundo termo.
(a + b) (a - b) = a2 - b2

Demonstração:
(a + b)(a-b) = a2-ab + ab-b2 (a + b) (a - b) = a2- b2 .

Generalizações do Produto da Soma pela Diferença:____________________


Podemos generalizar o produto da soma pela diferença entre dois termos, como
veremos a seguir.
a) Binômio Soma e Diferença com Potência
O produto de um binômio soma pelo binômio diferença é igual ao quadrado do
primeiro monômio menos o quadrado do segundo monômio.
(am + bn) (am - bn) = a2m - b2n

Demonstração:
(am + bn)-(am - bn) = a'i2m - am-bn + am-bn - b2n

(am + bnj (am - bn) = al2m


: _ b2n
Os Segredos da Álgebra para IME/ITA/OLIMPÍADAS 141

b) Generalização para uma Potência n.


O produto da diferença pela soma das potências de dois termos em PG é igual
à diferença entre quadrados com a potência consecutiva.

(a-b)- (a + b) (a2 + b2) (a4 + b4)-...-(a2n + b2n) = a2' _ b2'

Demonstração:
E = (a - b)(a + b) (a2+ b2)-(a4+ b4)-...-(a2n + b2n)

=> E = (a2 - b2)-(a2+ b2) (a4+ b4)....(a2" + b2")

=> E = (a4-b4).(a4+b4)-....(a2n+b2n)

= (a
=> E = 8-b
(a8 8)-...-(a
-bB 2n+b
)-...-(a2n+b;2n) =...

>n+1 on+1
(a-b)-(a + b) (a2 +b2)-(a4 +b4)-...-(a2n + b2n) = a2■' - b2

Exemplo Resolvido 157: Efetue (2a + 3b) (2a-3b).


Resolução: Podemos escrever:
(2a + 3b) ■ (2a- 3b) = (2a)2 -(3b)2 /. (2a + 3b) ■ (2a-3b) = 4a2-9b2.

Exemplo Resolvido 158: Efetue [m11n+pq).(mn-pq).

Resolução: Podemos escrever:


(mn + pq)(mn -pq) = (m11)2 -(pq)2 (mn + pq)(imn-pq) = m2n -P2q

Exemplo Resolvido 159: Efetue (x-2) (x + 2)-(x2 + 4)-... (x2m + 22m).

Resolução: Podemos escrever:


E = (x-2)-(x + 2)-(x2+4)-[x4+16)-...-[:x2m + 22m)

=> E = (x2 -4j-(x2 + 4)-(x4 +16) x2m + 22m)

E = (x4 -16) ■ (x4 +16) ■- (x2m +2',2m j

_ 22m’1
.-. (x-2)-(x + 2) (x2 +4)-(x4+16)-...-( x2m + 22m) = x2m+1
142 5 Produtos Notáveis

Veremos agora os produtos notáveis que tem relação com as equações do 2°


grau. São chamados de identidades de Stevin. São pouco usadas devido à sua
simplicidade, mas são identidades importantes numa prova que exija rapidez!

5.10) Identidades de Stevin para Dois Termos:


a) Produto entre dois binômios soma, com termo comum:
(x + a)(x + b) = x2+ (a + b)x + ab[

Demonstração:
(x + a)(x+ b) = x2+bx + ax+ ab (x + a)(x + b) = x2+ (a + b)x + ab .

b) Produto entre um binômio soma e um binômio diferença, com termo


comum:

(x a)-(x - b) = x2 + (a - b)x - ab

Demonstração:
(x+ a)(x-b) = x2-bx + ax-ab kx + a)(x-b) = x2 + (a-b)x-ab .

c) Produto entre dois binômios diferença, com termo comum:


(x - a)-(x - b) = x2 - (a + b)x + ab

Demonstração:
(x-a)(x-b) = x2-bx - ax + ab (x -a)(x-b) = x2 - (a+ b)x + ab .

Exemplo Resolvido 160: Efetue (x + 2) (x + 3).


Resolução: Podemos escrever:
(x + 2)-(x + 3) = x2+(2 + 3)x + 2-3 (x + 2)(x + 3) = x2+ 5x + 6 .

Exemplo Resolvido 161: Efetue (x + ab) (x-bc).


Resolução: Podemos escrever:
(x-s-ab)-(x-bc) = x2 + (ab-bc)x-abbc

(x + ab) (x - bc) = x2 +(a - c)bx-abzc .


Os Segredos da Álgebra para IME/ITA/OLIMPÍADAS 143

Exemplo Resolvido 162: Efetue (x-abc) -(x-bcd).


Resolução: Podemos escrever:
(x - abc) (x - bcd) = x2 - (abc + bcd)x + abc bcd
(x - abc) ■ (x - bcd) = x2 - (a + d)bcx + ab2c2d .

Consequência, para x = 1, temos:


Se substituirmos x = 1 nas identidades de Stevin, teremos identidades bem
interessantes, vejamos:

a) (1 + a)(1 + b) = 12 + (a + b)-1 + ab |(1 + a) (1 + b) = a + b + ab + l|.

b) (1 + a)-(1-b) = 12 + (a - b)-1-ab |(1 +a)-(1-b) = a-b-ab + l|.

(1-a)-(1-b) = 12-(a-rb)-1 + ab |(1-a) (1-b) = ab-a-b-rl[.


c)

Generalizações das Identidades de Stevin:


Podemos generalizar as identidades de Stevin, como segue:
a) Produto entre dois binômios soma, com termo comum:
(xm+ a)( xm + b) = x2m + (a + b)xm ab

Demonstração:
(xm+ a)-(xm+ b) = x2m + bxm + axm + ab

(xm + a) (xm+ b) = x2m + (a + b)x':m + ab .

b) Produto entre um binômio soma e um binômio diferença, com termo


comum:
(xm + a)-(xm- b) = x2m (a - b)xm - ab

Demonstração:
(xm + a) (xm - b) = x2m - bxm + axm - ab

(xm+ a) (xm- b) = x2m + (a b)xm - ab .


144 5 Produtos Notáveis

c) Produto entre dois binômios diferença, com termo comum:


(xm - a)-(xm- b) = x2m- (a + b)xm+ ab

Demonstração:
(x"-a).(x”-b) = x^m - bxm - axm + ab

^xm - a) (xm - b) - x:2m- (a + b)xm+ ab .

Exemplo Resolvido 163: Efetue ^2x7 + 3)^2x7 + 5)

Resolução: Podemos escrever:


8
(2x7 + 3) • (2x7 + 5) = (2x7 )2 + (3^5) • 2x7 + 3 ■ 5

(2x7 + 3) (2x7 + 5) = 4x14 + 16x7 +15.

Exemplo Resolvido 164: Efetue (a2016 + 2b3)-(a2016 3c4)

Resolução: Podemos escrever:


(a2016+?k
+ 2b3Vf =2016
3)-(a: -3c4) = (;a2016 )2+(2b3-3c4)at2016
: - 2b3 ■ 3c4

(a2016 + 2b3)(a,2016
: 3c4) = a'l4032 + (2b3-3c4)a2016 -6b3c4 .

Exemplo Resolvido 165: Efetue í pm >mn-by

Resolução: Podemos escrever:

>mn-ax r" -by ) = [pmn]2+(ax+by) ■pr” -ax.by

,mn - by
>mn -ax
p = p',2mn-(ax+by)prnn+axby.
Os Segredos da Álgebra para IME/ITA/OLIMPÍADAS 145

De um modo geral, podemos escrever as identidades de Stevin das seguintes


maneiras:
a) Produto entre dois binômios soma (genéricos), com termo comum:
(ax + b)(cx + d) = acx2 + (ad + bc)x + bd

Demonstração:
(ax + b)(cx + d) = acx2 + adx + bcx + bd
■■ E* + b) (cx + d) = acx2 + (ad + bc)x + bd

b) Produto entre dois binômios diferença (genéricos), com termo


comum:
(ax b)-(cx - d) = acx2 - (ad bcjx + bd

Demonstração:
(ax - b) (cx - d) = acx2 - adx - bcx + bd

|(ax - b) (cx - d) = acx2 - (ad + bc)x + bd .

c) Produto entre um binômio soma e um binômio diferença (genéricos),


com termo comum:
(ax + b) (cx - d) = acx2 + (bc - ad)x - bd

Demonstração:
(ax + b)-(cx - d) = acx2 - adx + bcx - bd
[(ãx + b) (cx - d) = acx2 + (bc - ad)x - bd|.

d) Produto entre um binômio diferença e um binômio soma (genéricos),


com termo comum:
(ax - b)-(cx d) = acx2 (ad - bc)x - bd

Demonstração:
(ax - b) (cx + d) = acx2 + adx - bcx - bd

(ax - b) (cx + d) = acx2 + (ad - bc)x - bd .


146 5 Produtos Notáveis

Exemplo Resolvido 166: Efetue (2x + 5)(3x + 4).

Resolução: Podemos escrever:


( 8 15 1
(2x + 5)(3: 4) = 2-3 x2+ 2-4 + 5-3 -x + 5-4

(2x + 5)■ (3x + 4) = 6x2 + 23x + 20 .

Exemplo Resolvido 167: Efetue (x-3)(7x-2).

Resolução: Podemos escrever:


(2 21 '
(x - 3) • (7x — 2) = 1-7 • x2 + 12 + 3-7 x + 3-2

(x-3)-(7x-2) = 7x2-23x + 6 .

Exemplo Resolvido 168: Efetue (5a + 2) • (9a - 3).

Resolução: Podemos escrever:


(18 15 '
(5a+ 2) (9a-3) = 5-9 a2 + 2-9-5-3 a-2-3

(5a + 2) (9a-3) = 45a2 + 3a - 6 .

Exemplo Resolvido 169: Efetue (ax -4) ■ (3x + b) .

Resolução: Podemos escrever:


(ax -4) (3x + b) = a -3-x2 +(a-b + 34)x + 3-2

(ax-4) (3x + b) = 3ax2 +(ab-12)x-4b.


Os Segredos da Álgebra para IME/ITA/OLIMPÍADAS 147

Problemas Propostos

Questão 5.19 (CN-1954)


Decomponha 16x4 -1 em três fatores.

Questão 5.20 (Harvard-MIT-2009)


Determine o valor da soma 112 -12 +122 -22 +132 -32 + ... + 202 -102

Questão 5.21 (Turquia-2007-Modificada)


Determine o valor de (lOO2 -992)(992 -982)...(32 -22)(22 -12) .

Questão 5.22 (OCM-1998-Modificada)


Determine o valor de 12 - 22 + 32 - 4 2 + ...-19982 +19992.

Questão 5.23 (Moscou 1945)


l2? — b2? (a + b)(a2 +b2)(a4 +b4)-...-(a2 + b26
Divida a' por

Questão 5.24 (Moscou 1945)

Divida a'i2k-b2k por


(a + b)(a2+b2)(a4+b4y...-^a2k '
AM
-b’

Questão 5.25 (Moscou 1946)


Prove que, depois de completar a multiplicação e agrupar os termos de
(l-x + x2 -x3 + ... ■-x"x100)-(l + x + x2 +... + x" + x100), não haverá

monômios de grau ímpar.

Questão 5.26 (CN-2005)


, a4 + b4 -6a2b2
Simplificando-se a fração , onde a > b, obtém-se:
a2 - b2 + 2ab
a) a2 - b2 - 2ab b) a2 -b2 + 2ab
c) a2 + b2 - 2ab d) a2 + b2 + 2ab
e) a2 +b2
148 5 Produtos Notáveis

Questão 5.27 (AHSME-1951 / CN-1998)____________________________

I ( x4 -1 i2 x2
Simplificando a expressão ,11+ -----— -—.para XeR , obtém-se:
V l 2x2
X4 -r X2 - 1 x4-x2-1
a) b) c)
2x2 2x2 2x2

x2 +1 , x2
d) ~2 e}T2

Questão 5.28
Simplifique (a + b - c + d)2 + (a + b - c - d)2 - 2^(c - a - b)2 - d2 j.

Questão 5.29
Simplifique (a + b)(a-b)(a2 +ab + b2)(a2 - ab + b2^a12 + a6b6 +b12)

Questão 5.30 (CN-1991)_____________________________________________ _


Simplificando a expressão abaixo, para os valores a, b e c que não anulam o
denominador, obtém-se:
(a2 -b2 -c2 -2bcj(a + b-c)

(a + b + c)(a2 +c2 -2ac-b2)

a) 1
b) 2
c) 3
d) a + b i-c
e) a - b + c
Os Segredos da Álgebra para IME/ITA/OLIMPÍADAS 149

Até agora, vimos os produtos notáveis de expoente dois, ou seja, elevados ao


quadrado, agora veremos os produtos de expoente três, ou seja, elevados ao
cubo, vamos lá!

5.11) Cubo da Soma de Dois Termos


O cubo da soma de dois termos é igual ao primeiro termo elevado ao cubo mais
três vezes o primeiro termo elevado ao quadrado vezes o segundo termo, mais
três vezes o primeiro termo vezes o segundo termo elevado ao quadrado, mais
o segundo termo elevado ao cubo.
(a + b)3 = a3 3a2b + 3abz b3 .

Demonstração:
(a + b)3 = (a + b)2 (a + b) <=> (a + b)3 = (a2 + 2ab + b2) ■ (a b)
<=> (a + b)3 = a3 + 2a2b + ab2 + a2b + 2ab2 b3

(a b)3 = a3 + 3a2b + 3ab2 + b3 .

Exemplo Resolvido 170: Desenvolva (x + 3)3 .

Resolução: Podemos escrever:


(x+ 3)3 = x3+3-x2 • 3 +3 ■ x-32+33 .-. (x + 3)3 = x3 + 9x2 + 27x + 27 .

Exemplo Resolvido 171: Desenvolva (3x + 2)3.

Resolução: Podemos escrever:


(3x + 2)3 = (3x)3+3 (3x)2-2 + 3-3X-22+23

.-. (3x + 2)3 = 27x3 + 54x2 + 36x + 8 .

5.12) Cubo da diferença de Dois Termos:


O cubo da diferença entre dois termos é igual ao primeiro termo elevado ao
cubo menos três vezes o primeiro termo elevado ao quadrado vezes o segundo
termo, mais três vezes o primeiro termo vezes o segundo termo elevado ao
quadrado, menos o segundo termo elevado ao cubo.
(a - b)3 = a3 - 3a2b + 3ab2 - b3 .
150 5 Produtos Notáveis

Demonstração:
(a-b)3 = (a-b)2 (a-b) (a - b)3 = (a2-2ab + b2) (a - b)

o (a - b)3 = a3 - 2a2b + ab2 - a2b + 2ab2 - b3

.-. (a - b)3 = a3 - 3a2b + 3ab2 - b3 .

Exemplo Resolvido 172: Desenvolva (x-2)3.

Resolução: Podemos escrever:


(x-2)3 =x3 +3 x2 2 + 3 x-22+23 (x-2)3 = x3-6x2+12x-8 .

Exemplo Resolvido 173: Desenvolva (x-2y)3.

Resolução: Podemos escrever:


4y2
(x-2y)3 = x3-3-x2-2y+ 3-x(2y)2-(2y)3

.-. (3x-2)3 = x3-6x2y + 12xy2-8y3 .

5.13) Cubo da Soma mais Cubo da Diferença:


Quando somamos o cubo da soma com o cubo da diferença, obtemos um
produto notável interessante: o dobro do primeiro termo que multiplica o
quadrado do primeiro termo mais três vezes o quadrado do segundo termo.
(a + b)3+(a - b)3 =2a(a2+3b2)

Demonstração:
(a + b)3 + (a - b)3 = a3 + 3a2b + 3ab2 + b3 + a3 - 3a2b + 3ab2 - b3
(a-í-b)3 + (a-b)3 = 2a3 + 6ab2 .-. (a + b).3: (a-b)3 = 2a(a2+3b2) .

Exemplo Resolvido 174: Efetue (x + 3)3 +(x-3)3 .

Resolução: Podemos escrever:


(x + 3)3 + (x-3)3 = 2-x(x2+3-32)=>(x + 3)3+(x-3)3 = 2x(x2+2?)

.-. (x + 3)3 + (x - 3)3 = 2x3 + 54x .


Os Segredos da Álgebra para IME/ITA/OLIMPÍADAS 151

Exemplo Resolvido 175: Efetue (2x + 5)3 (2x-5)3.

Resolução: Podemos escrever:


(2x + 5)3 + (2x — 5)3 = 2 - (2x)[)(2x)2 + 3 ■ 52 ) =>

(2x + 5)3 +(2x-5)3 =4x(4x2 +75)/. (2x + 5)3 + (2x - 5)3 =16x3 + 300x .

5.14) Cubo da Soma menos Cubo da Diferença:


Quando subtraímos o cubo da diferença do cubo da soma, obtemos um produto
notável interessante: o dobro do segundo termo que multiplica três vezes o
quadrado do primeiro termo mais o quadrado do segundo termo.
(a + b)3-(a-b)3 =2b(3a2+b2)

Demonstração:
(a + b)3 -(a-b)3 = a3 +3a2b + 3ab2 +b3 -(a3 -3a2b + 3ab2 -b3)

(a + b)3 - (a - b)3 = a3 + 3a2b + 3ab2 + b3 - a3 + 3a2b - 3ab2 + b3

(a + b)3-(a-b)3 = 6a2b + 2b3 .-. (a + b)3-(a-b)3 = 2b(3a2+b2)

Exemplo Resolvido 176: Efetue (x + 5)3-(x-5)3.

Resolução: Podemos escrever:


(x + 5)3 -(x-5)3 = 2-5(3-X2 + 52)=> (x + 5)3 -(x-5)3 = 10(3x2 +25)

=> (x + 5)3 -(x-5)3 =30x2+250.

Exemplo Resolvido 177: (3x + 7)3 -(3x-7)3 = 378x2 + 686.

Resolução: Podemos escrever:


(3x+ 7)3-(3x-7)3 =2-7^3-(3x)2+72l

=> (3x + 7)3 -(3x-7)3 = 14(27x2 +49)

=> (3x+ 7)3 - (3x - 7)3 = 378x2 + 686 .


152 5 Produtos Notáveis

5.15) Identidade de Cauchy (Soma):


Podemos escrever o cubo da soma de um modo mais cômodo, é a chamada
identidade de Cauchy:
(a + b)3 = a3 + b3 + 3ab(a
b)

Demonstração:
(a + b)3 = a3 + 3a2b + 3ab2 + b3 .-. (a + b)3 = a3+b3 + 3ab(a+ b) .

Exemplo Resolvido 178: Desenvolva (m + 10)3.

Resolução: Podemos escrever:


(m + 10)3 =m3 + 103 + 3-m-10(m + 10)

=> (m + 10)3 = m3 +1000+ 30m(m + 10).

Exemplo Resolvido 179: Desenvolva (3n + 4)3.

Resolução: Podemos escrever:


(3n + 4)3 = (3n)3 +43 +3(3n)-4(3n + 4)

=> (3n + 4)3 = 27n3 + 64 + 36n(3n + 4).

5.16) Identidade de Cauchy (Diferença):


Podemos escrever o cubo da diferença de um modo mais cômodo, é a
chamada identidade de Cauchy:
(a - b)3 = a3 - b3 - 3ab(a - b)

Demonstração:
(a-b)3 = a3 - 3a2b +3ab2-b3 (a-b)3 = a3 -b3 -3ab(a-b) .

Veremos como expressar a soma de cubos e a diferença de cubos. Podemos


escrevê-las a partir das identidades de Cauchy, temos também a forma
fatorada. que são chamados de produtos de Warring.
Os Segredos da Álgebra para IME/ITA/OLIMPÍADAS 153

a) Soma de cubos: a3 b3 =(a + b)3-3ab(a + b) .

Demonstração:
(a + b)3 =a3+b3+3ab(a + b) a3 + b3 = (a +b)3-3ab(a + b) .

b) Diferença de cubos: a3 - b3 = (a-b)3 + 3ab(a-b) .

Demonstração:
(a-b)3 = a3-b3 - 3ab(a-b) a3-b3=(a-b)3 3ab(a-b) .

c) Soma de cubos (Warring): a3+b3 = (a + b)(a2 -ab + b2) .

Demonstração:
a3+b3 = (a + b)3-3ab(a + b) <=> a3 +b3 = (a + b)|ja + b)2 -3ab

a3 +b3 = (a + b)|^a2 + 2ab + b2 -3ab^| a3 + b3 = (a + b)(a2 -ab + b2) .

d) Diferença de cubos (Warring): a3 -b3 =(a-b)(a2 + ab + b2j

Demonstração:
a3 - b3 = (a -b)3 + 3ab(a -b) a3 - b3 = (a - b)£(a - b)2 + 3ab"|

«=> a3 -b3 - (a-b)[a2 -2ab + b2+3abJ a3-b3 = (a-b)^a2 + ab + b2 j

Exemplo Resolvido 180: Efetue x3 + 43 .

Resolução: Podemos escrever:


x3 +43 = (x + 4)3 - 3-x-4(x + 4) => x3 + 43 = (x + 4)3 -12x(x + 4).
154 5 Produtos Notáveis

Exemplo Resolvido 181: Efetue (2m)3 - 33.

Resolução: Podemos escrever:


(2m)3 - 33 = (2m-3)3+3-(2m)-3(2m-3)

=> (2m)3 - 33 = (2m-3)3+18m(2m-3).

Exemplo Resolvido 182: Efetue (5a)3 +(2b)3.

Resolução: Podemos escrever:


(5a)3 +(2b)3 = (5a + 2b)[(5a)2 -(5a)-(2b) + (2b)2]

=> (5a)3 +(2b)3 = (5a + 2b)(25a2-10ab + 4b2).

Exemplo Resolvido 183: Efetue (11 x)3 - y3 .

Resolução: Podemos escrever:


(11 x)3 - y3 = (11 x - y )£(11 x)2 + (11 x) ■ y + y:
■!]
=> (11 x)3 - y3 =(11x-y)(l21 x2 +11 xy + y:
Os Segredos da Álgebra para IME/ITA/OLIMPÍADAS 155

Problemas Propostos

Questão 5.31 (Harvard-MIT-2007)


Sejam x e y dois números reais tais que x-y = 4 e x3-y',3 = 28 . Determine
xy.

Questão 5.32 (CN-2006)


x(x2 x-y) + y2(y + 1)
Simplificando-se a fração , x2 + y2 - xy « 0 , obtém-
x2 + y 2 -xy
se:
a) x-y + 1 b) x-y-1 c) x + y-1
d) 1 + x + y e) 1- x + y

Questão 5.33 (CN-1980)


(2x2 -4x + 8)(x2 -4)
Simplificando , vamos encontrar:
V2 • x3 + VÍ28

a) V2(x + 2) b) V2(x-2) c) V2(x2-4)

d) ,/2

Questão 5.34 (CN-1983)

Simplificando a fração
(x2z + zy2 + 2xyz)(x2

x3 +3x2y + 3xy2 +y,3


-Z) , vamos encontrar:

a) z(x + y) b) z(x-y) c) zx + y
d) zx - y e) z + y

Questão 5.35
(a + b)2 -4^a2 + b2 :
Simplifique
(a3+b3)2-(a2+b2)3 '
156 5 Produtos Notáveis

Questão 5.36 (AHSME-1952)


_________ —
2
(x + 1)2(x2 -x + l) (x-1)2(x2 + X + l)
, quando simplificado, é:
(x3+1)2 (xM2

a) (x + 1)4 b) (x3 +l)4


c)1

•>iwj
>2
d) [(x3 +1)(x3-1)]
Questão 5.37
Determine a3 - a2b - ab2 + b3 .

Questão 5.38 (Harvard/IVllT-2000)_______ _


Calcule 20003 -1999- 20002 -19992 ■ 2000 +19993 .

Questão 5.39 (Harvard/MIT-2007)


23-1 33-1 43-1 53-1 63-1
Calcule
23 -r-1 33 +1 43 +1 53 +1 63 +1

Questão 5.40 (Stanford-2012)


_ . ,. 233-1 333-1 43 -1 163 -1
Calcule —-------- -------- - ...—- .
23 +1 33 +1 43 -r-1 163+1
Os Segredos da Álgebra para IME/ITA/OLIMPÍADAS 157

Desenvolveremos agora a quarta potência de dois termos e a utilizaremos para


desenvolver potências de temos recíprocos.

5.17) Quarta Potência da Soma:


A quarta potência da soma é dada por
(a + b)4 = a4 + 4a3b + 6a2b2 + 4ab3 b4

Demonstração:
(a + b)4 =(a + b)3(a + b) <=■ (a + b)4 = (a3 + 3a2b + 3ab2 + b3)(a+ b)
(a + b)4 = a4 + 3a3b + 3a2b2 + ab3 + a3b + 3a2b2 + 3ab3 + b4
(a + b)4 = a4 + 4a3b + 6a2b2 + 4ab3 + b4 .

Exemplo Resolvido 184: Desenvolva (x + 2)4 .

Resolução: Podemos escrever:


4 8
(x + 2)4 = x4 + 4 • x3 2 + 6 x2 ■ 22 + 4 ■ x 23 + 24

=> (x + 2)4 = x4 +8x3 + 24x2 +32x4-16 .

Exemplo Resolvido 185: Desenvolva (2x + 3)4.

Resolução: Podemos escrever:


8x3 4x2 9 27
(2x + 3)4 = (2x)4 + 4 ■ (2x)3 • 3 + 6 ■ (2x)2 • 32 + 4 • (2x) ■ 33 + 34

.-. (2x + 3)4 = 16x4 + 96x3 + 216x2 + 216x + 81.

5.18) Quarta Potência da Diferença:


A quarta potência da diferença é dada por
(a - b)4 = a4 - 4a3b + 6a2b2 - 4ab3 + b4

Demonstração:
(a - b)4 = (a - b)3 • (a - b) (a-b)4 = (a3 -3a2b + 3ab2 -b3j(a-b)
158 5 Produtos Notáveis

■=> (a-b)4=a4- 3a3b + 3a2b2 - ab3 - a3b + 3a2b2 - 3ab3+b4

(a - b)4 = a4 - 4a3b + 6a2b2 - 4ab3 + b4 .

Exemplo Resolvido 186: Desevolva (x-1)4 .

Resolução: Podemos escrever:


(x — 1)4 = x4 - 4 ■ x3 -1 + 6 x2 -12 - 4 x- 13+14

=> (x-1)4 = x4 -4x3 +6x2 -4x + 1.

Exemplo Resolvido 187: Desenvolva (4m-3)4 .

Resolução: Podemos escrever:


64m3 16m2 g 27

(4m- 3)4 = (4m)4 -4 • (4m)3 ■ 3 + 6 ■ (4m)2 • 32 - 4 ■ (4m) ■ 33 + 34

(4m-3)4 = 256m4 -768m3 + 864m2 -432m + 81.

5.19) Identidade de Legendre (Soma):


Somando as duas identidades anteriores, chegamos à identidade da soma de
Legendre para á quarta potência.

(a + b)4 + (a - b)4 = 2^a2 + b2j + 8a2b2

Demonstração:
(a + b)4 + (a - b)4 = a4 + 4a3b + 6a2b2 + 4ab3 + b4 + a4 - 4a3b + 6a2b2 - 4ab3 + b4
(a + b)4 + (a - b)4 = 2a4 + 2a2b2 + 8a2b2 + 2a2b2 - 2b4 <=>
<=> (a + b)4-r(a - b)4 = 2a2(a2 + b2) + 2b2(a2+b2) + 8a2b2

(a + b)4 + (a - b)4 = 2(a2 + b2) + 8a2b2 .

Exemplo Resolvido 188: Efetue (3x + 4y)4 + (3x-4y)4 .


Os Segredos da Álgebra para IME/ITA/OLIMPÍADAS 159

Resolução: Podemos escrever:


9x2 !6y2
(3x + 4y)4 + (3x - 4y)4 = 2 -[(3x)2 + (4y)2]2 + 8 ■ (3x)2 ■ (4y)2

=> (3x + 4y)4 +(3x-4y)4 = 2(9x2 +16y2+1152x2y2

=> (3x + 4y)4 + (3x - 4y)4 = 2(81x4 + 288x2y2 + 256y4) + 1152x2y,2:

=> (3x + 4y)4 +(3x-4y)4 = 162x4 +576x2y2+512y4+1152x2y2

.-. (3x + 4y)4 +(3x-4y)4 = 162x4 + 1728x2y2 + 512y4 .

5.20) Identidade de Legendre (Diferença):


Subtraindo as duas identidades anteriores, chegamos á identidade da diferença
de Legendre para à quarta potência.
(a + b)4 - (a - b)4 = 8ab(a2 + b2j

Demonstração:
(a + b)4 - (a - b)4 = a4 + 4a3b + 6a2b2 + 4ab3 + b4 -
-(a4 - 4a3b + 6a2b2 - 4ab3 + b4 j
(a + b)4 - (a-b)4 = a4 + 4a3b + 6a2b2 + 4ab3 + b4 - a4 + 4a3b-
-6a2b2 +4ab3 -b4
(a + b)4 -(a-b)4 = 8a3b + 8ab3 (a + b)4 -(a-b)4 = 8ab(a2 + b2) .

Exemplo Resolvido 189: Efetue (7x + 2y)4 -(7x-2y)4 .

Resolução: Podemos escrever:


(7x + 2y)4 - (7x - 2y)4 = 8 ■ (7x) ■ (2y)[(7x)2 + (2y)2 ]

=> (7x + 2y)4 - (7x - 2y)4 = 112xy (49x2 +4y2).

Exemplo Resolvido 190: Efetue (11 m + 5n)4 - (11 m - 5n)4 .


160 5 Produtos Notáveis

Resolução: Podemos escrever:


(11 m + 5n)4 -(11 m -5n)4 = 8 • (11 m)■ (5n)[jl 1 m)2 + (5n)2]

=> (11 m + 5n)4 -(11 m -5n)4 = 440mnp21 m2 + 25n2).

5.21) Quinta Potência da Soma:_______________________________


A quinta potência da soma é dada por
(a + b)5 = a5 + 5a4b + 10a3b2 + 10a2b3 + 5ab4 + b5

Demonstração:
(a + b)5 = (a + b)4 (a + b)

(a + b)5 = (a4 + 4a3b + 6a2b2 + 4ab3 + b4) ■ (a + b)

(a + b)5 =a5 + 4a4b + 6a3b2 +4a2b3 + ab4 + a4b + 4a3b2 + 6a2b3 +4ab4 +b5

(a + b)5 = a5 + 5a4b + 10a3b2 + 10a2b3 + 5ab4 + b5 .

Exemplo Resolvido 191: Desenvolva (x + 1)5 .


Resolução: Podemos escrever:
(x + 1)5 = x5 + 5 x4 -1 + 10 x3 -12 +10 x2 -13 + 5 x • 14+15

=■ (x + 1)5 = x5+5x4+10x3+10x2+5x+ 1.

5.22) Vamos desenvolver a quinta potência da diferença:_______


A quinta potência da diferença é dada por
(a - b)5 = a5 - 5a4b + 10a3b2 - 10a2b3 + 5ab4 - b5

Demonstração:
(a - b)5 = (a - b)4 • (a - b)
(a - b)5 = (a4 - 4a3b + 6a2b2 - 4ab3 + b4 j ■ (a - b)
(a - b)5 = a5 - 4a4b + 6a3b2 - 4a2b3 + ab4 - a4b + 4a3b2 - 6a2b3 +
+ 4ab4 -b5
(a - b)5 = a5 - 5a4b + 10a3b2 - 10a2b3 + 5ab4 - b5 .
Os Segredos da Álgebra para IME/ITA/OLIMPÍADAS 161

Exemplo Resolvido 192: Desenvolva (x - 2)5 .

Resolução: Podemos escrever:


4 8 16
(x- 2)5 = x5 -5• x4 -2 +10 • x3 -22 —10 • x2 • 23 + 5 x ■ 24 — 2S

=> (x-2)5 = x5 -10x4 +40x3 -80a2 +80x-32.

5.23) A Soma das Quintas Potências da Soma e da Diferença:


A soma dos dois produtos notáveis anteriores é dada por
(a + b)5+(a - b)5 = 2a(a4 + 10a2b2 + 5b4)

Demonstração:
(a + b)5 (a -b)5 = a5 + 5à^b. + 10a3b2 + + 5ab4 + X + a5 -

-~5à^b. + 10a3b2 - JLQa^X + õab4 -X

(a + b)5 (a - b)5 = 2a5 + 20a3b2 + 10ab4

(a + b)|5
! +(a - b)5 = 2a(a4 + 10a2b2 + 5b4) .

Exemplo Resolvido 193: Determine (3x +1)5 + (3x -1)5 .


Resolução: Podemos escrever:

(3x + 1)5 (3x-1)5 = 2 (3x)- (3x)4 +10 (3x)2 -12 + 5 14

9x2

=> (3x + 1)5+(3x-1)5 =6x(81x4 +90x2 +5)

=> (3x + 1)5 +(3x-1)5 =486x5+540x3+30x.

Exemplo Resolvido 194: Determine (2x + 5)5 + (2x -5)5 .


Resolução: Podemos escrever:

(2x+ 5)5 (2x - 5)5 = 2 ■ (2x) • (2x)4 +10 ■ (2x)2 • 52 + 5 ■ 54

25
162 5 Produtos Notáveis

=> (2x + 5)5 + (2x-5)5 = 4x(l6x4 + 1000x2+ 3125)

=> (2x + 5)S+(2x-5)5 = 64x5 + 4000X3 + 12500X .

5.24) A Diferença das Quintas Potências da Soma e da Diferença:


A diferença entre os dois produtos notáveis anteriores é dada por
(a + b)5 + (a - b)5 = 2b(5a4 + 10a2b2 + b4)

Demonstração:
(a + b)5- (a-b)5 = ^ + 5a4b+10ãV_ + 10a2b3+ £àtí( + b5 -

- - 5a4b + iSãV. -10a2b3 + - b5)

(a + b)5 - (a - b)5 = 10a4b + 20a2b3 + 2b5

(a + b)5 + (a - b)5 = 2b(5a4 + 10a2b2 + b4) .

Exemplo Resolvido 195: Determine (2m + 3)5 +(2m-3)5 .

Resolução: Podemos escrever:

(2m + 3)5 + (2m-3)5 =2-3- 5(2m)4 + 10-(2m)2 -32 +34


'--- 'V' '--- 'T' 9
16m4 4m2
=• (2m + 3)5 + (2m-3)5 = 6(80m4 +360m2 + 8l)

=> (2m + 3)5 +(2m-3)5 = 480m4 +2160m2 +486.

Exemplo Resolvido 196: Determine (t +1)5 + (t -1)5.

Resolução: Podemos escrever:


(t + 1)5 + (t-1)5 = 2 • 1 ■ (5 ■ t4 + 10t2-12+14)

=> (t + 1)5+(t-1)5 =2-(5t4+10t2+l)

=> (t + 1)5+(t-1)5 = 10t4 + 20t2 + 2 .


Os Segredos da Álgebra para IME/ITA/OLIMPÍADAS 163

Problemas Propostos

Questão 5.41
Dado a2 + b2 = k e ab = x. Determine (a-s-b)4 e (a-b)4 .

Questão 5.42
17
Sejam x e y números reais positivos satisfazendo x2 + y',2 = 1„ e x 4 + y,44 = —
18
Determine xy.

Questão 5.43
x2 + y',2 x2 - y',2 x8+y'-8 x8 -y8
Dados x e y reais com = k, determine em
7^7J + x2 + y2 x8 - y8 rx8-.y8

termos de k.

Questão 5.44 (IMO-Longlist-1992-Adaptada)


5126-1
Simplifique
525-1 '

Questão 5.45
Sejam a e b números reais não nulos tais que x e y satisfazem o sistema
ax + by = 2
ax2 +by2 =20
ax3 + by3 = 56
ax4 + by4 = 272
Determine o valor de ax5 + by5.

Questão 5.46 (AIME-1990/Harvard-IVIIT-2009) ____________________________


Determine ax5 +by5 se os números reais a, b, x e y satisfazem as equações
ax + by = 3, ax2 + by2 = 7, ax3 + by3 =16, ax4 + by4 = 42 .
164 5 Produtos Notáveis

Questão 5.47 (OBM XXXI - 2a Fase - Nível 2)_____________________________


Determine ax5 +by5 se os números reais a, b, x e y satisfazem as equações

ax + by = 1, ax2 + by2 = 2, ax3 + by3 = 5, ax4 + by4 = 6 .

Questão 5.48 (AMC-2007)


Suponha que o número a satisfaça a equação a + a' = 4 . Qual o valor de
a4 4-a"4?
a) 169
b) 172
c) 192
d) 194
e) 212
Os Segredos da Álgebra para IME/ITA/OLIMPÍADAS 165

Vamos agora desenvolver a soma de termos recíprocos xn+-l, dada a

1
condição x + —= k. com k>2 Pnmeiramente iremos ver algumas regras
x
práticas, ao final veremos sua generalização.

5.25) Soma de Termos Recíprocos:


a) Soma dos quadrados de termos recíprocos, dada a sua soma.
A soma dos quadrados de dois termos recíprocos é dada por
x2+A- = k2-2
xz

Demonstração:
1 . ( n22 = k2
x + — = k <=> x + -
2
k2
x < xj

<=> x2 + 2 + ~ = k2 .-. x2 4 = k2-2 .


x2 x2

b) A soma dos cubos de termos recíprocos, dada a sua soma.


A soma dos cubos de termos recíprocos é dada por
X3 + 4~ = k3 -3k
x3

Demonstração:
3 2 3
‘4—H)’= k3 k2

<=> x3 +3x + 3^-J +4=k3 4 = k3


X3

<=> x3 +3k + -1- = k3 .-. |x3 + -L = k3 -3k .


x3 x3_______

c) A soma das quartas potências de termos recíprocos, dada a sua


soma.
A soma das quartas potências de termos recíprocos é dada por

x4=k4-4k2+ 2
x4
166 5 Produtos Notáveis

Demonstração:

1 .
x +- = k í 1 f,4 k4
x V x/
2 ,3 ,4

x4*4x3KM2KMxK)J*K)'" k4

_1_
<=> x4 + 4x2 + 6 + 4 + x2.
+ 6 + 4 = k4
x4
x4+4(k2-2) + 6 + 4 = k4 <=> x -8
X4 +4k2-8+ +C+ 4 = k4
6 + 14

x4 + 4k2 - 2 + 4 = k4 lx4 + 4 = k4-4k2 + 2 .


x4 X4
x

d) A soma das quintas potências de termos recíprocos, dada a sua


soma.
A soma das quintas potências de termos recíprocos é dada por
x5 +4 = k5 -5k3 +5k
x5
Demonstração:
x+’=k => f| x + -
n5
| =k5
X l xJ
,3 4 5

10x3 K) * k5

<=. x5 +5x3 +10X + 10-


KM?) M-rS
x5 + 5 x3
*?)MXK) ’?=RÍ
C2 xs+5(k3-3k) + 10k + 4 = k5 x5 + 5k3— 15k + 10k + 4- = k5
X5

<=> x5 +5k3 -5k + 4 = k5 x5+4 = k5-5k3 + 5k .


b5
xx x5______________

e) A soma das sextas potências de termos recíprocos, dada a sua soma.


A soma das sextas potências de termos recíprocos é dada por
x6 +4 = k6 -6k4 +9k2 -2
x6
Os Segredos da Álgebra para IME/ITA/OLIMPÍADAS 167

Demonstração:

íx + - | = X6+6x4+20 + 15X2+4- +A + -F
\ X x2 x4 x6

=> (x + 1)6 = x6+4


1 + 6Íx4
X6

=> k6=x6+ 1 6 (k4 -4k2 +2) + 15(k2 -2) +20


X6
=> k6 = x6 + _1_
X6 + 6k4 - 24k2 +12 + 15k2 - 30 + 20

=> k6 = x6 _1_ 6k4 -9k2 + 2 => x6 4- = k6 - 6k4 + 9k2 - 2 .


+ xs +
x6

Observação: Com um pouco de manipulação algébrica, podemos escrever a


soma das sextas potências da seguinte forma:
x6 +-1. = (k2 -2)(k4 -4k2 +l).

Generalizando por equação do 2o grau e lembrando que k 2: 2.


Podemos generalizar a soma da enésima potência resolvendo a equação do
segundo grau formada, assim sua solução geral é dada por:

Demonstração:
2=k x2 +1 = kx x2-kx + 1 = 0; A = (-k)2 -4-1-1
X

-(- k)± Vk2 -4 k±7k2 -4


a = k2 -4 > 0 ; x= X = -------------------
Ti 2

k + x/k2 - 4
Se x =---------------- , então:
2
,n
xn + A k + Tk2 —
-4
V
2
xn 2 k + 7k2 -4
168 5 Produtos Notáveis

Se x = , então:

n ,n
2
xn + —
xn k-7k2 -4

Observação: Os primeiros desenvolvimentos são formas práticas e rápidas,


fica a cargo do leitor decorá-las ou não.

■j
Vamos agora desenvolver a soma de termos recíprocos xn + — , dada a
xn

condição x-- = k, com k>2. Primeiramente iremos ver algumas regras


x
práticas, ao final veremos sua generalização.

5.26) Diferença de Termos Recíprocos:


a) Soma dos quadrados de termos recíprocos, dada a sua diferença.
A soma dos quadrados de dois termos recíprocos é dada por

X2
+4 = k2+2
x______

Demonstração:

1 f 1 f,2 = k2
x - - = k <=> x - - => mm = k2
2

x V xJ

=> x2_2 + 4 = k2 .-. Ix2 + ^- = k2+2 .


xz x2

b) A diferença dos cubos de termos recíprocos, dada a sua diferença.


A diferença dos cubos de termos recíprocos é dada por
x3-4- = k3+3k
x3

Demonstração:
,3 .2
x-l=k « 1 =k3 x3 -3x2 f—1 + 3x-
x—
X x t X)
Os Segredos da Álgebra para IME/ITA/OLIMPÍADAS 169

-4x3 = k3 -L = k3
x3
x3 - 3k —4 = k3 x3 —4 = k3 + 3k .
x3 x3

c) A soma das quartas potências de termos reciprocos, dada a sua


diferença.
A soma das quartas potências de termos recíprocos é dada por

x4 + 4- = k4 + 4k2 + 2
x4
Demonstração:
1Y*4 = k4
x —1 = .k o f x —
x V x}
k2 ,3 4
k4

<=> x4-4x2+6-4^4j + 4 = k4 <=> x4 -4 x2 + 6 + -4 = k4


x4 x4

o x4-4(k2+2) + 6 + 4- = k4 x4 -4k2 -8 + 6 + — = k4
x4

<=> x4 -4k2 -2 + x4 = k4 x4 + 4- = k4 +4k2 +2 .


x4

d) A diferença das quintas potências de termos reciprocos, dada a sua


diferença.
A diferença das quintas potências de termos reciprocos é dada por
x5 - A = k5 - 5k3 + 5k
x5
Demonstração:
( 1\5
1 .
x— = k cx> |x--| =k5
x k x)
.2 4 5
X5_5x4.['lj + 10X3 k5
170 5 Produtos Notáveis

x5 -5x3 +10X-10 + = k5

x5-5[x3-41 'l 1) 1
= k5

x5-õ(k3 + 3kj + 10k~4 = k5 «=> x5 - 5k3 -15k +10k - -!■ = k5


x5
xs-5k3-5k-4 = k5 x5-4 = k5+5k3 + 5k .
<=>
X5 x5 x3_____________

e) A soma das sextas potências de termos recíprocos, dada a sua


diferença.
A soma das sextas potências de termos recíprocos é dada por

X6 4 = k6+6k4+9k2+2
x6

Demonstração:

lx--] = x6-6x4 +15x2-20 + 4-4 + 4


k xJ x2 x4 x6
=> k6 = X6 +4-6fx4
x6 l
+ X4
1
+ x2 -20
=> k6 = x6 +4-6(k4 +4k2 +2) + 15(k2 + 2)-20

=> k6 = x6 +4--6k4-24k2-12 + 15k2 +30-20


x6
4-6k4-9k2-2 => x6+4 = k6 + 6k4 + 9k2 + 2 .
=> k6 = x6
x6 x6

Observação: Com um pouco de manipulação, podemos escrever:


x6 + 4 = (k2 +2)(k4 +4k2 +1).

Generalizando por equação do 2° grau e lembrando que k i> 2 .__________


Podemos generalizar a soma da enésima potência resolvendo a equação do
segundo grau formada, assim sua solução geral é dada por:
k±7k2 + 4
X =-------------------
2
Os Segredos da Álgebra para IME/ITA/OLIMPÍADAS 171

Demonstração:
x-- = k <=> x2 -1 = kx <=> x2 - kx -1 = 0; A = (— k)2 — 4 -1 - (—1)
x
-(-k)±x/k2 +4 k±x/k2+4
A = k2 + 4 > 0 ; x= x =----------------
24 2

Aqui temos que considerar dois casos:

Para n ímpar:
k + x/k2 + 4
Se x = , então:
2
n n
k + x/k2 + 4 __ 2
x" - — 2
xn k + x/k2 4

k-x/k2 -4
Se x = , então:
2
-----\n n
k-x/k2 -4 2
xn - —
xn 2 k - x/k2 - 4

Para n par:
k + x/k2 + 4
Se x = , então:
2
n
k + Vk2 +4 ____2
xn + —
xn ~~2 k + x/k^

k - x/k2 -4
Se x = , então:
2
n .n
1 k - x/k2 - 4 2
xn
xn 2 k - x/k2 - 4
172 5 Produtos Notáveis

Exemplo Resolvido 197: Sabendo que x + — = 16 , qual o valor de x2+ —?


x x
Resolução: Podemos escrever:
x2+1 = 162-2 => x2+-l = 256-2 => x2 + 4 = 254 .
x2 x2 x2

1 3
Exemplo Resolvido 198: Sabendo que X + — = 10 , qual o valor de x
X3 '

Resolução: Podemos escrever:


x3+4 = 103-3-10 =■ x3 + 4 = 100 - 30 => x3 + 4-= 970.
x3 x3 x3

Exemplo Resolvido 199: Sabendo que x + — = 6 , determine o valor de

“*7
Resolução: Podemos escrever:
x4 +—= 64 -4-62 +2 => X4 + 4 = 1296"4• 36 + 2
x4 x4
=> x4 +4 = 1298-144 => X4 -j1
= 1154.
x4 x4

1
Exemplo Resolvido 200: Sabendo que x + - = 5, determine o valor de
x

x5+ —
X5'

Resolução: Podemos escrever:


x5 + 4 = 5S-5-53+5-5 x5 + -1 = 3125-625 + 25
x5 x5
=> x5+ 1 = 2525.
x5
Os Segredos da Álgebra para IME/ITA/OLIMPlADAS 173

1
Exemplo Resolvido 201: Sabendo que x + — = 3 , determine o valor de
x

xe+4-
X6

Resolução: Podemos escrever:


x6+4 = 36-6-34 +9 32-2 => x6+4 729-6-81 + 9-9-2
x6 x6
=> x6 + 4- = 729-486 + 81 -2 => x6+4 = 322.
x6 x6

Exemplo Resolvido 202: Se x + — = 2 , qual o valor de x100 + —!—9


X X100 '

Resolução: Podemos escrever:


xioo ,100
1 2 v/22 -4 2
E = x100 => E =
+ xioo 2 2 +V22 -4
,100
E=1100 +1100 . x100
=3 E + x100 = 2'

Agora veremos as identidades para três variáveis, identidades muito úteis no


desenvolvimento das relações de Girard (polinômios) e também em fatorações.

5.27) Quadrado da Soma de Três Termos:


O quadrado da soma de três termos é igual ao quadrado de cada um dos três
termos mais o dobro do produtos tomados dois a dois.

b22 + c2 + 2(ab + ac + bc)


(a + b + c)2 = a2 + b

Demonstração:
(a + b + c),22 = (a + b + c)-(a + b + c) <=>
(a + b + c)2 = a2 + ab + ac + ab + b2 + bc + ac + bc + c2
<=> (a + b + c)2 = a2 + b2 + c2 + 2ab + 2ac + 2bc
(a + b + c)2 = a2 + b2 + c2 + 2(ab + ac + bc) .
174 5 Produtos Notáveis

Consequências do quadrado da soma de três termos:


a) O segundo termo é negativo:
(a - b + c)2 = a2 + b2 + c2 + 2(ac - ab - bc)

Demonstração:
(a - b + c)2 = (a - b + c) (a - b + c) <=>

(a - b + c)2 = a2 - ab + ac - ab + b2 - bc + ac - bc + c2
<=> (a - b + c)2 = a2 + b2 + c2 - 2ab + 2ac - 2bc
.. (a - b + c)2 = a2 + b2 + c2 + 2(ac - ab - bc)

b) O terceiro termo é negativo:


(a + b - c)2 = a2 b2 + c2 + 2(ab - ac - bc)

Demonstração:
(a + b - c)2 = (a + b - c)-(a + b - c) «=•
(a + b - c)2 = a2 + ab - ac + ab + b2 - bc - ac - bc + c2
<=> (a + b - c)2 = a2 + b2 + c2 + 2ab - 2ac - 2bc
(a + b - c)2 = a2 + b2 + c2 + 2(ab - ac - bc).

c) O segundo e o terceiro termos são negativos:


(a - b - c)2 = a2 + b2 + c2 + 2(bc - ab - ac)

Demonstração:
(a - b - c)2 = (a - b - c) (a - b - c)

(a - b - c)2 = a2 - ab - ac - ab + b2 + bc - ac + bc + c2

(a - b - c)2 = a2 + b2 + c2 - 2ab - 2ac + 2bc

(a - b- c)2 =a2 + b2 + c2 + 2(bc - ab - ac) .


Os Segredos da Álgebra para IME/ITA/OLIMPÍADAS 175

2
Exemplo Resolvido 203: Desenvolva (a + 2b + 3c) .
Resolução: Podemos escrever:
(a + 2b + 3c)2 = a2 + (2b)2 +(3c)2 +2[(a-2b) + (a-3c) + (2b-3c)]
=> (a + 2b + 3c)2 = a2 + 4b2 + 9c2 + 2(2ab + 3ac + 6bc)
=> (a + 2b + 3c)2 = a2 + 4b2 + 9c2 + 4ab + 6ac + 12bc .

Exemplo Resolvido 204: Desenvolva (4 - 2m + n)2.

Resolução: Podemos escrever:


(4-2m + n)2 = 42 +(2m)2 +n2 + 2[4n-(4-2m)-(2mn)]

=> (4 - 2m + n)2 = 16 + 4m2 +n2 + 2(4n - 8m - 2mn)

=> (4-2m + n)2=16 + 4m2 + n2 + 8n -16m-4mn .

Exemplo Resolvido 205: Desenvolva (x + y - 2z)2 .

Resolução: Podemos escrever:


(x + y-2z)2 = x2 +y2+(2z)2 +2(xy-x 2z-y-2z)

=> (x + y-2z)2 = x2+y2+4z2+ 2(xy-2xz-2yz)

=> (x + y-2z)2 =x2+y2 + 4z2 + 2xy-4xz-4yz.

Exemplo Resolvido 206: Desenvolva (1-x-y)2.

Resolução: Podemos escrever:


(1-x-y)2 =12 + x2 + y2 + 2(x-y-1x-1-y)

=> (1-x-y)2 =1 + x2+y2+2(xy-x-y)

=> (1-x-y)2 = 1 +x2+y2+2xy-2x-2y .

5.28) Identidade de Lagrange para Três Termos:


A identidade de Lagrange para três termos é útil em fatorações mais
rebuscadas, cuja percepção do leitor esteja bem apurada.
(ax-í-by + cz)2 + (ay -bx)2 + (az-cx)2 + (bz-cy)2 = (a2 +b2 + c2)(x2 + y2 + z2 j
176 5 Produtos Notáveis

Demonstração:
E = (ax + by + cz)2 + (ay-bx)2 + (az-cx)2 + (bz-cy)2

=> E = (ax)2 + (by)2 + (cz)2 + 2àx-by + ,2ax<íí + 2by-.'Cz + (ay)2 -

- 2~'ay-.bx + (bx)2 + (az)2 - ^a-z^cx + (cx)2 + (bz)2 - 2'';:bz-::cy + (cy)2

=> E = a2x2 + b2y2 + c2z2 + a2y2 + b2x2 + a2z2 + c2x2 + b2z2 + c2y2

=> E = a2 (x2 + y2 + z2) + b2 (y2 + x2 + z2) + c2 (z2 + x2 + y2)

(ax +by + cz)2 + (ay-bx)2 + (az-cx)2 + (bz - cy)2 = (a2 +b2 + c2)(x2 + y,22+z2)

Exemplo 207: Desenvolva (x + 2y + 3z)2 +(y-2x)2 +(z-3x)2 + (2z-3y)2 .


Resolução: Podemos escrever:
(x + 2y + 3z)2 + (y - 2x)2 + (z - 3x)2 + (2z - 3y)2 = (l2 + 22 + 32 )(x2 + y2 + z2)

(x + 2y + 3z)2 + (y - 2x)2 + (z - 3x)2 + (2z - 3y)2 = 14 (x2 + y2 + z2)

5.29) Produto Dois a Dois Elevado ao Quadrado:


Podemos também desenvolver a soma dos produtos dois a dois elevado ao
quadrado, segue o mesmo raciocínio da soma de três termos:
(ab + bc + ac)2 = (ab)2 + (bc)2 + (ac)2 + 2abc(a + b + c)

Demonstração:
E = (ab + bc x ac)2 => E = (ab +bc + ac) (ab +bc +ac) o

E = a2b2 + ab2c + a2bc + ab2c + b2c2 + abc2 + a2bc + abc2 + a2c2


(ab + bc + ac)2 = a2b2 +b2c2 + a2c2 + 2a2bc + 2ab2c + 2abcz

(ab + bc + ac)2 = (ab)2+ (bc)2+(ac)2+2abc(a + b + c) .

Exemplo Resolvido 208: Desenvolva (2y + yz + 2z)2.

Resolução: Podemos escrever:


(2y + yz - 2z)2 = (2y)2 + (yz)2 + (2z)2 + 2 ■ 2 • yz(2 + y + z)

=> (2y + yz + 2z)2 = 4y2 + y2z2 + 4z2 + 4yz(2 + y + z).


Os Segredos da Álgebra para IME/ITA/OLIMPÍADAS 177

2
Exemplo Resolvido 209: Desenvolva (m + 4mn + 4n) .

Resolução: Podemos escrever:


(m + m • 4n + 4n)2 = (1 • m)2 + (m ■ 4n)2 + (1 -4n)2 + 2 ■ 1 • m ■ 4n(1 + m + 4n)

=> (m + 4mn + 4n)2 = m2 +16m2n2 +16n2 + 8mn(1 + m + 4n).

Problemas Propostos

Questão 5.49 (AHSME-1952-1954 / CN-1986)

Se
7x + —7? = 3 , então x7 + -y1 e• igual a:
k xj xJ
a)1 b)2 c) 0
d) 3 e) 6

Questão 5.50 (CN-2014)


3
Seja x um número real, tal que x + - = 9 . Um possível valor de x-2 é 71.
x X
Sendo assim, a soma dos algarismos de "a" será:
a) 11 b) 12 c) 13 d) 14 e) 15

Questão 5.51
1 x3 + -1- = m e
Seja x um número real não nulo tal que x + - = 4 , se
x xJ
4 1 , . . , m+n
x + —— = n , determine o valor de--------.
x4 m -n

Questão 5.52 (AMC-2007)


Suponha que o número a satisfaça a equação 4 = a + a . Qual o valor de
a4 + a 4?
a) 169 b) 172 c) 192
d) 194 e) 212
178 5 Produtos Notáveis

Questão 5.53 (Stanford-2010)

Se x2 + -4 = 7 , determine o valor de x5 + 4.
xz xb

Questão 5.54

Se X + — = 1 , determine o valor de 5lx5 + X5 '


x V

Questão 5.55 (Singapura)

x6 1
+ x6
Se x2 - 4x +1 = 0, determine o valor de —
x3+-t-

Questão 5.56

Se x + - = , determine o valor de x7 + X7'


1
x

Questão 5.57
Seja r um número real, tal que x/r + 4 = 3 . Calcule o valor de r3 + 1
vr p-
Questão 5.58

Seja r um número real positivo, tal que -Jr - = 14 . Prove que x/r + — = 6 .
Vr x/r

Questão 5.59 (CN-1984)


- 2 2 2 £ £ 8
Se — + e x +y +z = 16 , o produto xyz é:
x y z yz xz xy 3
a) 192 b) 48 c) 32 d) 108 e) 96

Questão 5.60 (CN-1999)


Se m + n -p = 6 , mnp = 2 e mn + mp + np = 11, podemos dizer que o valor de
m P_ a
—-r— é:.
np mp mn
a) 1 b) 3 c)7 d) 18 e) 22
Os Segredos da Álgebra para IME/ITA/OLIMPÍADAS 179

Questão 5.61 (CN-2011)


_1_ 1 L. ■
Sejam a, b e c números reais, tais que — + + — = p, ab + bc + ac = r e
ab bc ac
a b c a b + - = q . O valor de q2 + 6q é sempre igual a:
b a a c c b
p2r2 + 9 P2f2 -9p
a) b) c) p2r2 -9
4 12
p2r2 -10
d) e) p2r2 -12p
4r

Questão 5.62 (Harvard/MIT-2008)

As raizes da equação x3 - 9x2 + 8x + 2 = 0 são p, q, r. Calcule — 1 1


P2 q2 r2

Questão 5.63 (AHSME-1981)


Para todo número positivo x, y e z, o produto
(x + y + z)’1 • (x-1 + y-1 + z"1) • (xy + yz + xzf1 • [(xy)-1 + (yz)
+y é

igual a:
a) x-2 •y •z-2 b) x“z + y-2

c) (x + y + z) d) —
xyz

e)-------- ---------
xy + yz + xz

Questão 5.64 (AHSME-1991)

Se x + Vx2-1 +----- J— = 20 , então x2 + 7x4 -1 +-------- 1


é igual a:
x - X2 — 1 X2+Vx4-1
a) 5,05 b) 20 c) 51,005 d) 61,25 e) 400

Questão 5.65
,5 ,5
5 + 721 5-V2Í
Calcule o valor de
2 2
180 5 Produtos Notáveis

Questão 5.66
>10 ,10
1 + >/5 1-75
Determine o valor de
2 2

Questão 5.67 (CN-1998)


/ i—\1997 ,1997 / »—\1997 ’ , <->1997
(2+73) + (2-73) (2+73)
Sejam x = e y= , o
2 73
valor de 4x2 - 3y2 é:
a) 1 b) 2 c) 3 d) 4 e) 5
Os Segredos da Álgebra para IME/ITA/OLIMPÍADAS 181

5.30) Identidades de Argand:


Outra identidade que requer uma boa criatividade do leitores é a identidade de
Argand, também encontrada em olimpíadas de Matemática e em colégios
militares. A identidade de Argand é dada por:
(a2 + ab + b2j(a2 - ab + b2)= a4 + a2b2 b4

Demonstração:
(a2 + ab + b2)(a2 - ab + b2 j = a4 -a3b + a2b2 + a3b - a2b2 + ab3 + a2b2 - ab3 + b4

(a2 + ab + bZ)(a2 - ab + t>2)= a4 + a2b2 + b4 .

Consequência para b = 1 :
Esse caso particular da identidade de Argand é bastante útil:
(a2 + a1 + 12)(a2 - a-1 + 12j= a4 + a2-12 + 14

(a2 + a + "l)(a2 - a + ij = a4 + a2 + 1 .

Generalizando a Identidade de Argand, temos:


Podemos generalizar a identidade de Argand para a soma de uma potência
qualquer, assim temos:
am • bn + b2n )(ai:2m - a'im • bn + b2n ) = a'4m + a2m ■b2n+ b4n
(ai2m + a'

Demonstração:
(a2m + am -bn +b2n)(a2m -am bn+b2n) = a'4m ,3j + a'
2n +

+ ,3l - a' ‘ ->t< + b4n


b2n
- +a2m'b

a2m + am ■ bn + b2n )(>“ -am-bn + b2n) = a4m + a2m ■b2n + b4n

Exemplo Resolvido 210: Efetue (4p2 + 2pq + q2)(4p2 -2pq + q2).

Resolução: Podemos escrever:


(4p2 + 2pq + q2 )(4p2 - 2pq + q2) = (2p)4 + (2p)2 ■ qI:2 +q4

=> (4p2 +2pq + q2)(4p2 -2pq + q2) = 16p4 +4p2q2 +q4 .


182 5 Produtos Notáveis

Exemplo Resolvido 211: Efetue (49t2 + 7t + l)(49t2 -7t + l).

Resolução: Podemos escrever:


(49t2 + 7t +1)(49t2 -7t + ij = (7t)4 + (7t)2 +1

=> (49t2 + 7t + l)(49t2 -7t +1) = 2401t4 + 49t2 +1.

Exemplo Resolvido 212: Efetue


[(xy)2m +(xy)m .(3z)n +(3z)2n][(xy)2m -(xy)m .(3z)n + (3z)2n]

Resolução: Podemos escrever:


E=[(xy)2m+(xy)m.(3z)n+(3z)2n][(xy)2m -(xy)m-(3z)n (3z)2n]
=> E = (xy)4m+ (xy)2m (3z)2n+(3z)4n
. c v4m..4m , 2oi,.2oi n2n_2n , o4n 4n
tz — a y
y ta y z +o z

Agora veremos a identidade para quatro variáveis ao quadrado, ou seja, o


quadrado da soma de quatro termos.

5.31) Quadrado da Soma de Quatro Termos:


O quadrado de quatro termos é igual á soma dos quadrados de cada termo
mais o dobro dos produtos tomados dois a dois.
I(a + b + c + d)2 = a2 + b2 + c2 + d2 + 2(ab + ac + ad + bc + bd + cd)

Demonstração:
2
(a + b + c + d) = (a + b + c + d)-(a + b + c + d)

(a + b + c + d)2 = a2 + ab + ac + ad + ab + b2 +bc + bd + ac + bc + c2 +
+ cd + ad + bd + cd + d2
(a + b + c + d)2 = a2 + b2 + c2 + d2 + 2ab + 2ac + 2ad + 2bc + 2bd + 2cd

.-. (a + b + c + d)2 = a2+b2+c2 + d2+2(ab + ac + ad + bc + bd + cd) .

Exemplo Resolvido 213: Desenvolva (x + 2y + 2z + 1)2 .


Os Segredos da Álgebra para IME/ITA/OLIMPÍADAS 183

Resolução: Podemos escrever.


(x + 2y+2z + 1)2 = x2 +(2y)2 + (2z)2 +12 +

+ 2[(x • 2y) + (x • 2z) +(x ■ 1) + (2y • 2z) + (2y ■ 1) + (2z-1)J


(x + 2y+ 2z + 1)2 = x2 +4y2 +4z2 + 1 +2(2xy + 2xz+ x + 4yz + 2y+ 2z).

O caso seguinte é a soma de três termos elevado ao cubo, ou seja (a b + c)3,


vamos desenvolvé-lo!

5.32) Cubo da Soma de Três Termos:


O cubo da soma de três termos é dado por:
(a + b + c)3 = a3 + b3 + c3 + 3ab2 + 3ac2 + 3a2b + 3a2c + 3b2c + 3bc2 + 6abc(

Demonstração:
(a + b + c)3 = (a + b + c)2(a + b + c)

<=> (a + b + c)3 = (a2 + b2 + c2 +2ab + 2ac + 2bcj (a + b + c)

<=> (a + b + c)3 = a3 + ab2 + ac2 + 2a2b + 2a2c + 2abc + a2b + b3 + bc2 +


+ 2ab2 + 2abc + 2b2c + a2c + b2c + c3 + 2abc + 2ac2 + 2bc2
(a + b + c)3 = a3 + b3 + c3 + 3ab2 + 3ac2 + 3a2b + 3a2c + 3b2c + 3bc2 + 6abc .

Podemos escrever essa soma de outras quatro formas, a saber:


a) Forma de soma dois a dois
(a + b + c)3 = a3 + b3 + c3 + 3a2 (b + c) + 3b2 (a + c) + 3c2 (a + b) + 6abc

Demonstração:
(a + b + c)3 = a3 + b3 + c3 + 3ab2 + 3ac2 + 3a2b + 3a2c + 3b2c + 3bc2 + 6abc

(a + b + c)3 = a3 + b3 + c3 + 3a2 (b + c) + 3b2 (a + c) + 3c2 (a + b) + 6abc .

b) Como soma dos quadrados da soma dos três termos:


(a + b + c)3 = 3 (a + b + c)(a2 + b2 + c2 j- 2 (a3 + b3 +c3) + 6abc
184 5 Produtos Notáveis

Demonstração:
(a + b + c)3 = a3 + b3 +c3 + 3ab2 + 3ac2 + 3a2b + 3a2c + 3b2c + 3bc2 + 6abc

(a + b + c)3 = 3a3 + 3b3 + 3c3 + 3ab2 + 3ac2 + 3a2b + 3a2c + 3b2c + 3bc2 +
+ 6abc - 2^a3 + b3 +c3)

(a + b + c)3 = 3a3 + 3b3 + 3c3 + 3ab2 + 3ac2 + 3a2b + 3a2c + 3b2c + 3bc2 -
-2^a3 + b3 + c3) + 6abc

(a + b + c)3 = 3a2 (a + b + c) + 3b2 (a + b + c) + 3c2 (a + b + c) - 2 (a3 + b3 + c3)

+ 6abc
(a + b -r c)3 = 3(a + b + c)(a2 + b2 + c2)-2(a3 +b3 + c3) + 6abc .

c) Como produto dois a dois:


(a + b + c)3 = a3 +b3 +c3 + 3(a + b + c)(ab + ac + bc)-3abc

Demonstração:
E = (a + b + c)3

E = a3 + b3 + c3 + 3ab2 + 3ac2 + 3a2b + 3a2c + 3b2c + 3bc2 + 6abc


E = a3 + b3 + c3 + 3ab2 + 3ac2 + 3a2b + 3a2c + 3b2c + 3bc2 + 3abc + 3abc
E = a3 + b3 + c3 + 3ab2 + 3ac2 + 3a2b + 3a2c + 3b2c + 3bc2 + 3abc + 3abc +
+ 3abc - 3abc
E = a3 + b3 + c3 + 3ab(a + b + c) + 3ac(a + b + c) + 3bc(a + b + c) - 3abc
|(a + b + c)3 = a3 + b3 + c3 + 3 (a + b + c)(ab + ac + bc) - 3abc .

d) Como produto da soma dois a dois:


(a + b-i-c)3 =a3+b3+c3 + 3(a-rb)(a + c)(b + c)

Demonstração:
(a + b + c)3 = a3 + b3 + c3 + 3ab2 + 3ac2 + 3a2b + 3a2c + 3b2c + 3bc2 + 6abc
(a + b + c )3 = a3 + b3 + c3 + 3ab2 + 3ac2 + 3a2b + 3a2c + 3b2c + 3bc2 + 3abc + 3abc
(a + b + c)3 = a3 + b3 + c3 + 3ab (a + b) + 3ac (a + c) + 3bc (a + c) + 3bc (a + b)
Os Segredos da Álgebra para IME/ITA/OLIMPÍADAS 185

(a + b + c)3 =a3 + b3 +c3 + 3b(a + b)(a + c) + 3c(a + c)(a + b)

|(a + b + c)3 = a3* b3T c3-r 3(a + b)(a + c)(b -r ç)[.

Consequências (Identidade de Gauss):


Podemos escrever a identidade de Gauss de duas formas:
a) Como soma de quadrados simples:
a3 +b3 +c3 -3abc = (a + b + c)(a2 + b2 + c2 -ab-ac -bcj

Demonstração:
a3 + b3 + c3 + 3 (a + b + c)(ab + ac + bc)-3abc = (a + b + c)3

a3+b3+c3+3(a+b + c)(ab + ac + bc)-3abc = (a + b + c)2(a + b-rc)

a3 + b3 + c3 = (a2 + b2 + c2 + 2ab + 2ac - 2bc)(a + b + c)-

- 3(a + b + c)(ab + ac + bc) + 3abc


a3 + b3 + c3 = (a + b + c) •
•|^a2 + b2 + c2 + 2ab + 2ac + 2bc -3(ab + ac +bc)”| + 3abc

a3 + b3 + c3 - 3abc = (a + b + c)(a2+b2+c2-ab-ac-bcj .

b) Como soma de quadrados da diferença:


(a + b + c)- (a - b)2 + (a-c)2 (b-c)2
a3 + b3 +c3 -3abc =
——— 2 ~

Demonstração:
(a + b + c) 2 Ía2+b2+c2-ab-ac-bc)
a3 + b3 + c3 - 3abc = - ------------ ------ 1’
2
(a + b + c)-Í2a2 + 2b2 + 2c2 - 2ab - 2ac - 2bc)
a3 + b3 + c3 - 3abc =---------------- ---------------------------------------------------- -
2
„ „ (a+ b + c)ía2 - 2ab+ b2 + a2 - 2ac + c2+b2 - 2bc + c2
a3 + b3 + c3 - 3abc =---------------------------------------------------------------------------------- -
2
186 5 Produtos Notáveis

(a + b + c)- (a-b)2 +(a-c)2 +(b-c)2


a3 + b3 + c3 - 3abc =
2 ~

Veremos agora a soma de quatro termos elevada ao cubo! É interessante a


forma com que podemos escrevè-la, você notará quando estiver nas
identidades condicionais lá na frente, vamos lá!

5.33) Soma de quatro termos elevado ao cubo:


A soma de quatro termos elevada ao cubo é dada por:
(a + b + c + d)3 = a3 +b3 +c3 +d3 -3ab(c + d)-3cd(a + b) +

+ 3 (a + b + c + d)-(ab + ac + ad +bc + bd + cd)


Demonstração:
(a-b + c + d)3 = (a +b +c +d)2 (a+ b + c +d) cs

(a + b + c + d)3 = (a2 + b2 + c2 + d2 + 2ab + 2ac + 2ad + 2bc + 2bd + 2cd) •


(a + b + c + d)
(a + b + c + d)3 = a3 + ba2 + ca2 + da2 + ab2 + b3 + cb2 + db2 + ac2 + bc2 + c3 +
+dc2 + ad2 + bd2 + cd2 + d3 + 2a2b + 2ab2 + 2abc + 2abd + 2a2c + 2abc + 2ac2 +
+2acd + 2a2d + 2abd + 2acd + 2ad2 +2abc + 2b2c + 2bc2 +2bcd + 2abd +
+2b2d + 2bcd + 2bd2 + 2acd + 2bcd + 2c2d + 2cd2
cs (a + b + c -í- d)3 = a3 + b3 + c3 + d3 + 3a2b + 3a2c + 3a2d + 3ab2 + 3b2c + 3b2d +
+3ac2 + 3bc2 + 3czd + 3ad2 + 3bd2 + 3cd2 + 6abc + 6abd + 6acd + 6bcd + 3abc -
-3abc + 3abd - 3abd + 3acd - 3acd + 3bcd - 3bcd
o (a + b + c + d)3 = a3 + b3 + c3 + d3 + 3ab (a + b + c + d) +
+ 3ac(a + b + c + d) + 3ad(a + b + c + d) + 3bc(a + b + c + d) +
+ 3bd (a + b + c + d) + 3cd (a + b + c + d) - 3abc - 3abd - 3acd - 3bcd
(a + b + c + d)3 = a3 + b3 +c3 +d3 -3ab(c + d)-3cd(a + b) +
+ 3(a + b + c + d) ■ (ab + ac + ad + bc + bd + cd)
Os Segredos da Álgebra para IME/ITA/OLIMPÍADAS 187

Veremos agora a quarta potência de três termos! Observe como podemos


escrevê-la, você notará quando estiver na parte de identidades condicionais,
vamos lá!

5.34) Quarta Potência de Três Termos:


A quarta potência de três termos é dada por:
(a + b + c)4 = a4 + b4 + c4 +4(a + b + c)2 (ab + ac + bc)-

-2(ab + ac + bc)2 -2abc(a + b + c)

Demonstração:
(a + b + c)4 =[ja + b + c)2j

=> (a + b + c)4 = |ja2 +b2 +c2j + 2(ab + ac + bc)j

=> (a + b + c)4 = a4+b4+c4+2(ab)2+2(ac)2+ 2(bc)2 +

+ 4(ab + ac + bc)2 + 4^(a + b + c)2 -2(ab + ac + bc)l(ab + ac + bc)

=> (a + b + c)4 = a4 +b4 +c4 + 2(ab)2 + 2(ac)2 + 2(bc)2-

-8(ab + ac + bc)2 +4 (a + b + c)2 (ab + ac + bc) + 4(ab + ac + bc)'.2

=> (a + b + c)4 = a4 +b4 +c4 + 2(ab)2 +2(ac)2 + 2(bc)2 -

-4(ab + ac + bc) +4(a + b + c) (ab + ac + bc)


=> (a + b + c)4 = a4 +b4 + c4 +2(ab)2 + 2(ac)2 +2(bc)2 +

+4(a + b + c)2(ab + ac + bc)-4^(ab)2 + (ac)2 + (bc)2 + 2abc(a + b + c)j

=> (a + b + c)4 = a4 +b4 + c4 -2(ab)2 -2(ac)2 -2(bc)2 -

-8abc(a + b + c) + 4(a + b + c) (ab + ac + bc)


=> (a + b + c)4 = a4 +b4 + c4 + 4(a +b + c)2 (ab + ac + bc)-

-2abc(a + b + c)-2^(ab)2 +(ac)2 +(bc)2 + 2abc(a + b + c)^j

(a + b + c)4 = a4 +b4 +c4 +4(a + b + c)2(ab + ac + bc)~

-2(ab + ac + bc)2 -2abc(a + b + c)


188 5 Produtos Notáveis

Problemas Propostos

Questão 5.68 (IMO-Longlist-1988 / AHSME-1975)


Se p, q e r são as raizes distintas da equação x3 - x2 x - 2 = 0, então
p3 +q3 +r3 é igual a:
a) -1 b) 1 c) 3 d) 5 e) NDA

Questão 5.69 (Putnam-1939-Modificada)


As raizes de x3 + ax2 + bx + c = 0 são a, P e y . Determine a3 + p3 + y3.

Questão 5.70 (AIME-2008)


Sejam r, s e t as très raizes da equação 8x3 +1001x + 2008 = 0 . Determine
(r + s)3+(s + t)3+ (t + r)3.

Questão 5.71 (Stanford-2007)


Se r + s +1 = 3, r2 +s2 + t2 =1 e r3 +s3 +13 = 3, calcule r • s ■ t.

Questão 5.72 (Stanford-2007)________________________________________


As raizes de x3 -7x2 -6x + 5 = 0 são a, b e c. Calcule (a + b)(a + c)(b + c).

Questão 5.73
(24 + 22 +l)(44 + 42 +l)(64 +62 +l)...(324 +322 +l)
Sabendo que A =
(l4 +12 +l)(34 +32 +l)(54 +52 +l)...(314 + 312 + l)

determine o valor de A-1053.

Questão 5.74
Mostre que (xy + yz + xz)3 = (xy)3 + (yz)3 + (xz)3 + 3xyz(x + y)(x + z)(y + z).

Questão 5.75 (Noruega-1996-Modificada)


Sejam x, y e z números naturais com x < < y y < z, tais que xyz = 78 e
x3 +. ..3
y3 + z3
x2 -i- yz + z2 = 206. Determine o valor de
xy + xz + yz
Os Segredos da Álgebra para IME/ITA/OLIMPÍADAS 189

Questão 5.76
b2)(a4-a2b2 + b4)...[a2n-a2n’1b2' + b2"
Efetue (a2 - ab + , para a * ±b.

Agora veremos uma importante identidade que relaciona produtos notáveis


com equações do terceiro grau. Nos exercícios veremos uma resolução usando
essa identidade!

5.35) Identidade de Stevin Para Três Termos:


A identidade de Stevin para três termos tem relação com as equações do 3°
grau, veremos as raizes positivas e as negativas.
a) Todas as Raizes Positivas
|(x + a)(x + b)(x + c) = x3 + (a + b + c)x2 + (ab + ac + bc)x + abcl

Demonstração:
(x + a)-(x + b)(x + c) = (x2+ax + bx + abj-(x + c)

(x + a) (x + b) (x + c) = x3 + ax2 + bx2 + abx + cx2 + acx + bcx + abc


■■■ |(x + a)(x + bM x + c) = x3+(a + b + c)x2 +(ab + ac + bc)x + abc .

b) Todas as Raizes Negativas


nx-a) (x - b)-(x -c) = x3 - (a + b + c)x2 +(ab + ac + bc)x -abc

Demonstração:
(x-a)-(x-b)(x-c) = (x2 - ax - bx + abj- (x - c)

(x - a) (x - b) (x- c) = x3 - ax2 - bx2 + abx -cx2 + acx + bcx - abc


.-. (x-a)(x-b)-(x-c) = x3-(a + b + c)x2+(ab + ac + bc)x - abc

c) Quatro Raízes Positivas


(x-a)(x-b')(x-c)-(x-d) = x4-(a + b + c + d)x3 +
+ (ab + ac + ad + bc + bd + cd)x2 - (abc-r abd + acd -bcd)x + abcd
190 5 Produtos Notáveis

Demonstração:
(x-a)(x-b){x-c)(x-d) = (x2-ax-bx + abj (x2 -cx-dx + cd)

(x-a)(x-b)(x - c)(x - d) = x4 -ax3 -bx3 + abx2 -cx3 + acx2 +

-rbcx2 - abcx - dx3 + adx2 + bdx2 - abdx + cdx2 - acdx - bcdx + abcd

(x-a)(x-b)(x-c)-(x-d) = x4-(a + b + c + d)x3 +

+(ab + ac + ad + bc + bd + cd)x2 -(abc + abd + acd + bcd)x + abcd

Consequência 01:
Para x = 1, temos:
a) (1 + a) • (1 + b) ■ (1 + c) = 13 + (a + b + c) • 12 + (ab + ac + bc) ■ 1 + abc

|(1 -r a) (1-rb)-(1-^c) = a + b + c -t-ab -i- ac + bc + abc +1|.

b) (1-a)(1-b)(1-c) = 13-(a + b + c)-12 + (ab + ac + bc) -1-abc


1(1 -a)(1-b)(1-c) = ab + ac + bc - a - b-c-abc + ll.

c) (1 — a)-(1 — b)-(1 — c)-(1 —d) = 14 - (a + b + c + d)-13 +


+ (ab + ac + ad + bc + bd <-cd)-12 - (abc + abd + acd + bcd)1 + abcd
(1-a) (1-b) (1-c) (1-d) = 1-a- b- c-d + ab + ac + ad + bc +
+bd + cd - abc - abd - acd - bcd + abcd

Consequência 02 (identidade de Gauss):


Para a, b e c, temos:
|(a + b)(a + c)(b + c) = (ab + ac + bc)(a + b + c) - abc

Demonstração:
(a +b)(a + c)(b + c) = (a2 +ac + ab + bcj(b + c)

(a + b)(a + c)(b + c) = a2b + abc + ab2 + b2c + a2c + ac2 + abc + bc2
(a-rb)(a + c)(b + c) = ab(a + c) + ac(a + c) + bc(a + c) + ab2+b2c + abc-abc
(a + b)(a + c)(b + c) = (a + c)(ab + ac +bc) + b(ab + ac + bc)-abc
Os Segredos da Álgebra para IME/ITA/OLIMPÍADAS 191

.. b)(a + c)(b + c) = (ab + ac + bc)(a + b + c) - abcl.

Exemplo Resolvido 214: Efetue (x + 3) (x + 6)-(x + 7).


Resolução: Podemos escrever:
(x + 3)• (x + 6)• (x + 7) = x3 + (3 + 6 + 7)x2 + (3 6 + 3-7 + 6• 7)x+ 3■ 6■ 7
=> (x + 3) ■ (x + 6) ■ (x + 7) = x3 + 16x2 + (18 + 21 + 42)x +126
=> (x + 3) • (x + 6) ■ (x + 7) = X3 +16x2 +81X + 126.

Exemplo Resolvido 215: Efetue (x-4)-(x-5)(x-11).

Resolução: Podemos escrever:


(x -4)(x - 5)(x-11) = x3 -(4 + 5 +11)x2 + (4 5 + 4 ■ 11 + 5 11)x -4 • 5-11
=> (x - 4) ■ (x-5) • (x -11) = X3 -20x2 + (20 + 44 + 55)x- 220
=> (x - 4) ■ (x - 5) ■ (x -11) = x3 - 20x2 +119x - 220 .

Exemplo Resolvido 216: Efetue (x-1)(x-2)(x-3)(x-4).

Resolução: Podemos escrever:


(x - 1)(x - 2)(x - 3)(x - 4) = x4 - (1 + 2 + 3 + 4)x3 +

+ (1-2 + 1- 3 + 1- 4 + 2- 3 + 2- 4 + 3-4)x2-
-(1-2-3+1-2-4 + 1-3-4 + 2-3-4)x + 1-2-3-4
=> (x - 1)(x - 2)(x - 3)(x - 4) = x4-10x3 + (2 + 3 + 4 + 6 + 8 + 12)x2 -
- (6 + 8 +12 + 24)x + 24
=> (x - 1)(x - 2)(x - 3)(x - 4) = x4 -10x3 +35x2 -50X + 24 .

5.36) Identidade de Sophie-Germain:


A identidade de Sophie-Germain é um produto notável muito útil em olimpíadas
internacionais, tem até aplicação em congruência modular. Ela se parece muito
com o quadrado da soma e é dada por:
a4+ 4b4 = (a2+ 2ab + 2b2)(a2- 2ab + 2b2)
192 5 Produtos Notáveis

Demonstração:
a4 + 4b4 = a4 + 4b4 + 4a2b2 -(2ab)2 a4 +4b4 = (a2 +2b2) ~(2ab)2

a4 + 4b4 = (a2 + 2b2 + 2abj(a2+ 2b2- 2ab) .

Consequências:
a) Para b = 1:
a4 + 4 14 = (a2+ 2 12+ 2a-l)(a2+ 2-12- 2a-1)

a4 + 4 = (a2 + 2a + 2)(a2 - 2a + 2) .

b) Para b = -:
2
2~ 2

a4 + 4 • (A) ° [a2+a+2’

+ a + iVa2- a +
2

Exemplo Resolvido 217: Efetue 16p4 + 324q4 .


Resolução: Podemos escrever:
16p4 + 324q4 = (2p)4 + 4 ■ (3q)4 =>
16p4 + 324q4 = [(2p)2 +2(3q)2 + 2(2p)(3q)] ■ [(2p)2 + 2(3q)2 -2(2p)(3q)]

=> 16p4 + 324q4 = ^4p2 + 18q2 +12pqj^4p2 +18q2 -12pq^.

Exemplo Resolvido 218: Efetue 625m4 +4 .


Resolução: Podemos escrever:
E = 625m4+4 => E = (5m)4+4

=> E = [(5m)2 + 2(5m ) + 2][(5m)2 -2(5m) + 2^

625m4+4 = (25m2+10m + 2)(25m2 -10m + 2).


Os Segredos da Álgebra para IME/ITA/OLIMPÍADAS 193

Exemplo Resolvido 219: Prove que 16a4 + = ^4a2 + 2a + 4a2 - 2a


4
i)
Resolução: Podemos escrever:
16a4 + —= (2a)4 + - =>16a4 +-
4 ' ' 4 4
= [(2a)2+- (2a) + lJ(3x)2-(2a)+l]
16a4 +- = f4a2 + 2a + -1 4a2 - 2a + —
4 2 2

5.37) Uma Identidade Interessante _____________________________________


Existe uma identidade muito parecida com a identidade de Sophie-Germain,
podemos dizer que é um caso particular, o seu raciocínio é o mesmo, mas
apenas um detalhe chama atenção. É dada por:
a4 + b4 = (a2 + ab72 + b4)(a2- ab72 + b2)

Demonstração:
a4 + b4 = a4 + b4 + 2a2b2-2a2b2 => a4 + b4 = (a2+b2 j -(ab72)

a4 + b4 = (a2 + ab72 +b2)(a2 -ab72 +b2 j .

Consequência:_____________________
Para b = 1, temos:
a4 + 1 = (a2 + a72 + l)(a2 - a 72 + 1)

Demonstração:
a4 + 1 = a4 + 1 + 2a2 - 2a2 a4 + 1 = (a2 + 1)2 - (a72)2

a4 + 1 = (a2 + a72 + l)(a2 - a 72 + ij .

Exemplo Resolvido 220: Efetue 81x4 +1 = ^9x2 + 3x72 + lj^9x2 -3x72 + lj.

Resolução: Podemos escrever:


81x4+1 = (3x)4+1 => 81x4+1 = ^(3x)2+(3x)-72+1^(3x)2-(3x)-72+l"|

81x4 +1 = (9x2 +3x72+ íj(9x2 -3x72+l).


194 5 Produtos Notáveis

5.38) Identidade de Chrystal:


Uma identidade diferente, que tem algumas aplicações em olimpíadas e é dada
por:
(b-c)2 +(b + c)2 + 2^b2 -c2)
1 2 1
b4-2b2c2+c4 (b-c)2 (b2-c2) (b + c)2

Demonstração:
(b - c)2 + (b + c)2 + 2^b2 - c2) (b-c)2 +(b + c)2 + 2^b2 — c2 i
E =-------------------------------------- => E =

E;
b4-2b2c2+c4

(b~c)2 I (b + c)2 , 2(b2 c2)


F4
(b2-c2)2 (b2-c2)2 (b2-c2)2

(b-c)2 | (b + c)2 [ 2
=> E =
[(b + c)(b-c)]2 [(b + c)(b-c)]2 (b2-C2)

=> E. (b~c)2 , (b+c)2 , 2


(b + c)2(b-c),22 (b + c)2 (b-c)2 (b2-c2)

(b - c)2 + (b + c)2 + 2^b2 - c2 j


1 2 1
b4-2b2c2+c4 (b-c)2 (b2-c2) (b + c)2

(x-1)2+(x + 1)2+2(x2-12j
Exemplo Resolvido 221: Calcule
x4 -2x2+1

Resolução: Podemos escrever:


(x-1)2 +(x + 1)2 +2(x2 -12) 1.
- 1 2
x4 - 2 x2 -12+14 (x-1)2+(x2-12) + (x + 1)2

(x-1)2+(x + 1)2 + 2(x2-12)


1 2 1
x4 -2x2 +1 = (x-1)2+(x2-l) + (x + 1)2'
Os Segredos da Álgebra para IME/ITA/OLIMPÍADAS 195

(2x-3)2 (2x + 3)2 + 2(4xz -9)


Exemplo Resolvido 222: Determine
16x4 -72x2 -81

Resolução: Podemos escrever:


(2x -3)2 + (2x + 3)2 + 2((2x)2 -32)
1 2 1
(2x)4 — 2-(2x)2 • 32 + 34 |2+[(2x)2-32) + (2x + 3),2:
(2x-3f
4x2 9

(2x - 3)2 + (2x + 3)2 + 2(4x2 - 9)


1 2 , 1
-----------7 +
16x4-72x2+81 (2x-3)2 (4x2-9) (2x + 3)2

Problemas Propostos

Questão 5.77_________________________
Determine o valor das expressões abaixo:
1 1 1
a) (a-b)(a-c) + (b-a)(b-c) (c-a)(c-b)

a b c__
b) (a -b)(a -c) + (b-aj(b-c) + (c-a)(c -b)

a2 b2 < c2
c)
(a-b)(a-c) (b-a)(b-c) (c-a)(c-b)
a3 b3 . c3
d)
(a-b)(a-c) (b-a)(b-c) (c-a)(c-b)

Questão 5.78__________________________________________
Sejam a, b e c números reais distintos dois a dois. Prove que
b+c a+c a+b
------------------------ 1---------------------------1----------------------- = u.
(a-b)(a-c) (b-a)(b-c) (c-a)(c-b)

Questão 5.79___________________________________________
Sejam a, b e c números reais distintos dois a dois. Prove que
a2+b + c a + b2+c a + b + c2
= 1.
(a-b)(a-c) (b-a)(b-c) (c-a)(c-b)
196 5 Produtos Notáveis

Questão 5.80__________________________________________________
Determine o valor das expressões abaixo:
________ 1________ __________ 1________ ________ 1________ +
a)
(a-b)(a-c)(a-d) (b - a) (b - c) (b - d) (c-a)(c-b)(c - d)
________ 1________
(d-a)(d-b)(d-c)
a b ________ c________
b) (a-b)(a-c)(a-d) + (b -a)(b - c)(b -d) (c - a)(c-b)(c-d)

________ d________
(d-a)(d-b)(d-c)
_______ a^_______ _______ b^_______ +________ o2_______ +
c) (a-b)(a-c)(a-d) + (b-a)(b-c)(b-d) (c-a)(c-b)(c - d)

_______ d2_______
(d-a)(d-b)(d-c)
a3 + b3 +_______ c3_______ +
d) (a-b)(a-c)(a-d) + (b-a)(b-c)(b-d) (c -a)(c-b)(c - d)

d3
(d-a)(d-b)(d-c)

Questão 5.81
1 1 1
Sejam a, b e c números reais, tais que - + , com os
a b c a+b+c
1 1 1 1
denominadores diferentes de zero. Prove que -5 + -
(a + b + c)5

Questão 5.82 (Finlândia 2002)


, 111 1 1 _1_ _1_ 1
Mostre que se - + - + - =------------ , , então — + ,desde
abc a+b+c an bn cn an + bn + cn
que n seja um inteiro positivo impar.

Questão 5.83
2 4b 4
Simplifique -------- +
2a - b b2 — 4a2 2a + b
Os Segredos da Álgebra para IME/ITA/OLIMPÍADAS 197

Questão 5.84
a+b 2a a-b
Simplifique a2-b2 +(a + b)2 ’
(a-b)2

Questão 5.85 (CN-1961)

------------<—+ -_y, 2
2x y • 1 + x
Efetue e simplifique
x + y y - x y.2 -x2J |_x + y x2-y2:

Questão 5.86 (CN-1978)__________________________


a4 -b4 2ab
Simplificando , para b # ±a ,
(a2+b2 + 2ab)(a2+b2-2ab) a2 -b2

obtemos:
a) 11 b)^È d)^
c) -
a -b a a -rb ■’5

Questão 5.87__________________________
a2 b2 (a + b)2 (ay-bx)2
Mostre que — +------ - -------— = ——------—
x y x+y xy(x
; + y) ’

Questão 5.88
(n + 1)4+4n4 (n-1)4 + 4n4
Mostre que o valor de = 0.
n2+(2n + 1)2 n2+(2n-1)2

Questão 5.89 (Grã-Bretânia-2014)


20144 + 4-20134 20124 + 4-20134
Determine o valor de
20132+4027z 20132+40252

Questão 5.90 (AIME-1987)


Seja m = (l04 + 324)(224 + 324)(344 +324)(464 +324)(584 +324) e

n = (44 + 324) (l 64 + 324) (284 + 324) )404 + 324) (524 + 324^. Determine —.
' n
198 5 Produtos Notáveis

Questão 5.91 (Moscou-Modificada)

Determine o valor de
4A 4J

HWH
Os Segredos da Álgebra para IME/ITA/OLIMPÍADAS 199

Agora chegou o momento das identidades condicionais, identidades


importantíssimas e muito frequentes em olimpíadas. Veremos as identidades
mais importantes nas competições, elas aparecem como exercícios para prová-
las, então preste bastante atenção nas demonstrações!

5.39) Identidades Condicionais:


Sea + b + c = 0, então temos:
a) Soma de Quadrados:
|a2 + b2 + c2 = -2(ab + ac + bc)|

Demonstração:
(a + b + c)2 = a2 + b2 + c2 + 2(ab + ac + bc)

=> (O)2 = a2 + b2 + c2 + 2(ab ac + bc)


.-. [a2 + b2 + c2 = -2(ab + ac + bc) .

b) Quadrado da Soma Dois a Dois:


(ab + ac + bc)2 = (ab)'>2 (bc)2 (ac)2

Demonstração:
(ab + bc + ac)2 = (ab)2 (bc)2 (ac)2 + 2abc - (a + b + c)

(ab + bc + ac)2 = (ab)2 (bc)2 +(ac)2 +2abc(0)

(ab + ac + bc)2 = (ab)2 (bc)2 + (ac)2 ■

c) Soma de Cubos Simples:


|a3 + b3 + c3 = 3abc]

Demonstração:
(a + b + c)3 = a3 + b3 + c3 + 3(a + b + c)(ab + ac + bc) - 3abc
=> (O)3 = a3 + b3 + c3 + 3(0)(ab + ac + bc) - 3abc
|a3 -r b3 + c3 = 3abc|.
200 5 Produtos Notáveis

d) Soma de Três Cubos da Diferença:


(a - b)3 + (b - c)3 + (c - a)3 3(a-b)(b-c)(c-a)

Demonstração:
a-b+b-c+c-a=0; x3 + y3 + z3 = 3xyz
=> x = a-b;y = b-c; z = c-a
.. (a - b)3 + (b -c)3 + (c-a)3 =3(a-b)(b-c)(c-a) .

e) Cubo da Soma Dois a Dois:


(ab + bc - ac)3 = (ab)3 + (bc)3 (ac)3 + 3(ab + bc)(ab + ac)(bc + ac)

Demonstração:
(a + b + c)3= a3 + b3 + c3 + 3(a + b)(a + c)(b + c)

(ab + bc + ac)3 = (ab)3 + (bc)3 + (ac)3 + 3(ab +bc)(ab + ac)(bc + ac)

f) Cubo da Soma Dois a Dois em Função do Produto dos Três:


(ab + bc + ac)3 = (ab)3 + (bc)3 + (ac)3 - 3a2b2c2

Demonstração:
(ab + bc + ac)3 = (ab)3 + (bc)3 + (ac)3 + 3(ab + bc)(ab + ac)(bc + ac)

=> (ab + bc + ac)3 = (ab)3 + (bc)3 + (ac)3 + 3abc(a + c)(b + c)(b + a)

=> (ab + bc + ac)3 = (ab)3 + (bc)3 + (ac)3 + 3abc(-b)(-a)(-c)

(ab + bc + ac)3 = (ab)3 + (bc)3 + (ac)3 - 3a2b2c2 .

g) Soma das Quartas Potências: (Stanford-2013)

a4 + b4 + c4 = 2(ab + ac + bc)',2

Demonstração:
(a + b + c)4 = a4 + b4 + c4 +4 (a + b + c)2 (ab + ac + bc) - 2(ab + ac + bc)2

- 2abc(a + b + c)
Os Segredos da Álgebra para IME/ITA/OLIMPÍADAS 201

(O)4 = a4 +b4 +c4 +4 - (O)2 • (ab + ac + bc) - 2(ab + ac + bc)2 - 2abc -(0)

=> 0 = a4 + b4 + c4 - 2(ab + ac+ bc)2

a4 + b4 + c4 = 2(ab + ac + bc)2 .

h) Quadrado da Soma de Três Quadrados:


(a2 + b2 + c2)2= 2(a4+ b4 + c4)

Demonstração:
(a2 + b2 + c2 )2 = a4 + b4 + c4 + 2[(ab)2 + (bc)2 + (ac)2 ]

[a2+b2+c2j = a4+b4+c4+2^(ab + bc + ac)2-2abc(a + b + c)J

(a2 +b2 +c2) = a4 + b4 + c4 + 2(ab + bc + ac)2 - 4abc(a + b + c)

a4 +b4 +c4
(a2 + b2 + c2) = a4 + b4 + c4 + a4 + b4 + c4 - 4abc(0)

(a2 + b2 + c2j = 2 (a4 + b4 + c4' .

i) Soma das Quintas Potências:


a5 + b5 + c5 = - 5abc(ab + ac + bc)

Demonstração:
(a2 +b2 + c2)(a3 + b3 +c3) = -2(ab + bc + ac)-3abc

a5 + a2b3 + a2c3 + a3b2 + b5 + b2c3 + a3c2 + b3c2 + c5 =


= -6a2b2c - 6a2bc2 - 6ab2c2
a5 + b5 + c5 = -5a2b2c - 5a2bc2 - 5ab2c2 -
- (a2b2c + a2bc2 + ab2c2 + a2b3 + a2c3 + a3b2 + b2c3 + a3c2 + b3c2 j

a5 + b5 + c5 = - 5abc (ab + ac + bc) -


- [a2b2(c + b + a) + a2c2(b + c + a) + b2c2 (a + c + b)"l
202 5 Produtos Notáveis

a5 + b5 + c5 = - 5abc (ab + ac + bc) - (a + b + c) • (a2b2 + a2c2 + b2c2)

a5 + b5 + c5 = - 5abc(ab + ac + bc)-(0)(a2b2 +a2c2 +b2c2)

|as + bs + c5 = - 5abc(ab + ac + bc)|.

Concluímos que:
(a2+b2+c2) (a3 + b3+c3)
a5 + b5 + c5
-------- -------- - - -------- - ------- - = -(ab + bc + ac) ■ abc =
5
. (a2+b2 + c2) (a3+b3+c3) a5+b5 + cs
2 ' 3 5 '

j) Soma das Sétimas Potências:


a7 +b7 +c7 = 7abc[(ab)2 + (ac)2 + (bc)2

Demonstração:
(a2 +b2 + c2 j(a5 +b5 +c5) = [-2(ab + bc + ac)][-5abc(ab + ac + bc)J
a7 + a5b2 + a5c2 + a2b5 + b7 + b2c5 + a2c5 + b2c5 + c7 =
= 10abc(ab + ac + bc)2
a7 + b7 + c7 = 10abc (ab + ac + bc)2 -

- (a5b2 + a5c2 + a2b5 + b2c5 + a2c5 + b2c5 )

a7 + b7 + c7 = 10abc[(ab)2 + (ac)2 + (bc)2J-

" [(ab)2 (a3 + b3) + (ac)2 (a3 + c3) + (bc)2 (b3 + c3)]

a7+b7+c7 = (ab)2[l0abc- (a3 + b3 + (ac)2 [l Oabc - (a3 + c3 +

+ (bc)2 [lOabc -
(b3-3)]
a7 + b7 + c7 = (ab)2 [7abc] + (ac)2 [7abc] + (bc)2 [7abc]

a7+b7+c7 = 7abc (ab)2 + (ac)2 + (bc)2 ou

a7 + b7 + c7 = 7abc(ab + ac + bc)2 .
Os Segredos da Álgebra para IME/ITA/OLIMPÍADAS 203

Concluímos que:
(a2+b2 + c2) (a5+b5+c5)
a7+b7+c7
= abc ■ (ab + bc + ac)2 =
2 5 7
. (a2+b2 + c2) (a5 + b5+c5)_a7 + b7+c7
2 ' 5 7

k) Produto de Potências Consecutivas:


2 2
a3 + b3+c3 a4+b4+c4 a5+b5 + c5
3 2 5

Demonstração:
ía3+b3+c3^2
(a4 + b4 + c4) = (abc)2 • 2(ab-s-bc+ ac)2
3

a3+b3+c3 |2 ía4+b4+c4]
- ------- - --------- = [(abc)(ab + bc + ac)]2
3
,2 , 2
a3+b3 + c3 a4+b4+c4 a5+b5+c5
3 2 5
2 2
a3tb3rc3 a4+b4+c4 a5+bs+cs
3 2 5

Concluímos que:

ía3 + b3+c3f a4+b4 + c4 = [(abc)(ab + bc + ac)]2


3 2
2 2
ja3+b3+c3 a4+b4+c4 as+b5+c5
2 5
2 2
a3+b3+c3 a4+b4+c4 a5+b5+c5
3 2 5
204 5 Produtos Notáveis

I) Cubo de Quatro Termos:


[a3 +b3 +c3 + d3 = 3ab(c + d) + 3cd(a + b)|

Demonstração:
(a + b + c + d)3 = a3+b3+c3+d3- 3ab (c + d) - 3cd (a + b) +

+ 3(a + b + c + d)- (ab + ac + ad + bc + bd + cd)


(O)3 = a3 +b3 + c3 +d3 +3(0)-(ab + ac + ad + bc + bd + cd)-

-3ab(c + d) -3cd(a + b)
=> a3 +b3 +c3 +d3 -3ab(c + d)-3cd(a + b) = 0

a3 + b3 + c3 + d3 = 3ab(c + d) + 3cd(a + b) .

Outra maneira:
a + b + c + d = 0 => a + b = -(c + d) => (a + b)3 = [-(c + d)]3

=> a3 + 3a2b + 3ab2 + b3 = -Fc3 + 3c2d + 2cd2 + d3 ]

=> a3+3ab(a+ b) +b3 =-c3 - 3cd(c + d)-d3

=> a3+b3+c3+d3= -3ab(a + b)-3cd(c + d)

=> a3 + b3 +c3 +d3 = -3ab[-(c + d)]-3cd[-(a + b)J

a3 t- b3 + c3 + d3 = 3ab(c + d) + 3cd(a + b) .

Vejamos algumas aplicações interessantes!

Exemplo Resolvido 223: Calcule Vlô-eV? + >/l6 + 6>/7 .

Resolução: Podemos escrever:


x = Vl6-6%/7 + 7l6 + 6>/7 => x - Vl6-6>/7 - 6 + 6>/7 = 0

=• x + (-x/l6-6%/7j + (->/l6 + 6>/7j = 0, x > 0.


_____ b_____ _____ c_____
a
X + (-V16-6V7 J + (->/l6 + 6V7 j = 0 => a + b + c = 0
Os Segredos da Álgebra para IME/ITA/OLIMPÍADAS 205

=> a2 +b2 + c2 = -2(ab + ac + bc) => a2 + b2 + c2 = -2[a(b + c) + bc]


=> x2 +16-677 + 16 + 677 =
__________ x__________
= —2 (716-677 + 716 + 677] + (-7l6 -677)(-Jl6 + 677)

=> x2+32 = -2 -x2 + 162 -(ô77)2

=> X2 +32 =-2[-x2 +(7256-252)] => x2 + 32 =-2[-x2 + 2]

=> x2+32 = 2x2-4 => 4 + 32 = 2x2-x2 => x2 = 36 x = 6.

Exemplo Resolvido 222: Calcule 744-1876 + 744 + 1876 .

Resolução: Podemos escrever:

x = ^44-1876+^44 + 1876 => x - 744 -1876 - 744 + 1876 = 0

=> x + (-744-1876 ) +(-744 + 18;7ê) = 0, x e R.


b c_____
a
X + (-744-1876) + (-744 +1876 ) = 0 => a + b + c = 0

=> a3 + b3 + c3 = 3abc
=> x3 - 44 +1876 - 44 -1876 = 3 ■ X ■ (-744 -1876 )• (-744 + 1876)

=> X3 - 88 = 3 • X-^442 -(1876)2}

=> X3 - 88 = 3 ■ X • (7l 936 -1944) => x3 -88 = 3■ x-(7-8)

=> x3 - 88 = 3 ■ x ■ (-2) => X3 - 88 = -6x =0 x3 + 6x - 88 = 0 x = 4.

Observação: Encontramos x = 4 pelo teorema do fator. Teorema que será


estudado em detalhes no capitulo de Fatoração.
206 5 Produtos Notáveis

Exemplo Resolvido 223: Se a + b + c = 3, determine o valor de


(a-1)3+(b-1)3+(C-1)3
15(a-1)(b-1)(c-1) '

Resolução: Podemos escrever:


a + b + c = 3 => (a-1)+(b-1) +(c-1) = 0
=> (a-1)3+(b-1)3+(c-1)3 = 3(a-1)(b-1)(c-1)

(a -1)3 + (b -1)3 + (c-1)3


=3
(a-1)(b-1)(c-1)
_1_ (a-1)3+(b-1)3+(c-1)3 = _1_ 3
15 (a-l)(b-1)(c-1) 15
(a-1)3+(b~1)3+(c-1)3 1
15(a-1)(b-1)(c-1) 5

Problemas Propostos

Questão 5.92 (IME-06/07)


a+b b+c a+c
Sejam a. b e c números reais não nulos. Sabendo que —=
a
a+b
determine o valor numérico de ------- .
c
Questão 5.93
1 1 + —!- = 6 .
Seja a. b e c, números reais tais que a + b + c = 5 e-------- h
b +c a +c a +b
, abc
Determine o valor de ----- +------- + —-.
b+c a+c a+b

Questão 5.94
2abc
Se a + b + c = 0, determine o valor de
(a2 + ac + bc + ab)(b + c)
Os Segredos da Álgebra para IME/ITA/OLIMPÍADAS 207

Questão 5.95
(a + b - 2c)2 (a + c-2b)2 (b-rc-2a)2
Se a + b + c = 0 , determine o valor de
a2+b2+c2

Questão 5.96 (Moscou 1949)


Prove que x2 + y2 + z2 = 2xyz para inteiros x, y, z, somente se x = y = z = O.

Questão 5.97 (lrã-1985)


Sejam x, y e z três números reais positivos, tais que x2 + y2 + z2 = xy + yz + zx.

Determine o valor de
Vx + ,/y + Vz

Questão 5.98
Sejam a, b e c inteiros positivos, tais que a = b + c. Prove que a4 + b4 + c4 é o
dobro do quadrado de um inteiro positivo.

Questão 5.99
(a - 2)3 + (b-2)3+(c - 2)3
Se a + b + c = 6 , determine o valor de
7(a-2)(b-2)(c-2)

Questão 5.100 (BMO-2007)


14 + 20074 + 20084
Determine o valor de
12 +20072 +20082 '

Questão 5.101 (Singapura-2014)


20143 -20133 -1
Determine o valor de
2013-2014

Questão 5.102

= 3 . Determine o valor de r3 _1_


Seja r um número real, tal que
3VF r3 '
208 5 Produtos Notáveis

Questão 5.103 (AMC-2011)


Qual dos valores abaixo é igual a yJ9- 672 + y/9 + 672 ?

. 7yÍ2
a) 372 b) 276 c) — d) 3j3 e) 6

Questão 5.104 (Princeton-2006)


Simplifique yj7 + 4y/3 + 7?-473 .

Questão 5.105 (AHSME-1970)


O número 1/3+ 272 - 73-272 é igual a:
a) 2 b) 273 c) 472 d) 7ê e) 2V2

Questão 5.106 (CN-1984)

1/3 + 2 72>/2 -1/3-2^272 é igual a:

a)1 b) 2 c) 3 d) 4 e) 5

Questão 5.107
Qual o valor de 7^72+7 -7õ72 -7 ?

Questão 5.108 (IMO-Longlist-1973)


O número 775 + 2 + 775 - 2 é racional ou irracional?

Questão 5.109 (AHSME-1980)


A soma 7s + 2i/Í3 + 75 - 27Í3 , é igual a:

1 3 765 1+^3
a)2 c)
4 '2
d) 72 e)1

Questão 5.110
Qual o valor de ^20 + 14^2 + ^20-14^2 ?
Os Segredos da Álgebra para IME/lTA/OLIMPlADAS 209

Questão 5.111 (Suécia-2001)


i 1
Mostre que (752+5)3 +(752-5)3 é irracional.

Questão 5.112 (Malásia-2010)

Mostre que existem inteiros m e n, tais que — = 375Õ + 7 - 7750 - 7 .


n

Questão 5.113 (Turquia-2007-Modificada)


Determinando o valor de x = ^11 + TsõT + x/l 1 — 7337, então x3 + 18x vale?

Questão 5.114 (Harvard/MIT-2008)______________________________________


Sejam a, b, c são números reais não nulos, tais que a + b + c = 0,
a3 + b3 + c3 = a5 + bs + c5 . Determine a2 + b2 + c2.

Questão 5.115
Simplifique (a-2b + c)4 (b - 2c + a)4 + (c - 2a + b)4.

Questão 5.116
(a + b + c)2 + (a - b - c)2
Se a2 + b2 + c2 = 5 , ache o valor de
2(5 + 2bc)
Questão 5.117
as + b5+c5
Se a + b + c 0, determine o valor de
abc(ac + bc + ab)

Questão 5.118
Se a + b + c = 0 , determine o valor de a6 + b6 + c6.

Questão 5.119
1 1 1
Sejam a, b e c números reais, tais que a + b + c = - + - + - = 0. Prove que
abc
a6+b6+c6
= abc.
a3+b3+c3
210 5 Produtos Notáveis

Questão 5.120 (Croàcia-2001)


a7 + b7 + c7
Se a + b + c = 0, determine o valor de
abc (a4 + b4 + c4 j

Questão 5.121
(ab)3 (bc)3 + (ac)3 - (ab + bc + ac)3
Se a + b + c = 0, determine
6abc-(a3 +b3 +c3 j

Questão 5.122 (IME-00/99)_____________________________________________


Considere quatro números inteiros a, b, c e d. Prove que o produto:
(a-b)(c-a)(d-a)(d-c)(d-b)(c-b) é divisível por 12.

Questão 5.123 (Peru 2009)


Mostre que, se a + b + c + d = 0, então
(ac-bd)(bc-ad)(cd-ab) = (a + b)(a + c)(a + d)(b + c)(b + d)(c-t-d).

Questão 5.124
a b c b-c c-a a -b
Se a + b + c = 0, mostre que = 9.
b-c c-a a -b a b c

Questão 5.125 (Rússia)


Sejam a. b e c números reais distintos dois a dois. Mostre que
a2 (c-b)-b2 (a-c) + c2 (b-a) é diferente de zero.
Os Segredos da Álgebra para IME/ITA/OLIMPÍADAS 211

Veremos agora tópicos avançados em produtos notáveis.


Observação: Essas identidades não serão demonstradas.

5.40) Tópicos Avançados.


Os tópicos avançados são identidades diferentes que serão úteis em algumas
questões, são identidades rebuscadas que muitos não conhecem. São elas-
TA1. Uma Identidade Interessante:
As identidades a seguir são binômios que podem aparecer facilmente em
questões de olimpíadas, bem como em escolas militares.
(a2 -k b2)(a2 - k- b2) = (a.| a2 + k b2)2 -k(a1 b2 +a2 b,)2 .
a)

b) (a2 + k b2 )(a2 + k - b2 j = (a, a2 + k b, b2 )2 + k(a, b2 -a2 b, )2 .

c) (a2 + k-b2j(a2 + k b2 ) = (a, a2 -k b, b2)2 + k(a-, b2 +a2 b^2 .

TA2, Identidade de Euler:


A identidade de Euler para quatro termos é uma identidade interessante, revela
a astúcia de um grande gênio. Essa identidade é dada por:
(a,2 + a| + a3 +a^)(b? + b2 + b| + b^) = (a1b5 - a2b2 a3b3 -a4b4)2 +

+ (aib2 + a2b-| + a3b4 - a4b3 )2 + (a-|b3 - a2b4 + a3b-| + a4b2 )2 +


2
+ (a1b4 + a2b3 - a3b2 + a4b-|)

TA3. Identidade de Binet-Cauchy (Generalização da Identidade de


Laqranqe):
Aqui temos a generalização da identidade de Lagrange.
' n V n 'n n ")
Eaící EM Xai di XbiCi = E (ai bj - aj bi)(ci dj - Cj d^
ki=1 Aí=1 ví=i A 1=1 1<i<j<n

TA4. Identidade de Lebesque:


A identidade de Lebesgue é pouco conhecida, relaciona o quadrado da soma
de quatro termos com o produto dois a dois. É dada por:
(a2 + b2 + c2 + d2 )2 = (a2 + b2 - c2 - d2 )2 + (2ac + 2bd)2 + (2ad - 2bc)2
212 5 Produtos Notáveis

TA5. Identidade de Liouville:


Essa é uma interessante identidade, que relaciona a soma das quartas
potências, permutadas em soma e diferença.
6 (a2 + b2 + c2 + d2 j = (a + b)4 +(a -b)4 +(a + c),44 +(a-c)4 + (a + d)4 +

(a-d)4+(b +c)4+(b-c)4 + (b + d)4 + (b-d)4 + (c + d)4 + (c - d)4

TA6. Identidade Trinomial:


Essa é a "identidade da equação do 2o grau”, relaciona o produto de equações
do 2° grau com seus coeficientes.
(x2 + axy + by2 )(m2 + amn + bn2) = (xm - byn)2 +

+ a(xm- byn)(ym+ xn + ayn) + b(ym + xn +- ayn)2

TA7. Identidade Cúbica:


Essa é uma interessante identidade com cubos.

(6a2 -4ab + 4b2j = (3a2 + 5ab-5b2 j +(4a3-4ab + 6lib2)\

+ (5a2 - 5ab-3lib2)3

TA8. Identidade de Ferrari:


Essa é a identidade das quartas potências, uma belissima identidade.

(a2 + 2ac - 2bc - b2j + (b2 - 2ab-2ac - c2) + (c2 + 2ab + 2bc-a2) =

= 2 (a2 + b2 + c2 -ab + ac + bc(

TA9. Identidade da Potência de 7:


Uma identidade bastante interessante que relaciona as potências de sete com
quadrado e produto da soma de binômios.
(a + b + c)7 - (a7 + b7 +c7) =

+ c)^(a2 + b2 + c2 + ab + ac + bc) +abc(a + b + c)^|


= 7(a + b)(a + c)(b

Observação: A identidade da Potência de 7 será demonstrada no capitulo de


polinõmios simétricos!
Os Segredos da Álgebra para IME/ITA/OLIMPÍADAS 213

Capítulo 06 - Fatoração

Introdução
Fatorar é colocar em produto de fatores primos. Por exemplo, fatorar 15 é
colocá-lo como um produto de fatores primos, no caso o 3 e o 5. Vamos fazer
isso com polinõmios agora, divirta-se!
Veremos os critérios de fatoração. vamos lá!

6.1) Critérios de Fatoração


Veremos os critérios mais interessantes para se fatorar polinõmios,
aprenderemos essas técnicas maravilhosas que ajudam o leitor a encontrar
raizes de polinõmios rapidamente. Divirtam-se!

O primeiro critério que estudaremos será a fatoração por agrupamento ou


“colocando em evidência". É um método que consiste em agrupar os termos
semelhantes buscando colocar a expressão em forma de produto.

6.2) Agrupamento ou “Evidência”:


Esse critério consiste em agrupar termos semelhantes até termos um produto.

Exemplo Resolvido 224: Fatore ax + by + bx + ay.


Resolução: Agrupando os termos semelhantes:
ax + by + bx + ay = x(a + b) + y(a + b)lax + by + bx +ay = (a + b)(x + y)l ■

Exemplo Resolvido 225: Fatore 2x + 4x2 - 2x4 - x3 .


Resolução: Podemos fazer quantos agrupamentos forem necessários:
E = 2x + 4x2 - 2x4 - x3 => E = 2x(1 + 2x) - x3 (1 + 2x) => E = (1 + 2x)(2x - x3)

2x + 4x2-2x4-x3 = x (1 + 2x)^2- x2 j .

Nosso segundo critério tem ligação com os produtos notáveis, são os


chamados quocientes notáveis, são muitas as aplicações desse critério
principalmente em simplificações.
214 6 Fatora ção

6.3) Quocientes Notáveis:


Esse critério consiste em usar os produtos notáveis para fazer a fatoraçâo.
Usaremos os mais comuns:
a2 -b2 a2 -b2
a) ou ----------- = a - b .
---------- ==aa++bb ou
a-b a+b
a3~b3 = a2+ab + b2.
b)
a-b
5-tbÍ = a2-ab + b2.
c)
a+b
a4-b4 a4-b4
d) = (a + b)(a2 + b2j ou ---------- = (a -b)(a2 + b2 j
a-b a+b
a5-b5
e) = a4 + a3b + a2b2 + ab3 + b4 .
a-b

-—-5- = a4 - a3b + a2b2 - ab3 + b4.


f)
a+b
a6 - b6
g) = (a + b) (a2 + ab + b2)(a2 -ab + b2).
a-b

h)
4^ = a4-a2b2+b4.
a2+b2

i) - ----- — = a6 + a5b + a4b2 + a3b3 + a2b4 + ab5 + b6 .


a-b
a7+b7
j) = a6 - a5b + a4b2 - a3b3 + a2b4 - ab5 + b6 .
a+b
a8-b8
k) = (a + b)(a2 + b2)(a4 + b4j ■
a-b
a9-b9
I) = (a2 + ab + b2)(a6 + a3b3 + b6 j.
a-b

m) - = (a2 -ab + b2 )(a6 -a3b3 +b6 j.

n) 5-^—= (a4 - a3b + a2b2 - ab3 + b4)(a4 + a3b + a2b2 +ab3 + b4).

a10+b10
o) = a8 - a6b2 + a4b4 - a2b6 + b8 .
a2+b2
Os Segredos da Álgebra para IME/ITA/OLIMPÍADAS 215

Generalização:
Podemos generalizar a soma e a diferença das enésimas potências.

?2sÈ2 = an-1+an-2b + ... + a- bn-2 + bn-1


a -b

?n+bn = an~1-an~2-b + ... -abn-2+bn-1.


a+b

Para n par, temos:


amn_bmn
= am(n-1) + a m (n-2)
k
lm(n-2) +bm|n‘1>.
’ ■bm + ... + amb'
am-bm

x4 -81
Exemplo Resolvido 226: Fatore
x-3

Resolução: Note que podemos usar o quociente notável:


a ~b = (a + b)(a2 +b2). Então temos:

x4 -81 x4 -34 x4 -81


= (x + 3)(x2+32)
x-3 x-3 x-3
x4 -81
= (x + 3)(x2 +9) .
x-3

32x5 -1
Exemplo Resolvido 227: Fatore
2x-1

Resolução: Note que podemos usar o quociente notável:


a ~b = a4 -i a3b + a2b2 + ab3 + b4. Então temos:
a -b

E=aêr =*=> E = => e=(2x)4+(2x)3-i+(2x)2-i2+(2x)-i3+i4


=> E = 16x4 + 8x3-1 + 4x2-1 + (2x)• 1 + 1 |e = 16x4 + 8x3 + 4x2 + 2x + l|.

64xs +729y6
Exemplo Resolvido 228: Fatore
4x2 +9y2
216 6 Fatoração

Resolução: Note que podemos usar o quociente notável:


a6+b6
= a4 - a2b2 + b4 . Então temos:
a2+b2
64x6 +729y6 (2x).60+(3y)'>6
E= => E = E = (2x)4-(2x)2-(3y)2 + (3y)4
4x2 +9y2 (2x)'l2 + (3y)2

64x6 +729y6
=> E = 16x4-4x2-9y2+81y4 .-. = 16x4 -36x2y2+81y4 .
4x2 + 9y2

O terceiro critério é "completando o produto notável", esse método consiste em


chegar aos produtos notáveis conhecidos, é um método bem avançado e pouco
usado devido à criatividade que o leitor deve ter para “enxergá-lo”.

6.4) Completando o Produto Notável:


Esse critério consiste em adicionar, subtrair, multiplicar ou dividir por termos
que completem um produto notável conhecido, esse critério é pouco usado,
devido à criatividade que o leitor deve ter para "enxergar" o produto notável.
Então, vamos conhecê-lo.

Exemplo Resolvido 229: Fatore x2 - 4 .

Resolução: Note que podemos usar o produto notável:


a2 -b2 = (a + b)(a -b). Então temos:

x2-4 = x2-22 X2 - 4 = (x + 2)(x -2) .

Exemplo Resolvido 230: Fatore 8x3 -27 .

Resolução: Note que podemos usar o produto notável:


a3 -b3 = (a -b)(a2 +ab + b2 j. Então temos:

E = 8x3-27 => E = 23x3-32 => E = (2x)3-32


=> E = (2x - 3)£(2x)2 + (2x) ■ 3 + 32 J =. E = (2x-3)(4x2 +6x + 9)

.-. 8x3 - 27 = (2x - 3)(4x2 + 6x + 9) .


Os Segredos da Álgebra para IME/ITA/OLIMPÍADAS 217

Exemplo Resolvido 231: Fatore x3-3x2+3x-1.


Resolução: Note que podemos usar o produto notável:
(a - b)3 = a3 - 3a2b + 3ab2 -b3. Então temos:

E = x3-3x2+3x-1 => E = x3-3x2-1 + 3x-12-13 E = (x-1)3 .

Exemplo Resolvido 232: Fatore x3 +6x2 -16 .


Resolução: Note que podemos usar o produto notável:
(a -b)3 = a3 - 3a2b + 3ab2 -b3. Então temos:

E = x3 + 6x2 -16 => E = x3 + 3x2 • 2 + 8 - 24 => E = x3 + 3x2 • 2 + 23 - 24


=> E = x3 +3x2-2 + 23 -24 + |12x|-[Í2x1 =>

E = x3+3x2 2 + |3x- 4 | + 23 - [Í2x] - 24 => E = (x + 2)3-12(x + 2)

=> E = (x + 2)^(x + 2)2-12 => E = (x + 2)[x2+4X + 4-12]

x3 + 6x2-16 = (x + 2)(x2+4x-8j .

Exemplo Resolvido 233: Fatore x4 +16 .


Resolução: Vamos resolver completando um trinômio quadrado perfeito,
lembrando que poderiamos usar o critério 2.
Note que podemos usar os produtos notáveis: (a + b)2 = a2+2ab + b2 e

a2 -b2 = (a + b)(a-b). Então temos:

x4 +16 = (x2)2 + 42 => x4 +16 = (x2)2 +42 +[8x2]-|8x2|

2x4
=> x4 +16 = (x2)2 + 8x2 t42 -[ãx2] => x4+16 = (x2+4) - |8x2 | =>

x4 +16 = (x2 + 4j2 - (2xV2)2 => x4 +16 = (x2 + 4 + 2Xv/2)(x2 + 4-2x72)

Observação: Poderiamos usar a identidade de Sophie-Germain para


a = x e b = V2 .
218 6 Fatoração

Problemas Propostos

Questão 6.1 (Harvard-MIT-2012)_______________________________


Determine a soma de todos os fatores primos distintos de 253 -272 .

Questão 6.2 (CN-1954)_____________


Decomponha 16x4 -1 em três fatores.

Questão 6.3 (AHSME-1954)


Os fatores de x4 + 64 são:
2
a)(x2+8j b) (x2 +8)(x2 -8}

c) íx2 + 2x + 4)(x2 -8X + 16) d) (x2-4x


- 4x + 8)(x2-4x-8j

e) (x2 + 4x + 8)(x2 -4x + 8)

Questão 6.4
Fatore 5ax + 3by-5ay-3bx.

Questão 6.5 (CN-1951)


Fatore x2 - 2xy + y2 - a2 .

Questão 6.6 (CN-1952)


Fatore 8x2 - 8xy - 3x + 3y .

Questão 6. 7
Fatore 16x4y6 -81a6b4 .

Questão 6.8
Fatore (2a - 3)4 - (a - 5)4.

Questão 6.9
a2 b2 c2
Se a + b + c = 0, qual o valor de — + — + — ?
bc ac ab
Os Segredos da Álgebra para IME/ITA/OLIMPÍADAS 219

Questão 6.10
27c2
Se a + b + c = 0, determine o valor de - - + 2,0
5bc 5ac 5ab

Questão 6.11 (AHSME-1952)


A expressão a3 - a’3 , é igual a:

a) Ía--Ya2 +1 + -!•']
V aA a2J
c) fa_lYa2_2 + jJ d)
k aA a2)

Questão 6.12 (AHSME-1955)

A fração
T7 -b-4 é igual a:
-b’2
a--
a) a 6 -b~s b) a + b' c) a -b' d) a2 + b2 e) a2 - b2

Questão 6.13 (AHSME-1950)


a2-b2 ab-b2
Simplifique
ab ab — a2

Questão 6.14
. b-c c - a a -b
Fatore ------ +
bc ca ab

Questão 6.15
- y2) + by(a2 + x2)
Fatore ax(b2 +

Questão 6.16

Fatore (a2 - 3a -r ab - 3b) - (a + b)2

Questão 6.17
Fatore a2c + 2abc + b2c + a2d + 2abd + b2d .
220 6 Fatoração

Questão 6.18__________________________________________
Fatore a2b + a2c + ab2 + b2c + ac2 + bc2 - a3 - b3 - c3 - 2abc.

Questão 6.19___________________
Fatore a2 - 8ab - 2ac +16b2 + 8bc -15c2.

Questão 6.20___________________
Fatore a4 + 5bc2 - a2b - a2c2 - 2b2 - 2c4 .

Questão 6.21_____________________
Fatore 2a4 +17a2b2 + 21 b4 + 8a2 + b2 -10 .

Questão 6.22_______________
a2 -bc-b2 -t-ac
Simplifique
a2 +b2 -c2 + 2ab

Questão 6.23 (AHSME-1960)


a2 + b2 -c2 + 2ab
Simplifique
a2 + cz - bz + 2ac

Questão 6.24
(a2-b2)3+(b2-c2)3 + (c2-a2)3
Simplifique
(a-b)3 +(b-c)3 +(c-a)3

Questão 6.25
(a5+a3b2)(a3-b3)(a2-b2)
Simplifique
(a4-b4)(a3 + a2b-ab2-b3)’

Questão 6.26______________
(a+ b)4 - (a - b)4
Fatore
(a2+b2)2-(a2-b2)2

Questão 6.27

a6-b6 a6 + b6 (a4~b4)
Fatore
a2-b2 a2+b2 (a2+b2)(a-b)
Os Segredos da Álgebra para IME/ITA/OLIMPÍADAS 221

Questão 6.28
a3-b3 a2 -b2 )
Simplifique
a2 -2ab + b2 a2 + ab J

Questão 6.29
(a-b)4 -ab(a-b)2 -2a2b2
Simplifique
(a-b)(a3-b3) + 2a2b2

Questão 6.30
Sejam a, b e c números reais não nulos, tais que a + b + c = 0, Prove que
a2+b2 , b2+c2 , c2+a2 a3^3^3
a+b b+c c+a bc ca ab

Questão 6.31
Se a * b # c, simplifique

M= _ a
i/V(a“b)(b-c)+V(a"b)(c“a)+V(b-c)(c-a)

a)1
«3 c) 3 d) Vã

Questão 6.32

Fatore 4b2c2 - (a2-b2 - c2) .

Questão 6.33
Fatore 2(a2b2 a2c2 b2c2)-(a4 + b4 +c4 j.

Questão 6.34 (A1ME-1986)


Calcule (Vê + Vê + V7)(>?5 + Vê - V7)( Võ - Vê + -fi)(-Vê + Vê + V7)

Questão 6.35 (Putnam-1938-Modificada)


Fatore (y2 - 3y + 2^ - 3(y,2: -3y + 2) + 2-y.
222 6 Fatoração

Questão 6.36 (CN-1998)


(x3 +y,33 +z3)2 -(x3 -y',3
A expressão é equivalente a:

a) 4x3 b) 4yx3 c) 4zx'.3 d) 4yzx3 e) 4xyz

Questão 6.37 (CN -1981-Modificada)

x (x4 -5x2 + 4)-2(x4 — 5x2 +4)


Fatore e simplifique a expressão —
(x3 -6x2 +12x-8)(x2 -l)

a)^ . x +1 ..x-2
b) — c) ------ d) ------- e) 1
x-2 x-1 x-2 x+2

Questão 6.38 (CN-1983)


(zx2 + zy2 + 2xyz)(x2 - y'’2)
Fatorando e simplificando a expressão —, tem-se:
x3 +3x2y + 3xy2 + y3
a) z(x + y) b) z(x-y) c) zx + y
d) zx - y e) y + z

Questão 6.39 (Harvard-MIT-2012)


Sejam a e b números complexos, tais que 2a + 3b = 10 e 4a2+9b2 =20,
determine o valor de ab.

Questão 6.40 (Harvard-MIT-2014)


Sejam a, b, c e x, números reais com (a + b)(b + c)(c+ a) * 0 que satisfaz
a2 b2 b2 c2 c2 a2
------ =-------+ 20, - ------ =-------- + 14 e
------- -------=--------+ x , qual o valor de x?
a+b a+c b+c b+a c+a c+b

Questão 6.41 (Kürschár-1959)

Se a, b e c são três números inteiros distintos e n é um inteiro positivo, mostre


an bn cn
que , é um número inteiro.
(a-b)(a-c) (b-a)(b-c) (c-a)(c-b)
Os Segredos da Álgebra para IME/ITA/OLIMPÍADAS 223

Questão 6.42
Determine o valor das expressões abaixo:
1 1 1
a)
(a-bj(a-c) (b-a)(b-c) (c-a)(c-b)
_____ a_____ _____ b_____ _____ c_____
b)
(a-b)(a-c) (b-a)(b-c) (c-a)(c-b)
a2 ! b2 + c2
C)
(a-b)(a-c) (b-a)(b-c) (c-a)(c-b)

a3 ; b3 c3
d)
(a-b)(a-c) (b-a)(b-c) (c-a)(c-b)

Questão 6.43 (Stanford-2013)


Sejam a = -~J3 + 75 + 77, b = 73 - 75 - 77,, 0 = 73 + 75 - 77. Determine
a4 b4 c4
(a -b)(a -c) + (b-c)(b-a) T (c -a)(c -b) '

Questão 6.44
2 2 2
Fatore -___ _______ +____ _______ +____ _______
(a-b)(a-c) (b-c)(b-a) (c-a)(c-b) a-b b-c c-a

Questão 6.45___________________________________
(x-b)(x-c) (x-c)(x-a) (x-a)(x-b)
Fatore
(a-b)(a-c) (b-c)(b-a) (c-a)(c-b)

Questão 6.46________________________________
- . a+1 b+1 c +1
Fatore------------------ +-------------------- +-------------------
(a-b)(a-c) (b-c)(b-a) (c-a)(c-b)

Questão 6.47___________________________________
a2bc ab2c abc2
Fatore
(a-b)(a-c) (b-c)(b-a) (c-a)(c-b)
224 6 Fatora çã o

Questão 6.48
Determine o valor das expressões abaixo:

(a - b)(a-c)(a-d) h (b - a)(b - c)(b - d) (c - a)(c - b)(c - d)


_________ 1_________
(d-a)(d-b)(d-c)
b. ________ a__________________ b________ +________ c________ +
(a-b)(a-c)(a-d) (b - a)(b - c)(b - d) (c-a)(c-b)(c-d)
________ d________
(d-a)(d-b)(d-c)

a2 +_______ b2_______ +________ c2_______ +


c)
(a-b)(a-c)(a-d) (b-a)(b-c)(b-d) (c-a)(c-b)(c - d)
+ d2
+ (d - a)(d - b)(d-c)

a3 ; b3 c3
d)
(a - b)(a-c)(a-d) (b-a)(b-c)(b-d) (c-a)(c-b)(c - d)
d3
(d-a)(d-b)(d-c)

Questão 6.49
Sabendo que a + b + c + d = 10, determine o valor de
_______ a4_______ +________ b4_______ +________ c4________ +
(a-b)(a-c)(a-d) (b-a)(b-c)(b-d) (c -a)(c - b)(c-d)
+_____ dj______
(d - a)(d- b)(d -c)

Questão 6.50
a5 - 2a4b + a3b2 - a2b3 - 2ab4 + b5
Fatore
a3 +a2b-ab2 -b3
Os Segredos da Álgebra para IME/ITA/OLIMPÍADAS 225

Questão 6.51
Sejam a, b e c números inteiros, tais que ab + bc + ca = 1. Prove que
(a2 +1)(b2 +l)(c2 +1) é um quadrado perfeito.

Questão 6.52
2
Sejam a, b e c números reais, tais que a+b+c=-+ + — = 0 . Prove que
a b c
a6+b6+c6
= abc.
a3+b3+c3

Questão 6.53
Fatore (ab + bc + ca)3 - abc(a + b + c)3 .

Questão 6.54
Se a + b + c = 0, determine o valor de
a2 + b2 c2
(a + b - c)(a - b + c) (c + b - a)(a + b-c) (c + b-a)(a-b + c)

Questão 6.55
Se a + b + c = O, determine o valor de
a4 [ b4 c4
a4 - (b2 - c2 )2 + b4 - (c2 - a2 )2 * c4 - (a2 - b2 )2 '

Questão 6.56
Se a + b + c = 0, Mostre que
2(ab + bc + ca)4 = a4 (b-c)4 + b4 (c-a)4 +c4 (a-b)4 .

Questão 6.57

Sejam a e b dois números reais, tais que —5—+ —— = 1. Mostre que


1+a 1 + b
a b
------ 7-------- 7 = a ~ b •
1 + b2 1 + a2
226 6 Fatora çã o

Questão 6.58
Sejam a. b e c números reais não nulos tais que (a+ b +c)2 = a2+b2+c2.

Prove que:
a2 b2 c2
a) --------4--------- i--------- = 1 •
a2-s-2bc b2 + 2ca c2+2ab
bc .ca + ab
b) = 1.
a2 + 2bc b2 + 2ca c2 + 2ab

No quarto critério, veremos a chamada “cruzadinha simples", uma ferramenta


fortíssima da fatoração que tem inúmeras aplicações nos polinômios e
simplificações, dentre outras. Vamos conhecê-lo.

6.5) Cruzadinha Simples___________________________


Esse quarto critério, denominado “cruzadinha simples", é uma ferramenta
fortíssima que usa soma e produto para encontrar os binômios que serão os
fatores. Não se esqueça das regras de sinais:
“(+).(+) = + : (-)■(-) = + ; (+)■(-) = -“.

A forma geral, para que tenhamos uma "cruzadinha simples”, é:


1 variável simples : f (x) = Ax2 + Bx + C.

1 variável generalizada : f (x) = Ax2m + Bxm + C.

2 variáveis simples : f (x) = Ax2 +Bx ■ y + Cy2.

2 variáveis generalizada : f (x) = Ax2m + Bxm yn +CyZn.

Para fatorar esse polinõmio, vamos realizar 3 passos, a saber:


Passo 01: Decompõem-se as duas extremidades em monómios, cujos
coeficientes são os divisores naturais dos coeficientes extremos.

Passo 02: Multiplicam-se em cruz os monómios decompostos de modo que o


resultado seja o termo central.

Passo 03: Cada linha forma um fator.

Observação: Sempre colocar o polinõmio a ser fatorado em ordem


decrescente de expoente.
Os Segredos da Álgebra para IME/ITA/OLIMPÍADAS 227

Veja o esquema abaixo:


1 Variável Simples:
Ax2 + Bx + C
■x c-,
82 ■ x C2

A operação (a1•x)•(c2)+ (a2 ■ x)• (c-j) deve resultar no termo central.

1 Variável Generalizada:
Ax2m + Bxm + C
a, ■ xm c,
a2xm c2

A operação (a, ■ xm j ■ c2 + (a2 • xm j ■ c-j deve resultar no termo central.

2 Variáveis Simples:
Ax2 + Bxy + Cy2
a, • x c, • y
a2 • x c2■y

A operação (a-, • x) ■ (c2 ■ y) + (a2 ■ x) • (c-| ■ y) deve resultar no termo central.

2 Variáveis Generalizada:
Ax2m + Bxm yn + Cy2n
a, • xm c, • yn
a2•xm c2yn

A operação (a, •xmj ^c2 -yn) + (a2 x'^•(ciV) deve resultar no termo
central.

Exemplo Resolvido 234: Fatore 6x2+7x + 2.


Resolução: Vamos resolver passo a passo:
Passo 01: Note que os divisores naturais de 2 são 1 e 2, então eles serão os
extremos da direita, colocaremos os divisores positivos, caso não dê o termo
central, colocaremos o negativo. Para os extremos da esquerda temos as
combinações (1 e 6); (2 e 3); (3 e 2); (6 e 1).
228 6 Fatoração

Passo 02: Vamos efetuar os produtos e somar os resultados:


6x2 +7x + 2 6x2 +7x + 2

X'^xTr 1 2x-~^>v1 <= linha "2 e 1"


6x2 3x-^><*2 <= linha "3 e 2"

x 2 = 2x e 6x1 = 6x 2x-2 = 4x e 3x -1 = 3x
s = 2x + 6x = 8x s = 4x + 3x = 7x

6x2 +7x + 2 6x2 +7x + 2


3x 6x 1
2x ^'>"'i'2 x '"‘■2
3x 2 = 6x e 2x -1 = 2x 6x2 = 12xex-1 = x
s = 6x + 2x = 8x s = 12x + x = 13x

Note que a segunda combinação dá o termo central, assim, temos:


Passo 03: linha "2 e 1" e "3 e 2", ou seja, 6x2 +7x + 2 = (2x + 1)(3x + 2).

Observação: Note que não foi necessário usar os divisores negativos.

Exemplo Resolvido 235: Fatore x2 + x-2 .

Resolução: Vamos resolver passo a passo:


Passo 01: Note que os divisores naturais de 2 são 1 e 2, então eles serão os
extremos da direita. Para os extremos da esquerda temos somente a
combinação (1 e 1).
Passos 02: Vamos efetuar o produto, notando que ele é negativo, e somar os
resultados:

x2 + x - 2
x -1 <= linha ”1 e -1"
x 2 <= linha "1 e 2"
x-2 = 2x e X-(-1) = -x
s = 2x - x = x

Note que essa combinação dá o termo central, logo temos:


Passo 03: linha ”1 e- 1" e "1 e 2", ou seja, x2 + x-2 = (x-1)(x + 2).

Exemplo Resolvido 236: Fatore x2n -5xn +6.


Os Segredos da Álgebra para IME/ITA/OLIMPÍADAS 229

Resolução: Vamos resolver passo a passo:


Passo 01: Note que os divisores naturais de 6 são 1,2, 3 e 6, então temos as
combinações da direita (1 e 6); (2 e 3); (3 e 2); (6 e 1). Para a esquerda só
temos (1 e 1).
Passo 02: Vamos efetuar o produto, notando que ele é positivo, e somar os
resultados, veja que é conveniente usar os termos negativos devido ao fato de
o termo central ser negativo:

x':2n - 5xn + 6
xn -2 «= linha "1 e-2"
xn -3 <= linha "1 e -3"

Note que a combinação que dá o termo central éo — 2 e o-3.


Passo 03: Os fatores são os da linha "1 e - 2" e “1 e - 3", ou seja,

X':2n - 5xn + 6 = (xn - 2^xn - 3).

Exemplo Resolvido 237: Fatore x6x +12x3x + 11.


Resolução: Vamos resolver passo a passo:
Passo 01: Note que os divisores naturais de 11 são 1 e 11, então temos uma
combinação da direita (1 e 11). Para a esquerda só temos (1 e 1).
Passo 02: Vamos efetuar o produto, notando que ele é positivo, e somar os
resultados, veja que é conveniente usar os termos positivos devido ao fato de
o termo central ser positivo:
x6x +12x3x + 11

X^x I <= linha "1 e 1”


X3x II <= linha "1 e 11"

Passo 03: Assim, as linhas "1 e 1" e "1 e 11" formam os fatores, ou seja,

+ 12x'3x+11 = (:x3x + 1
x6x+12x x3x + 11).

Exemplo Resolvido 238: Fatore x2m -2xmyn -15y2n .

Resolução: Vamos resolver passo a passo:


Passo 01: Note que os divisores de 15 são 1, 3, 5 e 15, então temos as
combinações da direita (1 e 15); (3 e 5); (5 e 3); (15 e 1). Para a esquerda só
temos (1 e 1).
230 6 Fatora çã o

Passos 02: Vamos efetuar o produto, notando que ele é negativo, e somar os
resultados:
x2m -2xmyn -15y2n

xm —5yn <= linha "1 e-5"


xm 3yn <= linha "1 e 3"

Passo 03: Logo, as linhas "1 e - 5" e "1 e 3” formam os fatores, ou seja,
x2m -2xmyn-15y2n =(xm-5yn)(xm + 3yn).

No quinto critério veremos como fatorar polinômios do 3o grau. Usaremos o


teorema do fator ou das raízes racionais e a cruzadinha simples.

6.6) Teorema do Fator ou das Raizes Racionais


Seja P(x) = an xn+an_1x':n-1 +---+a1 ■ x + a0 um polinômio de grau n. Então
temos:
a) Se x - a é fator de P(x), então P(a) = 0 . (Ida).

Demonstração (opcional):
Note que, se x - a é fator, então podemos escrever:
0
P(x) = (x-a)-q(x) => P(a) = (a-a)-q(a) => P(a) = 0.

Observação: Para mais detalhes, consulte um livro sobre divisão de


polinômios.

b) Se P(a) = 0 , então x - a é fator de P(x). (Volta),

Demonstração (opcional):
Pelo algoritmo da divisão (*), temos:
0
P(x) = (x - a) q(x) +r(x) => P(a) = (a - a) • q(a) +r(a) r(a) = 0.
Assim, se o resto é ZERO, então P(x) é divisível por q(x) e portanto x - a é fator
de P(x), ou seja, P(x) = (x - a) q(x).

(*) Observação: Para mais detalhes, consulte um livro sobre divisão de


polinômios.
Os Segredos da Álgebra para IME/ITA/OLIMPÍADAS 231

Teorema das Raizes Racionais


Considere ainda P(x) = an • xn +3,^ -xn'1 + •■• + a1 • x + a0 .
O Teorema das Raizes Racionais garante que, se essa equação admite o

número racional - como raiz (com p e Z, qe7/ e mdc(p,q) = 1, então a0 é


q
divisível por p e an é divisível por q.
qualquer divisor de a0
Se chamarmos - de a, então temos: a = Note que
q qualquer divisor de an
esse "a" é o mesmo do x - a acima.

Observações:
1) O teorema das raízes racionais não garante que o polinômio tenha raizes
racionais, mas caso existam, o teorema permite identificar todas as
raizes da equação;
2) Se an =1 e os outros coeficientes são todos inteiros, o polinômio possui
apenas raizes inteiras.
3) Se q = 1 e o polinômio admite raizes racionais, estas são inteiras e
divisoras de a0 .

6.7) Fatorando Polinômios do 3o grau


Vamos fatorar polinômios do 3o grau, usando o teorema do fator e a cruzadinha
simples de uma variável.

Seguiremos os seguintes passos:


Passo 01: Usar o teorema do fator ou das raizes racionais.
Passo 02: Como x - a é um fator e o polinômio é do 3o grau, então o outro fator
é do 2o grau. Para encontrá-lo podemos usar o dispositivo de Briot-Ruffini para
divisão de polinômios ou usar o artificio abaixo.
Artificio: Utilizamos a identidade de polinômios para obter o outro fator, veja
os exemplos para ficar mais claro.

Observação: Como o dispositivo de Briot-Rufinni é usado para divisão de


polinômios e esse assunto foge aos objetivos desse livro, então iremos usar o
artificio em todas as fatorações de polinômios de 3o grau por esse critério.

Passo 03: Usamos a cruzadinha simples.

Exemplo Resolvido 239: Fatore 3x3 -2x2 -7x-2.


232 6 Fatora çã o

Resolução: Vamos resolver passo a passo:


i1, i 2
Passo 01: Usando o teorema das raizes racionais, temos que a = $

1 2
Portanto, podemos ter como raizes: ±1, ±2, ±- e ± - . Assim, por inspeção,

temos que -1 é raiz, logo podemos escrever:


3x3 -2x2 -7x-2 = [x-(-1)](ax2 +bx + cj

=> 3x3 - 2x2-7x-2 = (x + 1)(ax2+bx+ c)

=> 3x3 - 2x2 - 7x - 2 = ax3 + bx2 + cx + ax2 + bx + c


=> 3x3 -2x2 - 7x-2 = ax3 +(a + b)x2 + (b + c)x + c .

Passo 02: Por identidade de polinômios, encontramos facilmente os valores de


a, b e c:
3x3 -2x2 -7x-2 = ax3 +(a + b)x2 +(b + c)x + c

|a - 3|, a + b = -2 => 3 + b = — 2 => |b = —5|, |c — —2|.

Logo, temos: 3x3 -2x2 -7x-2 = (x + 1)^3x2 -5x-2).

Passo 03: Usando a cruzadinha simples, para fatorar o termo do segundo grau,
temos: 3x3 -2x2 -7x-2 = (x + 1)(x-2)(3x + 1).

Exemplo Resolvido 240: Fatore 2x3 + 3x2 -4x - 6


Resolução: Vamos resolver passo a passo:
Passo 01: Usando o teorema das raízes racionais, temos que
±1, i 2, ± 3, ± 6
a=
±1, ± 2
1 3
Portanto, podemos ter como raízes: ±1, ± 2, ± 3, ± 6, ± - e ± - .

3
Assim, por inspeção, temos que -- é raiz, logo podemos escrever:

2x3 +3x2 -4x-6 ax2 + bx


Os Segredos da Álgebra para IME/ITA/OLIMPÍADAS 233

2x3 +3x2 -4x-6 = ^x + |j(ax2 + bx + c)

3a 2 3b
2x3 + 3x2 - 4x - 6 = ax3 + bx2 + cx + — 3c
x +—x+—
2 2 2~
2x3 + 3x2-4x-6 = ax3 + + b^x2 + + c^ 3c
2

Passo 02: Por identidade de polinômios, encontramos facilmente os valores de


a, b e c:
2x3 +3x2 - 4x-6 = ax3 + 3c
2
|a = 2|, — + b = 3 => 3 + b = 3 => |b = o|, 3b
------ 2 ------ 2 + ~

Logo, temos:
E = 2x3 + 3x2 - 4x - 6 E = ^x + |](2x2-4)
=> E = ^x + |V(x2 -2) . 2x3 + 3x2-4x-6 = (2x + 3)íx2-2j

Passo 03: Usamos a cruzadinha simples, para fatorar o termo do segundo


grau.

Exemplo Resolvido 241: Fatore 8x3m +22x2m -9xm-9.


Resolução: Vamos resolver passo a passo:
Passo 01: Usando o teorema das raizes racionais, temos que
±1, ± 3, ± 9
a =---------------
+1,i2, + 4,±8
Portanto, podemos ter como raizes:
±1, ±3, ±9, ± —, ± —, ± — ± i, ± —, + —+ —,+ — e ±—.
22244488 8
Assim, por inspeção, temos que 3 é raiz, logo podemos escrever:
8x3m+22x2m-9xm-9 = í:xm + 3^ax2m +bx
+ bxm
m +c) =>

8x3m + 22x,2m
2m - 9xm - 9 = ax3m + bx2m + cx,m
m + 3ax2m + 3bxm 3c
=> 8x3m + 22x2m - 9xm - 9 = ax3m + (3a + b)x2m +(3b + c)x':m + 3c.
234 6 Fatoração

Passo 02: Por identidade de polinômios, encontramos facilmente os valores de


a, b e c:
8x3m +22x2m!m -9xm - 9 = ax3m +(3a +b)x2m +(3b + c)xm +3c

|a = 8|, 3a+b = 22 => 24 + b = 22 => |b = -2|, 3c = -9 => |c = -3|.

Logo, temos: 8x3m + 22x2m -9xm -9 = íxm-3](8x2m-2x


-2x m-3).

Passo 03: Usando a cruzadinha simples, para fatorar o termo do segundo grau,
temos.
8x3m + 22x2m -9xm -9 = (xm -3)(2x2m +lj(4x':m-3).

No sexto critério, veremos a chamada "cruzadinha dupla", é como se fosse uma


generalização da cruzadinha simples, é muito eficaz para fatorar expressões
com duas variáveis.

6.8) Cruzadinha Dupla


Esse quinto critério, denominado "cruzadinha dupla", é uma ferramenta
fortíssima que usa soma e produto para encontrar os trinômios que serão os
fatores Não se esqueça das regras de sinais:
“(+)•(+)=+; (-)■(-) = + ; (+)(-)=-"•

A forma geral, para que tenhamos uma "cruzadinha dupla”, é.


2 variáveis simples: f (x) = Ax2 + Bx -y + Cy2 +Dx + Ey + F
2 variáveis generalizada: f (x) = Ax2m + Bxm • yn + Cy2n + Dxm + Eyn + F

Para fatorá-la, vamos realizar 4 passos, a saber:


Passo 01: Aplica-se cruzadinha simples para o trinômio Ax2 + Bxy + Cy2.

Passo 02: Aplica-se cruzadinha simples para o trinômio Cy2 +Ey+ F .


Passo 03: Aplica-se cruzadinha simples para os extremos, para a verificação
do termo Dx, não utilizado.

Passo 04: Cada linha forma um fator.

Observação: Sempre se deve colocar o polinómio a ser fatorado em ordem


decrescente de expoente, e, se faltar algum termo, completar com zeros.
Os Segredos da Álgebra para IME/ITA/OLIMPÍADAS 235

Veja o esquema abaixo:


2 Variáveis Simples
Ax2 + |Bxy| + Cy2 + Dx + [Êy] + F

cv y
a2 • x c2 • y f2

Ax2 + Bx ■ y + Cy2 + [Px] + Ey + F


a1 • x Í1
a2 ■ x f2

A operação (a, x)(c2 -y) + (a2 •x)(c1 -y) deve resultar no termo xy.

As operação (c2 •y)-(f1) + (c1-y)-(f2) deve resultar no termo y.


As operação (arx)-(f2)+(a2 x)-(f1) deve resultar no termo x.

2 Variáveis Generalizada
Ax2m + |Bxm ■ yn| + Cy2n + Dxm + |Ê/] + F

an xm c, • yn fi
a2 ■ xm c2 • yn f2

Ax2m + Bxm ■ yn + Cy2n + |pxm| + Eyn + F

aixm f,
a2-xm f2

A operação (a, xm)(c2 yn) + (a2 xmyn) deve resultar no termo xm • yn.

A operação (c2 ■ynj (f1) + (c1 yn) (f2) deve resultar no termo yn .

A operação (a1 ■ xm j (f2) +^a2 ■ xm j (f,) deve resultar no termo xm.

Exemplo Resolvido 242: Fatore x2 -4xy + 4y2 + 3x-6y .


Resolução: Vamos resolver passo a passo:
Passo 01: Aplica-se cruzadinha simples para o trinômio xz -4xy + 4y2 .
236 6 Fatoração

x2 -4xy+ 4y2
x -2y
x -2y

Passo 02: Aplica-se cruzadinha simples para o trinômio 4y2 - 6y + 0 .

4y2 - 6y + 0
-2y 0
-2y 3

Passo 03: Aplica-se cruzadinha simples para os extremos, para a verificação


do termo 3x , não utilizado.
x2 +3x + 0
x 0
x 3

Passo 04: Cada linha forma um fator:


x2 - 4xy + 4y2 + 3x - 6y + 0
x -2y 0 «= linha
x -2y 3 e= linha

Ou seja, x2 -4xy + 4y2 +3x-6y = (x-2y)(x-2y + 3).

Exemplo Resolvido 243: Fatore x2 +xy-12y2 + 8x + 11 y + 15 .

Resolução: Vamos resolver passo a passo:


Passo 01: Aplica-se cruzadinha simples para o trinômio x2 + xy -12y2 .

x2 +xy-12y2
x 4y
X -3y

Passo 02: Aplica-se cruzadinha simples para o trinômio -12y2 +11y +15.
Os Segredos da Álgebra para IME/ITA/OLIMPÍADAS 237

-12y2 +11y +15


4y 3
-3y 5

Passo 03: Aplica-se cruzadinha simples para os extremos para, a verificação


do termo 8x , não utilizado.
x2 + 8x + 15
x 3
x 5

Passo 04: Cada linha forma um fator:


x2 + xy -12y2 + 8x +11 y +15
x 4y 3 «= linha
x -3y 5 <= linha

Ou seja, x2 + xy-12y2 + 8x + 1 1 y + 15 = (x + 4y + 3)(x-3y + 5).

Exemplo Resolvido 244: Fatore 2x4 +17x2y2 +21 y4 + 8x2 + y'’2 -10.

Resolução: Vamos resolver passo a passo:


Passo 01: Aplica-se cruzadinha simples no trinômio 2x4 +17x2y2 +21 y4.

2x4 +17x2y2 +21y4


2x2 3y2
x2 7y2

Passo 02: Aplica-se cruzadinha simples para o trinômio 21 y4 +y2 -10 .

21y4+y2-10
3y2 -2
7y2 5

Passo 03: Aplica-se cruzadinha simples para os extremos, para a verificação


do termo 8x2, não utilizado.
238 6 Fatoração

2x4 + 8x2 -10

2x2 -2
x2''' *' 5

Passo 04: Cada linha forma um fator:


2x4 +17x2y2 +21 y4 +y2 + 8x2-10
2x23y2 -2 <= linha
x2 7y2 '■ ■*. 5 <= linha
Ou seja,
2x4 + 17x2y2 +21 y4 +8x2 +y2 -10 = (2x2+ 3y2 -2)(x2 +7y2 +õ).

O próximo critério é a chamada “cruzadinha dupla especial”, uma interessante


ferramenta para fatorar polinõmios do quarto grau.

6.9) Cruzadinha Dupla Especial:________________________________________


Nesse critério, denominado “cruzadinha dupla especial", aprenderemos a
fatorar polinõmios de quarto grau e usaremos as fatorações anteriores também,
mas com uma pequena diferença. Não se esqueça das regras de sinais:
“(+)•(+)=+; (-)■(-)=+; (+)(-)=-”■

A forma geral, para que tenhamos uma "cruzadinha dupla especial”, é:


1 variável simples : f (x) = Ax4 + Bx3 + Cx2 + Dx + E

1 variável generalizada : f (x) = Ax4m + Bx3m + Cx2m + Dxm + E

Para fatorá-la, vamos realizar 4 passos, a saber:

Passo 01: Aplica-se cruzadinha simples para os coeficientes extremos. Nessa


aplicação já iremos contabilizar uma parte do termo central.

Passo 02: Fazemos a diferença entre o termo central e o resultado já


contabilizado do passo 01.

Passo 03: Aplica-se cruzadinha simples com o resultado do passo 02, para a
verificação dos termos ainda não utilizados.

Passo 04: Cada linha forma um fator.

Observação: Sempre colocar o polinômio a ser fatorado em ordem


decrescente de expoente e, se faltar algum termo, completar com zeros.
Os Segredos da Álgebra para IME/ITA/OLIMPÍADAS 239

Veja o esquema abaixo:


1 Variável Simples
Ax4 + Bx3 - Cx2 + Dx + E

a, • x2 ——— •* e,
a2 ■ X2 —-------- e2

Rx2 = Cx2 - (a, x2 e2 + a2 • x2 ei)

I
Ax4 + Bx3 + Rx2 + Dx + E
a, ■ X2
rvx el
a2 -x2 r2x e2

A operação (a! ■ x2 j-(e2) + (a2 • x2 j ■ (e,) será parte do termo x2 .

A operação Rx2 = Cx2-(a! • x2 ■ e2 + a2 • x2 ■ e^ será o termo central de

referência para a cruzadinha simples.


Aoperação -x2)(r2 • x) + (a2 ■ x2)(rt ■ x) deve resultar no termo x3.

A operação (r2 ■ x) ■ e, + • x) ■ e2 deve resultar no termo x.

1 Variável Generalizada

Ax4m + Bx' + Dxm + E

a,■x2m
ei
a2 x2m e2
Rx2m = Cx2m -(a, -x'2m o j.a y
•C2 ~ Q2 ' *

l
Ax4m + Bx3m + Rx2m Dxm + E
a,x2m q ■ xm ei
a2 X2m r2 ■ xm e2

A operação (ad x':2m).(e2) + (a2-x:2m


: j (e1) será parte do termo x',2m

A operação Rx,2m
2m = Cx'2m - (a-, ■x2m e
= Cx + a2
e22+a x'2m e1) será o termo central de
2 ■x

referência para a cruzadinha simples.


240 6 Fatoração

A operação (a, • x',2m )(r2 xm) + (a2 x,2m


2m)(ri xm) deve resultar no termo x3m

A operação (r2 • xm > ■ e-| + (rd ■ xm) ■ e2 deve resultar no termo xm.

Exemplo Resolvido 245: Fatore x4 - 6x3 + 5x2 + 4x -4 .


Resolução: Vamos resolver passo a passo:
Passo 01: Aplica-se cruzadinha simples para os coeficientes extremos. Nessa
aplicação já iremos contabilizar uma parte do termo central.
Passo 02: Fazemos a diferença entre o termo central e o resultado já
contabilizado do passo 01.
Passo 03: Aplica-se cruzadinha simples com o resultado do passo 02, para a
verificação dos termos ainda não utilizados.
x4 -6x3 +5x2 +4x-4

x2 1

—>Rx2 =5x2-(x2-1 + x2(-4)J

=> Rx2=5x2+3x2 => Rx2 = 8x2

l
x4 -6x3 + 8x2 +4X-4
x2 _4x -4 <= linha
x2 -2x 1 <= linha

Passo 04: Cada linha forma um fator, ou seja,


x4 -6x3 +5x2 + 4x-4 = (x2 -4x-4)(x2 -2x + lj.

Exemplo Resolvido 246: (IME - Modificada) Fatore


x4 - 2x3 -11 x2 +18x +18 .

Resolução: Vamos resolver passo a passo:


Passo 01: Aplica-se cruzadinha simples para os coeficientes extremos. Nessa
aplicação já iremos contabilizar uma parte do termo central.
Passo 02: Fazemos a diferença entre o termo central e o resultado já
contabilizado do passo 01.
Os Segredos da Álgebra para IME/ITA/OLIMPÍADAS 241

Passo 03: Aplica-se cruzadinha simples com o resultado do passo 02, para a
verificação dos termos ainda não utilizados.
x4 - 2x3 -11 x2 +18X+18

x2 ———* -9
-------- *. -2
Rx2 = -11 x2 - (x2 ■ (-2) + x2 ■ (-9)]

=> Rx2 = -11 x2 +11 x2 Rx2 = Ox2

l
x4 - 2x3 + Ox2 +18x +18
x2 Ox -9 <= linha
x2 -2x-^><^ -2 <= linha

Passo 04: Cada linha forma um fator, ou seja.


x4 - 2x3 -11 x2 +18x +18 = (x2 - 9)(x2 - 2x - 2).

Exemplo Resolvido 247: (IME - Modificada) Fatore


x4 -12x3 +44x2 -32X-52.

Resolução: Vamos resolver passo a passo:


Passo 01: Aplica-se cruzadinha simples para os coeficientes extremos. Nessa
aplicação já iremos contabilizar uma parte do termo central.
Passo 02: Fazemos a diferença entre o termo central e o resultado já
contabilizado do passo 01.
Passo 03: Aplica-se cruzadinha simples com o resultado do passo 02. para a
verificação dos termos ainda não utilizados.
X4 -12x3 +44x2 -32X-52
X2 -2
x2 26
Rx2 = 44x2 - (x2 (-2) + x2 ■ 26)

24 x2
242 6 Fatoração

=> Rx2 = 44x2 — 24x2 Rx2 =20x2

X4 -12x3 l-20x2 -32x -52


x2 f -2x t »• -2 <= linha
x2 -10x26 <= linha

Passo 04: Cada linha forma um fator, ou seja,


x4 -12x3 + 44x2 - 32x - 52 = (x2 - 2x - 2)(x2 -1 Ox + 2ô).

Exemplo Resolvido 248: Fatore x4 -4x3 +16x2 + 40x + 52 .


Resolução: Vamos resolver passo a passo:
Passo 01: Aplica-se cruzadinha simples para os coeficientes extremos. Nessa
aplicação já iremos contabilizar uma parte do termo central.
Passo 02: Fazemos a diferença entre o termo central e o resultado já
contabilizado do passo 01.
Passo 03: Aplica-se cruzadinha simples com o resultado do passo 02, para a
verificação dos termos ainda não utilizados.
x4 -4x3 +16x2 +40X + 52

x2 ____ —► 2
26
Rx2 =16x2 -(x2 -2 + x2 -26)

28x2
=> Rx2 =16x2-28x2 Rx2 = -12x2

l
x4 -4x3 -12x2 +40X + 52
x2 2x 2 <= linha
x2 -6x 26 <= linha

Passo 04: Cada linha forma um fator, ou seja,


x4 - 4x3 +16x2 + 40x + 52 = (x2 + 2x + 2)(x2 - 6x + 26).

No próximo critério iremos fatorar polinômios do 5° grau. Usaremos o teorema


do fator ou das raizes racionais e a cruzadinha dupla especial.
Os Segredos da Álgebra para IME/ITA/OLIMPÍADAS 243

6.10) Fatorando Polinômios do 5o grau


Vamos fatorar polinômios do 5o grau, usando o teorema do fator e a cruzadinha
dupla especial.

Seguiremos os seguintes passos:


Passo 01: Usamos o teorema do fator ou das raizes racionais.
Passo 02: Como x - a é um fator e o polinômio é do 5o grau, então o outro fator
é do 4o grau. Para encontrá-lo podemos usar o dispositivo de Briot-Ruffini para
divisão de polinômios ou usar o artificio abaixo.
Artificio: Utilizamos a identidade de polinômios para obter o outro fator,
vejamos os exemplos para ficar mais claro.

Observação: Como o dispositivo de Briot-Rufinni é usado para divisão de


polinômios e esse assunto foge aos objetivos desse livro, então iremos usar o
artificio em todas as fatorações de polinômios de 5o grau por esse critério.

Passo 03: Usamos a cruzadinha dupla especial.

Exemplo Resolvido 249: Fatore 2x5 + x4 -16x3 - 8x2 + 30x + 15.

Resolução: Vamos resolver passo a passo:


Passo 01: Usando o teorema das raizes racionais, temos que
±1, ± 3, ± 5, ± 15
a= . Portanto, podermos ter como raizes:
±1,±2
±1, + 3,±5±15,±-1,±|,±| e . Assim, por inspeção, temos que é

raiz, logo podemos escrever:


2x5 + x4 -16x3 -8x2 + 30X + 15 = (x + ? j(ax4 +bx3 :x2 + dx + ej

=> 2x5+ x4-16x3-8x2+30X + 15 =

= ax5+bx4 :x3 + dx2 + ex + - x4 + - x3 + - x2 + - e


2 2 2 2 2
=> 2x5 + x4 -16x3 -8x2 +30X + 15

= ax5 Í£ + bV+fÈ + c e
12 J V2 2
244 6 Fatoração

Passo 02: Por identidade de polinõmios, encontramos facilmente os valores de


a, b, c, d e e:
I---------- g ._ K _ __ ____ .
a = 2,2 - + b = 1
----- |b = 0|. => -2 + c = -16 => ---------
=>------ |c = -16.

— + d= —8=> —----- + d = —8 => |d = 01, => —— 15 => |e = 30 [.


2 2 1-------- 1 2 '----------
Logo, temos:
2x5 + x4 -16x3 -8x2 +30x + 15 = ^x + ^(2x4 -16x2 +30)

2x5 + x4 -16x3 - 8x2 + 30x +15 = ^x + ^2 • (x4 - 8x2 +1 õ)

2x5 + x4 -16x3 -8x2 + 30x + 15 = (2x + 1)(x4 -8x2 +15).

Passo 03: Usamos a cruzadinha dupla especial, para fatorar o termo do quarto
grau.
2x5 + x4 -16x3 -8x2 +30X + 15 = (x + £)(2x4 -16x2 +3o)

2x5 + x4 -16x3 - 8x2 + 30x + 15 = íx + ^2 • (x4 - 8x2 +1 õ)

2x5 + x4 -16x3 - 8x2 + 30x +15 = (2x + 1)(x4 - 8x2 +15).

Exemplo Resolvido 250: Fatore 3x5 -5x4 -13x3 +19x2 +12X-12.


Resolução: Vamos resolver passo a passo:
Passo 01: Usando o teorema das raizes racionais, temos que
±1 ±2 ±3 ±4+6 ±12
a = —-———j. Portanto, podermos ter como raizes:

±1,±2,±3,±4,±6,±12,±i, ±-2 4
e ±- . Assim, por inspeção, temos que -1 é

raiz, logo podemos escrever:


3x5 - 5x4 -13x3 +19x2 +12x -12 = (x + 1)(ax4 + bx3 :x2 +dx + e)

=> 3x5-5x4-13x3+19x2 + 12x-12 =

= ax5 + bx4 + cx3 + dx2 + ex + ax4 + bx3 + c::x2 + dx + e


3x5-5x4-13x3 +19x2 +12X-12

= ax5+(a+ b)x4 +(b+ c)x3 + (c + d)x2 + (d+ e) x + e.


Os Segredos da Álgebra para IME/ITA/OLIMPlADAS 245

Passo 02: Por identidade de polinômios, encontramos facilmente os valores de


a, b, c, d e e:
|a = 3|, a + b = -5 => |b = —8|, => b + c = -13 => |c = —5|.
c + d = 19 =5 -5+d=19 => |d = 24|, |e = -12l
Logo, temos:
2xs + x4 -16x3 - 8x2 + 30x +15 = (x + 1)(3x4 - 8x3 - 5x2 + 24x -12).
Passo 03: Usamos a cruzadinha dupla especial, para fatorar o termo do quarto
grau.

Exemplo Resolvido 251: Fatore


6x5m - 29x4m +12x3rn +107x2m -144xm + 36 .

Resolução: Vamos resolver passo a passo:


Passo 01: Usando o teorema das raízes racionais, temos que
3_±1, ±2, ±3, ±4, ±6, ±9, ±12, ±18, ±36 . Portanto, podermos ter como
±1, ± 2, ±3, ±6
raizes:
1112 3 4 9
±1, ± 2, ± 3, ± 4, ± 6, ± 9, ± 12, ± 18, ± 36, ±—,±—,±—,±—,±±— e ±- . Assim,
—,±—
2 3 6 3 2 3 2
por inspeção, temos que 2 é raiz, logo podemos escrever:
6x5m - 29x4m +12x3m +107x2m -144xm + 36 =
= (xm -2j(ax4m +bx3m +cx
+ cx2m +dx m +e)
+ dxm

=> 6x5m -29x4m +12x3m +107x2m -144xm + 36 = ax5m + bx4m +


+ cx3m + dx2m + exm - 2ax4m - 2bx3m - 2cx2m - 2dxm - 2e
-2dx
=> 6x5m - 29x4m +12x3m +107x2m - 144xm + 36 =
= ax5m + (b - 2a) x4m + (c - 2b) x3m + (d - 2c) x2m + (e - 2d) xm -2e

Passo 02: Por identidade de polinômios, encontramos facilmente os valores de


a, b, c, d e e:
|a = 6|, b-2a = -29 => b-12 = -29 => |b = -17|, c - 2b = 12
=> c-2 (-17) = 12 => c + 34 = 12 => |c = -22|.
d-2c = 107 => d-2 (-22) = 107 => d + 44 = 107 => |d = 63|,
-2e = 36 => |e = -18|.
246 6 Fatoração

Logo, temos:
6x5m -29x4m + 12x3m +107x2m - 144xm + 36 =
= (xm - 2)(6x‘,m -17x3m -22x2m + 63xm -18 )

Passo 03: Usamos a cruzadinha dupla especial, para fatorar o termo do quarto
grau.

Agora veremos a chamada “cruzadinha tripla", uma fatoração para três


variáveis fortíssima que segue o mesmo raciocínio das fatorações anteriores,
vamos conhece-la!

6.11) Cruzadinha Tripla __________________


Esse sétimo critério, denominado "cruzadinha tripla”, é uma fatoração pouco
usada, veremos como utilizá-la. Não esqueça das regras de sinais:
“(+)•(+)=+; (-)■(-)=+; (+)■(-)=-"•

A forma geral, para que tenhamos uma "cruzadinha tripla”, é:

3 Variáveis Simples:
f (x) = Ax2 + Bxy + Cy2 + Dxz + Eyz + Fz2 + Mx + Ny + Pz + Q

3 Variáveis Generalizada:
f (x) = Ax2r + Bxrys + Cy2s + Dxrz‘ + Eysz‘ + Fz21 + Mxr + Nys + Pz‘ + Q

Para fatorá-la, vamos realizar 5 passos, a saber:


Passo 01: Aplica-se cruzadinha simples para o trinômio Ax2 + Bxy + Cy2.

Passo 02: Aplica-se cruzadinha simples para o trinômio Cy2 + Eyz + Fz2.

Passo 03: Aplica-se cruzadinha simples para o trinômio Fz2 + Pz + Q .

Passo 04: Verificam-se os termos que não foram usados.

Passo 05: Cada linha forma um fator.

Observação: Sempre colocar o polinômio a ser fatorado em ordem


decrescente de expoente e, se faltar algum termo, completar com zeros.
Os Segredos da Álgebra para IME/ITA/OLIMPlADAS 247

Veja o esquema abaixo:


Axz +{Bxy)-r Cy2 + Dxy i{Eyz)+ Fz2 + Mx 4 Ny +(P^- Q

a,x cry f,z


32 X C2y f2 Z 92

Ax2 + Bxy -r Cy2 +(Dxy)- Eyz 4- Fz2 y\ Pz - Q

a1 -x f,-z
32- x f2 z.
cvy + 9i
C2 y ► 92
3! • x ♦ 9i
32- X * 92

A operação (a, ■ x) ■ (c2 ■ y) + (a2 • x) ■ (c, ■ y) deve resultar no 2o termo.


A operação (c2 • y) ■ (t, ■ z) + (c-] ■ y) ■ (f2 ■ z) deve resultar no 5° termo.
A operação z)-(q2) + (f2 ■z) (q1) deve resultar no 9o termo.
Verifique se a operação (a! • x) (f2 z) + (a2 - x) - (f, z) resulta no 4o termo.
Verifique se a operação (c2 ■ y) ■ (q4) + (c1 ■ y) • (q2) resulta no 8o termo.
Verifique se a operação (a-j ■ x) ■ (q2) + (a2 ■ x) ■ (q1) resulta no 7° termo.

Exemplo Resolvido 252: Fatore


x2 - xy - 2y2 + xz + 7yz - 6z2 - x + 5y - 8z - 2.
Resolução: Vamos resolver passo a passo:
Passo 01: Aplica cruzadinha simples para o trinômio x2 - xy - 2y2 .

x2 - xy - 2y2
X^t>r -2y
x-' -* y

Passo 02: Aplica cruzadinha simples para o trinômio -2y2 + 7yz-6z2 .

-2y2 +7yz-6z2
-2y 3z
248 6 Fatoração

Passo 03: Aplica cruzadinha simples para o trinômio -6z2 - 8z-2 .


-6z2 -8z-2
3z 1
-2z -2

Passo 04: Verifica-se os termos que não foram usados.

Passo 05: Cada linha forma um fator, ou seja,


xz - xy-2y2 + xz + 7yz-6z2 - x + 5y-8z-2 = (x + y-2z-2)(x-2y + 3z + 1)

Exemplo Resolvido 253: Fatore


15x2 +14xy-49y2 -46xz + 42yz +16z2 +32x + 56y-74z + 9 .
Resolução: Vamos resolver passo a passo:
Passo 01: Aplica-se cruzadinha simples no trinômio 15x2 +14xy - 49y2 .

15x2 +14xy -49y2


3x 7y
5x -7y

Passo 02: Aplica-se cruzadinha simples no trinômio -49y2 + 42yz + 16z2.

-49y2 +42yz + 16z2


7y -....j.,,»' -8z
-7y -2z

Passo 03: Aplica-se cruzadinha simples no trinômio 16z2 -74z + 9 .


16z2 -74z + 9
-8z^t>r 1
-2z 9

Passo 04: Verificam-se os termos que não foram usados.

Passo 05: Cada linha forma um fator, ou seja,


15x2 + 14xy - 49y2 - 46xz + 42yz +16z2 + 32x + 56y - 74z + 9 =
= (3x + 7y - 8z + 1)(5x-7y-2z + 9).
Os Segredos da Álgebra para IME/ITA/OLIMPÍAOAS 249

Problemas Propostos

Questão 6.59 (Turquia-2005-Modificada)


Fatore x3 - 6x2 + 5 .

Questão 6.60 (Turquia-2009-Modificada)


Fatore x3 + 4x2 - x -10 .

Questão 6.61_________
Fatore x3 + 4x2 - x - 4 .

Questão 6.62_________
Fatore x3 +4x2 + x + 4.

Questão 6.63_______
Fatore 3x3 + 7x2 - 4.

Questão 6.64__________
Fatore x3 + 2x2 - 5x - 6 .

Questão 6.65 (Canadá-1996-IVlodificada)


Fatore 2x3 + 9x2 - 6x - 5.

Questão 6.66_______________
Fatore 4x3 - 24x2 + 35x -12.

Questão 6.67 (AHSME--|954-Modificada)


Fatore x3 + 6x2 +11x + 6 .

Questão 6.68 (AHSIVIE-1954-IVIodificada)


Fatore 4x3 - 8x2 - 63x - 9 .
250 6 Fatoração

Questão 6.69 (AHSME-1980-IVIodificada)


Fatore 8x3 - 4x2 - 42x + 45.

Questão 6.70 (Stanford-2008-Modificada)


Fatore x3 - 2x2 - 2x + 4 .

Questão 6.71
Fatore x3 -2x2 -x + 2 .

Questão 6.72
Fatore 6x3 +7x2 +13x + 2 .

Questão 6.73
Fatore x3 + 3x2 + 3x - 7.

Questão 6.74
Fatore x3 + x2 - 2 .

Questão 6.75
Fatore x3 + 4x2 -15x -18 .

Questão 6.76 (Espanha-2003-Modificada)


Se a é uma raiz de x3 + 2x2 +10x + 20 = 0. Mostre que a2 é um número par.

Questão 6.77
Fatore 2x2 - xy - 5x - 3y2 + 20y - 25.

Questão 6.78_____________________
Fatore -3x2 - 7xy - 2x - 2y2 - 9y + 5.

Questão 6.79______________________
Fatore 8x2 +10xy + 3y2-9y-14x + 6 .

Questão 6.80
Fatore x2 + 2xy + y2 - 3x - 3y - 4 .
Os Segredos da Álgebra para IME/ITA/OLIMPÍADAS 251

Questão 6.81
Fatore 4x2 + 4xy -15y2 - 8x + 76y - 96 .

Questão 6.82
Fatore 10x2 + xy - 33x - 2y2 +15y - 7 .

Questão 6.83
Fatore 3x2 -22xy -17x-16y2 -20y-6 .

Questão 6.84
Fatore 7x2 - 22xy + 41x + 3y2 - 23y + 30 .

Questão 6.85
Fatore 7x2 + 22xy +13x + 3y2 - y - 2 .

Questão 6.86
Fatore -3x2 - xy + 5x + y-2 .

Questão 6.87
Fatore 18x2 + 43xy + 24y2 +27x + 24y .

Questão 6.88 (Bósnia-2004-Modificada)


Fatore x4 - 56x +15 .

Questão 6.89 (Putnam-2001-Modificada)


Fatore x4-(2n-4)x2 +(n-2)2.

Questão 6.90 (Turquia-2001-Modificada)


Fatore x4 + 3x3 + 5x2 + 21 x -14 .

Questão 6.91 (Turquia-2004-Modificada)


Fatore x4 - 4x3 + 5x2 - 4x +1.
252 6 Fatoração

Questão 6.92 (Turquia-2009-Modificada)


Fatore x4 + 2x3 - 8x2 - 6x +15 .

Questão 6.93
Fatore x4 + 2x3 -16x2 - 2x +15 .

Questão 6.94 (Turquia-2009-Modificada)


Fatore n4 + 4n3 + 3n2 - 2n + 7.

Questão 6.95 (Turquia-2012-Modificada)


Fatore x4-7x3 + 14x2-14x + 4 .

Questão 6.96 (Turquia-2013-Modificada)


Fatore x4 -8x3 + 13x2 - 24x + 9 .

Questão 6.97 (Turquia-2013-IVIodificada)


Fatore n4 + 2n3 - 20n2 + 2n - 21.

Questão 6.98 (AHSME-1950-Modificada)


Fatore x4 - 7x3 + 14x2 - 8x .

Questão 6.99 (AHSME-1955-Modificada)


Fatore x4 + 2x2 + 9 .

Questão 6.100
Fatore x4 + 6x2 + 25 .

Questão 6.101
Fatore 3x4 + 4x2 + 28x + 5.

Questão 6.102 (Harvard/MIT-2006-Modificada)


Fatore x4 - 6x3 + 26x2 - 46x + 65 .

Questão 6.103 (Stanford-2013-Modificada)


Fatore x4 - x3 - 5x2 + 2x + 6 .
Os Segredos da Álgebra para IME/ITA/OLIMPÍADAS 253

Questão 6.104 (Stanford-2010)


Determine as raizes de 6x4 +17x3+7x2-8x-4 = 0.

Questão 6.105
Fatore 6x4 - x3 +17x2 - 3x - 3 .

Questão 6.106 (Stanford-2002)


Fatore o polinômio x4 - x3 - 5x2 + 3x + 6 .

Questão 6.107 (CN - 2004 - Modificada)

Fatore x2(x2 +13)-6x(x2 +2) + 4.

Questão 6.108
Fatore 3x4 - 8x3 + 3x2 + 22x - 24 .

Questão 6.109
Fatore 15x4 + 44x3 + 7x-6.

Questão 6.110
Fatore x4 -27x2 -14x + 120 .

Questão 6.111
Fatore x12n-''
- x'9n + 2x6n - x3n +1.

Questão 6.112
Fatore x4 + x3 + 2x2 + x +1.

Questão 6.113
Fatore x4 - 5nx3 + 4n2x2 + 7n3x - 3n4 .

Questão 6.114
Fatore 2x4 -3nx3 -7n2x2 -5n3x-3n4 .

Questão 6.115
Fatore 2x4 - 9x3 + 6x2 + 20x - 24 .
254 6 Fatoração

Questão 6.116
Fatore 2x4 - x3 + 3x2 - x +1.

Questão 6.117 (Harvard/MIT-2009 - Modificada)


Fatore x4 + 2x3 + 2x2 + 2x +1.

Questão 6.118
Fatore x4 + 2x3-13x2-14x + 24 .

Questão 6.119
Fatore 2x4 +5x3-41 x2-149X-105.

Questão 6.120
Fatore 4x4 - 23x3 + 30x2 - 207x - 54 .

Questão 6.121 (CN-2002 - Modificada)


Fatore as - 5a3 + 4a .

Questão 6.122 (Brasil-1995-Modificada)


Fatore x5 - x4 - 2x3 + 2x2 - 8x + 8.

Questão 6.123
Fatore x7 + x6 - 9x5 -13x4 + 8x3 +12x2 .

Questão 6.124 (Turquia-2001-Modificada)


Fatore x5 + x4 - x3 - x2 - 2x - 2 .

Questão 6.125 (Turquia-2002-Modificada)


Fatore a5 + 5a4 +1Oa3 + 2a2 -11a - 7 .

Questão 6.126
Fatore -x5 - x3 + x2 +1.

Questão 6.127 (Princeton-2006-Modificada)


Fatore x5 +5x4 -79x3 +64x2 +60x + 144 .
Os Segredos da Álgebra para IME/ITA/OLIMPÍADAS 255

Questão 6.128 (Turquia-2011-Modificada)


Fatore x5 + x4 - 4x3 - 7x2 - 7x - 2.

Questão 6.129
Fatore 12xs - 26x4 + 6x3 +13x2 - 3x - 2 .

Questão 6.130
Fatore 3x5 + 5x4 + 3x3 + 3x2 + 5x + 3 .

Questão 6.131
Fatore x5 + 5x4 + 7x3 - x2 - 8x - 4 .

Questão 6.132
Fatore x5 - 6x4 +16x3 - 32x2 + 48x - 32 .

Questão 6.133
Fatore x5 - 6x4 + 8x3 -16x2 - 48x - 32.

Questão 6.134
Fatore x2 + 3xz + 6x - 4y2 - 2yz + 8y + 2z2 +11 z + 5 .

Questão 6.135
Fatore 2x2 -5xy + 11 xz-3x + 3y2 -16yz + 6y + 5z2 +12z-9 .

Questão 6.136
Fatore -2x2 + xy + 4xz - 7x + y2 - yz - 5y - 2z2 + 7z + 4.

Questão 6.137
Fatore 2x2 + 5xz + 5x - 2y2 - 3yz - 3y + 2zz + 4z + 2.

Questão 6.138
Fatore 3x2 - xy + 8xz +4x-10y2 +17yz + 3y-3z2 +2z + 1.
256 6 Fatoração

6.12) Tópicos Avançados em Fatoração


Essa parte de fatorações é bem avançada, são as chamadas Fatorações de
Aurifeuillean. Há uma aplicação dela numa questão de olimpíada que
certamente poucos candidatos fizeram. Vejamos então essas fatorações!

Observação: Essas fatorações não serão demonstradas.

FA1) Potências de 2.
Fatoração para a potência de 2, um pequeno detalhe diferencia uma da outra,
confira!
24n+2 + 1 = ^22n+1 -2n+1+l)(22n+1+ 2n+1+l).

24n-2 + 1 = (22n’1 -2n+l)(22n-1 + 2n + l).

Aplicação da fatoração para potência de 2.


Prove que o número 21992 -1 pode ser escrito como produto de seis inteiros
maiores que 2248.

Resolução: Vamos resolver passo a passo.


Passo 01: Vamos usar o produto da soma pela diferença, então temos:
21992 _1 = (2996j2 21992 _1 = (2996 +^2996 _1j
12

21992 -1 =(2996 + 1)[(2'


,498^

_> 21992 -1
= (2"8 + 1)(2498 + 1)(2498 -1)

=> 2,1992 =(2996 + 1)(2498 +1)[(2249)|2: -121


21992 _1 _ (2996 + l)(2498 +l)(2249 +lj(2,249
;
-4
Passo 02: Note que 2996 +1 é da forma: a3 + b3 .
Assim, podemos escrever:
2
2996
+1
= (2332)3 +13 => 2996 +1 = (2332
+ 1)[(2332): -2332-1 + -Í2
. 2996
1 =(2332 + lV2664 _2332 4
Os Segredos da Álgebra para IME/ITA/OLIMPÍADAS 257

Passo 03: Usando a fatoração de Aurifeuilean para o 2498 +1, temos:


496
4n + 2 = 498 => 4n = 498-2 => n =----- In = 124
4
k = 2n +1 => k = 2 • 124 +1 => k = 248 +1 |k = 249|.
24n+2 + 1 = (22n+1' - 2n+1 + l)(22n+1 + 2n+1 +1), com n = 124

24124+2 + 1 = (22124+1
' _ 2124+1 + 1 , (22124+1 + 2124+1 + 1 j

2498 +1 = (2249 --2125+l)(2249 + 2125+l).

Portanto temos:
21992 _1 = (2996 + 1l)(2249 -2125 +l)(2249 + 212S+l)(2,249
: + 1)(2249 -1

Com 2996 + 1 ,332 ,664 _ 2332 + 1).

FA2) Potência de 3.
Na fatoração para a potência de 3, note que obtemos o produto de três fatores,
bastante interessante!
j6n-3 + 1 =(32n-1 + 13',2n-1 +3n +1)(32n-1 3n+l).

FA3) Potência de 5.
Na fatoração para a potência de 5, note que obtemos o produto de três fatores,
abaixo temos a aplicação dela, confira!
55n-1 = ^5n -l)-A B

Onde : A = 5'i2n + 3 • 5n -5k (5n +l) + 1, B = 52n + 3-5n + 5k (õn +lj + 1


e n = 2k -1.

Aplicação da fatoração para potência de 5.


(IMO-Longlist_1985) Fatore 51985 -1, como produto de três termos inteiros,
cada um maior que 5100 .

Resolução: Usando a fatoração de Aurifeuilean, temos:


l age _______ _
5n = 1985 => n = —-— |n = 397|,
258 6 F atoraçã o

n = 2k-1 => 2k-1 397 2k = 397 + 1 => k = — |k = 199|.


2 ----------
55n-1 = (5n

Onde : A = 52n + 3 ■ 5n - 5k (õn +1) +1, B = 52n + 3-5n + 5k (5n +l) + 1


e n = 2k - 1.
A = 5'i2n +3-5n-5k(5n+l) + 1

=> A = 52397 + 3-5397 - 5199(5397 + l) + 1

=> A = 5794 + 3 - 5397 _5199 (5397 + l) + 1.

B = 52n + 3 • 5n + 5k (5n +1) + 1

=> B = 52-397 + 3-5397 + 5199(5397 + l) + 1

=> B = 5794 + 3 ■ 5397 + 5199 (5397 + l) + 1.

55n-1 = (5n-l)-AB =>

51895 -1 = Í5397 -1 i).[5■794 + 3 5397 -5199 (5397 + 1) + 1]’


■[õ794 + 3 - 5397 + 51"(5397
+ 1) + 1]'
Observação: Há outras fatorações com potências maiores que 5, porém não
serão citadas.
Os Segredos da Álgebra para IME/ITA/OLIMPÍADAS 259

Capítulo 07 - Polinômios Simétricos

Introdução
Podemos fatorar polinômios usando os polinômios simétricos ou alternados. E
uma ferramenta bastante útil que facilita a fatoração. Vejamos como usar essa
ferramenta.

7.1) Polinômios Simétricos


Os polinômios simétricos são aqueles que não se alteram quando trocamos
simultaneamente qualquer par de variáveis.

Exemplo: P(x,y) = x2 + xy + y2 é um polinômio simétrico, pois, ao trocarmos

x por y e y por x, obteremos P(y,x) = y2 + yx + x2 =P(x,y).

Vejamos como são os casos genéricos dos polinômios simétricos.

7.2) Forma de um Polinômio Simétrico


Vejamos como se comportam alguns polinômios simétrico dependendo do grau
e da quantidade de variáveis.
Grau 1 e 2 Variáveis: P(x,y) = a(x1 +y1 j.

Grau 1 e 3 Variáveis: P(x,y,z) = a[x1 +y1 + z1 j.

Grau 2 e 2 Variáveis: P(x,y) = a(x2 + y2) + b(x1y1).

+ y2 + z2) + b(x1y1 +x1z1 +y1z1].


Grau 2 e 3 Variáveis: P(x,y,z) = a(x2 +y

Grau 3 e 2 Variáveis: P(x,y) = a(x3 + y,3) + b(x2y1 +x1y2j.


Grau 3 e 3 Variáveis:
P(x,y, z) = a(x3 +y3 + z3) + b(x2y + x2z + y2x + y2z + z2x + z2yj + c(xyz)

Grau 4 e 3 Variáveis:
P(x,y,z) = a(x4 + y4 +z4j+b(x3y+ x3z+ xy3 + y3z + xz':3+yz3)-
+ c(x2y2 + x2z2 + y2z2) + d( x2yz + xy2z + xyz2j.
260 7 Polinômios Simétricos

Grau 5 e 3 Variáveis:
P(x,y,z) = a(x5 + y,55 + z5) + b(x4y + x4z + xy4 +y4z + xz4 + yz4j +
+ c(x3y2 + x3z2 + x2y3 + y3z2 + x2z3 + y2z3) +
+ d(x3yz + xy3z + xyz3) + e(x2y2z + x2yz2 + xy2z2).

7.3) Propriedades dos Polinômios Simétricos


Vejamos algumas propriedades importantes dos polinômios simétricos.
P1) A soma, o produto e o quociente entre dois polinômios simétricos quaisquer
resultam num polinômio simétrico.

P2) (Ida) Seja um polinômio simétrico P(x, y, z...) nas variáveis x e y, z Se


P(x, y, z,...) for anulado para x = 0, então também será anulado para y = 0.
z = 0.... e vice-versa, ou seja, ele será divisível por x por y, por z,.... Assim, se
f>(x. y, z) for divisível por x, ele também será divisível por y e z.

P3) (Volta) Se um polinômio simétrico P(x, y, z...) for divisível por x, então será
divisível por y, z, .. e vice-versa, ou seja, se P(x, y. z) é anulado para x = 0,
y = 0 e z = 0. então x, y e z serão fatores.

P4) (Ida) Se um polinômio simétrico se anula para a igualdade entre duas de


suas variáveis, então se anulará para todas as combinações delas, ou seja,
será divisível pela diferença entre elas (x = y => x - y = 0). Além disso a
diferença entre essas duas variáveis será um fator desse polinômio. Os demais
fatores serão determinados de acordo com as expressões (formas) cíclicas
(fechadas) no sentido horário, como veremos nos exemplos.

P5) (Volta) Se um polinômio simétrico for divisível pela diferença entre duas de
suas variáveis, então será divisível por todas as combinações das outras, ou
seja, se anulará para a igualdade entre duas variáveis.

P6) (Ida) Se um polinômio simétrico se anula para a igualdade entre uma


variável e o oposto de outra, então se anulará para todas as combinações
delas, ou seja, será divisível pela soma delas (x = - y => x + y = 0). Além disso
a soma dessas duas variáveis será um fator desse polinômio.

Os demais fatores serão determinados de acordo com as expressões (formas)


cíclicas (fechadas) no sentido horário, como veremos nos exemplos.

P7) (Volta) Se um polinômio simétrico for divisível pela soma de duas variáveis,
então será divisível por todas as combinações das outras, ou seja, se anulará
para a igualdade entre uma variável e o oposto de outra.
Os Segredos da Álgebra para IME/ITA/OLIMPÍADAS 261

7.4) Fatoração por Polinômio Simétrico


Vamos aprender a fatorar polinômios, por meio dos polinômios simétricos, veja
como é uma ferramenta interessante, bastante simples e eficaz!

Para fatorar usando polinômios simétricos, vamos realizar os seguintes passos:


Passo 01: Verifique se o polinômio é simétrico.

Passo 02: Anulamos qualquer uma das variáveis para saber se haverá
monômios como fatores.

Passo 03: Igualamos duas variáveis quaisquer para saber se a diferença entre
elas é um fator, ou seja, x = y => x-y = 0.

Passo 04: Igualamos uma variável qualquer ao oposto de outra para saber se
a soma entre elas é um fator, ou seja, x = - y => x + y = 0.

Passo 05: Analisamos o grau do polinômio para colocar os fatores que faltam.

Passo 06: Damos valores ao polinômio parcialmente fatorado para encontrar


os coeficientes que faltam.

Exemplo Resolvido 254: Fatore (x + y)3 - x3 - y,33 .

Resolução: Vamos fatorar passo a passo:


Passo 01: Note que o polinômio é simétrico.
Passo 02: Note também que o polinômio se anula para x = 0 e, pela
propriedade P2, se anula para y = 0, ou seja, terá os monômios x e y como
fatores.
Passo 03: Veja que o polinômio se anula para x = - y , ou seja, x + y é fator.
Passo 04: Como o grau do polinômio é 3 e o produto também é do 3o grau,
podemos escrever: (x + y)3 -x3 - y3 = a■ xy(x + y).
Passo 05: Por fim. damos valores ao polinômio parcialmente fatorado para
encontrar o coeficiente que falta.
Para x = 1 e y = 2 , temos:
(x + y)3-x3-y3 = a • xy(x+ y) => (1 + 2)3-13-23 =a-1-2(1 + 2)

=> 33-13-23 = a-2-3 => 6a = 27-1-8 => 6a = 18 a = 3.


Logo (x + y)3 - x3 - y3 = 3xy (x + y).

Resposta: (x + y)3 -x3 -y3 = 3xy (x + y).


262 7 Polinômios Simétricos

Exemplo Resolvido 255: Fatore (x + y)4 - x4 - y44 .

Resolução: Vamos fatorar passo a passo:


Passo 01: Note que o polinõmio é simétrico.
Passo 02: Note também que o polinõmio se anula para x = 0 e, pela
propriedade P2. se anula para y = 0, ou seja, terá os monômios x e y como
fatores.
Passo 03: Como o grau do polinõmio é 4 e o polinõmio não se anula para os
passos 01 e 02, temos que o outro fator é do 2o grau, então podemos escrever:
(x + y)4 -x4 -y4 = xy[a(x2 +y2) + b ■ xy .

Passo 04: Por fim, damos valores ao polinõmio parcialmente fatorado para
encontrar os coeficientes que faltam.
Para x = 1 e y = —1, temos:
(jH-1))4 -14 -(-1)4 = 1 • (-1)[a • (l2 + (-1)2) + b • 1 • (-1)]
ò
=> -1-1 = (-1)[a(1 + 1)-b] |2a-b = 2|.

Para x = 2 e y = -1, temos:


(1.(-1))4 -14 -(-1)4 = 1 ■ (-i)[a• (12 + (-1)2) + b■ 1 ■ (-1)]
o
=> -1-1 = (-1)[a-(1 + 1)-b] |2a-b = 2|.

Assim, multiplicando a primeira equação por 2, podemos montar o sistema:


í 2a - b = 2 Í4a-2b = 4
T(-)
(5a - 2b = 8 [5a-2b = 8

4a-2b = 4 => 4-4-2b = 4 => 16-4 = 2b => 2b = 12 => |b = 6|.

Portanto, temos:
(x + y)4_x4_y4 = xy^a ■ (x2 + y2) + b • xy

(x + y)4_x4_y4 = xy|^4 -(x2 + y’‘2) + 6-xy]

(x + y)4-x4-y,4 = 2xy(2x2 + 2y2 +3xy).


Os Segredos da Álgebra para IME/ITA/OLIMPlADAS 263

Resposta: (x + y)4 - x4 -y4 = 2xy ^2x2 + 2y2 + 3xy).

Exemplo Resolvido 256: Fatore (x + y + z)3 - x3 - y'-3-z3.

Resolução: Vamos fatorar passo a passo:


Passo 01: Note que o polinômio é simétrico.
Passo 02: Veja que o polinômio se anula para x = - y, ou seja, x + y é fator.
Passo 03: Pela propriedade P6, x + z e y + z também são fatores.
Passo 04: Como o grau do polinômio é 3 e o produto dos fatores também é do
3o grau, podemos escrever:
(x + y + z)3 -x3 -y3 ~z3 = a(x + y)(x + z)(y + z).

Passo 05: Por fim, damos valores ao polinômio parcialmente fatorado para
encontrar o coeficiente que falta.
Para x = 0 , y = 1 e z = 2 , temos:
(0 + 1 + 2)3 - O3 -13 -23 =a (0 + 1)(0 + 2)(1 + 2)

=> 33-1-8 = a 1 ■ 2-3 => 6a = 27-9 => 6a = 18 a = 3.

Logo (x + y + z)3 - x3 - y’3 -z3 =3(x + y)(x + z)(y + z).

Resposta: (x + y + z)3 - x3 - y',3 z3 = 3(x + y)(x +z)(y + z).

7.5) Polinômios Alternados


Os polinômios alternados são aqueles cuja mudança simultânea de um par de
variáveis quaisquer, resulta no oposto do polinômio original.

Exemplo: P(x, y, z) = x(y-z) + y(z-x) + z(x-y) é um polinômio simétrico,


pois, ao trocarmos x por y e y por x, obteremos:
P(x, y, z) = y(x-z) + x(z-y) + z(y-x)
=> P(x,y,z) = -y(z-x)-x(y-z)-z(x-y)
=> P(x,y, z) = -[y(z-x) + x(y-z) + z(x-y)] = -P(x,y,z).
264 7 Polinômios Simétricos

Se trocarmos x por z e z por x, obteremos:


P(x. y. z) = z(y-x) + y(x-z) + x(z-y)
=• P(x,y.z) = -z(x-y)-y(z-x)-x(y-z)
=• P(x, y,z) = -[z(x-y) + y(z-x) + x(y-z)] = -P(x,y,z).
Se trocarmos y por z e z por y, obteremos:
P(x, y, z) = x(z-y) + z(y-x) + y(x-z)
=> P(x,y,z) = -x(y-z)-z(x-y)-y(z-x)
=> P(x,y,z) = -[x(y-z) + z(x-y) + y(z-x)] = -P(x, y.z).

7.6) Propriedades dos Polinômios Alternados


Vejamos algumas propriedades importantes dos polinômios alternados.
P1) Não existe polinômio alternado de grau 1 que tenha mais que duas
variáveis.

P2) Geralmente escrevemos os polinômios alternados em forma de diferenças


e eles são cíclicos (fechados).

P3) (Ida) Seja um polinômio alternado P(x, y, z...) nas variáveis x e y, z Se


P(x. y, z,...) for anulado para x = 0, então também será anulado para y = 0,
z = 0... e vice-versa, ou seja, ele será divisível por x por y, por z Assim, se
P(x, y, z) for divisível por x, ele também será divisível por y e z.

P4) (Volta) Se um polinômio alternado P(x, y, z...) for divisível por x, então será
divisível por y, z em outras palavras, se P(x, y, z) é anulado para x = 0,
y = 0 e z = 0, então x, y e z serão fatores.

P5) (Ida) Se um polinômio alternado se anula para a igualdade entre duas de


suas variáveis, então se anulará para todas as combinações delas, ou seja,
será divisível pela diferença entre elas (x = y => x - y = 0). Além disso a
diferença entre essas duas variáveis será um fator desse polinômio.
Os demais fatores serão determinados de acordo com as expressões (formas)
cíclicas (fechadas) no sentido horário, como veremos nos exemplos.

P6) (Volta) Se um polinômio alternado for divisível pela diferença entre duas de
suas variáveis, então será divisível por todas as combinações das outras, ou
seja, se anulará para a igualdade entre duas variáveis.
Os Segredos da Álgebra para IME/ITA/OLIMPÍADAS 265

P7) (Ida) Se um polinômio alternado se anula para a igualdade entre uma


variável e o oposto de outra, então se anulará para todas as combinações
delas, ou seja, será divisível pela soma delas (x = - y => x + y = 0). Além disso
a soma dessas duas variáveis será um fator desse polinômio. Os demais
fatores serão determinados de acordo com as expressões (formas) ciclicas
(fechadas) no sentido horário, como veremos nos exemplos.

P8) (Volta) Se um polinômio alternado for divisível pela soma de duas variáveis,
então será divisível por todas as combinações das outras, ou seja, se anulará
para a igualdade entre uma variável e o oposto de outra.

P9) O produto entre um polinômio simétrico e um alternado resulta em um


polinômio alternado.

7.7) Fatoração por Polinõmios Alternados


Nesta seção vamos aprender a fatorar polinõmios, por meio dos polinõmios
alternados, é uma abordagem interessante, bastante simples e rápida.

Para fatorar usando polinõmios alternados, vamos realizar os seguintes


passos:
Passo 01: Verifique se o polinômio é alternado.

Passo 02: Anulamos qualquer uma das variáveis para saber se haverá
monômios como fatores.

Passo 03: Igualamos duas variáveis quaisquer para saber se a diferença entre
elas é um fator, ou seja, x = y => x-y = 0.

Passo 04: Igualamos uma variável quaisquer ao oposto de outra para saber se
a soma entre elas é um fator, ou seja, x = - y => x + y = 0.

Passo 05: Analisamos o grau do polinômio para colocar os fatores que faltam.

Passo 06: Damos valores ao polinômio parcialmente fatorado para encontrar


os coeficientes que faltam.

Exemplo Resolvido 257: Fatore (x-y)3 (y-z)3+(z-x)3.


266 7 Polinõmios Simétricos

Resolução: Vamos fatorar passo a passo:


Passo 01: Note que o polinômio é alternado.
Passo 02: Veja que o polinômio se anula para x = y, ou seja, x - y é fator.
Passo 03: Pela propriedade P5, z - x e y - z também são fatores.
Passo 04: Como o grau do polinômio é 3 e o produto dos fatores também é do
3o grau, podemos escrever:
(x-y)3+(y-z)3+(z-x)3 =a(x-y)(y-z)(z-x).
Passo 05: Por fim, damos valores ao polinômio parcialmente fatorado para
encontrar o coeficiente que falta.
Para x = O, y = 1 e z = 2 , temos:
(x-y)3+(y-z)3+(z-x)3 =a(x-y)(y-z)(z-x)

=> (0-1)3+(1-2)3 +(2-0)3 =a(0-1)(1-2)(2-0)

=> — 1 — 1 + 8 = a • (—1) • (—1) • 2 =x> 2a = 6 |a = 3|.


Logo, temos:
(x-y)3+(y-z)3+(z-x)3 = a(x-y)(y-z)(z-x)

(x-y)3+(y-z)3+(z-x)3 =3(x-y)(y-z)(z-x).

Resposta: (x-y)3 +(y-z)3 +(z-x)3 =3(x-y)(y-z)(z-x).

Exemplo Resolvido 258: Fatore x2 (y - z) + y2 (z - x:) + z2(x-y).


Resolução: Vamos fatorar passo a passo:
Passo 01: Note que o polinômio é alternado.
Passo 02: Veja que o polinômio se anula para x = y, ou seja, x - y é fator.
Passo 03: Pela propriedade P5, z - x e y - z também são fatores.
Passo 04: Como o grau do polinômio é 3 e o produto dos fatores também é do
3o grau, podemos escrever:
x2(y-z) + y2(z-x) + z2 (x-y) = a(x-y)(y-z)(z-x).
Passo 05: Por fim, damos valores ao polinômio parcialmente fatorado para
encontrar o coeficiente que falta.
Para x = 0, y = 1 e z = 2 , temos:
x2 (y - z) + y2 (z - x) + z2 (x - y) = a(x - y)(y - z)(z - x)

=> 02 (1-2) +12 (2-0) + 22 (0-1) = a(0-1)(1 - 2)(2 - 0) =>

1-2 + 4 (-1) = a (-1) (-1)-2 => 2a = 2-4 => 2a =-2 |a = -l|.


Os Segredos da Álgebra para IME/ITA/OLIMPÍADAS 267

Logo, temos:
x2(y-z) + y2(z-x) + z2(x-y) = (-1)(x-y)(y-z)(z-x)
x2(y - z) +y2 (z - x) +z2 (x - y) = (x - y)(y-z)(x - z).

Resposta: x2 (y-z) + y2 (z -x) + z2 (x- y) = (x-y)(y-z)(x-z).

Exemplo Resolvido 259: Fatore x3(y-z) + y3(:’z-x)+z3(x-y).

Resolução: Vamos fatorar passo a passo:


Passo 01: Note que o polinômio é alternado.
Passo 02: Veja que o polinômio se anula para x = y, ou seja, x - y é fator.
Passo 03: Pela propriedade P5, z - x e y - z também são fatores.
Passo 04: Como o grau do polinômio é 4 e o produto dos fatores é do 3o grau,
podemos escrever:
x3(y-z) + y3(z-x) + z3(x-y) = (x-y)(y-z)(z-x)[a(x + y + z)].

Passo 05: Por fim, damos valores ao polinômio parcialmente fatorado para
encontrar o coeficiente que falta.
Para x = 0, y = 1 e z = 2 , temos:
x3(y-z) + y3(z-x) + z3(x-y) = (x-y)(y-z)(z-x)[a(x+y + z)]

=> O3(1 -2) +13(2-0) + 23(0-1) = a(0-1)(1-2)(2-0)[a(0+ 1 +2)]


=> 1-2 + 8 (-1) = 3a (-1)-(-!)■ 2 => 6a = 2-8 => 6a =-6 |a = -l|.

Logo, temos:
x3 (y - z) + y3 (z - x) + z3 (x - y) = (x - y)(y - z)(z - x)[a(x + y + z)] =>

x3 (y — z) + y3 (z — x) + z3 (x — y) = (x — y )(y — z)(z — x)[(-1)(x + y + z)]

x3(y-z) + y3(z-x) + z3(x-y) =(x- y)(y-z)(x- z)(x + y + z) .


268 7 Polinômios Simétricos

Problemas Propostos

Questão 7.1
Fatore (x + y)5 - x5 - y5 .

Questão 7.2 (CN-1995-IVIodificada)_____________________________________


Se (a + b + c)3 -a3 -b3 -c3 =k(a + b)[c2 + (a + b)c + abj , então o valor de k

é:
a)1 b)2
c) 3 d) 4
e)5

Questão 7.3 (Rússia)


Sejam a, b e c números reais distintos dois a dois. Mostre que
a2 (c-b) + b2 •(;a-c) + c2 -(b-a) é diferente de zero.

Questão 7.4 (CMO-2009-Modificada)


(xy + yz + zx)(x + y + z)
Fatore
(x + y)(y + z)(z + x) + xyz '

Questão 7.5
Determine o valor das expressões abaixo:
1 1 1
a)
(a-b)(a-c) (b-a)(b-c) (c-a)(c-b)
_____ a_____ b_____ _____ c_____
b)
(a-b)(a-c) (b-a)(b-c) (c-a)(c-b)

a2_______ b2 + c2
c) (a-bj(a-c) (b-a)(b-c) (c-a)(c-b)

a3 + b3 + c3
d) (a-bj(a-c) (b-a)(b-c) (c-a)(c-b)

a4 + b4 c4
e) (a-b)(a-c) (b-a)(b-c) (c-a)(c-b)
Os Segredos da Álgebra para IME/ITA/OLIMPÍADAS 269

a5 b5 + c5
f) (a-b)(a-c) (b-a)(b-c) (c-a)(c-b)

a6 b6 c6
9) (a-b)(a-c) (b-a)(b-c) (c-a)(c-b)

Questão 7.6
Determine o valor das expressões abaixo:
a-b c-a b-c
a) (a-b)(a-c) (b - a)(b-c) + (c-a)(c-b)

(a-b)2 < (c-a)2 | (b-c)2


b)
(c-a)(b-c) (a-b)(b-c) (a-b)(c-a)

(a-b)3 ' (c-a)3 ' (b-c)3


c)
(c-a)(b-c) (a-b)(b-c) (a-b)(c-a)

(a-b)4 (c-a)4 (b-c)4


d) (c -a)(b-c) + (a -b)(b-c) + (a -b)(c-a)

(a-b)5 (c-a)5 (b-c)5


e)
(c-a)(b-c) (a-b)(b-c) (a-b)(c-a)

Questão 7.7
Fatore x2y + x2z + xy2 + y2z + xz2 + yz2 + 2xyz .

Questão 7.8
Fatore x2y + x2z + xy2 + y2z + xz2 + yz2 + 3xyz .

Questão 7.9
Fatore 2z(x-y)2 (x + y)(x-z)(y-z) + 8xyz.

Questão 7.10
Fatore (x + 2y-3z)'>3 (y + 2z-3x)3 + (z + 2x-3y)3.
270 7 Polinõmios Simétricos

Questão 7.11
Fatore a3(b-c) + b3(c-a) + c3(a-b).

Questão 7.12_______________________
Fatore a4 (b-c) + b4 (c-a) + c4 (a-b).

Questão 7.13___________
Fatore a5(b-c) + b5(c-a'i) + c5 (a-b).

Questão 7.14
Fatore a(b-c)3 +b(c-a)3 +c(a-b)3 .

Questão 7.15
Fatore a(b-c)5 +b(c-a)5 +c(a-b)5 .

Questão 7.16
Fatore a2 (b - c)3 + b2 (c - a)3 + c2 (a - b)3 .

Questão 7.17
Fatore a2 (b - c)5 + b2 (c - a)5 + c2 (a - b)5 .

Questão 7.18
Fatore (x - y)(y + z)(z + x) + (y-z)(z + x)(x + y) + (z-x)(x + y)(y + z)

Questão 7.19 (BMO-1971)


Fatore (a + b)7 - a7 - b7.

Questão 7.20
Fatore (a + b + c)5 - a5 - b5 - c5 .

Questão 7.21 (União Soviética-1962)


Dados x, y e z distintos entre si. Prove que (x-y)5 + (y-z)5 +(z-x)5 é

divisível por 5(x-y)(y - z)(z-x).


Os Segredos da Álgebra para IME/ITA/OLIMPÍADAS 271

Questão 7.22
Fatore (ab + bc + ca)3 -abc(a + b + c)3.

Questão 7.23
Fatore (a + b + c)7 - a7 - b7 - c7 .

Questão 7.24
Fatore (x + y+ z)3 -(y + z-x)3 - (z + x-y)3 - (x + y-z)3.

Questão 7.25
Fatore (x + y+ z)5 - (y + z-x)5 - (z + x-y)5 -(x + y-z)5.
272 8 Somas de Newton

Capítulo 08 - Somas de Newton

Introdução
As famosas somas de Newton são importantíssimas para polinômios, quando
queremos encontrar as somas das n-ésimas potências. É incrível como elas
facilitam as contas e tornam a resolução concisa e elegante! Vamos aprender
com todos os detalhes essa maravilha que pode ser usada nos mais diversos
problemas, vamos lá!

8.1) Somas de Newton Para Dois Termos


Considere o polinômio P(x) = ax2 + bx + c , cujas raizes são q e r2. Note que,
se substituirmos q na equação, o resultado será zero, visto que q é raiz de
P(x). Então, temos:
P(q) = a ■ q2 + b ■ q + c = 0. (eq1)

Note também que, se substituirmos r2 na equação, o resultado será zero, visto


que r2 é raiz de P(x). Então, temos:
P(r2) = a r2 + br2 +c = 0. (eq2)

Somando (eq!) com (eq2): a ■ (q2 + l + b ■ (q + r2) + 2c = 0. (eq3)

Conclusões:
1) Como q2 + r2 tem grau 2, chamaremos de S2 (soma “2” de Newton). Assim

S2 = q2 + r2 .

2) Como q +r2 tem grau 1, chamaremos de S.| (soma "1" de Newton). Assim

S-| = q + r2 S, — q + r2 .

3) Como 2 tem grau 0, chamaremos de s0 (soma "0" de Newton). Assim


So = q° + r2 => So = 1 +1 => Sq = 2.

Então, podemos escrever: a ■ S2 + b ■ S-, + c • So = 0 .


Os Segredos da Álgebra para IME/ITA/OLIMPÍADAS 273

Generalização:
Se multiplicarmos (eq1) por r-, , temos:
ai?+b-if+c-r,=O. (eq4)

Se multiplicarmos (eq2) por r2 , temos:


ar3 +br2 + cr2 = 0. (eq5)

Somando (eq4) com (eq5):


a-^+r^ + b-^+rfJ + c • (r, + r2) = 0. (eq6)

Conclusões:
1) Como r3 +r2 tem grau 3, chamaremos de S3 (soma "3" de Newton). Assim

S3 = r3 + r3 ■

2) Como r2 + r2 tem grau 2, chamaremos de S2 (soma “2" de Newton). Assim

S2 = r2 + r2 .
3) Como r3 + r2 tem grau 1. chamaremos de St (soma “1" de Newton). Assim

S1 = h + r2 => Si = r, + r2 .

Então, podemos escrever: a S3+b S2+c S1=0.

Assim, seguindo o mesmo raciocínio para r2, r3 r," 2 . temos:

Se multiplicarmos (eq1) por r"-2 , temos:

a■ r2+n-2 + b• ri*"-2 + c • r"-2 = 0 => a■ r" + b• r^1 + c• rj1-2 = 0. (eq7)

Se multiplicarmos (eq2) por r2-2 , temos:

a-r2+n~2+br,1+n-2
2+n-2+c-r2-2 = 0 => ar£+br£~1+c-r2“2 =0. (eq8)

Somando (eq7) com (eq8):


a ■ (rf + r2 ) + b ■ (r^1 + r2"’1) + c • (r^2 + r2"~2) = 0. (eq9)

Conclusões:
1) Como r" + r2 tem grau n, chamaremos de Sn (soma “n” de Newton). Assim
= rin + r2 ■
274 8 Somas de Newton

2) Como r"-1 +rr2


2 *1 tem grau n-1, chamaremos de Sn-1 (soma “n - 1" de

Newton). Assim Sn_, = rf-1 +r2“1.

3) Como rf^ + rj-2 tem grau n - 2. chamaremos de Sn_2 (soma "n - 2” de

Newton). Assim Sn_2 = r"-2 + r2~2 .

Então, podemos escrever: a • Sn + b • Sn_-! + c ■ Sn_2 = 0 .

8.2) A Notação Sigma


Considere o polinômio P(x) = ax2 + bx + c, cujas raizes são r, e r2. Podemos
escrever:
P(x) = ax2 + bx + c P(x) = a- x2 ~ab x+j} (eq1)
Por outro lado, a forma fatorada de P(x) é:
P(x) = a (x-r1)-(x-r2) => P(x) = a [x2— (r, + r2)x + r1r2|. (eq2)

issim, igualando (eq1) com (eq2), temos:

P(x) = a- x2 +-x + = a -[x2 (ri+r2)x + rir2J.

Por comparação, podemos escrever:


. . b b . .
_(ri+r2) = - => r, .
rt+r2=—. (eq3)

rir2=f- (eq4)

Conclusões:
1) Como r-| +r2 tem grau 1, chamaremos de (soma "1" de Girard). Assim

CT1 = rt + r2 •
2) Como r-i r2 tem grau 2, chamaremos de o2 (soma "2” de Girard). Assim

°2 =r1 r2-
Então, podemos escrever:
P(x) = a ■ (x2 - ■ x + a2) = ° => x2 - a1 ■ x + o2 = 0. (eqõ)

Assim, juntando as duas notações, temos a generalização:


Sn - °1 Sn-1 + °2Sn-2 = 0 ou
Sn = Oi Sn-1 - o2 sn_2 . (eq6)
Os Segredos da Álgebra para IME/ITA/OLIMPÍADAS 275

Exemplo Resolvido 260: Sabendo que P(x) = x2 + px + q tem raizes a e b,

calcule a4 +b4 , em função de p e q.

Resolução: Do enunciado, temos:

= ~y => |oi °2 =+^ =>


|O1 =~P|. 02 |q2 =q|-

S0=a°+b° => So=1 + 1 => [S7~=2|.


S1 = a1 + b1 => S-j = o-j => |S-| = - p| ■

Assim, podemos escrever para n = 2, n = 3 e n = 4:


S? = CTj • S2_i — O2 ’ ^2-2 S2 = ’ $1 ~ g2 ’ ^0

=> S2 = (-p) (-p) + q-2 => |s2 =p2 + 2qj.


sn ■= 01 Sn_i-02’^n-2 $3 = °1 ■ S2 - °2 ■ $1
=> s3 =(-P) (p2 +2q)-q (-p) => S3 =-p3-2pq + pq

=> |s3 =-p3-pqj.


Sn = °1 ’Sn-1-o2 Sn-2 S4 = ■ S3 - O2 • S2
=> S4 =(-p) (-p,33 -pq)-q- (p2 +2q) => S4 =p4 -í-p2q - p2q - 2q2

=> S4 =p4 -2q2 => la4+b4 =p4-2q2|.

Resposta: a4 +b4 4 o_2


P -2q .

Exemplo Resolvido 261: Seja x + — = 3 , então x5


+ x5’
x

1 1
Resolução: Note que do enunciado x + — 3 e x — = 1, ou seja:
x
,0
r "i i
Oi = 3, <52 = 1, Sq = X° +1 — I => So = 1 +1 => Sq=2,

=> S.| = G-| => S-| = 3 .


276 8 Somas de Newton

Assim, podemos escrever para n = 2, n = 3, n = 4en = 5:


S2 = rq ■ S2_-| — o2 ■ S2_2 => S2 = <q ■ S1 — g2 ■ Sq S2 = 3 ■ 3 1-2
=> S2 = 9 - 2 => |S2 = 7|.
Sn = CT1'Sn_i -o2 • Sn_2 => S3 = o-, ■ S2 -a2 ■ S-j => S3=3-7-1-3

S3 =21-3 => js3 = 18^.


Sn = o, Sn_i-o2-Sn_2 => S4 = cq-S3-c2 S2 => S4 =3-18-1-7
=> S4 =54-7 => |S4 =47|.

Sn = °1 Sn-1 -o2 -Sn_2 => S5 = <q-S4-a2 ■ S3 => S5 3-47-1-18


=> S5 =141-18 => |Ss = 123|.

Resposta: a5 +b5 =123.

Vejamos agora para três raizes, ou seja, para um polinômio do terceiro grau.
Seguiremos o mesmo raciocínio anterior.

8.3) Somas de Newton para Três Termos


bonsidere o polinômio P(x) = ax3 + bx2 +cx + d, cujas raízes são q, r2 e r3.

Note que, se substituirmos q na equação, o resultado será zero, visto que q é


raiz de P(x). Então, temos:

P(q) = a ■ q3 + b ■ q2 + c ■ q + d = 0. (eq1)

Note também que, se substituirmos r2 na equação, o resultado será zero, visto


que r2 é raiz de P(x). Então, temos:
P(r2) = a - r2 + b ■ r2 + c • r2 + d = 0. (eq2)

Se substituirmos r3 na equação, o resultado será zero, visto que r3 é raiz de

P(x). Então, temos:

P(r3) = a • q3 + b • q2 + c ■ q + d = 0. (eq3)

Somando (eq1) com (eq2) com (eq3), temos:


a ■ (q3 + r23 + r33) + b • (q2 + r22 + r32 ) (q + r2+r3) + 3d = 0. (eq4)
Os Segredos da Álgebra para IME/ITA/OLIMPÍADAS 277

Conclusões:
1) Como q3 + r2 +r3 tem grau 3, chamaremos de S3 (soma "3" de Newton).

Assim S3 = q3 + r2 + r3 .

2) Como q2 +r2 +r2 tem grau 2, chamaremos de S2 (soma "2" de Newton).

Assim S2 = q2 + r2 + r2 .
3) Como q +r2 +r3 tem grau 1, chamaremos de St (soma "1" de Newton).
Assim Si = q1 + r2 + r3 => S, = q + r2 + r3 .
4) Como 3 tem grau 0, chamaremos de So (soma “0" de Newton). Assim
So = q° + r2° + r3° => So = 1 +1 +1 => So = 3 .

Então, podemos escrever: a ■ S3 + b • S2 + c ■ S! + d • So = 0 .

Generalização:
Se multiplicarmos (eq1) por q , temos:
a-r* + bq3 +c q2 + d-q = 0. (eq4)

Se multiplicarmos (eq2) por r2 , temos:

a-r2+b r2+c-r2 +d r2 . (eq5)


Se multiplicarmos (eq2) por r3 , temos:

ar3+b-r3+cr3+d-r3. (eq6)

Somando (eq4) com (eq5) com (eq6), teremos a (eq7):


a • (n4 + Í2 + r34) + b • (r,3 + r2 + rf) + c ■ (q2 + rf + r32) + d • (r, + r2 + r3) = 0.

Conclusões:
1) Como r4 +r2 +r3 tem grau 4, chamaremos de S4 (soma “4" de Newton).

Assim S4 = q4 + r2 + r3 .

2) Como q3 +r2 +r3 tem grau 3, chamaremos de S3 (soma "3” de Newton).

Assim S3 = q3 + r3 + r3 .

3) Como q2 +r2 +r3 tem grau 2, chamaremos de S2 (soma "2" de Newton).

Assim S2 = q2 + r2 + r3 .
278 8 Somas de Newton

4) Como S, = r,1 + r2 + r3 => S, = r, + r2 + r3 tem grau 1, chamaremos de S,

(soma "1'' de Newton). Assim S1 = r,1 +r2 + r3 => S-j = +r2 +r3 .

Então, podemos escrever: a ■ S4 + b ■ S3 + c ■ S2 + d ■ S-, = 0.

Assim, seguindo o mesmo raciocínio para r2, r3,..., r" 3 , temos:

Se multiplicarmos (eq1) por r"-3 , temos:

a ■ q3*"’3 + b • rf4"'3 + c ■ r^”’3 + c ■ r10+n-3 = 0


=> a r" +b-rf*1 + c r"-2 + d rf"3 = 0. (eq8)

Se multiplicarmos (eq2) por r2 3 , temos:

a • r23+n~3 + b • r2+n-3 + c ■ r^+n‘3 + c • r£+n-3 = 0


=> a r2+b r2"1+c-r2-2 + d r2-3 = 0. (eq9)

3e multiplicarmos (eq3) por r3 3 , temos:


a _3+n-3 , u r2+n-3 , r1+n-3 . r0+n-3 n
a • r3 + d • r3 + c • r3 t c • r3 —u

=> a r3+b r3-1+c r3~2+d r3~3 =0. (eq10)

Somando (eq8) com (eq9) com (eq10):


a ■ (rf + r2 + r3") + b • (r?'1 + r2"’1 + r3"-1) + c • (r^2 + r2n'-
' 2+r3"-2) +

+ d-(r1n’3 + r2~3 +•rJ-3) = 0. (eq11)

Conclusões:
1) Como r" +r2 +r3 tem grau n, chamaremos de Sn (soma “n” de Newton).

Assim Sn = r" + r2 + r3 .
2) Como rf1"1 + r2~1 + r3"1 tem grau n - 1, chamaremos de Sn_1 (soma “n - T

de Newton). Assim Sn_1 = r, + r2 “1 + r3 .

3) Como r"'2 + r2“2 + r3~2 tem grau n - 2, chamaremos de Sn_2 (soma "n-2"

de Newton). Assim Sn_2 = r"-2 + r2~2 + r3“2 .


Os Segredos da Álgebra para IME/ITA/OLIMPÍADAS 279

4) Como qn 3 +r2 3 +r3 3 tem grau n- 3, chamaremos de Sn_3 (soma "n-3"

de Newton). Assim Sn_3 = r"~3 + r2“3 + r3“3 .


Então, podemos escrever: a • Sn + b ■ Sn_, + c ■ Sn_2 + d ■ Sn_3 = 0 .

8.4) A Notação Sigma


Considere o polinõmio P(x) = ax3+bx2 +cx + d. cujas raizes são q, r2 e r3 •
Podemos escrever:
b 2
P(x) = ax3 + bx2+cx+ d => P(x) = a íx3 +-x c i)’ (eq1)
a a

Por outro lado, a forma fatorada de P(x) é:


P(x) = a (x-r,) (x-r2)-(x-r3) =>
P(x) = a [x3 - (r, + r2 + r3)x2 +(r, r2 + r, r3 + r2 -r3)x-rt -r2 -r3]. (eq2)

Assim, igualando (eq1) com (eq2), temos:


x3 + bx2+£x+dV
P(x) = a-
a a aJ
= a-[x3 -(q +r2 +r3)x2 + (q r2 + q -r3 +r2 r3)x-q r2 -r3]

Por comparação, podemos escrever:

-(q +r2 +r3) = - => q+r2+r3=-—. (eq3)

q r2+q-r3+r2 r3 =-. (eq4)


a
/ \ d d . cX
-(h r2 r3) = - => ri-r2-r3 = --. (eq5)

Conclusões:
1) Como r,+r2+r3 tem grau 1. chamaremos de o, (soma "1” de Girard).
Assim a, = q +r2 +r3.
2) Como r, r2+q r3+r2 r3 tem grau 2, chamaremos de o2 (soma “2” de
Girard). Assim o2 = q ■ r2 + q ■ r3 + r2 ■ r3 .
3) Como q ■ r2 ■ r3 tem grau 3, chamaremos de o3 (soma “3” de Girard). Assim
o3 = r, ■ r2 ■ r3 .
280 8 Somas de Newton

Então, podemos escrever a (eqõ):


P(x) = a (x3 - o, x2 + cr2 ■ x-cr3 j = 0 => x3 - o1 • x2 + a2 ■ x - 03 = 0.

Assim, juntando as duas notações, temos a generalização:


Sn ~ a1 ■ Sn-1 + °2 ■ Sn-2 _ °3 ■ Sn-3 = 0 ou
Sn = °1 ' Sn-1 - °2 ' Sn-2 + °3 ' Sn-3 • (ecI6)

Generalização para um Polinômio de Grau n.


Para um polinômio qualquer P(x) = an•xn + an_, • xn + • ■ ■ + a, • x + ag de grau
n, temos:
Sn “ ' Sn-1 + °2 ’ Sn-2 + ’ ’' + H)” °n ' S0 = 0 ■

Onde:
Sn = r,n + r2 + ••• + r"
= r1+r2+-- + rn
°2 = r1 r2 +r1 ’r3 +--- + rn-1 ’rn

(..................)
°n =r1 r2 ••• rn •

Observações:
1) Se o polinômio tem 2 raízes, então precisamos de duas somas para
encontrar as demais (nesse caso, precisamos de So e S,). Se o polinômio
tem 3 raizes, então precisamos de três somas para encontrar as demais
(nesse caso, precisamos de s0, St es2). E assim por diante.
2) Note que So sempre será igual à quantidade de raizes, visto que
So = <i° + r2 + + rn =* so = 1+1+--+1 So = n.
"n" vezes
3) Note que S, sempre será igual a cr,, visto que
S, = rj1+r2+... + ç! => S, = r,+r2+ ... + rn S, = a,.

Exemplo Resolvido 262: Prove que


axn+2+byn+2 =(x + y)(axn+1+byn+1j -xy(axn + byn)
Os Segredos da Álgebra para IME/ITA/OLIMPÍADAS 281

Resolução: Considere o polinômio genérico P(t) = A • (t2 - a, • t + a2 I, cujas

raizes são x e y.

Note que, se substituirmos x na equação, o resultado será zero, visto que x é


raiz de P(t). Então, temos:

P(x) = A-(x2 - cr-| ■ X + ü2 j = 0 => X2 - o-j • X + o2 = 0. (eq1)

Note também que, se substituirmos y na equação, o resultado será zero, visto


que y é raiz de P(t). Então, temos:

P(y) = A(y2 crry + cr2l = 0 => y2 - a-, • y + cr2 = 0. (eq2)

Se multiplicarmos (eq1) por axn , temos:


x2 -o-l • X + a2 = 0 => a xn+2-a arxn+1 + a a2 xn =0. (eq3)

Se multiplicarmos (eq2) por byn , temos:


y2-o1y + a2=0 => byn+2-b-o1-yn+1 + b-o2-yn =0. (eq4)

Somando (eq3) com (eq4):


axn+2 +byn*2 - a, ■ (axn+1 + byn+1 ) + a2 ■ (axn + byn j = 0
=> axn+2+byn+2 = (axn+1 +byn+1) -o2 -(axn +byn)

axn+2 + byn+2 := (x + y) (axn+1 + byn’1) - xy (axn + byn).

Como queríamos demonstrar.

Exemplo Resolvido 263: Prove que


axn+3 + byn+3 + czn+3 = (x + y + z) (axn+2 + byn+2 + czn+2) -

-(xy + xz + yz)(axn+1 + byn+1 + czn+1l + xyz(axn + byn + czn).

Resolução: Considere o polinômio genérico P(t) = A (t3 - a-j ■ t2 +a2t-a3).

cujas raízes são x, y e z.


282 8 Somas de Newton

Note que. se substituirmos x na equação, o resultado será zero, visto que x é


raiz de P(t). Então, temos:

P(x) = 0 => x - cq1• x2 + a2x-a3


3 -a
X3 0. (eq!)

Note também que, se substituirmos y na equação, o resultado será zero, visto


que y é raiz de P(t). Então, temos:

P(y) = 0 => y3-o,-y2+a2-y-a3 =0. (eq2)

Note também que, se substituirmos z na equação, o resultado será zero, visto


que z é raiz de P(t). Então, temos:

P(z) = 0 => z3 -o, z2 + o2 z-a3 = 0. (eq3)

Se multiplicarmos (eq1) por ax11, temos:


X3 -a, X2 + a2 X-a3 = 0

=> a • x11*3 - a ■ a,r• xn+2 + ao2-xn+1 -aa3 xn =0.


-aa (eq4)

Se multiplicarmos (eq2) por byn , temos:


3 ?
y - a, ■ y + a2 • y - a3 = 0
=> b yn~3-b a, yn+2+b-a2 yn+1-b a3 yn = 0. (eq5)

Se multiplicarmos (eq3) por czn , temos:


z3 - a, • z2 + a2 ■ Z - a3 = 0
=■ c ■ z”4'3 - c ■ a, • zn+2 + c • a2 ■ zn+1-c- G)3 • zn = 0. (eq6)

Somando as três últimas equações:


axn+3 + byn+3 + cz11’3 - a, ■ (axn+2 + byni+2 ;zn+2) +

+ a2 (axn+1 + byn+1 +czn+1 )-°3- (axn +byn +czn) = 0

«2n+3
=> axn,3+byr"^3+cz n+3=a1- (axn'2+byn+2+czni2j-

-a2-(axn+1+byn+1 4-CZ:n+1 j+a3 (axn+byn+czn j


Os Segredos da Álgebra para IME/ITA/OLIMPÍADAS 283

axn’3 + byn-3 + czn*3 = (x + y + z)(axn’2 + byn>2 :zn+2)-

-(xy + xz + yz)(axn+1 +byn+1 + czn+1 j + xyz (axn + byn + czn)

Como queríamos demonstrar.

Exemplo Resolvido 264: Sabendo que P(x) = 2x3-12x2-4x+ 6 tem raizes

a, b e c, calcule a5 + b5 + c5 .

Resolução: Do enunciado, temos:


-12 (_ 4) _______ 6
ai=-y- Iot = 6l, a2 -y- => |o2 = -2|, a3 = -
2
=. |o3 = -3|. So =a°+b0+c° => So =1 + 1 + 1 => |S0 = 3|.

Si=a1+b’+c1 => St = ot =» |St = 6|, S2=a2+b2+c2

=> S2 = (a + b + c)2 - 2(ab + ac + bc) => S2=c2-2o2

=> S2=62-2 (-2) => S2=36 + 4 => |S2 = 40|.

Assim, podemos escrever, para n = 3, n 4 e n=5:


Sn = oi • Sn_.| - cr2 ■ Sn_2 + o3 • Sn_3
S3 = o-| • S3_i - cr2 ' ^3-2 + ct3 ' S3-3
=> S3 = o1 ■ S2 - g2 • St + a3 • So => S3 = 6 ■ 40 — (-2) ■ 6 + (-3) - 3
=> S3 = 240 +12 - 9 => |S3 =243|.
54 = Oi • S4_i - a2 • S4_2 + ct3 • S4-3
=> S4 = a-, ■ S3 - a2 ■ S2 + a3 ■ St => S4 = 6 • 243 -(-2)-40 + (-3) • 6
=> S4 =1458 + 80-18 => |S4 = 1520|.

55 = Gt S5-I _ c2 ' S5-2 + a3 ■ S5-3


=> S5 =arS4 -a2 S3 +o3 S2 => S5 = 6-1 520-(-2)-243 +(-3)-40
=> S5 = 9 120 + 486-120 => |S5 = 9 486|.

Resposta: a5 + b5 + c5 = 9 486 .
284 8 Somas de Newton

Exemplo Resolvido 265: Sabendo que as raizes de P(x) = 2x3 + 5x2 + 4x-4

são a. b e c, determine a3 + b3 + c3.

Resolução: Do enunciado, temos:


5 _(-4)
o2 = + - => |g2 = 2|, o3 = => |°3 = 2|.
°1=~2 2
S0=a°+b° + c° => So =1 + 1 + 1 => |S0 =3|.

S, = a1 + b1 + c^ => S^ = o-, => S1 = — — , S2 = a2 + b2 + c2

=> S2 = (a+ b + c)2-2(ab + ac +bc) => S2=o2-2o2

=,s2=Í-5]2-2.2
-2 2 ^
=> s = ^-4^
S22 = S22 =
—-4 => s = ^ => s2 =-
2 l 2) 2 4 2 4 2 4

'Xssim, podemos escrever, para n = 3:


>n = 01 Sn.i - a2 ■ Sn_2 + o3 ■ Sn_3 => S3 = o1 ■ S3_j - o2 S3_2 + c3 ■ S3_3

S3 - o1 ■ S2 - o2 ' S, + c3 ■ Sq => S3
f 5^ 9 y f 5^ 2 3

45 45 -45 + 88 - S3 = ^
S3 =------+ 5 + 6 => S3 =-------- f 11 => S3 =
8 8 8 3 8

Resposta: a3 + b3 + c3 = .

Exemplo Resolvido 266: Se a + b + c = 0 , prove que a3 + b3 +c3 = 3abc.

Resolução: Sejam a, b e c as raízes da equação polinomial


P(x) = x3 -a, ■ x2 + o2 • x-o3 , então temos:

= a + b + c =5 |o1 = 0|,

So = a° + b° +c° => So =1 + 1 + 1 => |Sq = 3|.

S, = a1 +b1 + c1 zx> S1 = o1 => |S1 = o|. S2=a2+b2+c2

=> S2 = (a + b + c)2 -2(ab + ac + bc) S2 = — 2n2

=> S2=(0)2-2 a2 =x> |S2 = -20^].


Os Segredos da Álgebra para IME/ITA/OLIMPÍADAS 285

Assim, podemos escrever, para n = 3:


®n = ai ' Sn-1 “ °2 ■ Sn_2 + C3 ■ Sn_3
=> S3 = a-j ■ S3_-| - o2 ' S3-2 + a3 ’ S3-3
S3 ~ <r-| -82 —o2 ■ S-j + 03 • Sq S3 ~ 0■ (— 2o2) — ríQ -0 + 03 3
S3 ~ 303 ^83 = 3abc|.

Resposta: a3 + b3 + c3 = 3abc .

Vamos aos problemas propostos. Se você entendeu a resolução de todos os


exemplos resolvidos, vai conseguir resolver todos os exercícios desta lista.
Divirta-se com esses problemas e, se não conseguir, veja a resolução lá no
capitulo de resoluções!
286 8 Somas de Newlon

Problemas Propostos

Questão 8. 1

Se x + — = 1, determine o valor de 5/x5 _1_


+ x5 '
x V

Questão 8.2 (Stanford-2010)

Se x2 + -y = 7 , determine o valor de x5 + X5'


x4

Questão 8.3
1
Se x + - = V2 . determine o valor de x7 + X7'
x

Questão 8.4 (Stanford-2006-Modificada)


Sejam x + y = aexy = b. Calcule x6 + y,66 .

Questão 8.5
,5
Calcule o valor de 5-721
2

Questão 8.6
Se x2 - 3x +1 = 0 , determine o valor de x9 + x7 -9

Questão 8.7
Se a + b + c = 0, prove que a3 + b3 + c3 - 3abc .

Questão 8.8
Se a + b + c = 0, prove que a4 + b4 +c4 = 2(ab -i-bc + ca)2 .

Questão 8.9
(a2+b2+c2)2
Se a + b + c = 0, prove que a4 + b4 + c4 =
2
Os Segredos da Álgebra para IME/ITA/OLIMPÍADAS 287

Questão 8.10
Se a + b + c = 0, prove que a5 +b5 +c5 = -5abc(ab + ac + bc).

Questão 8.11
Sejam a, b e c inteiros positivos, tais que a = b + c . Prove que a4 + b4 + c4 é
o dobro do quadrado de um inteiro positivo.

Questão 8.12
a5+b5+cs
Se a + b + c = 0, determine o valor de
3abc(ac + bc + ab) '

Questão 8.13
(a2 + b2 + c2) ía3+b3+c3) a5 + b5_5
Se a + b + c = 0, prove que -------- - ------- - ■ - ------- -------- - =------- - -------

Questão 8.14 (Harvard/MIT-2008)____________________________


Sejam a, b, c são números reais não nulos, tais que a + b + c = 0,
a3 + b3 + c3 = a5 + b5 + c5 . Determine a2 + b2 + c2.

Questão 8.15
Se a + b + c = 0, prove que
2
^a3+b3 + c3j2 a4+b4 + c4 as + b5 + c5
2 5

Questão 8.16
Se a + b + c = 0, prove que a6 + b6 +c6 = 3a2b2c2 -2(ab + bc + ca)3.

Questão 8.17
Se a + b + c = 0, prove que

a6+b6+c6 =
(a3+b3 + c3)2
3 ’+
(a2+b2

4
4
288 8 Somas de Newton

Questão 8.18
Se a + b + c 0, prove que a7 + b7 + c7 = 7abc (ab + bc + ca )2.

Questão 8.19 (Croácia 2001)


a7+b7+c7
Se a + b + c = 0, determine o valor de
abc (a4 +b4 +c4 l

Questão 8.20
Se a + b + c = 0, prove que
+ c2)(a5+b5+c5)

Questão 8.21
2 (a3 + b3 + c3j
a7+b7+c7 a2-rb2+c2
Se a + b + c = 0, prove que
7 2 3
Questão 8.22
a7+b7+c7 (a3+b3 + c3) (a4+b4+c4)
Se a + b + c = 0, prove que
7 " 3 2

Questão 8.23
Se a + b + c = 0, prove que
a8 +b8 -c8 = 2(ab + bc + ca)[(ab + bc + ca)3 -4a2b2c2] .

Questão 8.24
Se a + b + c = 0, prove que

4^a2+b2+c2j(a3+b3+c3) (a4+b4+c4j
a8+b8+c8 =
9 + 2

Questão 8.25
Se a + b + c = 0, prove que a9 + b9 + c9 = 3abc[a2b2c2 -3(ab + bc + ca)3i
Os Segredos da Álgebra para IME/ITA/OLIMPÍADAS 289

Questão 8.26
Se a + b + c = 0, prove que
,9.b9.:9 (a3 + b3+c3)3[3(a2 + b2 + c2)(a3 + b3 c3)(a4 +b4 +c4j

9 4

Questão 8.27
Se a+b+c+d=0, prove que a3 +b3 +c3 +d3 = 3(abc + abd + acd-i-bcd).

Questão 8.28
Se a+b+c+d=0, prove que
a4 +b4 +c4 + d4 = 2(ab + ac + ad + bc + bd + cd)2 -4abcd .

Questão 8.29
Se a + b + c + d = 0, prove que
, , , . (a2 + b2 + o2 + d2)2
a4 + b4 + c4 + d4 = ------------------ -4abcd.
2

Questão 8.30
Se a+b+c+d=0, prove que
a5 + b5 + c5 + d5 = -5(ab + ac + ad + bc + bd + cd)(abc + abd + acd + bcd).

Questão 8.31
Se a + b + c + d = 0, prove que
a5+bs+cs+d5 a2+b2+c2+d2 a3+b3.c3 + d3
5 2 3

Questão 8.32
a6+b6+C6 + d6 ai
Se a+b+c+d=0, prove que -------------------------= a7 - a4 — + -—
6 4 4 3 2

Questão 8.33
Se a + b + c + d = 0, prove que a7 + b7 + c7 + d7 = 7a3 (a2 -a4).
290 8 Somas de Newton

Questão 8.34

Se a + b + c + d = 0, prove que a7-±b>c? td-í = + .Éà


8 4 3

Questão 8.35
Prove que xn + yn = (x + y) (xn-1 + yn-1 j - xy (xn-2 + yn-2 j.

Questão 8.36
Mostre que. se x + y = 0 . então 2xn + 2yn = (x2 + y2j (xn“2 + yn-2).

Questão 8.37
Prove que x2n+1 -y'’2n+1=(x2 + y2)(x2n-1 ,2n-3)
-y’2"-1)-x2y2(x2n-3 -y

Questão 8.38 (Putnam 1959)


,2n+1 _ 1 _1_ 1 1
•rove que x' = x2 + X2. x2n-1 X2n 3
x2n+1 x2n-1 x2n-3

Questão 8.39
Prove que xn + yn + zn = íx',n-1 + yn~1 j(x + y + z)-
-(xy + xz + yz)(xn~2 + yn-2 + zn-2j + xyz(xn*3 + yn-3 + zn‘3)

Questão 8.40
Mostre que, se a + b + c = 0, então an + bn + cn =
(a2 + b2 + c2 )(an’2 + bn'2 + o"’2) (a3 + b3 + c3 )(an~ bn-3 cn~3)
2 + 3

Questão 8.41 (AIME-1990)


Determine ax5 +by5 se os números reais a, b, x e y satisfazem as equações

ax + by = 3. ax2 +by2 = 7, ax3 +by3 = 16 , ax4 +by4 = 42 .


Os Segredos da Álgebra para IME/ITA/OLIMPÍADAS 291

Questão 8.42
Sejam a e b números reais não nulos tais que x e y satisfazem o sistema
ax + by = 2
ax2 +by2 =20
ax3 + by3 = 56
ax4 + by4 = 272

Determine o valor de ax5 +by5 .

Questão 8.43 (CN-1988)______________________________________


A equação do 2° grau x2 - 2x + m , m < 0 tem raizes x, e x2. Se

xi'2 + x2-2 = a e x?’1 + x2~1 = b , então x" + x2 é igual a:


a) 2a + mb
b) 2b-ma
c) ma + 2b
d) ma-2b
e) m(a-2b)

Questão 8.44
(a + b-2c)2 (a + c-2b)2 +(brc-2a):
Se a + b + c = 0, qual o valor de
a2 +b2 +c2

Questão 8.45 (IMO-Longlist-1988/AHSME-1975)_________________________


Se p, q e r são as raizes distintas da equação x3-x2 + x- 2 = 0, determine o
valor de p3 +q3 +r3.

Questão 8.46 (Putnam-1939-Modificada)


As raízes de x3 + ax2 + bx 0 são a, p e y. Determine a3 + p3 + y,33 .

Questão 8.47 (Singapura-2014)


Se a e p são as raizes da equação 3x2 + x -1 = 0 , onde a > p. determine o
a P
valor de — + — .
P a
292 8 Somas de Newton

Questão 8.48 (Turquia-2004)


Qual a soma dos cubos das raizes da equação x3 - 2x2 - x +1 = 0 ?

Questão 8.49 (Turquia-2012)


Sejam a, b e c são as raizes da equação x3 - 3x +1 = 0 . Determine
a8+b8-c®.

Questão 8.50 (AIME-2008)___________________________________________


Sejam r, s e t as três raizes da equação 8x3 +1001 x + 2008 = 0 . Determine

(t + r)
(r + s)3+(s +1)3 + (t+ 3.
r)<

Questão 8.51 (AHSME-1951)


1 1
Seres são as raizes da equação ax2 + bx + c = 0 , então o valor de
7+?
é:
b2 -4ac
a) b2 - 4ac b)
2a
b2 -4ac b2 - 2ac
c) d)

Questão 8.52 (AHSME-1955)


Se r e s as três raizes da equação x2 - px + q = 0 , então r2 + s2 é:

a) p2 + 2q b) p2 - 2q c) p2 + q2

d) p2 -q2 e) p2

Questão 8.53 (Harvard/MIT-2008)


A equação x3 - 9x2 + 8x + 2 = 0 , possui três raizes p, q, r. Determine
1 1 _1_
2 + 2 ~ 2 '
p q r

Questão 8.54 (BMO-2007)


14 + 20074 + 20084
Determine o valor de
12 +20072 + 20082 '

Questão 8.55 (Singapura-2014)


Os Segredos da Álgebra para IME/ITA/OLIMPÍADAS 293

20143 -20133 -1
Determine o valor de
2013-2014

Questão 8.56 (Stanford-2013)


Sejam a e b as soluções de x2 - 7x +17 = 0 . Determine a4 + b4 .

Questão 8.57 (Stanford-2007)


Sejam a, b, c as raizes da equação x3 -7x2 - 6x + 5 = 0 . Determine o valor de
(a + b)(a + c)(b + c).

Questão 8.58 (Stanford-2007)


Se r + s +1 = 3, r2 + s2 +12 = 1 e r3 + s3 +13 = 3 , calcule r • s • t.

Questão 8.59
Sejam a. b e c inteiros positivos, tais que a = b + c. Prove que a4 + b4 + c4 é
o dobro do quadrado de um inteiro positivo.

Questão 8.60
,10
Determine o valor de

Questão 8.61 (CN-1998)


/—J997 ,1997 , y—\1997 ’ , /-\1997
(2 +Vã) + (2-V3) (2 +Vã)
Sejam x = e y , o
2 73
valor de 4x2 -3y2 é:
a) 1 b) 2 c) 3 d) 4 e)5

Questão 8.62 (Áustria Competição Federal-2013)


Para três números a, b e c, seja Sn = an + bn + cn, sabendo que S, = 2, S2 = 6
e S3 = 14. Prove que, para todo número natural n > 1, temos
|sn _ Sn-i ■ Sn+i| = 8.
294 9 Respostas e Sugestões

Capítulo 09 - Respostas e Sugestões


5
a 12
Capítulo 01 - Potenciação 1.21) -— .
b
1.1) C. 3 _5
1.22) 4 a 4 b 6 .
1.2) 256abcd.
1.23) D.
1.3) 4a2 mm n" gb2mm.nn ,4gc2mm nn 1.24) A.
1.4) C. 1.25) E.
1.5) W40. 1.26) mm~1 (m2+l).
1.6) 10n‘1.
1.27) a+r|2 •b-3 •c<
1.7) 9.
, ,,nk+m+k n2+n3
1.8) (x2) 1.28) a 2
nn~1-n
1.9) (ab)a'b.
1.29) a n"1
1.10) ab(a’1’•b^"
.
Capítulo 02 - Radiciação
1.11) n = 4.
>310 080
1.12) 2'
2.1) 7^.
.
1.13) x,x*
x . 2.2) 1999®.
f x*~2-x2 2.3) D.
1.14) X 2.4) D.
o 50 o50 2.5) r = 4 e s = 5.
1.15) a3 -b3 .
2.6) 14.
,100 .100
1.16) a3 .b4,UÜ .c5’00
2.7) È.
n(n+1)( 2n+1) a
1.17) X 6 . 2.8) | .
n(n+1)(n+2)
1.18) x 3 2.9) a.
nfn^l)
,2 2.10) 18\/a79
2
1.19) x‘ 2.11) an.
n(n+1)(n+2)(n+3)
2-12) Ç.
1.20) X 4
2.13) 1.
Os Segredos da Álgebra para IME/ITA/OLIMPlADAS 295

2.14) 8.
2.15) a^aa+b ■ b .
f ®2*1 i
2.38) x' 6" i
2.16) 2.
2.17) 1. í líz1)
2.39) x 10"
2.18) (ip. 2.40) x(x‘1) x.

2.19) 5. M
fi-Vã '| 2.41) x 9 .
J 3 >
2.20) 33 2.42) B.
2.21) B. 2.43) E.
2.22) am+2.
2.44) 3.
2.45) 7.
2.23) 168 ^6-^49. 2.46) 5.
2.24) 24/ZÍ7 2.47) 5.
5 1 2.48) 1 + VÍ+ã.
2.25) a 6 b6 . 2.49) 6.
2.50) 46.
f-Ê] 2.51) n + a + x.
2.26) 8J.
2.52) 4.
2.27) x Vx3 .
2.53) ^.
2.28) B.
2.29) E. 2.54) Demonstração.
f n+1) 2.55) 5.
2.30) x 2 >. 2.56) 9.
2.31) 729. 2.57) 49.
2.32) 2250. 2.58) 2.
43 ___ 3
2.59) - .
2.33) 260 . ' 8 '
17
2.60) —.
2.34) 780 . x
2.35) C.
2.61) -lí— .
2.36) xl 3
t'] < 2.62) 2.
x

Í4n-l'| 2.63) 243.


2.37) x
4n I 2.64) 2o1V2O17 .
2.65) ^.
296 9 Respostas e Sugestões

2.66) 10. 3.12) V5-1 .


2 67)243 3.13) 2.
2.68) C. 1
2.69) 8. 314)3-
2.70) 75.
4 3.15) 1 + 75.
2.71) 525 . 3.16) 272 + 277 + 3 .
5 3.17) 77 + 7b.
2.72) 216
3.18) A.
^2 3.19) B.
2.73) 7 7 3.20) C.
5-^2 3.21) B.
2.74) 2 4 . 3.22) E.
2.75) 77 . 3.23) 2V2 + 7Õ + 2 75 + 727 .
2.76) 1313. 3.24) 14.
3.25) 1 + 75.
Capítulo 03 - Racionalização 3.26) 2-75.
3.27) 9 + 475 .
3.1)
cW 3.28) 75-1.
b
3.29) 3+275.
3.2)
73-75 3.30) 5-77 .
6
3.3) B. 3.31) 3-77 .
3.4) 75 + 75 - 74. 3.32) 75-2.
372-275+750 3.33) 2 + 75.
3.5)
4 3.34) 1+275.
3.6) 75 + 775 3.35) 275-3.
9 3.36) x = 3.
5 + 2770 3.37) M = 2.
3.7) 3.38) 7.
30
3.39) B.
6(4072-975+3577-12770)
3.8) 3.40) -277 .
215
3.41) -10 .
3.9) 72 + 74 .
3.42) -10 .
3.10) -2-275 .
3.43) 675-8.
3.11) -3-275. 3.44) 10.
Os Segredos da Álgebra para IME/ITA/OLIMPlADAS 297

3.45) 4.
3.46) B. Capitulo 05 - Produtos Notáveis
3 47) A.
3.48) A. 5.1) 711.
3.49) D. 5.2) 69.
3.50) B. 5.3) 5.
3.51)
1+ 75
2
3 + 77 5.5) C.
3.52)
2 5.6) 4.
3.53) Demonstração. \2
5.7) (x2 - n + 2] .
3.54) Demonstração.
3.55) 1. 5.8) X = 9
3.56) E. 5.9) B.
3.57) 2.
3.58) 75. 5.10)
3.59) B. 5.11) D.
3.60) 7Í3 . 1
5-12) -.
3.61) Demonstração.
3.62) Demonstração.
3.63) Demonstração. 5.13) .
3.64) 72 + 73. 5.14) 1.
3.65) 1 + 72 + 75. 5.15) Demonstração.
3.66) Demonstração. 5.16) 23.
3.67) 936 18
5.17) y .
3.68) 1+ 73 .
3.69) 73-1. k4 +24k2 +16
5.18)
3.70) 1 + 72 . 4k3+16k
3.71) 828. 5.19) (4x2 -l](2x + 1)(2x-1).
3.72) D.
5.20) 2100.
5.21) 199-197-195-...-3.
Capítulo 04 - Expressões
5.22) 5 993 002.
Algébricas
5.23) a-b.
5.24) a-b .
4.1) x3 -2x2 -6x + 27 . 5.25) Demonstração.
4.2) 3xjxy . 5.26) A.
298 9 Respostas e Sugestões

5.27) A. 5.56) ^2.


5.28) 4d2. 5.57) 5778.
5.29) a18 -b18. 5.58) Demonstração.
5.59) E.
5.30) A.
5.60) C.
5.31) -3.
5.61) C.
5.32) D.
5.62) 25.
5.33) B.
5.63) A.
5.34) B.
5.64) C.
5.35) —. 5.65) 2525.
a2b2
5.66) 123.
5.36) C. 5.67) D.
5.37) (a + b)(a-b)2. 5.68) D.
5.38) 3999. 5.69) -a3 +3ab-3c.
e 43 5.70) 753.
5.39) —.
63 ' 5.71) 4.
91 5.72) 5.
5.40) ----- . 5.73) 4.
136 '
5.74) Demonstração.
(a + b)4 =k2 +4kx + 4x2 e
_ 2414
5.41) 5.75) —- .
(a-b)4 =k2 -4kx + 4x2 97
on+1' +a2nb2" +b2"1
1 a
5.42) xy = - 5.76)
' 6' a2 + ab + b2
k4 +24k2+16 5.77)
5.43) a) 0. b) 0.
4k3+16k
c)1. d) a + b + c.
5.44) 5100 +57s + 5;!5O + 525 + 1.
5.45) 992.
5.78) Demonstração.
5.46) 20.
5.79) Demonstração.
5.47) 41.
5.80)
5.48) D.
a) 0. b) 0.
5.49) C.
c) 0. d)1.
5.50) E.
5.51) -123. 5.81) Demonstração.
71 5.82) Demonstração.
5.52) D.
5.83) ------ —.
5.53) ±123 . 2a+ b
5.54) 1
5.55) 116.
Os Segredos da Álgebra para 1ME/1TA/OLIMPÍADAS 299

,2
8ab2 9^(a-b)2 .(b-c)2+(c-a)2)
5.84)
(a2T^f 5.115)
2
5.116) 1.
5.85) X.
5.117) — 5.
5.86) D.
5.87) Demonstração. 5.118) 3a2b2c2 -2(ab + bc + ca)3.
5.88) 0. 5.119) Demonstração.
5.89) 0.
5.120) |.
5.90) 373.
5.91) 313.
2, se a + b + c»0 5.121)
5.92) O
-1, se a + b + c = 0
5.122) Demonstração.
5.93) 27. 5.123) Demonstração.
5.94) - 2. 5.124) Demonstração.
5.95) 9. 5.125) Demonstração.
5.96) Demonstração.
Capitulo 06 - Fatoração
597>5'
5.98) Demonstração. 6.1) 28
6.2) (4x2 +l)(2x + 1)(2x-1)
5.99)
6.3) E.
5.100) 4 030 057.
5.101) 3. 6.4) (5a-3b)(x-y).
5.102) 5778. 6.5) (x-y-a)(x-y + a).
5.103) B.
6.6) (x-y)(8x-3).
5.104) 4.
5.105) A. : -r-9a3b2j
6.7) (4x2y3-9a3b2)(4x2y'3
5.106) B.
5.107) 2. 6.8) (3a-8)(a + 2)(5a2-22a .34)
5.108) x/5. 6.9) 3.
5.109) E.
5.110) 4. 6.10) Ç.
5.111) Demonstração.
6.11) A.
5.112) Demonstração.
6.12) B.
5.113) B.
6.13) í? •
b
6.14) 0.
300 9 Respostas e Sugestões

6.15) (ay + bx)(ab + xy). 6.35) (y2 - 2y)(y2 - 4y + 2).

6.16) (a-4)(a-2)(a + b)2. 6.36) A.


2 6.37) A.
6.17) (a + b) (c+d). 6.38) B.
6.18) (a + b-c)(a-b + c)(-a + b-rc)
6.39) T'
6.19) (a-4b-5c)(a-4b + 3c).
6.40) -34 .
6.20) (a2 + b - 2c2 )(a2 - 2b + c2). 6.41) Demonstração.
6.42)
6.21) (2a2 + 3b2 - 2)(a2 + 7b2 + 5).
a) 0. b) 0.
a-b c)1. d) a+b+c.
6.22)
a+b-c
6.43) 30.
a +b-c
6.23) 6.44) 0.
a-b+c
6.45) 1.
6.24) (a + b)(a + c)(b + c). 6.46) 0.
6.47) 0.
a3 (a2 +ab + b2j
6.48)
6.25) ,2
(a + b)' a) 0. b) 0.
c) 0. d) 1.
6.26) — + — .
b a 6.49) 10.
6.27) 2a2b2 -a-b. 6.50) a2 +ab + b2.
a2 +ab + b2 6.51) Demonstração.
6.28)
a 6.52) Demonstração.
4ab 6.53) (a2 -bc)(b2 - ac )(c2 -abj.
6.29) 1-
a2 + b2
3
6.30) Demonstração. 6.54) -.
6.31) D.
3
6.55) ^.
6.32)
(a + b-c)(a-b + c)(a + b + c)-
6.56) Demostração.
(-a + b + c). 6.57) Demonstração.
6.33) 6.58) Demonstração.

(a + b + c)(a + b-c)(a-b + c)- 6.59) (x-1)(x2 -5x-~5)-

•(-a + b + c). 6.60) (x + 2)(x2 + 2xt-5).


6.34) 104.
Os Segredos da Álgebra para IME/ITA/OLIMPiADAS 301

6.61) (x-1)(x + 1)(x + 4). 6.87) (2x + 3y+ 3)(9x-r8y).

6.62) (x + 4)(x2 +l). 2 +4x + 15).


6.88) (x2-4x + l)(x2
2
6.63) (3x-2)(x + 1)(x + 2). 6.89) (x2 -n + 2'

6.64) (x + 1)(x-2)(x + 3). 6.90) (x2 +7)(x2 +3x-2).


6.65) (x-1)(2x + 1)(x + 5).
6.91) (x2 -3x + l)(x2 -x + l).
6.66) (2x-3)(2x-1)(x-4).
6.92) (x2 -3)(x2 +2x-5).
6.67) (x + 1)(x + 2)(x + 3).
6.68) (x + 3)(4x2 -20x-3). 6.93) (x-1)(x + 1)(x-3)(x + 5).
6.94) (n2 - n+ l)(n2 + 5n + 7).
6.69) (2x-3)2(2x + 5).
6.95) (x2-5x + 2)(x2-2x + 2).
6.70) (x-2)(x2-2).
6.96) (x2 - 7x + 3)(x2 - x + 3).
6.71) (x-2)(x-1)(x + 1).
6.72) (6x + 1)(x2 +x + 2). 6.97) (n2+l)(n2 + 2n-2l).

6.73) (x-1)(x2 + 4x + 7) . 6.98) x(x-1)(x-2)(x-4).

6.99) (x2-2x + 3)(x2 + 2x + 3).


6.74) (x — 1)(x2 + 2x + 2) .
6.100) (x2-2x + õ)(x2+2x + 5).
6.75) (x + 1)(x-3)(x + 6).
6.76) Demonstração. 6.101) (x2 - 2x + 5)(3x2 + 6x +1).
6.77) (2x-3y + 5)(x + y-5).
6.102) (x2-2x + 5)(x2-4x + 13)
6.78) (1-x-2y)(3x + y + 5) .

6.79) (2x + y-2)(4x + 3y-3). 6.103) (x2 -2)(x2 -x-3) .

6.80) (x + y-4)(x + y + 1). 1 2


6.104) X =-2, x=-1x = --,x = -,
3
6.81) (2x-3y + 8)(2x + 5y-12).
6.105) (3x +1)(2x - 1)(x2 + 3).
6.82) (5x-2y + 1)(2x + y-7).
6.83) (x-8y-6)(3x +2y+ 1) . 6.106) (x + 1)(x-2)[x2 -3).

6.84) (x-3y + 5)(7x-y + 6). 6.107) (x-2)2(x-1)2.


6.85) (7x + y-1)(x + 3y + 2).
6.108) (x2 - 3x + 4)(3x2 +x-6).
6.86) (-x + 1)(3x + y-2).
302 9 Respostas e Sugestões

6.109) (5x-2)(x + 3)[3x2 +x + l) 6.130) (x + 1)(x2 -x + lj(3x2 +5x + 3j.

6.110) (x-2)(x + 3)(x + 4)(x-5).


6.131) (x- 1)(x +1)2 (x + 2)2.
6 111)
‘ _ x2n + 1 x,6n
(x2n + l)(x,4n 6.132) (x-2)3(x2 +4).
1 _ *3n + 1).
6.133) (x-2)(x + 2)4.
6.112) (x2+l)(x2 + x + lj.
6.134) (x-2y+ z + 5)(x + 2y + 2z + 1)
6.113)
(x + n)(x - 3n)(x2 -3nx + n2).
6.135) (2x-3y + z + 3)(x-y + 5z-3).
6.114)
(x + n)(x - 3n)(2x2 + nx + n2 j. 6.136) (x-y-z + 4)(-2x-y + 2z + 1).

6.115) (x-2)3 (2x + 3). 6.137) (x + y + 2z + 2)(2x-2y + z + 1).

6.116) (x2 +l)(2x2 -x + l).


6.138) (3x + 5y- 2)(x + 2y + 3z + 1).
6.117) (x +1)2 (x2 +1).
Capitulo 07 - Polinômios
6.118) (x-1)(x + 2)(x-3)(x + 4). Simétricos
6.119) (2x + 7)(x + 1)(x + 3)(x-5).
6.120) (4x + 1)(x2 +9](x-6). 7.1) 5xy(x + y)(x2 + xy + y2).

6.121) (a-2)(a-1)a(a+1)(a + 2) 7.2) C.


7.3) Demonstração.
6.122) (x - 2)(x-1)(x + 2)(x2 + 2).
7.4) 1.
6.123) x2(x-3)(x-1)(x + 1)(x + 2)2. 7.5) Demonstração.
7.6) Demonstração.
6.124) (x + 1)(x2 + l)(x2 -2).
7.7) (x + y)(x + z)(y + z).
6.125) (a-1)(a+ 1)z (a2+4a+ 7). 7.8) (x + y + z)(xy + xz + yz).
6.126) (1-x)(x2+l)(x2+x + l). 7.9) (x + y)(x + z)(y + z).
7.10) 3(x + 2y-3z)(y + 2z-3x)-
6.127) (x-2)(x-6)(x + 12)[x2 +x + l).
(z + 2x-3y)

6.128) (x + 2)[x2-2x-l)(x2 + x + l) 7.11) (a-b)(c-a)(b-c)(a + b + c).

7.12) (a-b)(a-c)(b-c)-
6.129) (3x-2)(2x + 1)(x-1)(2x-2x-1).
(a2 + b2 +c2 +ab + ac + bc).
Os Segredos da Álgebra para 1ME/ITA/OLIMPÍADAS 303

7.13) (a-b)(c-a)(b-c)-
Capitulo 08 - Somas de Newton

■ Xa3 + Xa2b + abc ■


ksim sim ) 8.1) 1.
8.2) ±123.
7.14) (a -b)(c-a)(b -c)(a + b + c). 8.3) 2.
8.4) a6 - 6a2b + 7a2b2 - 2a2b3 - a4b - 2b'
7.15) (a-b)(c-a)(b-c)-

8.5) 2525.
• X a3 + X a2b - 9abc ■ 8.6) 6621.
\sim sim )
8.7) Demonstração.
8.8) Demonstração.
7.16)(a-b)(c-a)(b-c)(ab + bc + ac) 8 9) Demonstração.
8.10) Demonstração.
7.17) (a-b)(c-a)(b-c)- 8.11) Demonstração.
8.12) “|-
•í X a3b + X a2b2 _3X a2bc'j.
\sim sim sim ) 8.13) Demonstração.
7.18) (x-y)(x-z)(y-z). 8.14)
7.19) b
8.15) Demonstração.
7ab Xa5 +3Xa4b + 5Xa3b2 • 8.16) Demonstração.
\sim sim sim ) 8.17) Demonstração.
7.20) (a Ebjfa + cKb + c)- 8.18) Demonstração.
8.19) |.
(a2 + b2 + c2 + ab + ac + bc)

8.20) Demonstração.
7.21) Demonstração. 8.21) Demonstração.
8.22) Demonstração.
7.22) Xa3b3-zLa4bc- 8.23) Demonstração.
sim sim
8.24) Demonstração.
7.23) 7(a+ b)(a + c)(b + c)-
8.25) Demonstração.
( A2 8.26) Demonstração.
• ^a2 + ]Tab +abc(a + b + c) 8.27) Demonstração.
k sim sim ) 8.28) Demonstração.
8.29) Demonstração.
7.24) 24xyz.
8.30) Demonstração.
7.25) 80xyz(x2 + y',2
8.31) Demonstração.
304 9 Respostas e Sugestões

8.32) Demonstração.
8.33) Demonstração.
8.34) Demonstração.
8.35) Demonstração.
8.36) Demonstração.
8.37) Demonstração.
8.38) Demonstração.
8.39) Demonstração.
8.40) Demonstração.
8.41) 992.
8.42) 20.
8.43) B.
8.44) 9.
8.45) 4.
8.46) -a3 + 3ab-3c.

8.47) .

8.48) 11.
8.49) 186.
8.50) 753.
8.51) D.
8.52) B.
8.53) 25.
8.54) 4 030 057.
8.55) 3.
8.56) -353.
8.57) 4.
8.58) 4.
8.59) Demonstração.
8.60) 123.
8.61) D.
8.62) Demonstração.
Os Segredos da Álgebra para IME/ITA/OLIMPÍADAS 305

Capítulo 10 - Resoluções

Capítulo 01 - Potenciação

Questão 1.1 (AHSME-1952) - Resposta: Alternativa C


Resolução: Chamando a expressão de E, temos:
2n+4-2-(2n) 2n-24-2-(2n) 2n Í24 -2)
E= => E = => E =
2 (2n+3) 2-(2n-23) 2n -24

2^14-4 -2-(2n)
24 -2 7
=> E = =>E = 1^ = E=—
24 16 16 2-(2n*3) 8

Questão 1.2_________________________________
Resolução: Chamando a expressão de E, temos:
2d
1 „\abcd
2^ ' 2b ■ 2c ■ 2d £ _ 2^8000
- E=(28)
2d
= 256abcd

Questão 1.3_________________________________
Resolução: Chamando a expressão de E, temos:
,mm nn
=> E = 22®2 mm h" .32b2'mm n" ,y2c2 mm nn
E = ^22a2 -32b2 -72c2 j
/ 0\a2mm-nn / 9\b2 mm-nn / 9xc2 mm-nn
=>E = (22) ■(32)
•í72)
^22a2 ,32b2 72c2 ) _4;a2-mmnn gb2 ■ rnm. nn 4gc2mmn"

Questão 1.4 (AHSME-1971) - Resposta: Alternativa C_________


Resolução: Seja x = -2k e chamando a expressão de E, então:
2-(2k+1) _ 2,-(2k-1)
(2k-1) + --2k _ 2-2k-1 _ 2*2k+1 + 2~2k

!X+1+2X =• E = — -2x-2 + 2x => E = 2XÍ— 1 -2 + 1


=> E = 2: -2
2 Í.22
306 10 Resoluções

=> E = 2xf--1 => E = -2x-2 => E = -2X'1

. 2“(2k+1) _ 2~(2k-i) + 2~*■2k _ _2-(2k+1)

Questão 1.5________________________________
Resolução: Chamando a expressão de E, então:

104 • 1516 ■ 3311 • 7717 - 8413


E=
520.1430 ■ 3O40 1128

(2-5)4(3-5)16(3-11)11-(7H)17-(4-3-7)13
E=
520 (2-7)30 (2-3-5)40 -1128

E.Z. ■ |54 -3|51620.


-[_516 311 -1111 ■ |?171-11I17
________
230.[75õ].240.340.|540 32l28
.413-313[z2l

q4-t26 ^16+11+13 g4+16 _y17+13 ^^11+17


E=
230 240 • 340 .520.540.730.^28

1
E= => E = —
.240.^.^.54°.73tf'.^ 2',40 540

104 -1516 -3311 ■7717 -8413


=> E =----- ------ e = -2!_ = 10~4°
(2-5)40 104° 520 .1430.3O40 -1128

Questão 1.6
Resolução: Chamando a expressão de E, temos:
5^ 2n 2 ■ 5n + 5 ■ 2n
5n-1 +2',n-1 5 2 25
E= => E = => E =
51-n+21 5 2 5-2n +2-5n
5n 2n 2n -5n

5n~1 + 2',n-1
£ 2" 25n +5-2n
E= E= 5 2 => E = 2 5
51‘n +21 2_ 5■2n + 2 ■ 5n
5n 2n 2n -5n
2 ■ 5n + 5 ■ 2n 2n -5n 2n -5n E = 2n'1 ■ 5n’1
=> E = =. E =
2-5 5-2n +2-5n 2-5
Os Segredos da Álgebra para IME/ITA/OLIMPÍADAS 307

5n~1
E = (2.5)' = 10'
51-n + 21

Questão 1.7
Resolução: Chamando a expressão de E, temos:
.nk+2 „nkti +, v
xnnkk x nk(x2-x + l)
X - xnk X (*2 -X+1)
E= => E = E=
,nk-2 - Xnk + xnk xn” (x“2 -x (x-2 - x
x1 -x"1 + lj

(x2 - x + l) (x2 -x + 1)
=> E = => E = => E = (x2 -x + lj-
G í 1
| 1- x + x2 | 1 - X + X2 J
X2

xnk+2 - xnk
+X 2
= X .
xnk-2 - X^"1 + Xnk

Questão 1.8
nk
x vezes xnvezes xkvezes
E = xnk + Xnk + ... + x"k -xn + xn +...+ xn ■ xk + xk + ...+ xk

X2k = x2nk’2n-2k
E = xnk • Xnk • xn ■ xn • xk ■ xk = x2r|k -x2n ■X'

2^nk+n+kj _ / o\nk-n+k
=> E = x

Questão 1.9
Resolução: Chamando a expressão de E, temos:
bavezes a a vezes

E = ab + ab + . ,. + ab bb+bb+... + bb => E = ab ba-bb-aa E = aa’b ba*b

bavezes aavezes
'~+ ab
ab-ab +... • bb + bbí... + bb = (ab)a+b .

Questão 1.10
Resolução: Chamando a expressão de E, temos:
b3vezes abvezes
ab ., E = ab'a‘1’ _(b+1)
1 -ba ba-... ba
E = ab • ab •... • aib => E = ab ba ■ba' ■ba
308 10 Resoluções

Questão 1.11 (Harvard-MIT-2012)


Resolução: Podemos escrever:

£ 27
2,2
' (224)
128 222"
2128 2V
-
22 28 = 222n'7 229
.2^ 2n-7 = 9 => 2n =16
2n = 24 .-. n = 4 .

Questão 1.12
Resolução: Chamando a expressão de E, temos:
"2015"vezes

35
E- (2>‘J E = 2(35r6 E = 23
2016 .,10080
E = 23

Questão 1.13
Resolução: Podemos escrever:
x vezes x vezes
xx xx ... xx = x'x)X .-. xxxx-...xx = X xx

Questão 1.14
Resolução: Podemos escrever:
x vezes
xx+i+i_xi+n
X'X
E= xx^xx-...-(xx)Xj => E = x => E = x
x-1 J

f Xxt2 - X2
x' X’1
E =

Questão 1.15
Resolução: Chamando a expressão de E, temos:
"50'‘vezes "50" vezes "50" vezes

E = a3b4 a3b4 ... a3b4 => E = a3 a3 • ... a3 b4 b4 ■....b4


Os Segredos da Álgebra para IME/ITA/OLIMPÍADAS 309

"50“ vezes
=> E = a<3>5° ■ b(4>5° .-. a3b4 ■ a3b4 •...• a3b4 = a3*0 •b3o50 .

Questão 1.16
Resolução: Chamando a expressão de E, temos:
“100"vezes

E = a3b4c5 a3b4c5 •... a3b4c5


"100"vezes "100"vezes "100"vezes

=> E = a3 a3 .... a3 b4 b4 .... b4 c5 c5 .... c5

=> E = a(3)10° .b(<)10° b<5)100

100 vezes
■b4’00 e100
.-. a3b4c5 a3b4c5 ■,.. a3b4c5 = a3™ •c°

Questão 1.17_______________________________________
Sugestão: Use 12 + 22 +32 + ... + n2 = n^~— .
6

Resolução: Chamando a expressão de E, temos:


E = x - (x2)2 - (x3 )3 ■ (x4 f -... ■ (xnE = x • x22 • x33 • x4 4 .... x""
E _ x12+22+32+42+.. .+n2
=> E = x,12 . v22 v32 . v42
• X‘ • X' •X
. vn2
... • X
n(n+1)(2n-r1)
x-(x2)2.(x3)3.(x4)4.....(xn)n = X 6 .

Questão 1.18
n(n + 1)(n + 2)
Sugestão: Use 1-2 + 2- 3 + 3- 4 + ... + n (n + 1) =
3

Resolução: Chamando a expressão de E, temos:


E = x2.(x2)3.(x3)4.(x4)5.....(xn)n+1

=> E = x1 2 x2 3-x3 4.... xn (n+1> => E = x1 2+2'3+3' ■•+n-(n+1)

n(n+1)(n-2)
x2.(x2)3.(x3)4.(x4)5.....(xn)nt1 = X 3
310 10 Resoluções

Questão 1.19
2
Sugestão: Use 13 + 23 + 33 + ... + n3 ~n(n+1)~
2

Resolução: Chamando a expressão de E, temos:


3 4 ,n
E = X.[(x2)2] .[(X3)3] ,((/)^ -.{(xn)n)
E = x x2 2 2 x3 3 3 x4 4 4 •,.. xn n n

E = x,13 v X'33 y
X 23 v X 43 xn3 _ x13+23+33+43+...+n3

.n
X

Questão 1.20
Sugestão: Use
1-2-3+ 2-3-4+ 3-4 ■5 + ... + n(n + 1)(n +2) = n(n + 1)(n + 2)(n + 3)

Resolução: Chamando a expressão de E, temos:


.4 >5 / \n+2

=> E = x1 2 3 • x2 3 4 • x3 4 5 ■ .. • xn ’ <n+1) ’ <n+2>


x1 2-3 + 2 3 4 + 3 • 4 • 5+...+n • (n+1) (n+2)

n(n+1)(n+2)(n+3)

(<!)I((x2)3)4((xT)s-fr"r,r2- X 4

Questão 1.21
Resolução: Podemos escrever:
2 1
21 1 3 8-3 1-3 5
a3 -b2
E= => E = a3 4 b2 2 => E = a 12 -b 2 => E = a12-b‘
“1 3
a4 -b2
Os Segredos da Álgebra para IME/ITA/OLIMPÍADAS 311

2 1 5
a3.b2
1 3 b '
a4-b2

Questão 1.22
Resolução: Podemos escrever.

( 3/ V 3_3 3-6 _3
2a 4 2a" 2 4a4 2 4a 4 c 4a 4
E = => E = E =
< b1'3 A b/2 7" 2^3 E=—
b3 + 2 b23 b6

2aY Y2a"3'3 3 5
= 4■a 4 •b 6 .
k b/3 Á b/z
Questão 1.23 (AHSME-1954)
Resolução: Podemos escrever:

'
- 64’2
1 'l — (32)-45 1 + 1 -(26)’2 -(25)’5
=> E = -^
E = —-a0
16 + ll6j 16

1 _ 8-1
=> E = 1 — => E =------- E=I
8 8 8

Questão 1.24 (AHSME-1971)


Resolução: Podemos escrever:
_ 1_' _1_' r 1' r
1 + 2’32 1 + 2’16 1 + 2’8 1+2'2
S= 1+2 4

1-2 32
1 Y 1 ' 1/ 1'
Á. 1+2’32 1+2 8 1+2 2
S =Á 1 + 2 16 1+2 4
1-2’32
312 10 Resoluções

1-1 2
=> S = 2 S=— 2
r 1 ' 1' _ 1
1-2'32 1-2" 32 1-2 32

1 _i' 1 A £A
1 + 2 32 1 + 2'16 1+2 8 1+2 4
2 1-2 32
2

Questão 1.25
Resolução: Podemos escrever:

E = (-a6)-ar® .a32.a-32.(-a32) => E = a6-6+32-32+32

=5. E = a32
^.^y^yí-a1 a9.

Questão 1.26
Resolução: Podemos escrever:
m m m m
m2-1 m2-1
m+1 m-1 m+1 m-nm
m m +m m m m +m m

E= =>E =
2
2 m m-1 J2
rnm-1 mm +1 m m m

m m
m m
1 m2-1 m2-1
\m— m2-1
m*1 m-1 m m+1 m-1' m
E= m m + mi m m m +m m

nn1
=> E = m m +m m =. E = mm+1 + rnm“1 b m_m—1 (m2+l).
c =_
Os Segredos da Álgebra para IME/ITA/OLIMPÍADAS 313

Questão 1.27
Resolução: Podemos escrever:
■n4
E= => E = atn2 •b-"3
b"3 '

Questão 1.28
Resolução: Podemos escrever:

,a-(-2n).a+3n.a-(-4n) a
E=z => E = a+n

a-2n
a3n

: •a',3n
=> E = an a,2n ■a‘,n ...an2 g an+2n+3n+...+n2

rn2jn n2+n3
=> E = a 2 .-. E = a 2

Questão 1.29
Resolução: Podemos escrever:

an a-3.a-H4).....a
e=7 => E = a

a'n2

a"3

a-(-n)n
314 10 Resoluções

=> E = an an2 •a"3 ■a"4 w _ „n+n2+n3+...+nn


...-a""

n(nM)
nn+1-
=> E = a n-1 E = a "-1

Capítulo 02 - Radiciação

Questão 2,1 (CN-1964)________________________________________________


Resolução 01: Colocando todos os radicais no mesmo índice, ou seja, tirando
o mmc de todos os índices, temos:
7^Í8
72^
E= ___ => E = => E =
72Jã^ + 72J'a9
72^-72^ 7^24+8

7^ã33 7^a3®'~

72/
72^.72^ 7iO^ 724a
=> E = —
7^32
=> e = —, => e=
72/ 32-36

=> E = 7277r<T4) => E =


72 'a'3*4 E = 7^.

Resolução 02: Colocando os radicais no mesmo índice e simplificando cada


operação, temos:
12/Zã
. y-
E =. E = => E = => E =

•Jã
2^-24^ E = J^=
24/2-3 2ií^ => E =
E = -^= 7?l
=> E =
18/a8-9 7^

E = 7^H) 72/
=> E = E = 7^.
Os Segredos da Álgebra para IME/ITA/OLIMPlADAS 315

Questão 2.2 (CN-2000) - Resposta: Alternativa A


Resolução: Tirando os valores de x, y e z, temos:
x/x2 =1999® => x2 = (1999®)I3 => x = Vl99918 => x = 19999.

x/y=19994 => y = (l9994)2 => y = 19998.

5/Ç4 =1999® => z4 =(19998)5 => z = x/l99940 => z = 199910.

Assim, temos:
1
(xyz)3 = ^xyz =>> ^xyz = ^x y z => ^xyz = ^19999-19998-199910

=> ^xyz = x/"l 999',9+8-10 3/^ = 3/iggg27 => ^xyz =19999.

Questão 2.3 (AHSME-1956) - Resposta: Alternativa D


Resolução 01: Simplificando os indices com o expoente, temos:
l4 I4
E => E

=> E = (^)4-(^)4 => E = a2a2

Resolução 02: Efetuando a multiplicação dos indices e simplificando, temos:

E=[3Mf-[6^]|4 = =>

i4
c: /^,36 18/'a36
t = 18ya E = a2 a2 .-. [3/Wj4-|¥W]4 = a4 .
Questão 2.4 (AHSME-1998) - Resposta: Alternativa D
Resolução: Elevando a expressão ao quadrado, temos:
E= =/n => E = x/x/n N3" => E = x/n tj/tF

=> e = ^/n4 n9 => $’N WN =


316 10 Resoluções

Questão 2.5 (Harvard-MIT-1998)


Resolução: Elevando a expressão ao quadrado, temos:
r i2
r 2(5/2 1 Tíõ) 2(>/2+ >/iÕ)
s 5(73 + 5/5) 5(73 + 5/5)

2-5/2-5/10 + 10) 4(12 + 2-5/20)


= 0’^ 25(3 + 5/5) 25(3 + 5/5)

4(12 + 2-2 5/5) 2 4(12 + 45/5)

25(3 + 5/5) 25(3 + 5/5)

4-4((3 + 75) zr 2 <rf


=(íí- '
25(3 + >/5) U
1_6
25 IsJ 52'
Como r e s são inteiros primos relativos, concluímos que r = 4 e s = 5 .

Questão 2.6
Resolução: Podemos escrever:
2b -7a + 2a -7b
*7a-b 2a-b 2b -7b
E= => E = a-b
7b-a +2b-a 2^ • 7b j 2b • 7a
2a 7a
:' -jb 7s , 2a 7b ,2a •7a~
2a -7a
=> E = 3-b I => E = ^
2b ■ 7b 2^ • 7^ í 2^ ■ 7^ 2b , 7 b

(ya—b j 2a~b
=s E = —72a-b -7a-b E = 2-7 = 14.
7b-a +2b-a

Questão 2.7
Resolução: Podemos escrever:

laa+bbb+ba+baa aa-ab-bb+ba-bb-aa
=> E = fe=â
a':2b ba +b2a -ab ab -ab -ba + ba -ba -ab

aa bb (ab +ba) ^■b13


=> E = b^a b-a
=> E
Vab-ba-jab +ba) ab-ba - Vab'a
Os Segredos da Álgebra para IME/ITA/OLIMPÍADAS 317

aatb ■ bb + ba*b ■ aa b
. b-a
a'i2b ba +b2a -ab a

Questão 2.8
Resolução: Podemos escrever:

E=

=> E = a^y22b-a’b

= 2 + 2'
2 4+1 5
2 2 2

Questão 2.9
Resolução: Podemos escrever:

aa

E=

=> E =

=> E

=> E =
318 10 Resoluções

Questão 2.10
Resolução 01: Podemos escrever:

E^/^W.^a^ E = ^-5^- Va 4/

E = 36C,^-3íí/FW^W => E = ^ 3/4/ 3/4/5j

^
=> E = 3 -—

=> E = ~ E = —7^
=> E = 1^-3^.^ = —7^-—7^ —V?
==> E = —7a1+60+15+3 .'. E = —Tã7®
Resolução 02: Colocando os radicandos "para dentro das raízes", temos:

E = 777 3Ja 4I'a yja >/ã => E = 3 4 5 BJã ■ 3 4


Va va a6

=> E = => E = ,30

= E^Ti^a-7?7 => E = ^1-Va37 a120


=> E = ^ —VT37 = E = 17 => E = ~77“
180 ryK
■ 3 4 5l
3Ja $ 7a = Va

Questão 2.11
Resolução: Podemos escrever:
ígn2 »5n
E = n+:
jan2 => E = n+?/— (n2 +5n gn
=> E = n‘?
l_n2 +6n

a3n
'í 7a3"2
=. E = n‘^an2+6n-3n => E = n+7a"2’3n => E = n*7an(n+3) E = an.

Questão 2.12
Resolução: Podemos escrever:
^2n+1 -(2“n)2

2 r- n
E=" => E =
Os Segredos da Álgebra para IME/ITA/OLIMPÍADAS 319

j: 20*1-2n j 2 f92-2n
=. E = p — nl |4 ±____
V 22
-2n
= E = n-1^2^2 , £ __ 8nJ •2n => E = 2~4
=> E = 2
4/93 ^2~n ■ x/2n-1'1 _ i/3_
1 => E = 2L£_ n
=> E =
Ü2 2 i[Ã “2

(*) Observação: Veja essa técnica no capitulo de racionalização!

Questão 2.13
Resolução 01: Podemos escrever:

^16^4 |3VÍ6 .^4


E 3 E = 3I => E = 3
2^4-1^2 ^¥■^2 ^4-25 -1^2

1^220 l1^220 22
=> E = => E = 3
3 !5j221 •2
-1V2
^16^/4
=> E = 3-cL= => E = \/i 3 1.
^^22 2V41V2 -

Resolução 02: Colocando os radicandos "para dentro das raizes”, temos.


'^22-(24)5
' ^16^4
3
E=? => E = ? => E =
2^/4-1^ y 2V4-1^2 ^4.25'.1V2

=> E =
I 1^22 2,20: => E =
I I'fâ2
=> E = ?Pi= =1-
3 3 1^(27 )3 '2 Vx/?2
1^22 -25 j3 -1^2
320 10 Resoluções

Questão 2.14
Resolução: Podemos escrever:
,n

E="
'64n + 162n
=> E => E=
n82,H28)
8n + 32n V 8n + 32n V 8n +32n
kn
l82n+(23+5)’ 8n'n + (23-25)n Í8n ■ 8n +(8n -32n)
E=? => E = n => E = W
8n + 32n 8n + 32n 8n + 32n
lsn(8n + 32n)
64n +162n
=> E = f => E = ^8n .-. n___________ = 8.
8n +32n V 8n + 32"

Questão 2.15
Resolução: Podemos escrever:

E = a_bE = —>/a^aa2-b2ba-b
E = §tb.
V^b
E = ^a^a(3,b)(a^;
= a^a(a+b’b.

Questão 2.16
Resolução: Podemos escrever:

xy
h l^x b íl/x
a-o I___ . b-a i___ = (^)aÈ
V\/y W
b _
xy f xy i
ab(a-b)^X^ ab(b-a)J xab(a-b> xab(b-a)
|ab _ 2'-2ab-
a
yrbFb) yãb(b-Tj

_ a__ _ b__ a-b


xab(a-b)-ab(a-b)
b a Z
(JQL) ab(a-b)
X______
b-a
JAi
yab(a-b) ab(a-b) yab(a-b)

1 1
i—l
=> xab • yab = 2<2ab) =• xy = 2' 2 (xy) =2(2J ■■ l»-2|-
Os Segredos da Álgebra para IME/ITA/OLIMPlADAS 321

Questão 2.17
Resolução: Podemos escrever:

■a (;/
E= E=

^(75-a-TÍ) 1
J? _1
r^a
=> Ec = í va => E = í^a •a 2
' (a-1)2
( ra a _1 _1 1
=> E = Ui' ■a 2 => E = k/ã75° a 2 => E - j■ a 2

1 1 11
=> E = a2 a 2 => E = a2 2 => E = a° .-. E = 1.

Questão 2.18
Resolução: Podemos escrever:

—]W
E = tó 75
Í75 J
=> E = rf
■JPT 75 r
(2‘1)

^1-----
E =--------- \I2 • 2“' 2 ■ 2-75
■2
• 2^ 72

=> E =
tíí I21-J2 ,2-72 e = 1-^-J2-2~'^
(1-72)72
_ 2f 1—75), 2"75 f '-^2]'
E = 2^“^ 1-75
=> E => E = 2^
f 1-75 >2)
=> E = 2l ) . E
=> E = 2^ 3+3V2-V2-2)
1-2 J =>
; E = 2-(1+2'/5) .-. E =
2J
322 10 Resoluções

Questão 2.19
Resolução: Podemos escrever:
■5^°

sJs
5V5’^3
E=

-51
s'1.^5:

E = 5^
5" 5
' 5 lf
'■ 5 5
5

4
55
5 1 2
8
5’5 I 5s-55
(5^-5j25)
r 52
=> E = •5 25 =5 E = 5'

8 1,2 55 8 4 8
5+5 5—5 5
=5 E = 55 5.55 -55 => E= 55

4 4 4 4
55 541155 5“5+é
r 4
5
E = 55 => E = 5 => E = 5'

=> E = 5^°) => E = 5(1) E = 5.


Os Segredos da Álgebra para IME/ITA/OLIMPÍADAS 323

Questão 2.20
Resolução: Podemos escrever:

•3_,2-9Í/2

JJi) (?)

9^ '3^ r-
E = —V39'

2
,2-9 15
, ,2-9 V2
33
3 3

=> E = (33)l3^
JfJl 32^

33'3 K)-« 32^2

í9^-2-2*72' 9 15-2-615 j
3l 3 ( 3t-/2-3V2
3'

=> E = 33

í 3» 72-3^/2 í 3-j2-l‘j2
! 3
=. E = 33 = 33

( 3.72-31/2 11 í 3 1/2-2 1
E = 33 ‘ 3
= 3
324 10 Resoluções

Questão 2.21 - Resposta: Alternativa B


Resolução: Podemos escrever:

^bba-b
E=

a^aa2b~a2

c
=> E =
c» c='ÍsL
‘ j''1

=0 E =
c
[a\'aa2’b2-b-
c sF1
[Cc+1]'
cc

=> E = E = 1.

Questão 2.22
Resolução: Podemos escrever:

^(am+2)2 •^am+3J3
E= => E
^am-1

^a2m+4 a3m+9
E _ ^a2m+4+3m+9-(m+1)
=> E = => E = ^6^1
^am-1

=> E = 67a6<m+2’ ,m+2


^am-1
Os Segredos da Álgebra para IME/ITA/OLIMPÍADAS 325

Questão 2.23___________________________________________
Resolução: Podemos escrever:
E= • x/72 • 5/ãí => E = 23 ■ IÍ2 ■ 7 ■ x/72 ■ 3 • %3

=> E = 168-5/2 ll?2 1/3 • ?/8Í = 168 • l/ê • 5/49 .

Questão 2.24________________
Resolução: Podemos escrever:
.3
E=

=> E =

Questão 2.25________________
Resolução: Podemos escrever:

E= 1^=.^ => E = ?aM i38b43


w
rrú 2
2
b15
i3°b43
= Va3b .Í-2bà ’5 .180V^b43
a2b5

2-75 1 -7S _______


4 3
E = va3b ■ va !b 15 ■ 185/a3flb43 a3b-a 2b 60 .18Sya30b43

1 73
3/ •
5 -13
ro E = 3Va 2 -b 60 . 180V?;3°b43 va 2 b 60 .18^a30b43

18o/ f—1-60Í-1-3).6O
=> E = va'- 2> tí<60> • 18^3%43 = 18^° •b-13 ■a30b43
150 3£ 5 1
=> E = 18^FÍ2° ■ b30 a 180 .faiao ■ E = a ê - b6.
326 10 Resoluções

Questão 2.26
Resolução: Podemos escrever:

3
V x4 ' 5 _ 4 3 _2
~ X3
23-4 5 234 2-3 2
E= => E = x
X2

5 4-5 3-4-5 23-4-5 5 20 60 _120


2-3-4-5 + 23-4-5 23-4-5 120 120+120 120
23-4-5
E=x => E = x
75
120
., E = xH)
=> E = x’

Questão 2.27
Resolução: Podemos escrever:

E = 5Jx2^x4i/7
6
■X42 -ex’ 24 5
-71
24 2)
E= rx7
I Xz
f_6____ 5 ^4 5 J__A 245
x<2-4-5 2-4 5 + 2 4 5. => E = ^V^ 40 40+ 40
E=^ 6/ x7’8 +

^^2
H- X
5_
40
40
40.
=> E = S) => E = x —-í—l. 5 \40j

216 pn Í216-11
í—1 w
E=x
40 \40j .=> E = xl 40 > => E = x7° J => E = x7J
f32.3] E = xH)
=> E = x'8

Questão 2.28 Resposta: Alternativa B


Resolução: Podemos escrever:

l
“7
g
X ‘ _
x
(2wo.-10l)
I
n
2-2-...-2
"n"vezes
-(n~1)
2-2-...-2
"n-T* vezes
, 1
-+2
(21OO -101)

í n + 2(n-1) 22(n-2) + 2n
■■'+2" Í2100 -101)
I 2n 2^ 2^
=> x1- ' = X'
Os Segredos da Álgebra para IME/ITA/OLIMPÍADAS 327

J •2(n-1)+22(n-2)+...+2n
2n (2100--101)
= X'

Chamando essa soma de S, temos:


n + 2(n-1) + 22(n-2) + ... + 2n
S = ------- 1------ -------- i, coma = n, r = -1 e q = 2 .
2n
a(qn-l) rqr(n-1)-qn -nqn“1 +l]
q-1 (q-1)~2‘

S n(2n~1), (-1)-2[(n-'O •2n-n- 2n’1+l]


2-1 (2-1)'1
=> S = n ■ 2n - n + (-2) ■ (n • 2n - 2n - n • 2n"1 +1)

=> S = n ■ 2n - n - 2n ■ 2n + 2■ 2n + 2n ■ 2n~1 - 2 => S = 2-2n-n-2

S = 2n+1-(n + 2) .

Por comparação, temos:


xS=x(2,00-10l) x[2"*’ ■(nr2)]=x(2’00 -101]
=> n + 1 = 100
=> n = 100-1 => n = 99 .

Questão 2.29 - Resposta: Alternativa E

a (qn~l) rq(n-1)qn-rn-qn’1 +1
Sugestão: Use S = — ?2

Resolução: Podemos escrever:


"n" radicais

(n-1)l 2! , 1!
1 ■ 2-... (n-1) + 1-2*1
E=
328 10 Resoluções

n! _ (n-1)l + _ 21 II n° impar de "uns"


nl"(n-1)!+" 21*1! = x (1-1-.'..-1+1) ...
=> E = x E = x.

Questão 2.30
Resolução: Podemos escrever:
E = \/x tf*? ■ y/x?■ ...■yfx'' = ^x-x2 x3 ... x0

=> E = ^*1+2+3+.. ,+n


E = x'<’rl

Questão 2.31
Resolução: Podemos escrever:

E= = ^32 -34 -38 3,18

E = \/J,2+4+8+16 = 36 E = 729.

Questão 2.32
Resolução: Podemos escrever:
E = 1^.1^23.1^.1^7.....1^99 !
= 1^2 23-2s-....2,99

E 10/21+3+5+...+99

_ _ 250 3 E = 2250

Questão 2.33
Resolução: Podemos escrever:
E = ^2^2^W => E = V2 J I3I (3/4/

=• E = x^.^.2^.12^2 => E = 12^28° •12^2Õ.12^.120/2


12^260 220 .25 -2 => E = 12^260’20+5+1 => E = 12\/288
=5 E = •2'
86 43
E = 212° .'. E = 260.
Os Segredos da Álgebra para IME/ITA/OLIMPlADAS 329

Questão 2.34______________________________________________
Resolução: Podemos escrever:
E = ^7^7^7^7^ =» E = 6V7W -^?7

=> E = ^73^-12^7.36^.72?/7
=> E = 72^7 120 .72^7^. 72^7^. 72^72 ■ 72^7
ly120

=> E = 720l~Í2Õ •7'.24 7S 72 7 => E = 72^7120+24-6+2^1


153 51_ 17
=> E = 72^7153 => E = 7720 => E = 7240 E = 780.

Questão 2.35 Resposta: Alternativa C________________________________


Resolução: Da multiplicação e divisão de radicais em potência de 2. temos:
"n" radicais

x x... X x/x X2"-1


E
22 í—'l
A 3 J
32n-3-1+2n 4(t-2-")
22
E= ' 3
x
. I 3 2" J = X 3

Questão 2.36_____________________________________________________
Resolução: Da multiplicação de raizes na forma mn, podemos escrever:
"n" radicais
Z3n-l'
( 3n-1| 32 2
32 X ' 2;
E= => E = =0 E =

“n" radicais

=> E =

Questão 2.37___________________________________________ ____ _____


Resolução: Da multiplicação de raizes na forma mn , podemos escrever:
___ radicais

E=
<2 =. E =
330 10 Resoluções

"n“ radicais

'X4n-,
=> E =

Questão 2.38
Resolução: Da multiplicação de raízes na forma mn , podemos escrever:
"n" radicais
Bn-1

E =

Questão 2.39
Resolução: Da multiplicação de raízes na forma mn , podemos escrever:
"n" radicais

E = 10Jx9 10Jx9 10Jx9 ^x9-....1^9 =, E =

=> E=
10n I 9{l0g-1) => E = ~VZ^ /. E-xt ion J.
' X v 7

Questão 2.40
Resolução 01: Note que temos x - 1 termos, então, da multiplicação de raízes
na forma mn, podemos escrever:

r- ? '
E = Vx‘
:x2 yjxx3 x xx
VX
*
Vx
Xx

E = ^.x^xx3 í/.....^xxX =>E = xx


x)’ xx
X

=> E = xx xx X^xx4 ....•Vxx7

=> E = xx xx xx-X^ X/ xx ' xx


vx ... => E = xx-xx xx-... - X
"x-r‘vezes
xl x/ xx = xx’<x’1).
=> E = xx xx xx ... xx X Vx
Os Segredos da Álgebra para IME/ITA/OLIMPÍADAS 331

Resolução 02: Colocando os radicandos “para dentro das raízes”, temos:


_ X x2 x/ x3 x/ x4 X
x/ xx •X:
E = yx \x yx yx => E

=> E = yyx
X/x/ y3
•X
X3 x/ xx
VX => e=x
42x3í/ xl' xx
X

x4 x/ xx => E = ~i/x2x4 • x'• X4 X/ XX


=> E •X VX VX

xx )
(x-1).
... => E = —7x<x-1>xX => E = x'
xx-11

xl . ,(x-1)x
VX'

Questão 2.41
Resolução: Da multiplicação de raízes na forma mn. podemos escrever:
“n" radicais
S2[
E= => E =

92-1 9n-l'l
9° I 8
=> E = Vx
9-1
=5 E =
I 9"-1
X E=x
9" J

Questão 2.42 Resposta: Alternativa B

a (qn-l) rq[(n-1)-qn -n-qn"1-s-1


Sugestão: Use S = —
q-1-+ 2

Resolução 01: Podemos escrever:


... 7x2>/x => E = Vx" ■ \J Jxn 17x^”2 ...7x2\/>f
E = AjxnJxn-1Jx"-2
E = V/7.2^xn~1 7xn’2..Jx2Ví => E = x
l2^.22^-2...^
2 — x/------------ :-------- n
E = X2.X22 -27xn-2...x/Ã^ E = X2.X22
332 10 Resoluções

n n-1 n-2 n n-1 n-2 _1_


=> E = x2 X
22 • x 23 •2V-... 7x => E = x2.x 22 .x 23 • ...-X2n

í-1 ín-2n-1+(n-1)-2n-2
=• E = X l-2n

Chamando essa soma de S, temos:


S = n ■ 2n"1 + (n -1) ■ 2n"2 + ... + 1 = 1 + 2- 2 + 3-22 + ... + n- 2n"1,
com a = 1, r = 1 e q = 2 .
a(qn-l) rq[(n-1) qn -nqn“1
S=
Ç-1 (q-1)1;

l(2n-l) 1-2|:[(n-1)-2n-n-2n-1+l]
S= 2-1 + (2-1).2:

S = 2n-1 + 2- (n-2n-2n-n-2n~1+l)

=> S = 2n-1 + 2n 2n-2-2n-n-2n+2 S = n■ 2n -2n +1.

Logo, temos:
[ n S'
E = xL2n -I
n2n-2n*1 n-1+-
=> E = x 2n => E = X

E = x<n"1)+2'

Resolução 02: Colocando os radicandos “para dentro das raízes", temos:

E = ^xnJx^ E= xn-

xn-

E= xn-

=> E = Vxn-

=> E = J~y/xU2'2+3'22 + +n 2>n-1


' E = —^x1+2-2+3-22+. . +n-2f E = ^Ã
Os Segredos da Álgebra para IME/ITA/OLIMPÍADAS 333

Chamando essa soma de S, temos:


S = 1 + 2-2 +3-22+... + n • 2n-1, com a=1, r = 1eq = 2.
_a(qn-l) rq[(n-1)-qn-n-qn~1+l]
S
c’-1 (q-1)2

l(2n-l) 1-2|/(n — 1) - 2n -n-2n


=> S 2-1 +
(2-1).2:
=> S = 2n -1 + 2 (n ■ 2" - 2n - n ■ 2n-1 +1)

=> S = 2n -1 + 2n ■ 2n - 2 ■ 2n - n ■ 2n + 2 .-. S = n-2n-2n+1.

Logo, temos:
n.2n-2n-1 n-1+ 2n
E=^ => E = x 2" => E = x 2
.-. E = x<n-1)+2’n.

Questão 2.43 (AHSME-1954/Stanford-2010) - Resposta: Alternativa E


Resolução: Da soma de radicais simples, podemos escrever:
f i r f-------------------- 1 + 71 + 4-1 „ 1 + 7T74 1+75
'1 + 71
X = \J1 + ->/1 + V1 v1 + ■■■ => x = = ------- - ------- => —
X = X = ------------ .
2 2

Questão 2.44 (Harvard-MIT-2000)


Resolução: Da soma com termos em PA, podemos escrever:
^1 + 2^1 + 37i + 4^1 + 5VÍ4^ => x = Jl2+2^12+(1 + 2)/l2+(2-1 + 2)
x=

=> x = 1 + 2 x = 3.

Questão 2.45 (Harvard-MIT-2000) _______________________


Resolução: Da soma com termos em PA, podemos escrever:
x = /l6 + 3^16 + 7^16 + 11>/16 +15716+^ =>

x = j42+3j427(4 + 3)T42'+ (24 + 3)=> x = 4 + 3 .'. x = 7.


334 10 Resoluções

Questão 2.46
Resolução: Da soma com termos em PA, podemos escrever:
x = ^9 +2^9 4-5^9+ 8>^+lW9 + --- =>

x = J32 + 2 ^32 +(3 + 2)^32 +(2-3 + 2)77^


=> x = 3 + 2 x = 5.

Questão 2.47
Resolução: Da soma com termos em PA, podemos escrever:

x = \|16 + Jl6 +5\jl6+ 9^9 + 13716 + = =>

X = ^4 2 + 1^42 + (4 + 1)^42 +(2 4 + 1)\/...co


=> x = 4 + 1 /. x = 5.

Questão 2.48 __________________________ _______


Resolução: Da soma de radicais com termo fora da raiz, podemos escrever:
2 + ^22 + 4 a 2 + 74 +4a
x = \ia + 2\]a + 2>Jã+... => x = 2 => x- 2

2 + ^(1 +a) 2 + 27(1 +a) |------------ 7=1


=> X = => x =-------|x = 1 + 7l + a|.
2

Questão 2.49
Resolução: Da soma de radicais com termo fora da raiz, podemos escrever:

2 + V22 + 4-24
X = \/24 + 2^24 + 2>/24+T => x = — 2 + 74 + 96
2 X“ 2

2 + 7ÍÕÕ 2 + 10
|x = 6|
=> x =------------- => x = ---------
2 2

Questão 2.50
Resolução: Da soma de radicais simples, podemos escrever:
x = ^2070 + x/2070 + 72070 +~ — “ - 1 + + 4 2070 1+71 + 8280
2 =* X- 2

1 + 78281 1 + 91 92
=> x =------------
=> x =------- => x = — x = 46.
2 2 2
Os Segredos da Álgebra para IME/ITA/OLIMPÍADAS 335

Questão 2.51
Resolução: Fazendo A = n + aeB = x, na soma com termos em PA, podemos
escrever:

k = y/A2 + B^Ã^ÃTbJ^^T^aTb)^^

k = yj(n + a)2 + xj(n + a)2 + (n + a + x)^(n + a)2 +[2(n + a) + x "]>/...oo

k = ^(n
a)2 + > yj(n + a)2 + (n + a + x)^(n + a)2 + (2n + 2a

k =n+a+x.

Questão 2.52
Resolução: Podemos escrever:
x = ^60 + ^60 + ^60 + ?= =■ x = ^6Õ => x3 =60 + x x3-x-60 = 0.

Note que, pelo teorema do fator, 4 é raiz dessa equação. Como as outras raizes
são complexas (verifique!), temos que a resposta é 4.

Questão 2.53
Resolução: Da diferença de radicais simples, podemos escrever.
' . 21
-1 + , 1 + 4----
21 V"
V 16
x = . 1-
16 2
4 + 21
4
=> X = => x =
2 2 2
5 -2 + 5 3
+2 2 3 1 3
=> x = => x = => x = — => x =------ X = —.
2 2 2 2 2

Questão 2.54 (Rússia/IMO-Longlist-1969)____________


Resolução: Dos radicais alternados, podemos escrever:
x - Ja - bja + b a - b\la + ... => x2 = a-b\/a + bx

= a-x2 => b2 (a + bx) = (a - x2)

=> ab2 + b3x = a2-2ax2 + x4 => x4 - 2ax2 + a2 - b3x - ab2 = 0 .


336 10 Resoluções

Somando e subtraindo bx3, b2x2 e abx , temos:


x4 - j2ax21 + a2 - b3x - ab2 = 0

=> x^-ax2 -ax2 + a2 - b3x - ab2 - bx3 + bx3 + b2x2 - b2x2 +


+ abx - abx = 0
x^(x2 - a + bx + b2) -bx(b2 x2 +bx ~a)-ã(x2 -a + b2 +bxj = 0
=> (x2+bx + b2-a)(x2-bx-a) = 0

x2 +bx + b2 -a = 0 OU x2 - bx-a =0
=> x2+bx + b2-a = 0 => A = b2-4.1.(b2-a) => A = b2 -4b2 +4a

-b±V4a-3b =-b . V4a - 3b


A = 4a - 3b2 ; => x = 2.1 X“ 2 + 2

-b I4a-3b 3b2 b .
=> x = — + x= 'a------
2 4 4 2 '
=> x2 - bx-a = 0 => A = (-b)2 -4.1.(-a) .-. A = b2+4a;
-(-b)± y/b? -b Vb2 + 4a
+ 4a
=> x = => x = — +-----------
2.1 2 2
-b b2 + 4a
=> x = — + x= íaaT- —
4 - 2-
2 4

Do enunciado, note que:


b^ b2+^ b2 5b2
a > b2 => a + — >
4 4 4 4

Da condição de existência do radical, temos:

a + —>0 => < o => b2 < 0 , absurdo!


4 4

Logo yja-b^t 3b2 b


'a---------
4 2
Os Segredos da Álgebra para IME/ITA/OLIMPÍADAS 337

Questão 2.58
Resolução: Dos radicais alternados simples, para a > 1, temos:

X = y7 — ^7 + \/7 - ...eo _1 + 74-7-3 -1 + 728-3


=> x =------------------- => —
— x=
2 2
-1+725 -1 + 5 4
=> X = --------------- => x =-------- =■ x = - /. E = 2.
2 2 2

Questão 2.59
Resolução: Dos radicais alternados simples, para 0 < a < 1, temos:
=
r—-------- .1 + , .1 + 4
a 33
---- -1+P
x=
'33 '33 I33 v
-------- ...CO => x = 1 64 => x = —1_16
64 64 64 2 2

-1+ /16i~33 Í49 7


V 16 => X =
V16 => x = 4
2 2 2
-4 + 7 3
4 33 '33 '33 3
=> X = => X = ------ ...oo = —
2 2 64 64 64 8

Questão 2.60

Resolução 01: Sejam a e b, tais que a = 7JX4+7JX4+Vx4 ...CO e

b 4Jx4 4Jx4 Vx4...o= , então temos: E =


b
.

Assim:
a = ^x4+7\^Mx*Mx4+...«>
a= a7 = x4 => a8 = x4

=> a = a = Jx|.

ÈÍ = x4
b
=> b3 = x4 => b = => [b = x-^|.

Logo, temos:
E = -^- .-.E = ^
E= " => E =
b x x.^? X
338 10 Resoluções

Resolução 02: Da a multiplicação e divisão de radicais infinitos, temos:

E=
.00 —77
=> E =----- — => E
77
-77 = 3V7
r- 'J* r-
=> E =——
=x X

Questão 2.61

Resolução 01: Sejam a = \J


26
'x976JxM x9^26J777 e
b=i6J7ZF 16Jx9 17777 , então temos: E = —
b
Assim:
a = 26\/x9.2fx9-26^
'x9.27 X9 -r...CO => a = 277
=> a‘26=x9-?a => a'i27 = x9 => £7^.

b= x9’^ X9 ...CO => b = Wb => b16 = x9.b


l1 6
b_ = x9 =, b15=x9 =, b =
b
177
Logo, temos:
=>E = ^ 15/^ 15)
E=— => E = -LÍ= =0 E = E = -í—-
b 77 15V? 177-177 x

Resolução 02: Da a multiplicação e divisão de radicais infinitos, temos:

2&,
.OT 2Vx9
E = => E = => E = -t=

=> E = =■ e = -t== => E =


, E.Í2
’77 X
Os Segredos da Álgebra para IME/ITA/OLIMPÍADAS 339

Questão 2.62
Resolução: Podemos escrever:
E= O2 = —x/32 => E = V32 .'. |E = 2|.

32

0
Questão 2.63
Resolução: Podemos escrever:

V 729
E = 3-?29
í—] => E = (36)W =>
=> E = 729^5+1'1 E = 35 |E = 243|.

Questão 2.64
Resolução: Podemos escrever:

X xx 2017 => X2017 2017 201y2ÕÍ7|.

Questão 2.65
Resolução: Podemos escrever:

xx X = ^3.
X = 27 => x27 = 27 => x= 2T27 => x

Questão 2.66
Resolução: Podemos escrever:

X=W => x=isyrõx Vx =1SyiÕ x = 10.

Questão 2.67
Resolução: Podemos escrever:
243/57;'
x = 2W^ X = 24\/243* = 24^243 x = 243 .
340 10 Resoluções

Questão 2.68________________
Resolução: Podemos escrever:
M= => M = 5^)
=> M = 5t'2
fl.JL1
M = 75
í^)
.-. |m^ = 751
= ^77 =0 M-^=^

Assim, por comparação, temos:


^7í = 7s => ^ = ^75 .-. x = 5|.

Questão 2.69______________________
Resolução: Podemos escrever:

•§/2
x = 2>/2 => X = 2-22 => x = 7s x=8.

Questão 2.70________________
Resolução: Podemos escrever:
'3 5*
,7^SV3
I-----
3-52
x = 5\/3 => x = => x = 573 x = 75.

Questão 2.71________________
Resolução: Podemos escrever:

_1_ J625
x*x =625 => x '625 = 625 => x = 625-^25 ^25 = 625 625

25
=> X = (54)B25 (A).
=> x = 54 ’
_4_
X = 525 .

Questão 2.72________________
Resolução: Podemos escrever:

1 >71024
= 1024 => x '1024 = 1024 => x = 1024 '1024 => x = 1024 1024

=> X =
, 32
(21O)1O24 x=2
10'1
(A).
'
_5_
X = 216 .
Os Segredos da Álgebra para IME/ITA/OLIMPlADAS 341

Questão 2.73
Resolução: Podemos escrever:
1^?16-1
= 7 => x^=7 => x = a^
=> x = 7 7 x=7 7

Questão 2.74
Resolução: Podemos escrever:
J/ _i_
x =32 => x^33 =32 => X = 32^32 => x = 25fe)
f3/
5 v
í23-2
5-^2
22
=> x = 2 x =2 4

Questão 2.75
Resolução: Podemos escrever:

{ÍZ x 7
x’x = xx .-. 7'7 =77 .

Questão 2.76
Resolução: Podemos escrever:

x'x = x*
x iXn""
13 ’13 1313

Capitulo 03 - Racionalização

Questão 3.1 (CN-1954)


Resolução: Podemos escrever:

E--S- = .=> c-Vb7^ c-Vb3


■ E= => E =

C.^
=o E =
c-Vb3 c
b
342 10 Resoluções

Questão 3.2
Resolução: Chamando a expressão de E, podemos escrever:
E 1 = _______ 1_______ = 1
tftftfz-tftfjl 77T23 tf??-tf??
E=- 1 --- 1 1 75,6-3.26"4
7s1’2 -23+1 7s3■24 7s3■24 756'3 26-4
E___i__ = T^F \l?-tf?
7s3 ■ 24 7s3 ■ 22 7s3+3 • 24+2 73®-26

7?-tf?
c =--------------
1 73 72
3 2 -T^TÍ 6

Questão 3.3 (CN-1976) - Resposta: Alternativa B


Resolução: Chamando a expressão de E, podemos escrever:

E=
A7Ã-373 => E =
A7Ã-373 7Ã + 73
7Ã-75 ” 7Ã-7ã Tã + 73
2 2
A(7Ã) +a7Ã x^-373-7Ã-3(73)
E=

R2-(<
AA +a73Ã-3x/3Ã-3-3 A2 +(A-3)73Ã-32
=> E =
” Ã^3
(7ã)2-(73)2
e_ (A - 3)(A + 3) + (A - 3)73Ã (A-3](a + 3 + 73Ã)
=> E =-------------------------------
’ Ã^3 A-3

E = A + 3 + 73Ã.

Questão 3.4 (CN-1999-Modificada)


Resolução: Chamando a expressão de E, podemos escrever:
2 2 2 -J5+J3 2 tf?
E= > E=
75-73 72 ” 75 - 73 75 + 73 72 7?
Os Segredos da Álgebra para IME/ITA/OLIMPÍADAS 343

2(75 + V3) 2^^ E 2(V5 + V3) 23/4


=> E =
(Vã)2-(Vã)2’ 37? 5-3 2

E = 75 + 73-^4 .

Questão 3.5_____________________________________
Resolução: Usando a forma prática, temos:
N n-(7ã-7b-7c)-(a + b-c + 27ãb)
VÃ-Vb + Vc (A + B-C)2-4AB

3 3(75-Vã-V2)-(5 + 3-
E=
Tã-Tã + Tã (5 + 3-2)2 - 4-5-3

3(75-7ã-72)(6 + 2-7Í5) 3-2(75-Vã-V2)(3 + VT5)


E= E=
62 -60 -24

(-75 + 7ã + 7ã) • (3 + Tis) -3VÕ ■»373 + 3V2 - V75 + V45 - Vãõ


4 4
-375+3Tã +372-573+ 375 + Vãõ _ 3V2 -2>^ 4-
4 4

Questão 3.6
Resolução: Podemos escrever:

E = —_ E=
7ã => E
75
2V8+3V5-7V2 2-2-72 + 375-772 4-j2^3</5-7j2

Vã Vs + Tã 7ã-(75 + 72) Tis + Tê


=> E = =>E = - => E =
3V5-3V2 V5+V2 3-[5-2]
3

E=
7Í5 + 76
9

Questão 3.24
Resolução 01: Podemos escrever:
73 + 78 => A = 3 e B = 8, C = 732-8 => C = 79=8 .-. |C = 1|
344 10 Resoluções

=> 7a +Vb =
v 2
-
2
=> 7^vã = vvM
2
13-1
2

73 +Vã = 42 +1.

Resolução 02: Podemos escrever:


73+Vã = 72 + 272+1 => ^3 + Vã = )2 + 2 ■ >/2 • 1 +12

=> V3 +Vã = J(V2+1)2 => 73 +Vã = |V2 + 1|, como V2+1>0

7s +Vã = 42+a.

Questão 3.25
Resolução 01: Podemos escrever:

'p-4V2 = V6-72 42 .> 76-472 = 76-732


. = 6 e B = 32, C = 762 -32 => C = 736 - 32 => ,C = 2|.

76 - 472 = 46-4^2 = 2 - 42.


V 2

Resolução 02: Podemos escrever:

76-472 =74-2-272+2 => 76-472 =^22-2-1 >^ + (72)2

=> 76 - 472 =^(2-72)2 => 76-472 =|2-^|, como 2-72 >0

76 - 472 = 2 - 72.

Questão 3.26
Resolução 01: Podemos escrever:

7l61 + 72Vã = 7l61 - 72 722 => 7161 + 75^5184 = 7l61-725920


A = 161 e B = 25920, C = 7õ2 -25920 => C = 725921-25920

|C = 1| => 7ã^ = Jãv2+JãH£


V 2 V 2
Os Segredos da Álgebra para IME/ITA/OLIMPÍADAS 345

1161-1
7161 + 7275 = 7i 61+ 7275 =

=> 7161 + 7275 = 781 + TãÕ 7l61+ 7275 = 9 + 4^.

Resolução 02: Quando os números são grandes, desconfie. Note que:

7l61 + 72s/5 = ,161 + 236 7õ => 7"! 61+ 7275 = 781 + 2-3675+80
V M(5)
=> 7l61 + 727õ = 781 + 2-9-475+80
2
=> >/l61 +-72x/5 = yJ9z +2 9-475+(4V5)

=> 7l 61+ 7275 = J(l9 + 4x/5)2 => Vl61 + 72T5 = |9 + 47s|, como

9 + 475 >0 7l61 + 72y/5 =9 + 475.

Questão 3.27
Resolução 01: Podemos escrever:

74-7Í2 => A = 4 e B = 12, C = 7a2-B => C = xM2 -12


=> 0 = 716-12 => 0 = 7? => |C = 2|.
74-TÍ2 = .
N 2 \
.-. 74-7Í2 = 75-1.

Resolução 02: Podemos escrever:

•^4 - \/l2 = >/l -2x/3 + 3 => ^4 - -2 • 1 ■ >/3 + (■^3)

=> 7á-7Í2 = ^(l-73)2 => 74-7Í2 = |1 - 731, como 1 - 73 < 0

=■ 74-TÍ2 =-(1-73) 74 - 7Í2 =73-1.


346 10 Resoluções

Questão 3.28
Resolução 01: Podemos escrever:
7l7 + 12T2 = 7l7 + 72-122 => 7l7 +1277 = 7l7 + 7288
A = 17 e B- 288, C = 7l72 -288 => C = 7289-288 |C = 1|
EHS 717 + 1272 = 117-1
=> TÃ77b = J—
N 2 2 V 2 V 2
7l7 + 12T7 = 3 + 78

.-. 717+12T2 = 3 + 272.

Resolução 02: Podemos escrever:


7l7 + 12T2 = 79 + 2-672 + 8 =>
7l7 + 12T2 = ^32 + 2-3 -2^+ (2\^)2 => 7l7 + 12T2 = ^3 + 2^2^

=> 717 +1272 = |3 + 2721, como 3 + 272 > 0


7l7 + 12T2 =3 + 277.

Questão 3.29
Resolução 01: Podemos escrever:
732-1077 = 732-77-102 => 732-1077 = 732 - 77ÕÕ
A = 32 e B = 700, C = 7322 -700 => C = 71024-700

=> c = 7324 .-. |c = 18| => 7a + 7b = ,+ Ia-c


V 2 'I 2
^32-1077 = '32 + 18 732-1077
2
=> 732-io77 = 725-77 .-. ^32-1077 = 5-77.

Resolução 02: Podemos escrever:


732-1077 = 725 - 2-5^ + 7 =>
------------- / 2
732-io77 = J52-2-5-77+(77) => >/32-io77 = ^(5-77)2

=> 732-1077 = |5 - 77|, como 5-77 > o .-. 732-1o77 = 5 - 77.


Os Segredos da Álgebra para IME/ITA/OLIMPÍADAS 347

Questão 3.30
Resolução 01: Podemos escrever:

7l6-677 = >/l6-77-62 => 7l6-677 = 7l6-7252


A = 16 e B = 252, C = Ta2 -B => C = 7l62 -252

=> C = 7256-252 => C = 7Ã => |C = 2|.

=■ 7i6-677 = 70-77 .-. 7i6-e77 = 3 - 77.

Resolução 02: Podemos escrever:


7l6-6x/7 = ^9 - 2 ■ 377 + 7 => 716-677 = ^32 -2-3-77-r(77)2

=> Vl6 -6-?7 = 7(3 - 77 )2 => 7l 6 — 677 = |3 — 771, como

3-77>o 7i 6 - 677 = 3-77.

Questão 3.31
Resolução 01: Podemos escrever:

Ti 0-476 = 73 - 72-22 => Ti o - 476 = 73 - 78


A = 3 e B = 8, C = 7a2 -B => C = Ts2 -8 => |C =1|

7ÍFZ776=,^+I-xÍE1 => TiO-476 = 72-1.


N 2 V 2

Resolução 02: Podemos escrever:

TiO-476 = Ta-2-276+ 6 - 710 - 476 =^22 -2-2-T6+(76)2

=> Ti 0-476 = ^(2 - Te )2 => /10-476 = |2- Te|, como 2 - 7ê < 0

=> TiO-476 = -(2- Te) .'. 0-476 =76-2.


348 10 Resoluções

Questão 3.32
Resolução 01: Podemos escrever:

79 + 475 = 79 + 7õ-42 => 79 + 475 = 7s+ VsÕ


792 -80 => C = 781-80
A = 9 e B = 80, C = |C = 1|.
'a+~c '^2 7^7775 = ^ + '9-1
,9 + 4x/5 =
=. 7a-7b =
2 2 V 2 2

79 + 475 = 75 + 2.

Resolução 02: Podemos escrever:


79 + 475 = 74 + 2-275 + 5 => 79 + 475 = ^22 + 2 • 2 • Tõ + (Tis)2

79 + 475 = J^2 + Ts)2 => 79 + 475 = |2 + Ts|, como

2 + 75 > 0 => 79 + 475 =2 + 75.

Questão 3.33
Resolução 01: Podemos escrever:

713 + 748 => A = 13 e B = 48, C = 7l32 -48 => C = 7169-48


=> C = 7Í2Í |C = 11| => 7a-Tb = J---Í - J-ÍA-C
2
=> 713+748
V 2 V 2 V 2 V2
=> 713 + 748 = 7Í2 + 7Í /. 713 + 748 = 273 +1.

Resolução 02: Podemos escrever:


7l3+ 748 = 713 + 473 => 7l3 + 748 = 7l + 2-273 +12 =s>
7l3 + T48 = ^12 +2-1-273 +(2T3)2 => 7l3 + 748 = ^(l + 2^3 'f

=> 713 + 748 = |1 + 273|, como 1 + 273 > 0 => 7l3 + 748 =1 + 275.
Os Segredos da Álgebra para IME/ITA/OLIMPÍADAS 349

Questão 3.47 (IMO-Longlist-1988 - Modificada)


Resolução: Podemos escrever:
.2
m = 11 + 672 => 01 = 9 + 2-372 +2 => m = (3 + 72)

n = 11-672 => n=9-2-3>/2+2 n = (3-72)2

p = 445 + 2 + 775 -2 = p - 775+2 - 775 -2 = 0


=> - 445 + 2 = q; - 445 - 2 - r; => |p + q + r =~Õ].
p+q+r=O => p2+q2+r2 =-2(pq + qr + rp).

=> (p)2 + (-775 + 2)2 + (775-2^ =

= -2((-775+2 )(-775-2 ) + p • (-775 + 2) + p ■ (-775-2))

=> p2 + 75 + 2 + 75 - 2 = -2^(75)2 -22 + p■ (-775+2- 775-2 i

=> P2+ 275 = -2(75 - 4 + p ■ (-p)] => p2+2^5 =-2(l-p•:2)

P2 =275 + 2 => p2 = 2(75+1)


=> p2 + 245 =-2 + 2p2 => p

=> p = ^2(75 +1) 445 +2 + 445 -2 = 42- 445 +1.


(11 + 672)■ 7l 1-672 - (11 - 672) ■ 7l 1 + 672
X=
(775 + 2 + 775 - 2) + (775 + 1)

(3 + 72)2 - J(3-72)2 - (3 - 72)2 - J(3 + 72)2


(72-775+i)+(775+i)
(3 + 72)2-(3-72)-(3-72)2.(3 + 72)
x= 72
(3 + 72).(32-(72)2]-(3-72).[32-(^)2)
=> X =

(3+ 72) (9-2)-(3-72) (9-2) 7(3 + T2)-7(3-T2)
X=
x= 72 72
21 + 772 -21 + 772 =>x = l^
42 42
350 10 Resoluções

Questão 3.48 (AHSME-1976) - Resposta: Alternativa A


Resolução: Podemos escrever:
a = 775 r 2 + 775 - 2 => a - 775 + 2 - 775-2 = 0 => -775 + 2 = b;
-775 - 2 = c; => |a + b + c=~Õ].
a + b + c = 0 => a2 + b2 +c2 = -2(ab + bc + ca).

=> (a)2 +(-775 + 2)2 +(775-2)2 =

= -2^(-775 +2^(-775 - 2j + a- (-775 + 2 ) + a-(-775-2^

=> a2 + 75 + 2+75-2 =-2 \|(75)2 - 22+a- (-775 + 2-775-2)

=> a2 +275 =-2(75-4 + a-• (-a)] => a2 +275 =-2(l-a2)


=> a2+275 =-2 +2a2 => a2 = 275+2 =5 a2 =2(75+1)

Í2(75+1) I775 + 2 + 77F - 2 = 72 ■ 775 +11.

m = 73 - 272 => m = 72-2-1-72+1 => m = ^(72 -1)2

=> m = ^72 - 1|, como 72 -1 > 0 .-. |73- 2T2 = 7ã-1| -

775 + 2 + 775 - 2 y, _ N 72-775+1


N= - 73 - 2^2 => N = (72-1)
775 + 1 775 + 1
N = 72 - 72+1 .-. |N = 1|.

Questão 3.49
Resolução 01: Podemos montara seguinte equação:
-A
72 + 75 => 4m3-3m (7A2-bJ- => 4m
A == 00 => 4m3
3 -3m-^22-õ)-2= 0
=> 4m.33 -3m (74-5)-2 = 0 => 4m.33-3m-(7^í) -2 = 0

=> 4m3 - 3m (-1)- 2 = 0 .’. 4m3 + 3m-2 = 0.

Usando o teorema do fator (*). note que a única raiz real dessa equação
*1
4m3 + 3m-2 = 0 é-,ou 1 . Assim, substituindo na outra expressão,
seja m = -
2 2
temos:
Os Segredos da Álgebra para IME/ITA/OLIMPÍADAS 351

2
n = m2 - n = l-(-1) n = —+ 1
4
1+4 5
=> n =------ n=—.
4 4
Logo: ^A + Vb = m + Vn ^27?5=1
V4 2 2

(*) Veja o teorema do fator no capitulo de Fatoração.

Resolução 02: Podemos escrever:


^2 + ^5 = ^2 + ^ = 3^2 + 2 = ^(A-tTb)3

=> (A2 +3B)A + (3A2 + b)7b = 2 + 2-

Por comparação, temos:


JI-
B = — => 3A2+B = 2 => 3A2+ —= 2 => 3A2=2--
4 44
=> 3A2 = -—— => 3A2 = — => A2 = — => A = +- (eq1) e
4 4 4 2 ' '
(A2+3BjA = 2 => A2+3-|!a = 2 => 4A3+15A = 8 (eq2)

Note que A = —i não satisfaz (eq2). Portanto A = 1


2
r r—x3
Logo: '2 + ^5 = 3 - + -
yl2 ’4J •2 2

Questão 3.50 (CN-1982) - Resposta Alternativa D


Resolução 01: Podemos montar a seguinte equação:
x/lO + 6V3 = \/10 + 'fa-62 = |^10 + x/lÕ8 ]

4m3 - 3m • (\/a2-b)-A = 0 => 4m3-3m- x/lO2 - 10810 = 0

=> 4m.33 -3m-(?/l00-108)- 10 = 0 => 4m3 - 3m ■ -10 = 0

=> 4m3 - 3m- (-2)-10 = 0 => 4m3+6m-10 = 0.


352 10 Resoluções

Usando o teorema do fator (*), note que a única raiz real dessa equação
4m3 +6m-10 = 0 é1,ou seja m = 1. Assim, substituindo na outra expressão,
temos:

n = m2 3^102 -108) => n = 12 - V(-8) => n = 1-(-2) => n = 3.

Logo: ^A-í- Vb = m + Vn V1O + VÍÕ8 = 1 + Vã.

(*) Veja o teorema do fator no capítulo de Fatoração.

Resolução 02: Podemos escrever:


Vio-r6Vã = ^(a + Vb)3 => (a2 + 3B)A + (3A2 + B) VÊ = 10 + 6 Vã .

Por comparação, temos:


B = 3 => 3A2+B = 6 => 3A2 + 3 = 6 => 3A2 = 6-3 => 3A2 =3

=> A2 =1 => A = +1 (eq1) e )a2 +3b)a = 10 => (a2+3-3)a 10

=> A3+9A = 10 (eq2)

Note que A = -1 não satisfaz (eq2). Portanto A = 1.

Logo: ^10 + 6^3 = ^(1 +Vã)3


=> Vio + sVã = 1 + Vã.

Questão 3.51 (CN-2011) - Resposta Alternativa B


Resolução 01: Podemos montar a seguinte equação:
V26-15V3 = x/26 - V3 152 = |V26 - V675 |

4m3-3m■ ^Va2-b)-A = 0 => 4m3 - 3m • í x/262 -675 -26 = 0

=> 4m.3J - 3m • (V676 - 675) - 26 = 0 => 4m3-3m(VÍ) -26 = 0

=> 4m3 - 3m-1 - 26 = 0 => 4m3-3m-26 = 0.


Usando o teorema do fator (*), note que a única raiz real dessa equação
4m3 - 3m - 26 = 0 é 2, ou seja m = 2. Assim, substituindo na outra expressão
temos:
n = m2 -3^262 -675) => n = 22-Vi => n = 4-1 => n = 3.
Os Segredos da Álgebra para IME/ITA/OLIMPÍADAS 353

Logo: ^A-7b = m-7n => ^26-1573 = 2-73.

(*) Veja o teorema do fator no capítulo de Fatoração.

Resolução 02: Podemos escrever:


^26 -1573 = ^(A-7b)3 => (A2 + 3B)A-(3A2 +BpB = 26-1573 .

Por comparação, temos:


B = 3 => 3A2+B = 15 => 3A2+3 = 15 => 3A2=15-3
=> 3A'2 = 12 => A2 = 4 => A = ±2 (eq1) e
(a2 +3B)a = 26 => (a2+3-3)a = 26 => A3 + 9A = 26 (eq2)
Note que A = -2 não satisfaz (eq2). Portanto A = 2.

Logo: ^26-1573 =^(2-73)3 => ^26-1573 = 2 - 73 .

Questão 3.52 (Stanford-2008)


Resolução 01: Podemos montar a seguinte equação:
'1777^45 J45 , 1777 = 3/45 + L/17Y2
E=? => £ = 3/— +
4 Y 4 4 V 44
V V l 4 J

C 3 45 L 289 c ,45 '2023


=> E = 3/---- + I7------- => E = ?l—4 +
V 4 16 16

4m3 -3m■ ^7a2 -B^ - A = 0 => 4m3 - 3m • ,12025 2023 ^= 0


V 16 16 4

=> 4m3-3m.^] ^ = 0
T = ° = 4m3-3m{3^)- 4

=> 4m3-3m-^j 45
— = 0 => 16m'13 - 6m - 45 = 0.
4
Usando o teorema do fator (*), note que a única raiz real dessa equação
3
16m 3-6m-45 =
16m-6m-45 =0 0 é
é ou seja m = -. Assim, substituindo na outra
2
expressão temos:
354 10 Resoluções

2023 7312 ,í~2 9 1


n = m2 => n = — - 3/— => n =------
16 \2) V16 4 2
9-2 7
=> n =------ n = —.
4 4
3ll 777 + 45 = 3+ /7 . 3ll7^745 = 3^T7
Logo: 7A + -Jb = m + Vn => ~2V4 "V 4 “2+ 2 ’
V 4

(*) Veja o teorema do fator no capítulo de Fatoração.

Resolução 02: Podemos escrever:


3Í17s/7+45 = Jl7^+45 Í45 + 17 /7=J—
V 4 V 4 4 V 4 2 V4 VV !
=> (a2 + 3B)a + (3A2+B)Vb=^ + Ç.^.

Por comparação, temos:


B = - => 3A2+B = — => 3A22 + - = — => 3A2
2 = —--
4 2 4 2 2 4
34-7
=> 3A2 = => 3A2 = — => A2 = - => A = ±- (eq1) e
4 4 4 2 1 1
(a2+3b)a = 45 fA2+3.Z^A = ^ 45
4A3+21A = 45 (eq2)
4 l 4 4

3 3
Note que A = — não satisfaz (eq2). Portanto A = -.
2 2

F 3 <
h7^7+45 J3 17 ] . 3 17^7+45 _ 3 V7
Logo: ?/ ^2 Y4J ” V 44 ” 2+ 2 ’
4
Os Segredos da Álgebra para IME/ITA/OLIMPÍADAS 355

Capítulo 04 - Expressões Algébricas

Questão 4.1 (CN-1952)


Resolução: Do enunciado, temos:
(x2 -5x + 9^(x + 3) = x3 -5x2 + 9x + 3x2 -15x + 27

[x2-5x + 9j(x + 3) = x3-2x2-6x + 27.

Questão 4.2 (CN-1952)______________________________________


Resolução: Do enunciado, temos:
>/l6x3y - ^25xy3 -(x-5y)• Jxy = 4x,/xy -5yyxy - x^/xy + 5y,/xy

.-. 7l6x3y-725xy3 -(x-5y)-7*y = 3x,/xy •

Capítulo 05 - Produtos Notáveis

Questão 5.1 (Noruega-1999)


Resolução: Do enunciado, temos:

7772 -662 I(777 — 66)(777 + 66) 7772 -662


= 777-66
777 + 66 777 + 66 777 + 66
7772 - 662
= 711.
777 + 66

Questão 5.2 (Noruega-1999)


Resolução: Do enunciado, temos:
x2y + xy2+x + y = 63 => xy(x + y) + (x + y) = 63 => (x + y)(xy+ 1) = 63

(x +y)(6+ 1) = 63 => 7(x + y) = 63 => x + y = — |x + y = 9|.

Então, podemos escrever:


x2+y2 = (x + y)2-2xy => x2+y2=92-2-6 x2+y',2 = 81-12

.-. |x2 +y2 =69|.


356 10 Resoluções

Questão 5.3 (Noruega-1998)

Resolução: Do enunciado, podemos escrever:


(a-b)2 = a2 +b2-2ab => (a-b)2 = 31-2-3 => (a-b)2=31-6

=. (a-b)2 =25 => a-b = ±>/25 .-. a-b = ±5.

Como a > b => a - b > 0, temos: a — b = 5 .

Questão 5.4 (Harvard-MIT-2012)


Resolução: Elevando 2a + 3b = 10 ao quadrado, temos:
2a + 3b = 10 => (2a + 3b)2=102 => 4a2 + 12ab + 9b2 = 100

=> 20 + 12ab = 100 => 12ab = 100-20 => 12ab = 80


u 80 .20
12 3

Questão 5.5 (AHSME-1958) - Resposta: Alternativa C.


Resolução: Do enunciado, temos:
_L+_L=a => x2^y2 = a => (x+y)2~2xy = a (x+y)2~2b
= a => = a => =a
x2 y2 x2y,22 (xy)2 (b)2

=> (x + y)2 - 2b = ab2 => (x + y)2 = ab2 + 2b .-. (x + y)2 = b(ab +2).

Questão 5.6 (Noruega-1998)


Resolução: Note que:
9991 = 10000-9 => 9991 = 1002 -32 => 9991 = (100 - 3)(100 + 3)
9991 = 97-103.

Observe que 97 e 103 são primos. Como p é o maior, temos p = 103, assim a
soma dos algarismos depél + 0 + 3 = 4.

Questão 5.7 (Putnam-2001-Modificada)


Resolução: Note que:
x4 ~(2n-4)x2 + (n-2)2 = (x2)2 -2-(n-2)-x2 +(n-2)2

a2 a
Os Segredos da Álgebra para IME/ITA/OLIMPÍADAS 357

=> x4-(2n-4)x2 + (n-2)2 = x2-(n-2)


. a '~b-'.

x4 -(2n-4)x2 +(n-2)2 =(x2 - n + 2)2.

Questão 5.8
Resolução: Note que:
-|2
E-f — 7x + 2 + 272x E-j = X
+ 2-72-Vx+(V2j2 => E, = 7x + 72
b a —- a b
!R2 (^)
Vx + 2 + 2>/2x =|Vx + >/2|.

E2 = 7l1 + 3\/8 => E2 = Vl1 +3-2^2 E2 = 79 +3-2^2+ 2 =>

E2 = 9 +2 >/2-3 + (^)2 => E2 = 3 + 72 7l 1 + 3^8 = |3 + 72|.


a b .
1(3)2 b 3

Vx + 2 + 2V2x = Vl1 + 3>/8 => |Vx + >/2| = |3 + ^|

í Vx + V2 = 3 + V2 7x = 3
|Ví + V2 = -(3 + 72) 7x + 72 =-3-72
í x=9
[Vx = -3 - 2x/2 <0 (não serve).

Resposta: x = 9.

Questão 5.9 (AHSME-1955) - Resposta: Alternativa B.


Resolução: Do enunciado, temos:
b (-p) c
r + s = — => r + s = -i—- => r + s = p; rs = - => rs = - => rs = q .
a 1 a 1
Então, podemos escrever:
r2 + s2 = (r + s)2 -2rs r2+s2=p2-2q.
358 10 Resoluções

Questão 5.10 (Singapura-2014)


Resolução: Do enunciado, temos:
_1
a+0=~- => a + 0 = -—;a0 = — => a0 = — => a0
a 3 a 3 3
Então, podemos escrever:

a 0 a2 +02 a 0 (a + 0)2 -2a0 « j


P a aP p a aP 0 a 2
3
1 2 1+ 6
“ , P . 9+3 “ + P=_9_ - “+P=Z.f_3'|
0 a _2 0 a _1 0 a 9 l ij
3 3
a^0=_21 f3'| • “+0=_Z
0+a 9\lJ "0a 3

Questão 5.11 (AHSME-1951) - Resposta: Alternativa D.


Resolução: Do enunciado, temos:
b c
r + s = —; rs = -.
aa
Então, podemos escrever:
1 1 r2 + s2 1 1 (r + s)2 - 2rs
r2+s2 = r2s2 => -j+-2 =
r s (^)2
b2 -2C
1 1 til -2-f^
,2
VaJ 1 1 a2 a

CJ
c^
a2
b2 -2ac
1 1 zzz c2
1 1
~2+~2 =
r s
b2 - 2ac a2
a2 Z2
1 1 b2 - 2ac
Os Segredos da Álgebra para IME/ITA/OLIMPlADAS 359

Questão 5.12 (Hungria)


Resolução: Do enunciado, temos:
(ac + bd)2 + (ad -bc)2 = (a2 + b2 )(c2 + d2)

=o + (ad-bc)2 =(1)(1) => (ad-bc)2=1-|

=> (ad-bc)2=l => ad-bc = ^ 1


ad-bc = —
2

Questão 5,13 (Harvard/MIT-2008)


Resolução: Do enunciado, temos:
(ac + bd)2 + (ab - cd)2 = (ac)2 + 2acbd + (bd)2 + (ab)2 - 2abcd + (cd)2

(ac + bd)2 + (ab - cd)2 = (ac)2 + (bd)2 + (ab)2 + (cd)2

(ac -t- bd)2 + (ab - cd)2 = c2(a2 + d2) + b2 (d2 +a2)

(ac + bd)2 +(ab - cd)2 = (a2 + d2 )(c2 + b2 j

=> (j) +(ab-cd)2 =(1)(1) => (ab-cd)2 = 1-1

=> (ab-cd)2 = — => ab-cd = ,p .-. ab - cd = .


v ' 9 V9 3

Questão 5.14 (Eõtvõs-1933)_______________________________


Resolução: Do enunciado, temos:
(ac + bd)2 + (ab - cd)2 = (a2 + d2 )(c2 + b2 j

=> O2 +(ab-cd)2 = (1)(1) => (ab-cd)2 =1 ab-cd = 1.

Questão 5.15 (Índia-1998-Modificada)______________________


Resolução: Do enunciado, temos:
E = (ac + 3bd)2 + 3(ad-bc)2
=> E = (ac)2 + 2ac • 3bd + (3bd)2 + 3^(ad)2 -2ad■ bc + (bc)2^|

=> E = (ac)2 + 6acbd + 9(bd)2 + 3(ad)2 -6adbc + 3(bc)2


=> E = (ac)2 + 9(bd)2 + 3(ad)2 + 3(bc)2
360 10 Resoluções

=> E = a2 (c2 + 3d2) + 3b2 (3d2 + c2)

(ac + 3bd)2+ 3(ad-bc)2 = (a2 + 3b2)(c2 + 3d2)

Questão 5.16 (Hong Kong-2002)


Resolução: Podemos escrever:
(ac + 5bd)2 + 5(ad -bc)2 = ^a2 + 5b2 )(c2 +5d2)

(xlYl + 5x2y2)2 +5(x2y-| -xiY2)2 =(x,2 +5x|)(yf +5y^)

=> (VlÕ5)2+5(5)2 =(10)(y2+5y2) => 10(y2+5y2) = 105+125

=> io(y2 + 5y2) = 230 => y2+5yl=^y .-. y2+5y2=23.

Questão 5.17 (Eslovênia-2010/ Kosovo-2013)


Resolução: Tirando o MMC, temos que:
a^b = 6 (a + b)2+(a-b)2 2(a2+b2]) a2 +b2
a-b^a + b (a-b)(a + b) = 6 => a2-b2 = 2 => a2-b2 = 3

=• a2+b2 = 3^a2-b2) => a2+b2 = 3a2-3b2 => 3b2+b2 = 3a2-a2

=> 4b2 = 2a2 => 2b2 = a2 => |sb6 =a6|.

Assim, chamando a expressão de E, temos:


(a3+b3)2+(a3-b3)2 2(a6+b6)
c a3+b3 a3-b3
tz = ------------ +------------- => E = => E =
a3-b3 a3+b3 (a3-b3)(a3+b3) a6-b6

2(8b6 +b6) 2 (9b6) . |a3+b3 . a3 - b3


18
=. E = => E = 7b6 " ’ ’ + " "
8b6 - b6 a3-b3 a3+b3 7

Questão 5.18 (Júnior Balkan-1997)


Resolução: Tirando o MMC, temos que:

(x2+y2) +(x2-y2) 2(x4 +y4)


x2 + y2 x2 -y2
= k => =k
x2-y2 x2 + y',2 (x2-y2)(x2 + y2) x4-y4
Os Segredos da Álgebra para IME/ITA/OLIMPÍADAS 361

x4+y4 k x4 -y4 2
" x4-y4 2 x4 k

x4 + y4 x4 - y4 k +2 (x4+y -4)2+(x4-y4)2 k2 +4
x4-y4 ' x4 + y4 2+k (x4 2k
-y4Xx4+y4)
2(x8 +y8)
k2 +4 x8 + ya k2 + 4 4k
=>
x8-y8 2k x8-y8 4k x8 + y8 k2 + 4 '

Assim, chamando a expressão de E, temos:


(k2+4)2+(4k)2
x8 + y8 x8-y8 k2+4 4k
E= => E = E=
Ãv x8 + y8 4k "k2+4 4k(k2+4)

k4 +8k2 +16 + 16k2 x8 + y8 x8!-y8 k4 +24k2 +16


=> E = x8-y8+i°
4k3+16k x8 + y8 4k3+16k

Questão 5.19 (CN-1954)_____________________________________ ___


Resolução: Usando o produto da soma pela diferença, temos que.
E = 16x4-1 => E = (4x2)2-12 => E = (4x2+l)(4x2-1)

=> E = (4x2 + l]((2x)2 -ij .-. 16x4-1 = (4x2 +l}(2x + 1)(2x-1)

Questão 5.20 (Harvard-MIT-2009)_____________________________


Resolução: Usando o produto da soma pela diferença, temos que:
E = 112 -12 +122 - 22 +132 - 32 +... + 202 -102
=> E = (11 +1)(11 - 1) + (12 + 2)(12 - 2) +... + (20 + 10)(20 - 10)
=> E = (12) (10) + (14) (10) + ... + (30) (10)
=> E = 120 + 140 + 160 + ... + 300 E = 2100.

Questão 5.21 (Turquia-2007-IVIodificada)


Resolução: Usando o produto da soma pela diferença, temos que:
E = (l002 -992)(992 -982)...(32 -22)(22 -12)

=> E = (100 +99)(100-99)(99 + 98)(99-98)...(2+ 1)(2-1)


=> E = (199)-(1) (197) (1)-...-(3) (1) .-. E = 199-197-195-...-3.
362 10 Resoluções

Questão 5.22 (OCM-1998-Modificada)_____________________________


Resolução: Usando o produto da soma pela diferença, temos que:
E = 12-22+32 -42 + ...-19982 +19992

E = 12 + (-22 +32) + (-42 +52)-62 + ... + (-19982 +19992) =>

E = 12 + (3 + 2)(3 - 2) + (5 + 4)(5 -4)+ ... + (1999 + 1998)(1999-1998)

=> E = 1 + (5) (1) + (9)-(1) + ... + (3997)-(1) => E = 1 + 5 + 9 + ... +3997


.-. E = 5993002.

Questão 5.23 (Moscou 1945)______________________________________


Resolução: Usando a generalização do produto da soma pela diferença,
temos que:
b)(a + b)(a2 + b2)(a4 + b4 )■...• (a2 +b26
-b'2? =(a-
a',2?-h

a2?-b2?
= |a-b|.
b)(a2 +b2j(a4 +b4)-...^a,6
‘ +b26

Questão 5.24 (Moscou 1945)_____________________________________


Resolução: Usando a generalização do produto pela soma, temos que:
a2k -b2k = (a- b)(a + b)(a2 + b2j(a4 + b4)-...(a2 +b2 |

,2k h2k
_________________ a" -b‘__________
= |a - b|.
(a + b)(a2 +b2^a4 +b4)-...(a2k 1 + b2k'1

Questão 2.25 (Moscou 1946)


Resolução: Podemos escrever de modo a obtermos o produto da soma pela
diferença, logo:
E = (l- x + x2 - x3 +... • -x:" + x100).(l + x + x2 +... + x99 + x100)

E = [(l + x2+... + x1°°) + (-x...-x:")].[(l


‘ + x2+...+ x
x100
10°)+(:x X:")]
E = [(l + x2+... + x10°)-(x + ...+ x:")]-[(l + x2+... + x100) + (x ... + x")]
,2
=> E = Í1 + x2+...+ x100)2-(x + x3... + X,99
Os Segredos da Álgebra para IME/ITA/OLIMPÍADAS 363

2 -i2
’l-(x102
--i)]2 rx(xi°°-i)7 '(x102-1 x101-x)
=>E = => E =
X2 -1 X2 -1 x2-1 x2-1

|)2 (x101 2 -)■ 2


(x102 01
= E.l02-1 2 => E =
(x2-1)' (x2^)2

(*;102_1 + x101_x.j(x102_1_xW1
=> E =
(x2-1)2

[x101 (x + 1)-(x + 1)][x101 (x-


1) + (x-1)]
=> E

= H
„ (x + 1)(x-1)[x:101-l][x101 (x101 .101
=> E =-----------------
.2 X2 — 1
12)
(x2-t)'

E = -*':202 -l)
E (x2-1)[(x2),00+(x2)"+... + (x2)1]
x2 -1 x2-1 X2 -1

+ x198 7777],
Questão 5.26 (CN-2005) - Resposta: Alternativa A.
Resolução: Podemos escrever:
(a2 -b2 j -(2ab)2
a4 +b4 -6a2b2 E a4 + b4 - 2a2b2 - 4a2b2
E= => E = =>E =
a2 - b2 + 2ab a2 - b2 + 2ab a2 - b2 2ab
(a2-b2 +2ab)(a2-b2-2ab) a4+b4 6a2b2
= a2 -b2 -2ab .
a2-b2+2ab a2-b2+2ab

Questão 5.27 (AHSME-1951 / CN-1998) - Resposta: Alternativa A.


Resolução: Podemos escrever:
~2
E = 11 +
x4 -1 1 x8-2x4+1 x2
=> E = ,1 +
2x2 2 4x4 2
364 10 Resoluções

Mx4+x8-2x4+1 x2 x8+2x4+1 x2
=> E = => E =
4? T 4? T
T c x4 +1 x2
x4 +1 x4 +1-x4
=> E =
■E = è-
= E-^“
=> E =
2x2 2 2x2

Questão 5.28
Resolução: Podemos escrever:
E = (a + b-c + d)2 + (a + b- c-d)2-2^(c -a-b)2 -dzJ
=> E = (a + b-c + d)2 (a + b- c-d)2-2[c-a-b + d][c-a-b-d]

=> E = (a + b-c + d)2 (a + b-c-d)2-2(a + b-c-d)(a + b-c + d)

=> E = [(a + b- d) - (a + b-c -d)]Z

E = [a + b- c + d- a- b + c + d]2 => E = [2d]2 E = 4d2 .

Questão 5.29
Resolução: Note que:
(a-b)(a2 +ab + b2) = a3-b3 e (a+ b)(a2-ab+ b2) = a3 + b3

E = (a + b)(a -b)(a2 + ab + b2)(a2 -ab


- + b2)(a12 +a6b6 +b12)

=> E = (a3 + b3)(a3-b3)[(a6)2 a6b6 + (b6)2]

E = (a6-b6)[(a6)2+asb6+(b6)2] => E = (a6)3-(b6)3 .-. E = a18-b,18


18.

Questão 5.30 (CN-1991) - Resposta: Alternativa A.


Resolução: Podemos escrever:
(a2 -b2 -c2 -2bcj(a + b - c) |^a2 -(b2 +c2 -r-2bc^(a + b-c)
=>E
(a + b + c)(a2 + c2 -2ac - b2 j (a + b + c) (a -c)2 - b2 j

|^a2 -(b + c)2J(a + b-c)


(a + b + c)(a -b - c)
E |E = 1|.
(a + b + c)(a - c + b)(a - c - b) (a+ b+ c)(a-c-b)
Os Segredos da Álgebra para IME/ITA/OLIMPÍADAS 365

Questão 5.31 (Harvard-MIT-2007)


Resolução: Podemos escrever:
x3-y3=28 => (x - y)(x2 + xy + y,2 ) = 28 => 4(x2 + xy + y 2 ) = 28

=> x2 + xy + y2 = 7 |x2 + y>2


: = 7 - xy].

Assim, elevando a primeira equação ao quadrado, temos:


(x-y)2 = 42 => x2 -2xy + y2 =16 => 7-xy-2xy = 16 => 7-16 = 3xy

9 i----------- 1
=> 3xy = -9 => xy = ~3 ■ lxy = ~3I•

Questão 5.32 (CN-2006) - Resposta: Alternativa D.


Resolução: Podemos escrever:
x^x2 + x - yj + y2 (y +1) x3 + x2 - xy + y3 + y2
E= => E =
x2+y2-xy x2 +y2 - xy

(x + y)(x2 - xy+ y2 ) + x2 - xy + y2 (x2 -xy + y2][(x + y) + l]


E= E = ^—
Ml
x2 + y2 - xy x2 + y2 - xy

E = x + y +1.

Questão 5.33 (CN-1980) - Resposta: Alternativa B.


Resolução: Podemos escrever:
(2x2 -4x + 8)(x2 -4) 2(x2 -2x + 4j(x-2)(x + 2)
E= => E =
V2-x3 + VÍ28 ■72 x3 +8^2

2(x2 -2x + 4j(x-2)(x + prz^2K+4^(x- 2)


2) E = -|=-
=> E =
V2.(x3+8) V2

.. E = 72 (x-2).

Questão 5.34 (CN-1983) - Resposta: Alternativa B.


Resolução: Podemos escrever:
(x2z + zy2 +2xyz)(x2 -y2) z(x + y)2(x-y)(x + y).E = z(x_y)
E= =>E =
x3 +3x2y + 3xy2 +y3 (x + y)3
366 10 Resoluções

Questão 5.35
Resolução: Podemos escrever:
(a + b)2 -4^a2 + b2)
______ a2 + 2ab + b2 - 4a2 - 4b2______
E= => E =
(a3 + b3)2_(a2+b2)3 a6 + 2a3b3 + bs-)a6 + 3a4b2 + 3a2b4 + b6)

2ab-3a2 -3b2 2ab-3a2 -3b2


=> E = => E =
2a3b3 -3a“b2 -3a2b4 2a3b3 -3a4b2 -3a2b4
2ab-3a2 -3b2
=> E =
a2b2 )2ab-3a2 -3b2) a2b2

Questão 5.36 (AHSME-1951) - Resposta: Alternativa C.


Resolução: Podemos escrever:
\2 2 2
(x + 1)2-(x2 -X + 1I (x-1)2 -)x2 + X + l)
E=
(x3+1)2

2
(M 2

=> E
(x + 1)-(x2 -x + 1
)]_ [<»-<) R
(x3+l)2

2 2

=> E = (>3-f
(x3+1f_ (x3+lf (x3-l)4

Questão 5.37
Resolução: Podemos escrever:
E = a3-a2b-ab2+ b3 = a(a2-b2)-b(a2-b2) =

= (a-b)(a2-b2) = (a-b)(a-b)(a + b) E = (a + b)(a-b)2.

Questão 5.38 (Harvard/MIT-2000)


Resolução: Usando o resultado anterior para a = 2000 e b = 1999, temos:
20003 - 1999 ■ 20002 -19992 • 2000 + 19993 = (2000 + 1999) • (2000 - 1999)2

= 3999-12 = 3999.
Os Segredos da Álgebra para IME/ITA/OLIMPÍADAS 367

Questâo 5.39 (Harvard/MIT-2007)


Resolução: Podemos escrever:
a3-1

a3 +1
____ K
(a-1)(a2+a + l)

(a + 1)(a2 - a +1)
23 -1 33-1 43 -1 53 -1 63 -1

23 +1 33 +1 43 +1 53 +1 63 +1

(2-1)(22 + 2 + l) (3-1){32 + 3 + l) (4-1)[42+4 + l)


(2 + 1)(22- 2 + l) (3 + 1){32 -3 + l) (4 + 1)(42-4 + l)

(5~1)(52 + 5 + l) (6-1)(62+6 + 1)
(5 + 1)(52 -5 + l) (6 + 1)(62-6 + 1)

23 -1 33-1 43 -1 53 -1 63 -1 E-x
EX MM x M 0-x m 43
M X tf*
23 +1 33 +1 43 +1 53 +1 63 +1 [U 3 J/f-X ^X 0'X

Note que esses primeiros termos (destacados) se cancelam, sobrando o 3 e o


7 no denominador.
Por outro lado, os termos não destacados também se cancelam, sobrando o 43
no numerador e o 3 no denominador. Assim, temos:
23 -1 33-1 43 -1 53 -1 63 -1 43 Í431
23+1 33 +1 43 + 1 53 + 1 63+1 3-3-7 [63]’

Questão 5.40 (Stanford-2012)___________________


Resolução: Podemos escrever:
a3 ~1 _ (a-1)(a2+a + l) 33 -1 43-1 163 -1
23 -1 ___________

a3+1 (a + 1)(a2-a + lj 23 +1 33 +1 43 +1 163 +1

(2-1){22 +2 + l) (3-1)(32 +3 + l) (4-1)(42 +4+l)

(2 + 1)(22-2 + l) (3 + 1)(32-3 + l) (4 + 1)(42-4 + l)

(14-1)(142 +14 + l) (15-1)(l52+15 + l) (16-1)(l62+16 + 1)

(14 + 1)(142-14 + l) (15 + 1)(l52-15 + lj (16 + 1)(l62 -16 +1)

23 -1 33 -1 43 -1 163 -1
23 +1 33 +1 43 +1 163 +1
368 10 Resoluções

B X 2-X ■X X M■ 273
Jãí-3 g-x IsJ-X ■X. ELzbX'
Note que esses primeiros termos (destacados) se cancelam, sobrando o 2 no
numerador e o produto 16-17 no denominador.
Por outro lado, os termos não destacados também se cancelam, sobrando o
273 no numerador e o 3 no denominador. Assim, temos:
1 , _ 91
23 -1 33 -1 43 -1 163 -1 1-X X’ [~9Í~|
23-r1 33 +1 43 +1 163 +1 X-17 X h36r
"a T”

Questão 5.41
Resolução: Do trinómio quadrado perfeito, temos:
(a+ b)2 = a2 + 2ab + b2 => (a+ b)2 = k + 2x => (a+b)4 = (k + 2x)2

.-. (a-r b)4 = k2+4kx + 4x2 .

(a-b)2 = a2 - 2ab +b2 => (a-b)Z=k-2x => (a - b)4 = (k - 2x)2

(a-b)4 = k2 - 4kx+ 4x2 .

Questão 5.42
Resolução: Elevando ao quadrado, temos:
(x2 + y2)2=12 x4 +2x2y2 +y'4=1 => 2x2y2+ —= 1 => 2x2y2=1- —
18 18
1
=> 2x2y2 = — => x2y2= — xy = -
18 36

Questão 5.43
Resolução: Tirando o MMC, temos que:

(x2 + y2)2+(x2-y2)'2 2(x4


x2 + y2 x2-y,2:
= k => = k => = k =>
x2+y2 (x2-yz)(x2 + y2) x4 — y,4

2(x4 + y4 ) = k(x4-/’4) => 2x4 + 2y4 = kx4 - ky''4 => ky4 + 2y4 = kx4 -2x4

(k + 2)y4 = (k-2)x4 => y'.4


=fMX
lk + 2j 4 y
.16 fk-2Y
lk + 2jfx16
Os Segredos da Álgebra para IME/ITA/OLIMPÍADAS 369

Assim, chamando a expressão de E, temos:

x 8 + y8x8 -y8 l )2+(x8-y8)2


(x8 + y'8 2(x16+y16)
E= => E = => E =
x8-y8 + x8 + y8 x16-y16
(x8 -y8)(x8+y8)
(4 (k + 2)4 x16 + (k - 2)4 x16
2 x16+f^Y X16 2
lk + 2j (k^)4
=> E = => E =
x^-ffS^fxie (k + 2)4 x16 - (k - 2)4 x16
Vk + 2 J (k + 2)4

2-[(k + 2)4 +(k-2)4] 2 |V(22 +k2)2 +8-22 -k2J


=> E => E
[(k + 2)4-(k-2)4] 8-2-k-(22 +k2)

2 |^2-(4 + k2)2 + 32k2


4-|jl6 + 8k2 +k4) + 16k2J
=> E => E
16k(4+k2) 16k-(4 + k2)

16 + 8k2 +k4 +16k2 x8+y',8 x8-y8 k4 + 24k2 +16


=> E =
4k(4 + k2) " x8-yJ
l + x8 + y8 4k3 +16k

Questão 5.44 (IMO-Longlist-1992-Adaptada)


Resolução: Fazendo 525 = x , temos:

E=
5125-1
525 -1
=> E =
525 5 -1
525 -1
=> E = n5-152S-1
=> E-——
x-1
(x-1)(x4+x3 + x2 + x + 1)
= x4 + x3 + x2 + x + 1
x-1
E = (525)4 +(525)3 +(525)2 + 525
+1

5125-1
= 51°° + 575 + 550 + 525 +1.
" 525 -1
370 10 Resoluções

Questão 5.45_____________________________________________ ___


Resolução: Multiplicando cada equação por x + y. temos:
ax + by = 2 => (ax + by)(x + y) = 2(x+ y)
=> ax2 + axy + bxy+ by2 = 2(x + y) => 20+ (a+ b)xy = 2(x + y).
ax2+by2=20 => (ax2+by2 j(x + y) = 20(x + y) =>

ax3+ax2y + bxy2+by3 = 20(x + y) => 56 + (ax+ by)xy = 20(x + y)

=> 56 + 2xy = 20(x + y) => |28 + xy = 10(x + y)|. (eq1)

ax3+by3=56 => (ax3+by3 j(x+ y) = 56(x + y) =>

ax4 +ax3y+ bxy3 + by4 =56(x + y) => 272 + (ax2 +by2 jxy = 56(x + y)

=> 272 + 20xy = 56(x + y) => |68 + 5xy = 14(x + y)|. (eq2)

Resolvendo o sistema formado por (eq1) e (eq2), temos:


x + y = 2exy = -8.
ax4+by4 =272 => (ax4+by4 j(x + y) = 272(x + y)

ax5+ax4y + bxy4+by5 =272-(x + y)

=> ax5 + by5 + (ax3+by3 jxy = 272-(x +y)

=> ax5 + by5 + 56 (-8) = 272-2 => ax5+by5 -448 = 544

=> ax5 + by5 = 544 + 448 |ax5 +by5 = 992|.

Questão 5.46 (AIME-1990/Harvard-MIT-2009)______________________


Resolução: Multiplicando cada equação por x + y, temos:
ax + by = 3 => (ax + by)(x + y) = 3(x + y)
ax2 + axy + bxy+ by2 = 3(x + y) => 7 +(a+ b)xy = 3(x + y).
ax2+by2=7 => (ax2 + by2j(x + y) = 7(x + y)

=> ax3 + ax2y+ bxy2 + by3 = 7(x + y) => 16 + (ax + by)xy = 7(x + y)

=> |l 6-r 3xy = 7(x + y)|. (eq1)

ax3+by3=16 => (ax3 + by3j(x + y) = 16(x + y) =>

ax4+ax3y+ bxy3+by4 = 16(x +y) => 42 + (ax2 + by2jxy = 16(x + y)


Os Segredos da Álgebra para IME/ITA/OLIMPÍADAS 371

=> |42 + 7xy = 16(x + y)|. (eq2)


Resolvendo o sistema formado por (eq1) e (eq2), temos:
x + y = -14 e xy = -38.
ax4+by4=42 => (ax4 +by4)(x + y) = 42(x + y)

=> ax5+ax4y + bxy4 +by5 =42-(x + y)

=> ax5+by5 + (ax3+by3 jxy = 42-(x + y)

=> ax5 +by5 + 16 (-38) = 42 (-14) => ax5+by5 -608 =-588

=> ax5 + by5 = -544 + 608 |ax5 +by5 = 2o|.

Questão 5.47 (OBM XXXI - 2a Fase - Nível 2)__________


Resolução: Multiplicando cada equação por x + y, temos:
ax + by = 1 => (ax+ by)(x + y) = x + y
=> ax2+axy + bxy + by2 = x + y => 2 + (a+ b)xy = x + y.

ax2+by2=2 => (ax2 + by2j(x + y) = 2(x + y)

=> ax3 + ax2y + bxy2 + by3 = 2(x + y) => 5 +(a: by)xy = 2(x + y)

=> |5 + xy = 2(x Zyj]. (eq1)


ax3+by3=5 => (ax3+by3j(x + y) = 5(x + y)

=> ax4+ax3y+ bxy3 + by4 = 5(x + y) => 6 + (ax2 + by2)xy = 5(x + y)

=> |6 + 2xy = 5(x + y)|. (eq2)

Resolvendo o sistema formado por (eq1) e (eq2), temos:


x + y = -4 e xy = -13.
ax4+by4=6 => (ax4 + by4 j(x + y) = 6(x + y)

=> ax5 + ax4y + bxy4 +by5 =6 (x + y) =>

ax5 + by5 + (ax3 + by3)xy = 6 ■ (x + y) => ax5 +by5 + 5 ■ (-13) = 6 • (-4)

=> ax5+by5-65 =-24 => ax5 + by5 =-24+ 65 |axs +by5 = 4l|.
372 10 Resoluções

Questão 5.48 (AMC-2007) - Resposta: Alternativa D.


Resolução: Elevando ao quadrado duas vezes, temos:

a + a‘ = 4 =>(a + a~1) =42 => a2 + 2a a"1 + a‘ 2 =16 => a2 + a


+ a" = 16-2

a2 +a = 14 => (a2 +a'r2)2=142 : a4 + 2a2 a' + a‘ = 196


a4 +a = 196-2 a4+a' = 194.

Questão 5.49 (AHSME-1952-1954 / CN-1986) - Resposta: Alternativa C.


Resolução: Do enunciado, temos:
1 ,2 1 1 .3
X+— = 3 => x 4— = x/3, x -i— >0 =>
X I x x
.2 .
=» x3+3x2.i+3x.m + 4- = 3v^ 1 3x/3
+ x3
x <X J1 xJ

x3+-^ + 3-73=373 /. x3
x3 r°
Questão 5.50 (CN-2014) - Resposta: Alternativa E
Resolução: Do enunciado, temos:
3
x + - = 9 =>
x ÍX + Í=92 => x2 +2 x-
X
+ 4- = 81 => x2 A = 81-6
x2 x2
=> x2+4-
+4t = 75.
x2
Então, podemos escrever:

3r
rx — ( 3>2
= x2 +—9--2x
_ • 3— I X — — I =75-6 =>
l xJ x2 x v xJ
x---- = ±x/69 .
x
Assim, a soma dos algarismos dea = 6 + 9 = 15.

Questão 5.51
Resolução: Usando a forma prática, temos:

x - = k => x3 +-1-= k3 -3k => m


m== 43-3-4 m = 64 -12 m = 52.
x x3
Os Segredos da Álgebra para IME/ITA/OLIMPÍADAS 373

x + - = k=> x4 + —= k4-4k2+2 => n = 44 -4-42 +2 => n = 256-64 + 2


X X4

/. n = 194.

Então, podemos escrever:


m + n 52 + 194 m+n 246 m +n 123
----- =----------------------
m-n 52-194 m-n -142 m-n 71

Questão 5.52 (AMC-2007) - Resposta: Alternativa D.


Resolução: Usando a forma prática, temos:
x + i = k => X4 + 4 =
=kk44 -4k + 2 => a4 +-Jr = 44 - 4 ■ 42 + 2
-4k22 +2
x x4 a4
=> a4+4-= 256-64+ 2 .-. a4 1
+ 4 = 194.
a4 a4

Questão 5.53 (Stanford-2010)


Resolução: Usando a forma prática, temos:
x2+4 = k2-2 => 7 = k2-2 => k2 = 9 => k = ±v/9 k = ±3.
x2
x5+4
. = k5-5k3_ x5
=> J+4
+5k - = (±3)5-5-(±3)3+5(±3)
xü xb
x5+-4r = ±243 + 5-27±15 x5 + 4 = ±123.
xb x5

Questão 5.54
Resolução: Usando a forma prática, temos:
x5 + 4 = k5-5k3 + 5k =>
=>xx55 +-1-= (1)5 - 5 ■ (1)3 +-5 ■ (1) => x5+4 = 1"5+ 5
xb xs X’ X
+ 4=1 => 5 X5 T = 1.
= X5 4 = 5/í 5 x5
x5 V x5 V 7
Questão 5.55 (Singapura)
Resolução: Usando a forma prática, temos:
x2 +1 .
x2-4x + 1 = 0 => x2+1 = 4x => --------- = 4 x+-=4 .
x x
374 10 Resoluções

,3
=> x3 + ^-= k3 - 3k => x3 = k3-3k-k3
x3 x3

• x3+ —- 2 |3 = —3k.
X3 X

x + - = k => x6 + 4 = k6-6k4+9k2-2
X x6

= x6+4 x6
(x + ^j +2 = k6-6k4+9k2-2-kB+2

.-. x6
1 ( 1V 6 + 2 = -6k4 +9k2.
x6 l xj

Então, podemos escrever:

X6 _1 _ í n6
E=—
+ X6
lx+xj +2 => E =
-6k4 + 9k2
=> E = 2k3-3k
x3+i-
X3
(lX+xJn3 -3k

\’.2
x6
+ x6
=> E = 2 43-3-4 => E = 128-12 — = 116.
x3 1
X3

Questão 5.56
Resolução: Usando a forma prática, temos:
x + - = k => x3+4- = k3-3k => :x3 + -L = (^)3-3.^
x x3

=> x3+4 = 272-3^2 .-. x3 + 4- = -j2.


x3 x3
x + - = k => x4 +4 = k4-4k2 +2 -x4+A = (72)4 -4(^)2+2
x x4
=> x4 + 4 = 4- 8 + 2 .-. x4
+ 4 = "2 ■
x4 x4
Os Segredos da Álgebra para IME/ITA/OLIMPÍADAS 375

Então, podemos escrever:


x7+1 + x + A = 2V2
X X7

=> X7+ 72+4- = 2^2 => x7+4- = 2V2-^


X7 x
■ x7
+ 4 = 72.
X

Questão 5.57
Resolução: Usando a forma prática, temos:
x + - = k => x3+4- = k3-3k => (?/r)3+—!—= 33-3-3 => r + - = 27 —9
x x3 r

1
r + - = 18 .
r
Repetindo o procedimento, temos que:
x3+4-=k
= k3-3k =>
3-3k 3 +4 = 183-3-18
=> rr3 r3 + 4 = 5832 - 54
x3 r3
r3 + 1 = 5778.

Questão 5.58
Resolução: Tirando o MMC dos Índices, temos:
1
= 14 1^3-4==14 => m3- = 14
tf 1 1 i

67F J_= 6.
+Í=6~ R2
(Prove)

Fazendo 1\/r = x , podemos escrever:


tf = 14 => (W----- 1í- = 14 => x3-A = 14
(12<
= 6 => (1^)2 1 4- = 6 . (Prove)
= 6 => x2
tf
(W xz
376 10 Resoluções

Usando a forma prática, temos:


x — — = k => x3—— = k3+3k k3 +3k = 14 k3 + 3k-14 = 0.
x x3

Por inspeção (teorema do fator), temos que k = 2, assim:


x-- = k => x2+^- = k2+2 => x2+^- = 22+2 .-. x2
x x2 x2
+4=
x2
6.
Como queríamos provar.

Observação: Veja o teorema do fator no capítulo de Fatoração!

Questão 5.59 (CN-1984) - Resposta: Alternativa E.


Resolução: Do enunciado, temos:
x+y+z=16 => (x + y + z)2=162 |x2 + y2 + z2 + 2(xy + xz + yz) = 256|.

Então, podemos escrever:


2 2 2 x y
X + _y_ z
z 8 2xy + 2xz + 2yz + x2 + y,22 + z2 8
+—
x y z yz xz xy 3 xyz 3

256 8
=> 8xyz = 768 => xyz = xyz = 96.
xyz 3

Questão 5.60 (CN-1999) - Resposta: Alternativa C.


Resolução: Do enunciado, temos:
m+n+p=6 => (m + n + p)2 = 62

=> m2 + n2 +p2 +2(mn + mp + np) = 36 => m2+n2+p2+2(11) = 36 .

=. m2+n2 + p2 =36-22 [jm2 +n2 +p2 — 14|.

Então, podemos escrever:


m n p m2 + n2 + p2 m n p 14 m n P
np mp mn mnp np mp mn 2 np mp mn
Os Segredos da Álgebra para IME/ITA/OLIMPÍADAS 377

Questão 5.61 (CN-2011) - Resposta: Alternativa C.


Resolução: Do enunciado, temos:
111 a+b+c a b c a b c
----- 1-------+f — — = p => = p; =q
ab bc ac abc b a a c c b
=>-b + [a l- + l|_a— + -c -c —b I[aa b
a c a
q+ —
b c
c b c a b c
1 (a
=> - (a + b + c) + i- (a + b + c) + - . + b + c) = q + 1 + 1 + 1
c '
ab - ac + bc
+ — l = q + 3=>(a + b + c)l = q+3
<a + b+^ lã+b abc
(a+b+c 2 2
■ (ab + ac + bc) = q +3 => pr=q+3 => (pr) =(q + 3)
l abc
=> q2 + 6q + 9 = p2r2 q2 + 6q = p2r2 - 9 .

Questão 5.62 (Harvard/MIT-2008)


Resolução: Do enunciado, temos:
b (-9)
p + q + r = - —- => |p + q + r = 9|;
p + q + r = — => p+q+r=--
a 1
c 8
pq + pr + qr = - => pq + pr + qr = - => |pq + pr + gr = 8|;
a 1
d 2
pqr = — => pqr = --
a
|pqr =
3=
Então, podemos escrever:
E= 1 1 p = P2q2 + P2r2 +q2r2
E=
~P2 q2 r2 ’ p2q2r2

(pq + pr + qr)2 -2pqr(p + q + r) 82-2(-2)-9 64-36


E= => E =---------- - => E =
(PAr)2 (-2)2 4

100
=> E =----- • — + — + — = 25.
4 P2 q2 r2

Questão 5.63 (AHSME-1981) - Resposta: Alternativa A.


Resolução: Podemos escrever:
E = (x + y z)~1(x + z' )(*y yz + xz) 1[(xy) 1 + (yz) 1 + (xz)
+ y'
378 10 Resoluções

_1_
=> E =
í 1
Vx+y+z X7HK 1
xy + yz + xz Mxy
1U + A.
yz xz

=> E = f x + y1 + z ) f xy + yz + xz)II í xy + yz1 + xz 'j ("x + y + z


J l xyz J xyz

=• E = •z
??? A E = X ■y

Questão 5.64 (AHSME-1991) - Resposta: Alternativa C.


Resolução: Fazendo a = x + 7x2 -1 e b = x - 7x2 -1, temos:

a + - = 20
=> a + a = 20 => 2a = 20 => a = 10.
b
| ab = 1
x + \/x2-1 = 10 =. x2-1=10-x => x2-1 = (10-x)2
x2 -1 = 102 -20x + x2 => 20x = 100 + 1
icrT
X =------- ou |x =5,05|.
20
E = x2 + 7x4 -1 +------ 1=^ => E = x2 x4 -1 + x2 -Vx4 -1
x2 + 7x4-1
=. E = 2x2 => E = 2(5,05)2 :=> E = 2(25,5025) .'. |E = 51,005|.

Questão 5.65
5 + 721
Resolução: Fazendo = x, podemos escrever:
2

2 1 2 2 1 2 5-721 2 2(5-721)
x 5 + 721 x 5 + 721 x ~ 5 + 721 5-721 x 52-(72Í)2
2

2 2(5-721) 2(5-721) 2
x 25-21 x 4 X

2 =-----------------
5 + ^1 5-721
F x+-
10
X 2 2 X 2

Usando a forma prática, temos:


x + - = k => x5+4-= k5-5k3 + 5k x5 + 4- = 55-5-53 +5-5
x x5 xb
x5 +4-= 3125-625+ 25 .-. x5 + 1 2525
x5 x5 "
Os Segredos da Álgebra para IME/ITA/OLIMPÍADAS 379

Questão 5.66

Resolução: Fazendo
1 + 75 = x , podemos escrever:
2
2 1 2 2 1 _ 2 1-^5 2 2(1-75)
X 1 + 75 X 1 + 75 x - 1 + 75 1-75 X
2
_ 2(1-75) 1 2(1-75)
=> —
1 2 1-75
X 1-5 x X 2

X + —=
1 1+75 1-75 1;; 275
X+ X “ 2
X 2 2

Usando a forma prática, temos:


x + - = k => x5+-^L = k5-5k3+5k => x5+^- = (75)5 -5-(75)3 + 5-\Í5
x x5 x^

=> x5+4-= 2575-2575+ 575 x5+4-


+ x5
= õ75
x3 xs

- X5 j =(575)2 = 125
X5.

=> x10
1 = 125-2 .'. x10+—— = 123
X10 x10

Questão 5.67 (CN-1998) - Resposta: Alternativa D.


Resolução: Note que:
1 = 1 2-75 1 2-75 1 2-75
2+75 ~ 2 + 75 2-75 2 + 75" 22-(73)2 ” 2 + ^3 4-3

1
= 2-75 ^2 + 73)f"7 = (2-73)
. -.1997
2 + 75
r ,
= (2-75)
y-s1997
" / /—\1997
(2 + 75)
Fazendo (2 + 73 )1997 = k , podemos escrever:

, <-..1997
_(2+^) + (2-75)
x => 2x = k + -.
2 k
380 10 Resoluções

/ r~\1997 ’ / /—\1997 k-1


(2+^) yVã = k-i .
y=
73 y = 7F
4x2-3y2=(2x)2-(y73)2 4x2 - 3y2 = (2x + y 73 )(2x - y73)
/2
=■ 4x2-3y2 = fk +- +k--Yl,1.1
k+—k+- 4x2 -3y2 = (2k) l-
l k kA k k
,4x2 -3y2 =4].

Questão 68 (IMO-Longlist-1988 / AHSME) - Resposta: Alternativa D.


Resolução: Do enunciado, temos:
B (~1)
p + q + r = -- => p + q + r = -
--í—4 |p + q + r = 1|;
A 1
D 1
pq + pr + qr = — => pq + pr + qr = - |pq + pr + qr = 1|;

D
Pqr = -_ => pqr = - H) |pqr = 2|
1

Então, podemos escrever:


(a + b + c)3 = a3 + b3 +c3 + 3(a + b + c)(ab -ac + bc) -3abc

=> (p + q + r)3 = p3 + q3 + r3 + 3(p + q + r)(pq + pr + qr)-3pqr

=> 13 = p3 + q3+r3+3-1-1-3-2 => 1 = p3 + q3 + r3 + 3-6

3 3 r3 = 1 + 3 |p3 +q3 +r3 = 4|.


=> p +q +

Questão 5.69 (Putnam-1939-Modificada)


Resolução: Do enunciado, temos:
n B „ a
a + P + r = -— => a + p + y = --

° ... b
ap + ay + Py = — => ap + ay + py = — => |ap + ay + py = b|;
1
D c
aPy = -~ => apy = -- R7
Os Segredos da Álgebra para IME/ITA/OLIMPÍADAS 381

Então, podemos escrever:


(a + b + c)3 = a3 + b3 + c3 + 3(a + b + c)(ab + ac + bc)-3abc

(a + P + y)3 = a3 + p3 + y3 + 3(a + p +y)(ap +ay + Py)-3aPy

(-a)3 =a3+p3+y3+3(-a)(b)-3(-c) => -a3 = a3+p3+ y3-3ab + 3c

|a3 + p3 + y3 = -a3 + 3ab - 3c|.

Questão 5.70 (AIME-2008)___________________


Resolução: Do enunciado, temos:
B Q __________
r + s +1 =----- => r + s +1 = — => Ir + s + t = 0|;
A 1
. 1001
rs + _ . D
rt + st = — rs + rt + st =-------
A 8
rst = -° => rst = - 2008 |rst = -251|.
A 8

Então, podemos escrever:


r + s + t = O => r + s = -t; => r +1 = -s ; => s +1 = -r ;
r + s +1 = 0 => r3 + s3 +13 = 3rst.
E = (r + s)3+(s +1)3+(t+ r)3 => E = (-t)3 + (-r),33 + (-s)3

=> E = -(t3 + r3+s3) => E = -(3rst) => E = -3(-251) .-. |E = 753|.

Questão 5.71 (Stanford-2007)____________________________


Resolução: Podemos escrever:
(a + b + c)3 = 3(a + b + c)(a2 +b2 + c2)-2^a3 + b3 + c3j- 6abc

=> (r + s + t)3 = 3(r + s + t)ír2+ s2+t2j-2^r3+s3+ t3j + 6rst

=> 33 = 3-31-2-3 + 6rst => 27 = 9-6 + 6rst => 6rst = 27-3

=> rst = — |rst = 4|.

Questão 5.72 (Stanford-2007)____________________


Resolução: Do enunciado, temos:
B (~ 7) í------------------ 1
a + b + c =-----=> a + b + c = ---- ----- => a + b - c = 7r,
A 1 -----------------
382 10 Resoluções

. D (-6)
ab + ac + bc = — => ab + ac + bc = lab + ac + bc = —6|;
A 1
abc = - — => abc = -- => |abc = -51.
A 1
Então, podemos escrever:
a + b + c = 0 => a + b = -c; => a + c = -b; => b + c = -a;
E = (a + b)(a+ c)(b+ c) => E = (-c)(-b)(-a) => E = -(abc) => E = -(-5)
|(a + b)(a + c)(b + c) = 5|.

Questão 5.73
Resolução: Da identidade de Argand, podemos escrever:
a4 + a2 +1 = (a2 - a + l)(a2 + a + l)

(24 + 22 +l)(44 +42 +l)(64 +62 +l)...(324 +322 +l)


A=
(l4 +12 +l)(34 +32 +l)(54 +52 +l)...(314 +312 + l)

(22 -2 + l)(2z +2 + l)...(322 -32 + l)(322 +32 + 1)


=> A =
(l2-1 + l)(l2 +1 + i)...(312-31 + i)(312 +31 + 1)

_ 3-7-13-21...993-1057
=> A = 1057.
” 1-3-7-13-21...931-993

Assim, temos: A-1053 = 1057-1053 = 4.

Questão 5.74
Resolução: Podemos escrever:
(a + b + c)3 = a3 + b3 + c3 + 3 (ab2 + ac2 + a2b + a2c + b2c + bc2 j + 6abc

(a + b )3 = a3 + b3 + c3 + 3ab2 + 3ac2 + 3a2b + 3a2c + 3b2c + 3bc2


+ 3abc + 3abc
(a + b + c)3 = a3 + b3 + c3 + 3ab(a + b) + 3ac(a + c) + 3bc(a + c) + 3bc(a + b)

(a + b + c)3 = a3 + b3 + c3 + 3b(a + b)(a + c) + 3c(a + c)(a + b)

(a + b + c)3 = a3 + b3 + c3 + 3(a + b)(a + c)(b + c)

Fazendo: a = xy, b = yz e c = xz, temos :


(xy + yz + xz)3 = (xy)3 ■ (yZ)3 + (xz)3 + 3(xy + yz)(xy + xz)(yz +xz).
Os Segredos da Álgebra para IME/ITA/OLIMPlADAS 383

Questão 5.75 (Noruega-1996-Modificada)


Resolução: Decompondo o 78 e usando os dados do enunciado, temos por
comparação:
xyz = 78 => xyz = 2-3-13 => |x = 2|; |y = 3|; |z = 13|.

Verificando na segunda equação:


x2+y2+z2 = 22 + 32 +132 => x2 + y2+z2 = 4+ 9 + 169
x2 + y2+z2 =182.

Observe que não condiz com o resultado dado, o que leva o leitor a pensar em
erro de digitação, mas CUIDADO! Note que a questão é sutil, ela diz que x, y e
z são números naturais, mas não diz que eles são primos, fazendo uma nova
inspeção, encontramos:
xyz = 78 => xyz = 1-6-13 => |x = 1|; |y = 6|; |z = 13|.

Verificando na segunda equação:


x2+y2+z2 =12 + 62 +132 => x2+y2+ z2 =1 + 36 + 169
x2 + y2 + z2 = 206 .
Agora sim, temos o resultado esperado, assim o valor pedido é:
E X3 + y3 + z3 13 + 63 +133 E 1 + 216 + 2197 2414
xy +xz + yz=>=> E = 1-6 + 1-13 + 6-13 =. E- 6 + 13 + 78
,
97

Questão 5.76
Resolução: Note que:
a2 - ab + b2 = a3 + b3 ; a4-a2b2+b4 = (a2)2-a2b2-í-(b2)2
a+b
" .3 ---------------------------------------
a32+b32
=> a4-a2b2+b4 = a4 - a2b2 + b4 =
a2+b2 a2+b2 •

a8-a4b4+b8=(a4)2-a4b4+(b4)2

a322 +b322
=> a8-a4b4+b8 = a8-a4b4+b8 =
a4+b4 a;?2 + b2*
•jn nH—1 on-1
,2
ai2‘ -a2 b2 + b2" -a2"V + b2 '
384 10 Resoluções

=. a2" -a2"'^2"'1 +b2"


^2"'iy+^a2n'1J ,3

a2”1 tb2"-1

a3-2n'1 + b3 2n-1
a2"-a^b2"'1+b2n
on-1 on-1
a2 +b2

Assim, podemos escrever:


E = (a2 -ab + b2j(a4 -a2b2 + b4)...^ a2" -a2n'1b2"'1 +b,2: ”

a3 2n’1 + b3 2n‘1 '


ía3+b3 a32 + b32
=. E =
I a+b a2+b2 on-1 on-1
a2 +b2 J
on-1 on-1
(a3+b3).[(a3)2 + (b3)2]....|(a3)i ; +M
=> E =
(a+ b)-(a2+b2)-...ía2 +b2 j

on-1
+(b3)2
a3-b3
E=
a3-b3 (a + b)(a2+b2j-...^a2 +b2

on-1
(b3)
E=
(a3-b3j(a + b)(a2 +b2)-...-(a2n 1 + bo2 n-i \

=> E =---------------
__ Pf-(<__
>n-1 or
(a2 +ab + b2j(a-b)(a + b)-(a2 +b2 j'...-(a2' + b2

[a2nJ-(b2ny
/ n
(a2"
-b2"] (a20)'
2
-a: rV-^b2"}
.2

E= =>E =------
(a2+ab +b2)^a2 -b',2n (a2 +ab + b2^a',2”
Os Segredos da Álgebra para IME/ITA/OLIMPÍADAS 385

^a2'2" -a^b2"+b2'2") a2"*' 2^*1


a2"b2" + b:
=> E = E=
+ ab + b2j (a2 + ab + b2 j

Questão 5.77_______________________ _
a) resolução: Tirando o MMC. temos:
E =___ 1___ +____ 1
-___ +____ 1___
(a-b)(a-c) (b-a)(b-c) (c-a)(c-b)

(b- c) -(a - c) + (a -b) b-c-a+c+a-b


=> E =
(a - b)(a -c)(b - c) (a -b)(a -c)(b -c)

1
. .------------------------------------- F 1 1 ■ ■ -f- ----------------------------------
1 “ Q#
(a-b)(a-c) (b-a)(b-c) (c-a)(c-b)

b) resolução: Tirando o MMC, temos:


a b c
t —------------------- +--------------------- r-------------------
(a-b)(a-c) (b-a)(b-c) (c-a)(c-b)

a(b-c)-b(a-c) + c(a-b) ab - ac - ab + bc + ac - bc
=> E = => E =
(a -b)(a -c)(b-c) (a-b)(a-c)(b-c)
a b , c
= 0.
(a-b)(a-c) (b-a)(b-c) (c-a)(c-b)

c) resolução: Tirando o MMC, temos:


a2 + b2 ! c2
E = (a-b)(a-c) + (b-a)(b-c) + (c-a)(c-b)

a2 (b - c) - b2 (a - c) + c2 (a - b)
=> E =
(a-b)(a-c)(b-c)

a2(b-c)-a(b2 -c2) + bc(b-c)


=> E =-------------------------------------------------
(a-b)(a-c)(b-c)

(b - c)[a2 - a(b + c) + bc i r (b-c)(a-b)(a-c)


=> E = 1.
(a -b)(a -c)(b -c) (a -b)(a -c)(b -c)
386 10 Resoluções

d) resolução: Da identidade de Stevin, temos:


(x-a)(x-b)-(x-c) = x3 -(a + b + c)x2 + (ab + ac + bc)x - abc

x3 (a + b + c)x2 (ab + ac + bc)x abc


x-a = (x-b)(x-c) (x-b)(x-c)+ (x-b)(x-c) (x-b)(x-c)

Para x = a, temos:
a3 (a + b + c)a2 (ab + ac + bc)a abc
(a-b)(a-c) (a-b)(a-c) (a-b)(a-c) (a-b)(a-c)

a3 (a + b + c)a2 (ab + ac + bc)a abc


= 0.
(a-b)(a-c) (a-b)(a-c) (a-b)(a-c) (a-b)(a-c)

Fazendo o mesmo para x = b e x = c, obtemos:


b3 (a + b + c)b2 (ab + ac + bc)b abc
=0
(b-a)(b-c) (b-a)(b-c) (b-a)(b-c) (b-a)(b-c)
c3 (a + b + c)c2 (ab + ac + bc)c abc
= 0.
(c-a)(c-b) (c-a)(c-b) (c-a)(c-b) (c-a)(c-b)

Chamando a expressão pedida de E e somando as três equações


encontradas, membro a membro:
a2 + b2 ! c2
=■ E - (a + b + c) (a-b)(a-c) + (b — a)(b -c) + (c -a)(c -b)

a b c
(ab + ac + bc) (a -b)(a-c) + (b -a)(b -c) + (c -a)(c -b)

1 . 1 1
-abc (a-b)(a -c) + (b-a)(b-c)+ (c-a)(c -b) =0+0+0

=> E - (a + b + c)[1] + (ab + ac + bc)[0]-abc[0] = 0 => E-(a + b + c) = 0


|E = a + b +~c].
Os Segredos da Álgebra para IME/ITA/OLIMPÍADAS 387

Questão 5.78
Resolução: Tirando o MMC, temos:
E b+c a+c a+b
(a-b)(a-c) (b-a)(b-c) + (c-a)(c-b)

E (b + c)(b-c)-(a + c)(a-c) + (a + b)(a-b)


(a-b)(a-c)(b-c)
b2-c2-(a2-c2) + a2-b2 b2_c2_a2+c2^a2_b2
E= => E = E = 0.
(a-b)(a-c)(b-c) (a-b)(a-c)(b-c)

Como queríamos provar.

Questão 5.79
Resolução: Tirando o MMC, temos:
a2 + b + c a + b2 rc a + b + c2
E=
(a-b)(a-c) (b-a)(b-c) (c-a)(c-b)
(a2 + b + cj(b-c) ~(b2 + a + c)(a -c) + (c2 + a + bj(a-b)
=> E =
(a -b)(a - c)(b -c)

a2 (b -c) + (b2 -c2)-b2 (a -c)-(a2 -c2) + c2(a -b) + (a2 -b2)


(a -b)(a - c)(b - c)
E a2 (b-c)-b2 (a-c) + c2 (a-b) .
(a-b)(a-c)(b-c)
Questão 5.81
Resolução: Tirando o MMC e usando a identidade de Gauss, temos:
1 1 1 1 ab + bc + ca 1
a b c a+b+c abc a+b+c
=> (ab + bc + ca)(a + b + c) = abc => (a + b)(a +c)(b + c) +abc = abc
=> (a + b)(a + c)(b +c) = 0 => a + b = 0 => a = -b ou
a + c = 0 => a = -c ou b + c = 0 => b = -c.
Assim, podemos escrever:
,.111
t =------ 1------- 1------ => E = —L.
1 1
+ b5+c5
=> E = _± + A + _L
a5 b5 c5 (-b)5 b5 b5 c5

^e = A=>e. 1 1 1 _1_ 1
(0 + c)5 A as+b5+c5~ (a + b + c)S
388 10 Resoluções

Questão 5.82 (Finlândia 2002)


Resolução: Tirando o MMC, temos:
111 1 ab + bc + ca 1
--- r — = ------------ => =
ab c a + b + c--------------- abc---------- a + b + c
(ab + bc + ca)(a + b + c) = abc => (a + b)(a + c)(b + c) + abc = abc
(a + b)(a + c)(b + c) = 0 => a + b = 0 => a =-b ou
a + c = 0 => a = —c ou b + c = 0 => b = -c.

Assim, podemos escrever:


n ímpar
1 1 _1_ 1 1 1
E =A+ 1
=> E =----+ => e = -A
an bn cn (-b)" bn cn bn bn cn
1 1 1 1 1
=> E = — => E =
cn (0 + c)',n 7" b" cn (a + b + c)n

Questão 5.83
Resolução: Tirando o MMC, temos:
2 4b 4 2(2a + b)-4b-4(2a-b)
r~ =---------- 1-------------------------- => E =
2a - b b2 4a2 2a + b 4a2 -b2
4a + 2b - 4b - 8a + 4b 2b-4a -2(2a-b)
=> E = => E = => E =
4a2-b2 4a2 - b2 (2a-b)(2a + b)
2 4b 4 2
2a - b b2 - 4a2 2a+ b 2a + b

Questão 5.84
Resolução: Tirando o MMC, temos:
(a + b)3 - 2a(a2 - b2 j + (a - b)3
a + b__ 2a a-b
E= |2 a2-b2+(a + b)' => E =
(a-b)'

2a(a2 +3b2)-2a(a2 -b2) 2a3 + 6ab2 - 2a3 + 2ab2


E

a+b 2a a-b 8ab2


(a-b)2 a2 -b2 "(a + b)'
Os Segredos da Álgebra para IME/ITA/OLIMPÍADAS 389

Questão 8.85 (CN-1961)


Resolução: Tirando o MMC, temos:
2x y y2 1 t x
E=
x+y y-x + y2 -x2_ x + y + x2 -y2

2x(y-x)-y(x + y) + y2 f(y-x)- x
=> E =
y2-x2 ' '. y2 - x2 .
2xy - 2x2 - xy - y2 + ' y- x - x
=> E = y^2 J y2 - x2
xy-2x2 y-2x x(y-2x)y2 - x2
________
=> E = E=
y2 - x2 y2 - x2 y2 -x2 J [ y-2x
2x
x + y' y-x
+ y2 -x
l.í[x + y
- „ 2Jj L
1 , x
x2-y,2:
= x.

Questão 5.86 (CN-1978) - Resposta: Alternativa D.


Resolução: Tirando o MMC, temos:

_________ a4 - b4__________ 2ab


E=
(a2 + b2 + 2abj(a2 +b2 - 2abj a2-b2

E-— a4-h4 2ab (*2-b2)(a2+b2) 2ab


=>E =
(a + b)2(a-b)2 a2-b2 (a2-b2)2 a2-b2

a2 + b2 2ab a2 +b2 -2ab (a-b)2


=> E = => E = => E =
^b2-?-^2 a2-b2 (a + b)(a-b)
a4 - b4 2ab a-b
[a2 + b2 + 2ab)[a2 + b2 - 2abj a2-b2 a+b

Questão 5.87
Resolução: Tirando o MMC, temos:

E_ a2 [ b2 (a + b)2 a2y(x + y) + b2x(x + y)-xy(a + b)2


=> E =
x y x+y " xy(x + y)
390 10 Resoluções

a2xy + a2y2 + b2x2 + b2xy - xy (a2 + 2ab + b2)


=> E =
xy(x + y)
E a2xy + a2y2 + b2x2 + b2xy - xya2 - 2xyab - xyb2
xy(x + y)
a2y2 + b2x2 -2xyab E (ay)2 -2(ay)(bx) + (bx)2
=o E = => E =
xy(x + y) xy(x + y)
a2 ' b2 (a + b)2 (ay-bx)2
x y x+y xy(x + y) '

Questão 5.88
Resolução: Podemos escrever:
(n + 1)4+4n4 n4+4n3+6n2+4n + 1 + 4n4
a -------- ----------- => a =
= -----------------------------------------
n2+(2n + 1)2 n2+4n2+4n + 1

(n + 1)4 + 4n4 5n4 + 4n3 + 6n2 + 4n +1


n2 + (2n + 1)2 5n2 + 4n +1
(n-1)4+4n4 n4 - 4n3 + 6n2 - 4n +1 + 4n4
b =----------------- —
n2 +(2n-1)2 n2 +4n2 -4n + 1
(n-1)4 +4n4 5n4 - 4n3 + 6n2 - 4n +1
n2 + (2n-1)2 5n2 -4n + 1
E (n + 1)4+4n4 (n-1)4 4n4
n2 + (2n + 1)2 n2+(2n-1):

5n4 +4n3 -r6n2 +4n + 1 5n4 - 4n3 + 6n2 - 4n +1


=> E =
5n2 +4n + 1 5n2 -4n + 1

_ E 5n4+4n3+n2+5n2+4n + 1 5n4-4n3+n2 + 5n2-4n+ 1


5nz + 4n + 1 5n2 -4n + 1

n2 ^5n2 + 4n ++ 5n2 + 4n +1 n2 5n2 - 4n +1 + 5n2 - 4n +1


E
5n2 + 4n +1 5n2 - 4n +1
Os Segredos da Álgebra para 1ME/ITA/OLIMPÍADAS 391

(n2+l)(5n2 + 4n + l) (n2+l)(5n2 -4n + l)


=> E =
5n2 + 4n + 1 5n2 -4n + 1
(n +1)4 + 4n4 (n -1)4 + 4n4
=> E = (n2+l)-(n2+l) .-. =0
n2+(2n + 1)2 n2+(2n-1)2

Como queríamos demonstrar.

Questão 5.89 (Grã-Bretânia-2014)


Resolução: Esta questão é uma aplicação do resultado da questão anterior,
para n = 2013.

Questão 5.92 (IME-06/07)


Resolução: Do enunciado, temos duas opções:
Caso 01: Se a + b + c^0.
a+b b+c a+c a +b+b+ c+a +c 2a + 2b + 2c 2(a-b + c)
c a b c+a+b a i b i c a + b rc
a +b b + c a +c a+b
=> -- =2 --- = 2
c a b C

Caso 02: Se a + b + c = 0 => a + b = -c


a+b a+b 1
c c c

Questão 5.93
Resolução: Do enunciado, temos:
a+b+c = 5 a = 5-(b + c)
a + b + c = 5 => b = 5-(a + c)
a + b + c = 5 => c = 5-(a + b)

Assim, substituindo, temos:


b c E 5-(b + c) | 5-(a + c) , 5-(a + b)
E = -?- + => E =
b+c a+c a+b b+c a+c a+b
(b + c) 5 (a + c) + 55 (a + b)
=> E = —
b+c b+c a+c a+c a+b
a +b a+b
392 10 Resoluções

1 1 1
=> E = 5- -1-1-1 => E = 5 (6)-3
b+c a+c a+b
=> E = 30-3 .-. |E = 27|.

Questão 5.94
Resolução: Usando o dado do enunciado e chamando a expressão de E,
temos:
„ , 2abc
a + b + c = 0 => a + b = -c; => E = +—--------------------- r----------
Ia2 + aç + bc + abl(b + c)

2abc________ 2abc p. 2abc


=> E = r------ E=—
=> E = ------------ => c - — .-. E =-2.
[a(a + c + b) + bc](b + c) [bc](-a)----------------- abc

luestão 5.95
Resolução: Usando o dado do enunciado e chamando a expressão de E,
temos:
a + b + c = 0 => a + b = -c; a + c = -b => ; b + c = -a;
(a + b - 2c)2 + (a + c - 2b)2 + (b + c -2a)2
=> E =
a2+b2+c2
(-c-2c)2 r(-b-2b)2 +(-a-2a)2
=> E =
a2+b2+c2
(-3c)2!+(-3b)2+(-3a)2 9c2 + 9b2 + 9a2
=> E = => E =
a2+b2+c2 a2+b2+c2
9(c2+b2+a2) (a + b-2c)2 (a + c-2b)2 +(b -2a)'
E= —=9.
a2+b2+c2 '■ a2+b2+c2

Questão 5.96 (Moscou 1949)


Resolução: Do enunciado, temos:
x2 + y2 + z2 = 2xyz => x2 - 2xyz + y2 + z2 = 0
=> x2 - 2 • x • yz + (yz)2 - (yz)2 + y2 + z2 = 0

=> (x-yz)2-(yz)2+y2+z2 =0.


Os Segredos da Álgebra para IME/ITA/OLIMPÍADAS 393

Note que, para uma soma de "quadrados" ser nula, todos os termos quadráticos
devem ser nulos, assim:
(x-yz)2 =0
x = yz = 0
. (yz)2 = o yz = 0
y2 = 0 y=0
z=0
z2 =0

Questão 5.97 (lrã-1985)_____________________________________


Resolução: Multiplicando por 2, temos:
x2 + y2 + z2 = xy + yz + zx => 2x2 + 2y2 + 2z2 = 2xy + 2yz + 2zx
=> x2 + x2 + y2 + y2 + z2 + z2 = 2xy + 2yz + 2zx
=> x2 - 2xy + y2 + y2 - 2yz + z2 + x2 - 2zx + z2 = 0
= (x - y)2 + (y-z)2 + (x - z)2 = 0
x - y = 0 => |x = y|; y - z = 0 => |y = z|; x - z = 0 => |x = z|.

Assim, podemos escrever:


_ 'J* Vx \/x -Jx x/x 2
x/x + x/y + x/z x/x + x/x + x/x x/x + x/y + x/z 3x/x x/x + ,/y + 7z 3

Questão 5.98___________________________________
Resolução: Do enunciado, temos:
x + y + z = 0 => x4 + y4 + z4 = 2(xy + xz + yz)2

a = b + c => a-b-c = 0 => a + (~b) + (-c) = 0


(a)4 + (-b)4 + (-c)4 = 2[(a)(-b) + (a)(-c) + (-b)(-c)j12

.'. a4 + b4 + c4 = 2(bc - ab - ac)2 .

Assim, bc - ab - ac é inteiro já que a, b e c são inteiros.

Questão 5.99
Resolução: Fazendo uma mudança de variável, temos:
a - 2 = x; b - 2 = y; c - 2 = z => x + y + z = a- 2 + b- 2 + c- 2 = 0.
x + y + z = 0 => x3 + y3 + z3 = 3xyz.
.'. (a - 2)3 + (b - 2)3 + (c - 2)3 = 3(a - 2)(b - 2)(c - 2).
394 10 Resoluções

Assim, temos:
(a-2)3+(b-2)3+(c-2): 3(a - 2)(b - 2)(c - 2) e = 3
E= — => e =
7(a - 2)(b - 2)(c - 2) 7(a - 2)(b - 2)(c - 2) ” 7

Questão 5.100 (BMQ-2007)


Resolução: Seja x = -1, y = -2007 e z = 2008 , temos:

x + y + z = 0 => (x2+y.22+z2) =2^x4+y4+z4j

x + y + z = 0 => x2 + y2 + z2 =-2(xy + xz+yz)

x4+y4 z4
2(x4 +y4 +z4) (£+y,2:
E= => E = 2(x2 + y2+z2j => E =
x2 + y2 z2 2(x2 + y2+z2)

(*2 + y2+z2) -2(xy + xz + yz) .


=> E = —- -------- <=>
=> E = — ------ — => E =-(xy+xz +yz)

=> E = -[x(y + z) + yz]=>E = -[(-1)(-2007 + 2008) + (-2007)(2008j]

14 + 20074 + 20084
E =-[-1-4 030 056] = 4 030 057
12 + 20072 +20082

Questão 5.101 (Singapura-2014)


Resolução: Seja x = -1, y = -2013 e z = 2014 , temos:

x + y + z =-1-2013 + 2014 => x + y + z = 0 => x3 + y,33 + z3 = 3xyz


20143 -20133 -1 20143 +(-2013)3 +(-1)3
E= => E =
2013-2014 (-1) (-2013)-2014

x3+y3+z3 => E = ^ 20143 -20133 -1


=> E = =3
xyz xyz 2013-2014

Questão 5.102
Resolução: Fazendo uma mudança de variável, temos:

^ + -í = 3 => ^ + A-3 = 0 => 377 = x; A = y; -3 = z;


K/r vr W
z = \/r +
_1_
x+y -3 = 0.
Os Segredos da Álgebra para IME/ITA/OLIMPÍADAS 395

x + y + z = 0 => xJ3 + y,3:J + z3 = 3xyz.


3
+ ("3)3 => r+--27 =-9
r
1 1
=> r + --27 + 9 = 0 r + --18 = 0.

(-18)3=3.(r3).[A](_18) : r3 + -1 - 5832 = -54

1
=> r3 + -1 = 5832-54 .-. r3 + -j = 5778.

Questão 5.103 (AMC-2011) - Resposta: Alternativa B.


Resolução: Fazendo uma mudança de variável, temos:
x = 79-672+79+ 672 => X - 79 - 672 - 79 + 672 = 0
=> - 79-672 = y; - 79 + 672 = z;
=> x+y+z=x-79 - 672 - 79 + 672 = 0.
x + y + z = 0 => x2 + y2 + z2 =-2(xy + yz +zx).
(x)2 + (79-672 j2 +(79 + 672^ =

= 2^(79-672^(79 + 672^ + X ■ ^9-6^2 + X ■ 79 + 6^21

=> X2 +9-672 + 9 + 672 = 2 J92 -[672^ + x■ (79-672 +^9+ 672 j"| =>

__
x2 +18 = 2[V81-72 + x-x] => x2+18 = 2 79 + xH=> x2+18 = 6 + 2x2

=> 3x2 =12 => x2 =4 => x = ±2 79-672 +79 + 672 =2.

Questão 5.104 (Princeton-2006)_____________________


Resolução: Fazendo uma mudança de variável, temos:
X = 77 + 473 + 77-473 => x-77 + 473 - 77 - 473 =0
=> - 77 + 473 = y; - 77 - 473 = z;
=> x + y + z = x-77 + 473 - 77 - 473 = 0.
396 10 Resoluções

x+y+z=0 => x2 + y2 + z2 = -2(xy + yz + zx).

=> (x)2 + (-7?+ 475 )2 + (-77-4V3)2 =

= -2|J-V7 +4,/3 )(->/7-4>/3) + x ■ (-7?+ 475 | + x ■ (-77-475)]

=> x2+7 + 475 + 7-475 =-2 ^72 - (475)2 - X • (7? + 475 +/? - 475 ) =>

x2 +14 = -2[v/49-48-x-x] => x2 + 14 =-2(l-x2) => x2+14 = -2 + 2x2


=> x2 = 14 + 2 => x2 = 16 => X = +4 y/7 + 4^3 + ^7 - 475 = 4 .

Questão 5.105 (AHSME-1970) - Resposta: Alternativa A.


Resolução: Fazendo uma mudança de variável, temos:
x = 73 + 2 72 - 73-2 75 => x- 73 + 2 72 + 73-2 72 = 0
=> - 73 + 2 72 = y ; 73-2 72 = z; => |x + y + z = Õ]
x + y + z = 0 => x2 + y2 + z2 = -2(xy + yz + zx).

=> (x)2 + (-73 + 272 j2 + (73 - 272 j2 =

= -2^(-73 +272^(73 - 272 j + X ■ (-73 +272) + X ■ 73-275] =>

X2 +3+275 + 3-275 = -2 Í32 -(272)2 +x-(-73 +275 + 73 - 275)

=> x2 +6 = -2[-79-8 +x (-x)] => X2+6 = -2(-1-x2)

=> x2 + 6 = 2 + 2x2 => x2 = 4 => X2 = 4 => x = +2

.-. 73 + 275 - J3-2J2 = 2 .

Questão 5.106 (CN-1984) - Resposta: Alternativa B.


Resolução: Fazendo uma mudança de variável, temos:

X= 7 ++ 22 ^275
^3 x/ÍTI -^3-2
- J3 - 2 ^275
^2^2 => V3 ++ 22 x/x/í?2 -\/3-2V
=> xx == \/3 -
=> X = 7 + 2 75 - 73 - 2 75 => X - 73 + 2 75 + 73-2 V2 = 0
=> - 73 + 2 72 = y ; 7-2 75 = z; => |x + y + z = o|.
x + y + z = 0 => x2 + y2 + z2 =-2(xy + yz + zx).
Os Segredos da Álgebra para IME/ITA/OLIMPÍADAS 397

=> (x)2 + (-73 + 2^2 j2 + (V3- 2^2 )2 =

= -2^-x/3 + 2^jp3-2>^j + x-(-73 + 2V2j V3-2x/2 j =>

x2 +3 + 272 + 3-272 = -2 •^32-(2x/2)2 + x ■ (-73 + 272 + 73-272 j

=> x2 + 6 = -2[-79^8 + x-(-x)j => x2+6 = -2(-1-x2)


=> x2+6 = 2 + 2x2 => x2 =4 => x2=4 => x = ±2
V3 + 2\/2v5 -^3-2^2^ = 2.

Questão 5.107
Resolução: Fazendo uma mudança de variável, temos:
x = ^5V2+7-^5>/2-7 => x - ^5>?2 + 7 + ^5-/2 -7=0

=> - ^5^2 + 7 = y ; ^5^2-7 = z;

=> x+y+z= x - ^5y/2 + 7 + ^5>/2-7 = 0.


x+y+z=0 => x3 + y3 + z3 = 3xyz.

=> (x)3 + (-^/5s/2 + 7)3 + ^5x^-7)3 = 3(x)(-7õx^ + 7)(75x/2-7)

=> x3 - 5x/2 - 7 + 5V2 - 7 = -3x^(5V2)2 - 72 j

=> x3 -14 =-3x(\/50- 49 j => x3-14 = -3x => x3+3x-14 = 0.

Pelo teorema do fator (veja detalhes no capitulo de fatoração) ou por inspeção,


temos que 2 é raiz dessa equação.

Assim: ^5-72+7-^5x/2-7 = 2 .

Questão 5.114 (Harvard/MIT-2008)


Resolução: Da teoria, podemos escrever:
(a2+b2+c2) (a3+b3 + c3)_a5 + b5+c5
2 3~ 5
398 10 Resoluções

(a2+b2 + c2j (a5 + b5 + c5)_a5 + b5+c5 (a2 + b2+c2)


2
2 3 5 6 5
a2 + b2 + c2 = ?.
5

Questão 5.115
Resolução: Fazendo uma mudança de variável, temos:
a - 2b + c = x; b-2c + a = y; c - 2a + b = z;
=> x + y + z = a-2b + c + b-2cí-a + c-2a + b = 0.
x + y + z = O => x4 + y4 +z4 =2(xy + xz + yz)2.

=> E = (a-2b + c)4 +(b-2c + a)4 +(c-2a + b)4

=> E = 2[(a-2b + c)(b-2c + a) + (a-2b + c)(c-2a + b) +

+(b-2c + a)(c-2a + b)]2

E = 2 ab-2ac -ra2 -2b2 + 4bc -2ab + bc -2c2 + ac + ac -2a2 +ab -2bc +

•r4ab- 2b2 + c2 - 2ac + bc + bc - 2ab + b2 - 2c2 +4ac-2bc + ac - 2a2 +abl


r__ = -|2
=> E = 2 3ab + 3ac + 3bc - 3a2 - 3b2 - 3c2

=> E = 2^3 (ab + ac + bc-a2-b2-c2j]


2
(2a2 + 2b2 + 2c2 -2ab-2ac-2bcj
=0 E = 2-9
2
2
(a - b)2 + (a - c)2 +(b -c)2
=. E = 18
2

18^(a-b)2 +(a-c)2 +(b-c)2]


=> E " 4 ’

9 [(a-b)2 + (a-c)2+(b-c)2]
E =
2
Os Segredos da Álgebra para IME/ITA/OLIMPÍADAS 3S9

Questão 5.116
Resolução: Da teoria, podemos escrever:
(a + b + c)2 + (a -b -c)'j2 [a + (b + c)]2+[a-(b + c)]2
2(5 + 2bc) 2(5 + 2bc)

E_2[a2 + (b + c)2] 2^a2 + b2 + 2bc + c2 :


E 2(5 + 2bc)
EJ5-t2bC.] .-. E = 1.
2(5 + 2bc) 5 + 2bc

Questão 5.117
Resolução: Da teoria, temos: a5 + b5 + c5 = -5abc (ab + ac + bc). Logo:

a5 + b5+c5 -5abc(ab + ac + bc) a5+b5 + c5


abc(ac + bc + ab) abc(ac + bc + ab) " abc(ac + bc + ab)

Questão 5.118
Resolução: Da teoria, podemos escrever:
a3+b3 + c3 = 3abc => (a3+b3 + c3) = (3abc)2

=> a6 + b6 + c6 + 2(a3b3 + a3c3 + b3c3j = 9a2b2c2


=> a6+b6 c6 + 2^(ab + ac + bc)3 + 3a2b2c2J = 9a2b2c2

=> a6 + b6 + c6 + 2(ab + ac + bc)3 = 9a2b2c2 - 6a2b2c2


a6 + b6 +c6 = 3a2b2c2 -2(ab + bc + ca)3 .

Questão 5.119
Resolução: Da teoria, podemos escrever:
a + b + c = 0 => a3 + b3 + c3 = 3abc => (a3 + b3 + c3j = (3abc)2.

2 l + l = o=> ab + bc + ac
= 0 => ab + bc + ac = 0.
a b c abc
=> a6+b6 + c6+2(a3b3 + a3c3 + b3c3)2 = 9a2b2c2

0
=> a6 + b6 + c6 + 2 (ab + ac + bc)3 + 3a2b2c2 = 9a2b2c2 =>
400 10 Resoluções

a6 + b6 + c6 + 6a2b2c2 = 9a2b2c2 => a6 + b6 + c6 = 9a2b2c2 - 6a2b2c2 =.

a6 + b6 + c6 = 3a2b2c2 => a6+b6+ c6 = (3abc)abc =>


a6 + b6 + cs
= abc
3abc
a6 + b6 + c6
■■ ra^=abc-
Questão 5.120 (Croácia-2001)
Observação: Temos abaixo a primeira resolução desta questão, a segunda
resolução está no capitulo 8, cuja resolução é por somas de Newton.
Resolução: Da teoria, podemos escrever:
a4 +b4+c4
a4 + b4 + c4 = 2(ab + ac + bc)2 => (ab + ac + bc)2 =
2
a4+b4+c4
a7 + b7 + c7 = 7abc (ab + ac + bc)2 => a7 + b7 + c7 = 7abc
2
a7 + b7+c7 7
=> a7 + b7+c7 = — abc(a4+b4+c4)
abc(a4 +b4 + c4j 2

Questão 5.121

Resolução: Da teoria, podemos escrever:

a + b + c = 0 => (ab + bc + ac)3 =(ab)3+ (bc)3+ (ac)3 - 3a2b2c2

=> (ab)3 + (bc)3 + (ac)3 = (ab + bc + ac)3 + 3a2b2c2 .

a + b + c = 0 => a3 + b3+c3 = 3abc.

(ab)3 + (bc)3 + (ac)3 - (ab + bc + ac)3


6abc(a3 + b3 +c3j

, (ab + bc + ac)3 + 3a2b2c2 - (ab + bc + ac)3 3a2b2c2


E=
6abc (3abc) 18a2b2c2

(ab)3 + (bc)3 + (ac)3 - (ab + bc + ac)3 1


6abc(a3 + b3 +c3) 6
Os Segredos da Álgebra para IME/ITA/OLIMPÍADAS 401

Questão 5,123 (Peru 2009)


Resolução: Podemos escrever:
x = ac-bd = ac - b(-a-b-c) :=> x = ac + ab + b2 + bc
=> x = a(b + c) + b(b + c) => |x = (a +b)(b + c)|.

y = bc-ad = bc-a(-a-b-c) => y = bc + a2 + ab+ ac

=> y = a(a + b) + c(a + b) => ly = (a + b)(a + cH.

z = cd-ab = -c(-a-b-c)-ab => z =-(ab+ ac +bc+ c2 j


=> z = -[a(b + c) + c(b + c)] => |z =-(a + c)(b +c)|.

Logo, temos:
E = (ac - bd)(bc - ad)(cd - ab)
=> E = - (a + b)-(a + c)-(a + b)-(b + c)-(a + c)-(b + c)

=> E = -(a + b) ■ (a + c) (-1)(a + d)■ (b + c)• (-1)(b + d)• (-1)(c + d)

E = (a + b)(a + c)(a + d)(b + c)(b + d)(c + d).

Questão 5.125 (Rússia)____________________________ ___


Resolução: Podemos escrever:
E = a2 ■ (c - b) + b2 • (a - c) + c2 • (b - a)
=> E = a2 (c - b) + ab2 - b2c + bc2 - ac2
=> E - a2(c-b)-a(c2-b2j + bc(c-b)

=> E = (c-b)(a2-a(b + c) + bc(| E = (c-b)(a-b)(a-c).

Como a, b e c são números reais distintos dois a dois, temos que a diferença
não é nula, o que conclui a nossa demonstração.

Capitulo 06 - Fatoração

Questão 6.1 (Harvard-MIT-2012)________________________________


Resolução: Note que 253 -272 pode ser fatorado da seguinte forma:
253 - 272 = (52)3 - (33 )2 = (õ3)2 - (33 = (õ3 + 33](õ3 - 33)
402 10 Resoluções

=> 253 -272 = (125 + 27)(5-3)(õ2 + 5.3 + 32)

=. 253 - 272 = (152)(2)(25 +15 + 9) => 253 - 272 =(23 -19) (2)(49)

=> 253 - 272 =24■72■19 .

Assim, a soma pedida vale: 2 + 7 + 19 = 28.

Questão 6.2 (CN-1954)________________________________________________


Resolução: Fatorando por diferença de quadrados e chamando a expressão
de E. temos:
E = 16x4-1 => E = (2x)4-14 => E = |j2x)2+12p2x)2-12]

=■ E = [4x2+l][(2x) + l][(2x)-l] .'. E = {4x2+l](2x + 1)(2x-1).

Questão 6.3 (AHSME-1954) - Resposta: Alternativa E.___________________


Resolução: Usando o artificio de somar e subtrair a mesma quantidade e
chamando a expressão de E, temos:
E = x4 + 64 => E = (x2)2+82 => E = (x2)2 + 82 + 2 • x2 ■ 8 - 2 ■ x2 ■ 8

= (x2 + 8)2-16x2 => E = [x2 + 8]2-(4x)2


E

=> E = (x2 + 8 + 4x)(x2 + 8-4x) /. E = (x2+4x + 8J(x2 - 4x + 8).


Questão 6.4__________________________________________________________
Resolução: Agrupando os termos semelhantes e chamando a expressão de E,
temos:
E = 5ax + 3by - 5ay - 3bx => E = 5a(x-y)-3b(x-y) E = (x-y)(5a-3b).

Questão 6.5 (CN-1951)_________________________________________________


Resolução: Fatorando por diferença de quadrados e chamando a expressão
de E, temos:
E = x2-2xy+ y2-a2 => E = (x-y)2-a2 => E = [(x - y) + a][(x - y) - a]

E = (x-y-a)(x-y + a).

Questão 6.6 (CN-1952)_________________________________________________


Resolução: Agrupando os termos semelhantes e chamando a expressão de E,
temos:
E = 8x2 -8xy-3x + 3y E = 8x(x-y)-3(x-y) E = (x-y)(8x-3).
Os Segredos da Álgebra para IME/ITA/OLIMPÍADAS 403

Questão 6.7
Resolução: Fatorando por diferença de quadrados e chamando a expressão
de E, temos:
E = 16x4y6 - 81a6b4 => E = (4x2y3)2 -(9a3b2)2

/. E = (4x2y3 + 9a3b2^4x2y3 -9a3b2j.

Questão 6.8
Resolução: Fatorando por diferença de quadrados e chamando a expressão
de E, temos:
E = (2a-3)4 -(a-5)4 => E =[(2a -3)2 + (a-5)2^(2a -3)2 - (a -5)2 J

=> E = (2a-3)2+(a-5)2][(2a-3) + (a-5)][(2a-3)-(a-5)J

=> E = 4a2 -12a + 9+ a2 - 10a + 2õJ(3a-8)(a + 2)

.-. E = (3a-8)(a + 2)(5a2-22a + 34).

Questão 6.9
Resolução: Usando o artifício de multiplicar e dividir a mesma quantidade e
chamando a expressão de E, temos:
_c^_
a2 b2 c2 a2-a b2 b c2 c a3
b =----- 4---------- 1------- => E =-------------- 1--------------- 1------------- => E =--------- 4- lí
bc ac
ac ab bca acb abc abc abc abc
a3+b3+c3 3abc ■■■ |E = 3|.
abc abc

Questão 6.10
Resolução: Usando o artifício de multiplicar e dividir a mesma quantidade e
chamando a expressão de E, temos:
27a2 27b2 27c2 27a2 a 27b2-b 27c2 c
=> E =
5bc 5ac 5ab 5bca 5acb 5ab c

27a3 27b3 27c3 27 (a3+b3+c3) 27(3abc)


=> E =-------------i---------------+
, E= => E =
5abc 5abc 5abc 5abc 5abc

27a2 27b2 27c2 81


-------------1----------- 4-------------- = ------
5bc 5ac 5ab 5
404 10 Resoluções

Questão 6.11 (AHSME-1952) - Resposta: Alternativa A.


Resolução: Fatorando por diferença de cubos e chamando a expressão de E,
temos:

E = a3 - a => E = (a-a a2 +aa'

=> E a3-a'

Questão 6.12 (AHSME-1955) - Resposta: Alternativa B.

Resolução: Fatorando por diferença de quadrados e chamando a expressão

de E, temos:

E =
a
a
~2
-b
-b 2
=. E = H±í a'2-b“2
=> E =
a’2 + b'2

a -b
-2-b

a - b-4
=a +b
a -b"2

Questão 6.13 (AHSME-1950)


Resolução: Chamando a expressão de E, temos:

a2-b2 ab-b2 a2 -b2 + b2 - ab a2-b2 b(b-a)


E = => E = => E = +^a(b-a)
ab ab-a2 ab ab-a2 ab

a2-b2 b a2-b2 bb a2-b2+b2 a^ a


=> E = E=
ab a ab a b ab ab b

Questão 6.14

Resolução: Usando o artificio de multiplicar e dividir a mesma quantidade e

chamando a expressão de E, temos:

E = ^ + c - ai a-b (b-c)a (c-a)b (a-b)-c


’+ ab
bc ca bca cab abc
ab - ac bc - ab ac-bc ab - ac + bc - ab + ac - bc
=> E = ■ +--------------- =-------------------------------------------------------
=> E --
abc abc abc abc
b - c: c-a
. +----- +--------
a-b
= 0.
bc ca ab
Os Segredos da Álgebra para IME/ITA/OLIMPÍADAS 405

Questão 6.15
Resolução: Chamando a expressão de E, temos:
E = ax(b2 + y2) + by(a2 + x2) => E = axb2 + axy2 + bya2 + byx2
=> E = bx(ab + xy) + ay(xy + ab) => E = (ab + xy)(bx + ay)
ax(b2 + y2 j + by(a2 + x2) = (ay + bx)(ab + xy).

Questão 6.16_________________________________________________________
Resolução: Fatorando por diferença de quadrados e chamando a expressão
de E, temos:
E = (a2 -3a + ab-3b)2 -(a + b)2 =>E = [a(a-3) + b(a-3)]2 -(a+ b)2

=> E = |^(a - 3)(a + b)J2 - (a + b)2

=> E=[(a-3)(a + b) + (a + b)][(a-3)(a + b)-(a + b)]


=> E = [(a + b)(a-3 + 1)][(a + b)(a-3-1)J E = (a-4)(a-2)(a + b)2 .

Questão 6.17
Resolução: Chamando a expressão de E, temos:
E = a2c + 2abc + b2c + a2d + 2abd + b2d
=> E = c(a2 + 2ab + b2j + d(a2 +2ab + b2í => E = (c + d)(a2 + 2ab + b2 j

E = (c + d)(a + b)2 .

Questão 6.22
Resolução: Chamando a expressão de E, temos:
a2 - bc - b2 + ac __ (a - b)(a + b) + c(a-b)
E= => E =
a2+b2-c2 + 2ab (a + b)2-c2

(a - b)(a + b + c) a2 - bc -b2 + ac a-b


=> E =
(a + b-c)(a + b + c) " |a2 + b2 -c2 + 2ab a + b-c

Questão 6.23 (AHSME-1960)_________________


Resolução: Chamando a expressão de E, temos:
E = a2 +b2 -c2 + 2ab _ (a + b)2 -c2 (a + b -c)(a + b + c)
E= => E =
a2+c2-b2 + 2ac (a + c)2-b2 (a + c - b)(a + b + c)
406 10 Resoluções

! a2 + b2 - c2 + 2ab a + b-c
|a2 +c2 -b2 + 2ac a-b + c

Questão 6.24
Resolução: Note que a soma dos termos em parênteses dá zero, então,
usando o produto condicional e chamando a expressão de E, temos:
a2 -b2 + b2 -c2 + c2 -a2 = 0 => condicional
a-b i-b-c + c- a = 0 => condicional
(a2 _ b2 ? + (b2 _c2’)3 + /c2 _a2 j3 3/a2 _ b2 ^2 _ c2 W..2 _ g2 \
E= (a-b)3+(b-c)3+(c-a)3 => E=
E= 3(a-b)(b-c)(c-a)

3(a-b)(a + b)(b-c)(b + c)(c-a)(c + a) .


3(a-b)(b-c)(c-a) \ )

Questão 6.25
Resolução: Podemos escrever:
(a5+a3b2)(a3-b3)(a2-b2)

(a4 - b4 )(a3 + a2b-ab2 -b3 j

a3(a2+b2)(a3-b3)(a2-b2) a3(a3-b3)
E= =o E =
(a2-b2)(a2 +b2^a2(a + b)-b2(a + b)J (a + b)(a2-b2)]
a3 (a - b)(a2 + ab + b2) a3 (a2 + ab + b2 j
=> E = E=
[(a + b)(a + b)(a-b)] 1 '
(a + b)'
Questão 6.26
Resolução: Podemos escrever:
(a + b)4 - (a - b)4 8ab(a2 +b2)
E=
(a2+b2)2-(a2-b2)2

8ab(az + b2) 8ab(a2 +b2 j 2(a2+b2)


=> E = => E = 4a2b2 ~ E=
(b2+b2)(a2+a2 j ab
Os Segredos da Álgebra para IME/ITA/OLIMPÍADAS 407

c 2a2 2b2 (a + b)4-(a-b)4 2a 2b


=> E =----- +-------
ab ab (a2 +b2)2 -(a2 -b2)2 b a

Questão 6.27
Resolução: Chamando a expressão de E, temos:
4^J = a4+a2b2+b4
a6-b6=(a2-b2)(a4+a2b2+b4) =»
a2-b2
4i4 = a4-a2b2+b4
a6 +b6 = (a2 + b2)(a4-a2b2 +b4) =>
a2+b2

c a6-b6 a6+b6
t ---------------------------------
a2-b2 a2+b2
_E±L_
(a2+b2)(a-b)

(*2-b2)(*2+b2)
E = (a4 + a2b2 + b4)-(a4-a2b2 +b4)
(a2 + b2)(a-b)

^---b^a + bl =, E = 2a2b2-(a + b)
=> E = 2a2b2 -
a-b '
a6-b6 _ a6+b6 _ (a4-b4)
= 2a2b2 - a - b .
a2-b2 a2+b2 (az+b2j(a-b)

Questão 6.28
Resolução: Chamando a expressão de E, temos:
(a2 + ab + b2)
a3-b3 a2-b2
E= => E =
a2 - 2ab + b2 a2 +ab (a^b)2' afã^Q

a2 + ab + b2
E=
a

Questão 6.29
Resolução: Chamando a expressão de E, temos:
E (a-b)4 -ab(a-b)2 -2a2b2
(a-b)[a3 -b3)+2a2b2
408 10 Resoluções

a4 -4a3b + 6a2b2 -4ab3 + b4 -ab(a2 -2ab + b2)-2a2b2

a4 - ab3 - a3b + b4 + 2a2b2


a4 - 4a3b + 4a2b2 - 4ab3 + b4 - ab(a2 - 2ab + b2)

(a2+b2) -ab(a2+b2)

a4 - 4a3b + 4a2b2 - 4ab3 + b4 -a3b + 2a2b2 - ab3


=> E =
(a2 + b2 )|^a2 + b2 - abj
(a2 + b2 j|^a2 + b2 - abj - 4a3b + 4a2b2 - 4ab3

(a2 +b2^a2 +b2 -abj

4ab(a'
=> E = =z
Ia' 2 ^-ab (a2 + b2j ^a^JdsSt^abJ

(a -b)4 - ab(a -b)2 -2a2b2 4ab


(a -b)(a3 -b3) + 2a2b2 ^Tb2 '

Questão 6.30
Resolução: Podemos escrever:
a2+b2 b2+c2 c2+a2 a2+b2 ' b2+c2 t c2+a2
E =-------------- 1---------------- 1-------------- => E =
arb b+c c+a —c -a -b

-ab(a2 + b2)-bc(b2 +c2)-ca(c2 + a2)


=> E =
abc

c -a3b-ab3 -b3c-bc3 -ac3 -a3c


=> t =------------------------------------------------------------
abc

a3 (-b- c) + b3 (-a -c) + c3 (-a - b) a3 a + b3 - b + c3 c


E= E=
abc abc

a3 a b3 • b c3 -c c a3 b3 c3
=> E = --------- H-------------1----------
abc abc abc bc ca ab
Os Segredos da Álgebra para IME/ITA/OLIMPlADAS 409

Questão 6.31 - Resposta: Alternativa D.


Resolução: Elevando ao quadrado temos, temos:

, . va — u -r vu — -r vo — a
M= , -............... =
77(a-b)(b-c) + V(a-b)(c-a) +
, r------r i-------- i--------\2
=» M2 = _ r. ,

[77(a-b)(b_c)+V(a-b)(c-a)+>/(b"c)íc_a)]
a-b + b- c + c- a + 27prõdute~2_a__2)
=> M2 =
~b )(c - ãj
=> M2 = 2 .-. |M = y/2|.

Questão 6.32
Resolução: Chamando a expressão de E, temos:
E = 4b2c2 - (a2 - b2 - c2)

=> E = [(2bc) - (a2 - b2 - c2 j^(2bc) + (a2

=> E = |~b2 + 2bc + c2 - a2 j^a2 - (b2 - 2bc +


■=’)]
=> E =^(b + c)2 -a2]^a2 -(b-c)2]

=> E = [(b + c - a)(b + c + a)][ja + b - c)[a - (b - c)]J

.-. E = (a + b-c)(a-b + c)(a + b + c)(-a + b + c).

Questão 6.33
Resolução: Chamando a expressão de E, temos:
E = 2^a2b2 + a2c2 + b2c2 j - (a4 + b4 + c4)

=> E = 2a2b2 + 2a2c2 + 2b2c2 - a4 - b4 - c4


=> E = c2(2a2+2b2)-(a2-b2)2-c4

=> E = c2[\a+ b)2+(a-b)2]-(a2-b2) -c4


410 10 Resoluções

=> E = -[c4 + c2[(a + b)2 + (a-b)2] + [(;a-b)(a + b)J2]

=> E = -[c2-(a-b)2][c2-(a + b)2]

=> E = [c2-(a-b)2^(a + b)2-c2]

=> E = [[c-(a-b)](c + a-b)][(a + b-c)[a-(b-c)]J

E = (a + b - c)(a - b + c)(a + b + c)(-a + b + c).

Questão 6.34 (AIME-1986)


Resolução: Note que a expressão pedida é uma aplicação do resultado da
questão anterior, então temos:
E = (Vê + Vê + V7)(Vê + Vê - V7)(>/5 - Vê + V7 )(-Vê + Vê + V7)
=> E = (a+ b - c)(a-b + c)(a + b+ c)(-a+ b + c) =>
E = 2(a2b2 + a2c2 + b2c2)-(a4 + b4 + c4), a = Vê; b = Vê; c = V7;

a2b2 + a2c2 + b2c2 = (^)2 (Vê)2 + (Vê)2 (V7)2 + (Vê)2 (V7 )2

=■ a2b2 + a2c2 + b2c2 =30 + 35 + 42 => |a2b2 +a2c2 +b2c2 =10?].

a4 +b4 +c4 = (Vê) +(Vê) +('/7) => a4 + b4 + c4 = 25 + 36 + 49

=• |a4 +b4 + c4 = 11o|.

E = 2(a2b2 + a2c2 +b2c2)-(a4 +b4 +c4)=> E = 2 ■ (107)-(110)

=> E = 214-110 |E = 104|.

Questão 6.35 (Putnam-1938-Modificada)


Resolução: Desenvolvendo, temos:
E = (y2 3y + 2)2 - 3(y2 - 3y + 2) + 2-y

=> E = (y2 -3y + 2)(y2 -3y + 2)-3y2 + 9y - 6 +2-y

=> E = y4-3y3+2y2-3y’3: + 9y2 - 6y + 2y2 - 6y + 4 - 3y2 + 8y - 4

=> E = y4-6y3+10y2-4y => E = y4 - 4y3 - 2y3 + 2y2 + 8y2 - 4y

=> E = y2(y2 4y + 2)-2y(y2 - 4y + 2).-. E = (y2-4y + 2)(y2 - 2y) .


Os Segredos da Álgebra para IME/ITA/OLIMPlADAS 411

Questão 6.39 (Harvard-IVIIT-2012)________________________


Resolução: Elevando ao quadrado, temos:
2a + 3b = 10 => (2a + 3b)2=102 => 4a2+12ab + 9b2 = 100

RO 20
=• 20 + 12ab = 100 => 12ab = 80 ab = — ab = — .
12 3

Questão 6.40 (Harvard-IVIIT-2014)_________________


Resolução: Somando as equações, temos:
a2 a2 on b2 b2 „ c2 c2
a +b a +c b+c b+a c+a c+b
a2 b2 c2 b2 c2 a2
a+b b+c c+a a+c b+a c+b
a2~b2 , b2-c2 , c2-a2
= 20 + 14 + x
a+b b+c c+a
(a-b)(a + b) . (b-c)(b + c) , (c-a)(c+ a) *
a+b b+c c+a
=> a-b + b- c + c- a = 34 + x => 0 = 34 + x |x = —34|.

Questão 6.51
Resolução: Do enunciado, temos:
a2+1 = a2+ab + bc + ca => a2+1 = a^ + ab+ bc+ ça

=> a2 +1 = a(a + c) +b(a+ c) .-. a2 +1 = (a + c)(a + b) .

b2+1 = b2+ab + bc + ca => b2 +1 = b2 + ab + bc + ca


=> b2+1 = b(b + c) + a(b + c) [b2 +1 = (a + b)(b + c)[.

c2 +1 = c2 + ab + bc + ca => c2 +1 = c2 + ab + bc + ca

=> c2 + 1 = c(a + c) + b(a + c) .-. c2 + 1 = (b + c)(a+ c) .

Assim, substituindo, temos:


(a2 + l)(b2 + l)(c2 +1) = (a + c)(a + b)(a + b)(b + c)-(b + c)(a +c)

(a2 +l)(b2 +l)(c2 +l) = [(a + b)(b + c)(a + c)]2.


412 10 Resoluções

Questão 6.54
Resolução: Do enunciado, temos:
a2 b2 c2
t =-------------------------------- 1-------------------------------- 4---------------------------------
(a + b - c)(a - b + c) (c + b - a)(a + b - c) (c + b - a)(a-b + c)
c a2 b2 c2
(~c-c)(-b-b) (-a-a)(-c-c) (-a-a)(-b-b)
a2 b2 c2 a3+b3+c3 _ 3abc
3abc _ 3
E => E = => t =-------- /. t=—
4bc 4ac 4ab 4abc 4abc
' ‘ 4J

Questão 6.55
Resolução: Usando a notação sigma (*), temos:
a = ab +bc + ca ; a + b + c = 0 => a2 + b2+c2=-2o

• x = a4 -(b2 -c2) => x = (a2 + b2 - c2)(a2 -b2 + c2)

=> x = (-2o-c2-c2)(-2o-b2-b2j => x = 4^o + c2^o + b2j

o
• o + c2 = ab + bc + ca + c2 c2 = ab + c(b + a + c) c2 = ab
o
b2 =ab + bc + ca + b2 => o +a2 = ca + b(a + c + b) o + b2=ca
2
x = 4(o + c2)(o + b2) => x = 4(ab)(ca) a4-(b2-c2) = 4a2bc .

.2
• y = b4 -(c2-a2) => y = (b2 + c2-a2)(b2-c2 + a2)

=> y = (-2o-a2-a2^-2a-c2-c2) => y = 4(c + a2)(a + c2)

0
a2 = ab + bc + ca + a2 => a2 = bc+ a(b+ c + a) o + a2=bc

y = 4(0 + a2)(a + c2) => y = 4(bc)(ab) b4-(c2-a2) = 4ab2c .

•z = c4-(a2-b2)2 => z = (c2 + a2-b2)(c2-a2+b2)

=> Z = (-2o-b2-b2^-2o-a2-a2) => z = 4(a +b2)(o + a2)

=> z = 4(ca)(bc) c4-(a2-b2)2 = 4abc2 .


Os Segredos da Álgebra para IME/ITA/OLIMPÍADAS 413

P a4 b4 c4
a4-(b2-c2)2 b4-(c2-a2)2 c4-(a2-b2)2

a4 , b4 c4 c_ a3+b3+c3 3abc
=> E = => E => E =-------
4a2bc 4ab2c 4abc2 4abc 4abc
~a4 b4 t c4 ' 3
a4 - (b2 - c2 )2 + b4 (c2 - a2 )2 + c4 - (a2 - b2 )2 4

(*) Veja todos os detalhes da notação sigma no capitulo de Somas de Newton


(capitulo 8)!

Questão 6.57
Resolução: Podemos escrever:
a + b a(1 + b) + b(1 + a)
= 1 => a(1 + b) + b(1+ a) = (1 + a)(1+ b)
1+a 1+b (1 + a)(1 + b)
=> a + ab + b + ab = 1 + a + b + ab .-. |ab = 1|.

a (l + a2)-b(l + b2) a + a3 - b-b3


E = —- ---------- — => E = — E=
1 + b2 1 + a2 (l + a2)(l + b2) 1 + a2 +b2 +a2b2

(a-b) + (a-b)(a2 + ab + b2j (a - b) 1 + [ai>-4+V


=> E = E =---------- —
1 + a2+b2+12

a b
--------- = a — b
1 + b2 1 + a2 '

Capitulo 07 - Polinômios Simétricos

Questão 7.1
Resolução: Vamos fatorar passo a passo
Passo 01: Note que o polinômio é simétrico.
Passo 02: Veja que o polinômio se anula para a = 0, ou seja, a é fator.
Passo 03: Pela propriedade P2, b também é fator.
Passo 04: Veja que o polinômio se anula para a = — b, ou seja, a + b é fator.
414 10 Resoluções

Passo 05: Como o grau do polinômio é 5 e o produto dos fatores é do 3o grau,


podemos escrever:
(a + b)5 - a5 - b5 = ab(a + b)i x• (a2 + b2) + y • abj ■

Passo 06: Por fim, damos valores ao polinômio parcialmente fatorado para
encontrar o coeficiente que falta.
Para a = 1 e b = 1, temos:
(a + b)5 -a5 -b5 = ab(a + b)|^x(a2 +b2) + yabj

=> (1 + 1)5 -15 -15 =1-1(1 + 1)[x(l2 + 12) + y-1.l]


=> 25 — 1 — 1 = 2 ■ [2x + y] => 32-2 = 2(2x + y) 2(2x + y) = 30
.-. 2x + y = 15.

Para a = 1 e b = 2, temos:
(a + b)5 - a5 -b5 = ab(a + b)^x(a2 + b2) + yabj

=> (1 + 2)5 -15-25 = 1-2-(1 + 2)|\(l2 + 22) + y-1-2]


=> 35 -1 - 32 = 2 ■ 3 ■ [5x + 2y] => 243 - 33 = 6(5x + 2y)

=> 6(5x + 2y) = 210 .'. 5x + 2y = 35.

Resolvendo, encontramos x = 5 e y = 5. Logo, temos:


(a + b)5 - a5 -b5 = ab(a + b)[\(a2 + b2) + y abj

=> (a + b)5 -a5 -b5 = ab(a + b)[ô(a2 + b2) + 5ab]

.-. (a + b)5 - a5 - b5 = 5ab (a + b) (a2 + ab + b2).

Questão 7.2 (CN-1995-Modificada)______________________________________


Resposta: Alternativa C.
Resolução: Vamos fatorar passo a passo:
Passo 01: Note que o polinômio é simétrico.
Passo 02: Veja que o polinômio se anula para a = -b , ou seja, a + b é fator.
Passo 03: Pela propriedade P6, a + c e b + c também são fatores.
Passo 04: Como o grau do polinômio é 3 e o produto dos fatores também é do
3o grau, podemos escrever:
(a + b + c)3 - a3 - b3 - c3 = x- (a + b)(a + c)(b + c).
Os Segredos da Álgebra para IME/ITA/OUMPÍADAS 415

Passo 05: Por fim, damos valores ao polinômio parcialmente fatorado para
encontrar o coeficiente que falta.
Para a = 0 , b = 1 e c = 2 , temos:
(0 + 1 + 2)3 - O3 -13 -23 =x (0 + 1)(0 + 2)(1 + 2)

=> 33—1 —8 = x-1-2-3 => 6x = 27-9 => 6x = 18 x = 3.


Logo (a + b + c)3-a3-b3-c3 = 3(a + b)(a + c)(b + c).

Comparando com o que foi dado no enunciado, temos:


(a + b + c)3-a3-b3-c3=3(a-rb)(a + c)(b + c)

(a + b + c)3-a3-b3-c3 = k(a + b)[^c2 + (a + b)c + abj

k(a + b)^c2 + (a + b)c + abj = 3(a + b)(a + c)(b + c)

=> k^c2 + ac + bc + abj = 3(a + c)(b+ c)

=> k[c(c + a) + b(c + a)] = 3(a + c)(b + c)

=> k(c + a)(c + b) = 3(a + c)(b + c) .'. |k = 3|.

Questão 7.3 (Rússia)


Resolução: Vamos fatorar passo a passo:
Passo 01: Note que o polinômio é alternado.
Passo 02: Veja que o polinômio se anula para a = b, ou seja, a - b é fator.
Passo 03: Pela propriedade P6. a - c e b - c também são fatores.
Passo 04: Como o grau do polinômio é 3 e o produto dos fatores também é do
3o grau, podemos escrever:
a2(c-b) + b2(a-c) + c2(b-a) = x (a-b)(a-c)(b-c).

Passo 05: Por fim, damos valores ao polinômio parcialmente fatorado para
encontrar o coeficiente que falta.

Para a = 0, b = 1 e c = 2 , temos:
O2 (2 -1) +12 (0-2) + 22 (1 - 0) = x ■ (0-1)(0 - 2)(1 - 2)
— 2 + 4 — X • (—1) (—2) (—1) => 2 =-2 .'. |x = —1|.
416 10 Resoluções

Logo:
a2(c-b) + b2(a-c) + c2(b-a) = x- (a-b)(a-c)(b-c)

=> a2(c-b) + b2(a-c) + c2(b-a) = (-l)(a-b)(a-c)(b-c).

.-. a2 (c - b) + b2 (a -c) + c2 (b - a) = (a -b)(c - a)(b - c)

Note que esse produto acima é inteiro, pois a, b e c são reais e distintos dois
a dois.

Questão 7.11
Resolução: Vamos fatorar passo a passo:
Passo 01: Note que o polinômio é alternado.
Passo 02: Veja que o polinômio se anula para a = b, ou seja, a - b é fator.
Passo 03: Pela propriedade P6, a - c e b - c também são fatores.
Passo 04: Como o grau do polinômio é 3 e o produto dos fatores é do 4o grau,
podemos escrever:
a3 (b-c) + b3 (c-a) + c3 (a-b) = (a-b)(a -c)(b-c)-[k(a + b + c)].

Passo 05: Por fim, damos valores ao polinômio parcialmente fatorado para
encontrar o coeficiente que falta.
Para a = 0, b = 1 e c = 2 , temos:
a3(b-c) + b3(c-a) + c3(a-b) = (a-b)(a-c)(b-c)[k(a + b + c)].

03(1-2) + 13(2-0) + 23(0-1) = (0-1)(0-2)(1-2)-[k(0 + 1 + 2)]

0 + 2-8 = (-2)(3k) => -6 = -k |k = 1|.

Logo, temos:
a3 (b-c) + b3 (c-a) + c3 (a-b) = (a - b)(a-c)(b -c) [k(a + b + c)J

a3 (b - c)-í-b3 (c-a) + c3 (a-b) = (a-b)(a-c)(b-c)(a + b + c) .

Questão 7.14
Resolução: Vamos fatorar passo a passo:
Passo 01: Note que o polinômio é alternado.
Passo 02: Veja que o polinômio se anula para a = b, ou seja, a - b é fator.
Passo 03: Pela propriedade P5, a - c e b - c também são fatores.
Os Segredos da Álgebra para IME/ITA/OLIMPÍADAS 417

Passo 04: Como o grau do polinômio é 4 e o produto dos fatores é do 3o grau,


podemos escrever:
a(b -c)3 + b(c-a)3+c(a-b)3=x(a-b)(a-c)(b-c)(a + b + c).

Passo 05: Por fim, damos valores ao polinômio parcialmente fatorado para
encontrar o coeficiente que falta.

Para a = 0. b = 1 e c = 2, temos:
a(b-c),33 +b(c-a)3 + c(a-b)3=x-(a-b)(a-c)(b-c)(a + b + c)

=> 0 • (1 - 2)3 +1-(2 - O)3 + 2-(0 -1)3 = x (0-1)(0 - 2)(1-2)(0 +1 +2)

=> 1-23+2 (-1)3 =x (-1) (-2) (-1)-3 => -6x = 8-2 .-. |x = 1|.

Logo a(b-c)3 b(c-a)3 +c(a-b)3 = (a-b)(a-c)(b-c)(a + b-r-c).

Questão 7.16
Resolução: Vamos fatorar passo a passo:
Passo 01: Note que o polinômio é alternado.
Passo 02: Veja que o polinômio se anula para a = b, ou seja, a - b é fator.
Passo 03: Pela propriedade P5, a - c e b - c também são fatores.
Passo 04: Como o grau do polinômio é 5 e o produto dos fatores é do 3o grau,
podemos escrever:
a2 (b-c)3 +b2 (c - a)3 + c2 (a-b)3 = (a-b)(a-c)(b-c)-
■|^m(a2 + b2 + c2)+ n(ab + bc + ac)"|

Passo 05: Por fim, damos valores ao polinômio parcialmente fatorado para
encontrar o coeficiente que falta.

Para a = 0, b = 1 e c = 2, temos:
O2 (1 - 2)3 +12 (2 - O)3 + 22(0 -1)3 = (0 -1)(0 - 2)(1 - 2) ■
|m(o2+12+22) + n(O-1 + 1-2 + O-2)]

1 • 23 + 4 • (-1)3 = (-1) • (-2) ■ (-1) {m(1 + 4) + n(1-2)]

=> (-2)(5m + 2n) = 8- 4 .-. |5m+2n = -2l.


418 10 Resoluções

Para a = 0, b = -1 e c = 2 , temos:
02(-1-2)3 +(-1)2(2-0)3 +22(0-(-1))3 =(0-(-1))(0-2)(-1-2)-

■ |^m(o2 + (-1)2 + 22) + n(0 ■ (-1) + (-1) ■ 2 + 0 • 2) j

=> 1 23 + 4-13 =1(-2)-(-3)-[m(1 + 4) + n(-1-2)] => 6(5m-2n) = 8 + 4

.-. |5m-2n = 2|.

Passo 06: Resolvendo o sistema encontramos m = 0en = ~1.


Logo a2 (b-c)3 +b2 (c-a)3 + c2 (a-b)3 = (a -b)(c -a)(b -c)(ab + bc + ac).

Questão 7.20____________________________________________________
Resolução: Vamos fatorar passo a passo:
Passo 01: Note que o polinòmio é simétrico.
Passo 02: Veja que o polinòmio se anula para a = - b, ou seja, a + b é fator.
Passo 03: Pela propriedade P6, a + c e b + c também são fatores.
Passo 04: Como o grau do polinòmio é 5 e o produto dos fatores é do 3° grau,
podemos escrever:
(a + b + c)S -a5 -b5 -c5 -
= (a + b)(a + c)(b + c)|^x(a2 +b2 + c2) + y(ab + bc + ca)J

Passo 05: Damos valores ao polinòmio parcialmente fatorado para encontrar o


coeficiente que falta.
Para a = 0 , b = 1 e c = 2 , temos:
(0 + 1 + 2)5-O5-15-25 =
= (0 +1)(0 + 2)(1 + 2)[x(o2 + 12 +22j + y(0.1 + 1-2+2-0)]

=> 243-1-32 ==1-2-3[5x + 2y] => 6(5x + 2y) = 210


.-. 5x + 2y = 35 (eq1).
Para a = 0, b = 1 e c = 1, temos:
(0 + 1 + 1)5 -O5 -15 -15 =

= (O + 1)(O + 1)(1 + 1)fx(o2 +12 +12) + y(0-1 + 1-1+ 1-


0)]
=> 32-1-1 = 2-1-1[2x + y] => 2(2x + y) = 30
2x + y = 15 (eq2).
Os Segredos da Álgebra para IME/ITA/OLIMPÍADAS 419

Passo 06: Resolvendo o sistema, encontramos: x = 5 e y = 5.


Logo: (a + b + c)5 -a5 -bs -cs =

= (a + b)(a + c)(b + c)^õ(a2 + b2 + c2) + 5(ab + bc + ca)j•

Observação: Podemos escrever essa fatoração da seguinte forma:


(a + b + c)5 -a5 -b5 - c5 =

= |(a + b)(a + c)(b + c)^(a + b)2+(b + c)2 + (c + a)2j

Questão 7.21 (União Soviética-1962)___________________________________


Resolução: Vamos fatorar passo a passo:
Passo 01: Note que o polinômio é alternado.
Passo 02: Veja que o polinômio se anula para x = y, ou seja, x - y é fator.
Passo 03: Pela propriedade P5, z - x e y - z também são fatores.
Passo 04: Como o grau do polinômio é 5 e o produto dos fatores é do 3o grau,
podemos escrever:
(x-y)5 + (y-z)5+(z-x)5 =(x-y)(y-z)(z-x)-
• [m(x2 + y2 + z2) + n(xy + yz + zx)j

Passo 05: Por fim, damos valores ao polinômio parcialmente fatorado para
encontrar o coeficiente que falta.

Para x = 0 , y = 1 e z = 2 , temos:

(0-1)5 + (1-2)5+(2-0)S=(0-1)(1-2)(2-0)-
• pn(o2+12 + 22 j + n(0-1 + 1-2 +0-2)J

=* (-1)5 + (-1)5 + 25 = (-1)• (-1) ■ 2 ■ [m(1 + 4) + n(1 • 2)]

=> 2(5m + 2n) = -1 -1 + 32 |5m+ 2n = 15|.

Para x = 0, y = -1 e z = 2 , temos:
(0-(_1))5+(-1-2)5+(2-0)5=(0-(-1))(-1-2)(2-0).

■Jjn^O2+ (-1)2+ 22j + n(0(-1) + (-1)-2 + 0-2


420 10 Resoluções

=> 15 +(-3)S + 25 = 1-(-3)-2-[m(1 + 4) + n(-1-2)]


=> - 6(5m - 2n) = 1 - 245 + 32 |5m-2n = 3~5].

Passo 06: Resolvendo o sistema encontramos m = 5en = -5.

Logo (x-y)s+(y-z)5+(z-x)5=5(x-y)(y-z)(z-x)-

•[(x2+y2 + z2)-(xy + yz + zx)].

Como 5(x-y)(y-z)(z-x) é fator, concluímos que


(x-y)5 + (y-z)5 +(z-x)5 é divisível por 5(x- y)(y - z)(z- x).

Como queríamos provar.

Questão 7.24
Resolução: Vamos fatorar passo a passo:
Passo 01: Fazendo uma mudança de variável, temos:
y + z- x = a;z + x- y = b;x + y- z = c; => x + y + z = a + b + c.
E = (x + y + z)3 -(y + z-x)3-(z +x-y)3-(x +y-z)3

=> E = (a+ b + c)3-a3 -b3-c3.

Passo 02: Note que o polinômio simétrico foi fatorado na questão 7.2, usemos
o seu resultado:
a + b = (y + z-x) + (z + x-y) => a + b = 2z.
b + c = (z + x-y) + (x + y-z) => b + c = 2x.
a + c = (y + z-x) + (x + y-z) => a + c = 2y.
E = (a + b + c)3-a3-b3-c3 => E = 3(a + b)(a + c)(b + c)

=> E = 3 (2z)-(2y) (2x) .-. |E = 24xyz|.


Os Segredos da Álgebra para IME/ITA/OLIMPÍADAS 421

Questão 7.25
Resolução: Vamos fatorar passo a passo:

Passo 01: Fazendo uma mudança de variável, temos:


y + z- x = a ; z + x-y = b; x + y- z = c;=> x + y + z = a + b + c.
E = (x + y + z)S -(y + z - x)5 - (z + x - y)5 - (x + y -z)S

=> E = (a + b + c)5-a5-b5-c5.

Passo 02: Note que o polinõmio simétrico foi fatorado na questão 7.20, usemos
o resultado da observação:
a + b = (y + z-x) + (z + x-y) => a + b = 2z.
b + c = (z + x - y) +(x + y-z) => b + c = 2x.
a + c = (y + z-x)+ (x + y-z) => a + c = 2y.
E = (x + y + z)5 -(y + z-x)5 -(z + x-y)5-(x + y-z)5

=■ E = j(a + b)(a + c)(b + c)^(a + b)2 + (b + c)2+(c + a)2J

E = |(2z)(2y)(2x)[(2z)2+(2x)2+(2y)2]
=> E = 20xyz|^4x2+4yz+4z2J => E = 20xyz-4^x2 + y2 + z2]
.'. E = 80xyz(x2 + y2+z2j .

Capítulo 08 - Somas de Newton

Questão 8.7
Resolução: Sejam a, b e c as raizes da equação
3 2
X - X +(J2 -X-O3 = 0, então podemos escrever:
a + b + c = 0 => |S-| = Oi = 0|, So = a° + b° + c° = 3 => |Sq — 3[.

S2=o2-2ct2 => S2=(0)2-2<j2 |S2 = -2<t2|.

Da notação generalizada, temos:


S3 = Gi • S3_-| - a2 ' $3-2 + a3 ' $3-3 $3 = CT1' $2 “ CT2 ' $1 + CT3 ' $0
=> S3 = 0 ■ S2 - o2 • 0 + o3 • 3 => |S3 = 3o31 |S3 = 3abc|.
422 10 Resoluções

Questão 8.8
Resolução: Aproveitando a resolução anterior e da notação generalizada,
podemos escrever:
53 = • S3_i - o2 ’ $3-2 + ct3 ‘ $3-3 S3 = o-| • S2 - o2 • S-| a3 - So
=> S3 = 0 • S2 - o2 • 0 + o3 • 3 => |S3 = 3o^]

54 = (J-j ■ S4_-j — o2 • S4_2 + o3 • 84.3 => S4 = CT-| ■ S3 — o2 • S2 + o3 • S-|


=> S4 = 0-S3 -o2 >(-2q2) + a3 0 => |s4 = 2a||

S4 = 2(ab + bc + ca)2 .

Questão 8.9
Resolução: Do raciocínio da resolução anterior, podemos escrever:

S2 — —2c>2 c?2 —
TS
2
Da notação generalizada, temos:
S4 — Gj ■ S4_-j — o2 ■ S4_2 + 03 ■ S4-3 => S4 — cr-j ■ S3 — o2 ■ S2 + <73 ■ S,

=> S4 = 0 • S3 - o2 ■ (~2o2) + <33 ■ 0 => S4 — 2o2 —S4


= 2'í--Í
k 2 )
(a2+b2+c2)2
S? S,
=> S4 = 2-^- => S4=-^ S4 =
2

Questão 8.10
Resolução: Da notação generalizada, temos:
54 = o-| • S4_-| — a2 • S4_2 +03 • S4_3 => S4 = o-, ■ S3 — o2 ■ S2 +03-81
=> S4 = 0■ S3 — o2 ■ (—2o2) + o3 • 0 => |s4 - 2o||.

55 = C1 ■ S5_1 - o2 ■ S5_2 + o3 • S5_3 =5 S5 = a-j ■ S4 - o2 • S3 + o3 • S2


=> S5 = 0-S4— o2 •(3a3) + a3 (-2a2) => |Ss = - 5■ o2 ■ a31

.-. |S5 = - 5abc(ab + bc + ca)|.

Questão 8.12______________________________________________________
Resolução: Aproveitando o resultado da questão 8.10, temos:
a5 + b5+c5 - 5abc(ab + bc + ca) a5+b5+c5 5
3abc(ac + bc+ ab) 3abc(ac + bc + ab) 3abc(ac + bc+ ab) 3
Os Segredos da Álgebra para IME/ITA/OLIMPÍADAS 423

Questão 8.13
Resolução: Dos resultados anteriores, podemos escrever:
s3
S2 — —2<?2 r?2 S3 = 3o3 => g3 =—
2
S5=-5-O2-o3 => S5=-5-^-^J => Sg _ s2 s3
5 2 3
a5 + b5+c5 _(a2 + b2+c2) (a3+b3+c3)
5 ~ 2 3 '

Questão 8.15
Resolução: Dos resultados anteriores, podemos escrever:
2
S3 —2o2 S3 - 3o3 => a3 Ê3 o?=f~
3 l 3 J
S4 = 2o2 =>

Por outro lado, temos:


S5 — — 5 ■ 02 ■ 03 Sg — 5 • 02 ■ o3 í => S2 -■ 25 • o2 ■ o2

2 2

25
= t 2 J t 3 J => V 5 J =ÍMÍM
V2H3J
2 2
a5+b5-ecs a3+b3+c3 a4+b4+c4
5 3 2

Questão 8.16
Resolução: Podemos escrever:
Sg = <*1 ■ S6_-| - o2 ■ S6_2 + a3 • S6_3 => S6 = c-, • S5 - o2 • S4 °3 ’S3
=> S6 = 0 ■ S5 - a2 ■ ) + o3 ■ (3a3) => |s6 = 303-202!

S6 = 3a2b2c2 - 2(ab + bc+ ca)3 .


424 10 Resoluções

Questão 8.17
Resolução: Aproveitando a resolução anterior, podemos escrever:

Sg — 3o3 — 2 => S6 = 3 ■ $3
,2
-2-ÍM
3
=>S6=3-
$3 -2- $1
3 l2J 9 8

o S3 st . s
-s6 = ^- T ' •---------- 3------------------- 4--------

Questão 8.18
Resolução: Podemos escrever:
Sy = G, S7-1 -o2 S7-2 +03 Sy_3 => Sy = Ot ■ Sg - o2 • S5 + o3 ■ S4 =>
Sy = 0 ■ Sg — O 2 ‘ 5'02 ‘ O3 ) + O3 ■ (2o2 ) —1* Sy = 5 o7 ’O3 + 2o3 ’O3

=> |s7 =7-02 ■o3| .-. S7 = 7abc(ab + bc + ca)2 .

Questão 8.19 (Croácia 2001)


Observação: Temos abaixo a segunda resolução desta questão, a primeira
resolução está no capitulo 5, cuja resolução é por produtos notáveis.

Resolução: Da notação generalizada, temos:


S3 = 01 ■ S3_i - o2 ■ S3_2 + o3 ■ S3_3 => S3 = o, ■ S2 - o2 ■ S, + o3 ■ So
=> S3 = 0 • S2 - o2 ■ 0 + o3 • 3 => |S3 = 803],

$4 = a1 ' $4-1 ct2 ' $4-2 + ct3 ' $4-3 $4 = • S3 - O2 • S2 + o3 • Sj


=> S4 = 0 ■ S3 - o2-(-2o2) +o3 ■ 0 => S4 = 2o| | S4 = 2(ab+ bc + ca)2 .

Também podemos escrever:


Sy — O, ■ Sy -j — 02 ‘ Sy_2 +03- Sy_3 —> Sy = O-| ■ Sg — O2 ’ Sg + 03 - S4
=> Sy — 0 ■ Sg — 02 ■ ( — 5 ■ o3 ■ 03 ) + c3 ■ S4

=> Sy = 5 ■ 02 03 + o3 • S4 => Sy = 5 03 + — • o3 • S4
2
7 S7 7 a7+b7+c7 7
=> Sy - - O3 S4 =>
°3 ' $4 2 abc(a4 + b4 +c4 j 2
Os Segredos da Álgebra para IME/ITA/OLIMPÍADAS 425

Questão 8.27
Resolução: Podemos escrever:
cr2 = S2 4- 2cf2 |~S2 ~ -2cf21.
0
O-j = Sg 4- 3 CF-] • C?2 — ^^3 ^83 — 3cF3 I.
0 0

Questão 8.28
Resolução: Sendo a, b, c e d as raízes da equação genérica
x4 = o1 • x3 - o2 • x2 + a3 • x - o4 , podemos escrever:

o2 = S2 + 2o2 |s2 = -2o21-


0
o3 = S3 + 3o-| • o2 ~|s3 = 303].
0 0
84=^- S4_.j - o2 • ^4_2 + <73 ■ S4_3 - cf4 • S4_4
—84 = CJ.| • S3 — c?2 ■ 82 4" o3 • S-j — O4 • Sq —> S4 = —o2 ' ^2 — O4 ■ Sq
0 0
=> S4 = -c2 (-2o2)-a4 -4 |s4 = 2o2 - 4g41.

Questão 8.29
Resolução: Sendo a, b, c e d as raizes da equação genérica
x4 - a-, ■ x3 - o2 • x2 + o3 ■ x - o4, podemos escrever:

: — S2 ~ 2o2 —S2 — —2c2 a2 = —


s7
2
0

cr3 = S3 + 3 cr-i ■ cr2 - 3a3 => S3 ,3a3


S3
a3 - —
0 o
S4 = Oi ■ S4_1 - a2 ■ S4_2 + a3 ■ S4_3 - ct4 • S4_4
S4 = CT-| ■ S3 — CF2 ‘ CT3 ^1 ~ ^4 " S0 ^4 = ~O"2 ^2 ~ CT4 S0
0 0

= -^-y-^S2-°4’4 S2
S4=^--4a4.
s4
2
426 10 Resoluções

Questão 8.30
Resolução: Sendo a, b. ce d as raízes da equação genérica
x4 = c-| - x3 - o2 - x2 + o3 • x - , podemos escrever:

c2 = S2 + 2o2 |S2 = -2Õ2].


0
o3 — S3 + 3 o-, • cj2 —■ 3g3 [Sg — 3o3 ]•

0 0
$4 = o-| • S4_-| - o2 • S4_2 + o3 ■ S4_3 - o4 • S4_4
=> S4 = cr-, • S3 — o2 • S2 + 03 • S-| — o4 ■ Sq => S4 = —o2 ■ $2 — g4 • Sq
0 0
=> S4 = -a2 • (-2g2 ) - 04 • 4 |s4 = 2o2 - 4ct4 |.

85=01- S5-1 - o2 • S5_2 + o3 • S5_3 - o4 • S5_4


S5 = c>i • S4 — o2 • S3 + 03 ■ S2 — o4 ■ S1 => S5 = — o2 ■ $3 + 03 • S2
'õ "õ w
=> $5 = -o2 (303) —o3 *(-2o2) |S5 = -5 • a2 • g3 |

Questão 8.31
Resolução: Sendo a, b, c e d as raízes da equação genérica
x4 = o-] • x3 - a2 • x2 + <j3 • x - <j4 , podemos escrever:

cr2 = S2 2o2 Js2 = -2o~2 [.

0
c?i — S3 3^1 • c?2 — 3g>3 | S3 — 3cy3"|.
0 0
54 = 01 • S4_i - ct2 • S4_2 + ct3 • S4_3 - o4 • S4_4
—> S4 = O1 • S3 C2 • S2 + O3 • S1 — <J4 • Sq S4 = —Cí2 ' S2 — G4 ■ Sq
0 0
=> S4 = — o2 • 2o2 ) — c4 • 4 |s4 = 2o2 - 4ct4 | ■

55 = CT1 • S5_1 - a2 • $5-2 + ct3 ■ $5-3 ” ct4 ■ $5-4


=> 85= Gi • S4 - o2 • S3 + (53 • s2 - o4 • S1 => S5 = -O2 • S3 + CT3 • S2
"õ “ * 0
=> S5 = ~ct2 (3ct3)“ct3 -(-2ct2) S5 = -5 • o2 • CT3

• $5 _ $2 $3
5 ~ 2 3 '
Os Segredos da Álgebra para IME/ITA/OLIMPÍADAS 427

Questão 35
Resolução: Sendo x e y as raízes de um polinõmio quadrático genérico, então,
da expressão geral, é imediato que:
Sn =o1Sn_1-oz.Sn_2 xn+y" =(x + y)(xn-1+yn-1) -xy(xn'-2 +yn'2)-

Questão 36
Resolução: Sendo x e y as raízes de um polinõmio quadrático genérico, então,
da expressão geral, temos que:
_ S2
a2 = s2 + 2g2 => S2 = —2o2
o
g2~~T
Sn - o, ■ Sn_1 - a2 ' Sn_2 => Sn--a2'Sn-2
o
sn=-(-t}s"-2
=> 2Sn = S2 Sn_2 2xn + 2yn = (x2 + y''2)(xn-2 + yn-2).

Questão 8.37__________________________________________
Resolução: Sendo x e y as raízes de um polinõmio quadrático genérico,
podemos escrever:
x2 = a, ■ x - a2 => (x2)2 = (a, x-o2)2 => x4 =G2 x2-2o1a2- d
(xy)'2
=> x4=(x + y)2x2-2(x + y)(xy)x + (xy)
=> x4 = (x2 + 2xy + y2 jx2-2(x + y)x2y + x2y2 =>

x4 = (x2 + y2)•x2 + 2x3y - 2x3y - 2x2y■2+x2y2=> x4 = (x2+y2)x2-x2y:2

=> x4 x2n"3 =(x2 +y2) x2 x2n-3 -x2y2x2n"3

y2n+1 = (x2 +y2)- x2n-1 - x2y2 x2n"3 (eq1)

Fazendo o mesmo para o y, temos:


y2=a1y-a2 => (y2)2 =(ary-a2)2 => y4 =G2 ■ y2 -2a, -a2 y + a2

=> y4 =(x + y)2 y2-2(x + y)-(xy) y + (xy)2

=> y4 = (x2 + 2xy + y2)-y2-2(x+y)-xy2+(xy)2 =>

y4 = (x2 + y2) y2 +2xy3 -2x2y2 -2xy3 +x2y2 => y'4 = (x2 + y2)-y'2
: -x2y2
428 10 Resoluções

y4y■2n-3=(x2 + y2) . y2 .y2n—3 -x2y2 y2n-3

■■■ y,2n+1 = (x2 + y'’2) . y2n-1 - x2y2 ■ y2n-3 (eq2)

Fazendo (eq1) - (eq2):


X2n<l_y2n+1 = (x2+y2 (x2+y2) y2n 1+x2y2 y2n 3
)-:2n"1-x2y2x2n’3
x2n-1 _y2n+1 = (x2 + y'2)(x,2n-1
: _ y2n-1 )-x2y2(x2"-3-y2n-3).
-y

Questão 8.38 (Putnam 1959)

1
Resolução: Fazendo y = —, no resultado anterior, facilmente encontramos:
x
x2n-r1 - y2n+1 = (x2 +y2)(x2n-1 _y2n-1 )-x2y2(x2n-3 ,2n-3)
-y
z 1 x 2n-1 ■
x2n+1 2 ,2n+1 = X2 x2n-1
,2
x2n-3
X lx J
1
X^n+1
—= fx2 + x2 x2n-1 X2n-3 1
x2n+1 l X^n-1 x2n-3

Questão 8.39___________________________________________________________________
Resolução: Sendo x, y e z as raízes de um polinômio quadrático genérico,
então, da expressão geral, é imediato que:
Sn = °1 ■ Sn_i - o2 ' Sn-2 + ct3 ' Sn-3

xn+yn + zn = (x + y+ z)(xn~1+
+y “1
yn~ + n~1)-
1 +z

- (xy + xz + yz)(xn~2 + yn~2 +zn“2 j + xyz (xn-3+yn-3+zn-3)

Questão 8.40
Resolução: Dos resultados anteriores, podemos escrever:

= S3
S2 =-2q2 a2 = • S3 = 3cr3 a3
3
Sn = CT1' Sn_-| - o2 • Sn_2 + ■ Sn_3 => Sn = -o2 ■ Sn_2 + cr3 ■ Sn_3
0
= “^“■^•Sn-2 +~^'Sn-3 ■■■ Sn
_ S2 ' Sn-2 | S3 ' Sn-3
Sn
2 3
Os Segredos da Álgebra para IME/ITA/OLIMPÍADAS 429

Questão 8.43 (CN-1988)___________________________________________ ____


Resolução: Sendo x1 e x2 as raizes da equação, então, da expressão geral,
temos que:
o (‘2) ■

CT1 =------ 1— -
=> |oi = 2|: °2 = y => |°2 = m|-

sn = a, ■ Sn_1 -a2 • Sn_2 => Sn = 2• b-m■ a .-. |x? + xg = 2b-ma|.

Questão 8.45 (!MO-Longlist-1988 / AHSME-1975)


Resolução: Sendo p, q e r as raizes da equação, então, da expressão geral,
temos que:
_____ -j _____
„ 1 -- l°i=1I; n2=- => EEE3; °3 = —— => EZHD •
Q1 - ------ ~
(~2) i---------- 1

= S2 + 2o2 => 12 = S2 + 2 • 1 => S2 = 1 - 2 => |S2 = -l].

Sn = 01 • Sn_.| - a2 ■ Sn_2 + o3 ■ Sn_3 => S3 = 1 • S3_-| -1 • S3_2 + 2 • S3_3


=> S3 = S2 — S-j + 2• Sq => S3 = —1 — 1 + 2-3 /. |S3 = 4[.

Questão 8.46 (Putnam-1939-Wlodificada)


Resolução: Sendo n, P e 'I as raizes da equação, então, da expressão geral.
temos que:
a
ai=-- => Ei 3 ' =7 l°2 =bl: °3 = ”7 =* EI
= S2 + 2o2 (-a)2=S2+2b |s2 =a2-2b|.
Sn = CT1 ’ Sn-1 " a2 ' Sn-2 + CT3 ' Sn-3 => S3 = ("a) ’ S3-1 “ b ’ S3-2 + ("c) ’ S3-3

=0 S3 — Ç—-a) • S2 — b • — c ■ Sq S3 =(-a)[a2-2bj-b(-a)-c-3

=> S3 = -a2 + 2ab + ab - 3c |s3 = -a2 +3ab - 3c|.

Questão 46 (Singapura-2014)
Resolução: Sendo a e 0 as raizes da equação, então, da expressão geral,
temos que:

°1=“Í;CT2=T =* CT2 2
3
430 10 Resoluções

<?1 — S2 + 2cr2 - = S2--


l 3; 9 2 3

=> S2 = - + - => s2= — s2 = -


2 9 3 2 9 2 9
7
a ! p a2 + p2 H+P =_9_ => “ + P=-^ a p =_7
Pa aP P ' a 1 p a 9 p+a 3
3

Questão 8.62
Resolução: Dos produtos notáveis, podemos escrever:
= S2 -r 2g2 => 22 = 6 + 2ct2 => 2o2 = —2 /. |a2 - ~l] •
<j-| = S3 + 3o1o2 — Sog => 2^ = 14 + 3 • 2 • 1) ~ 3^3 [03 = Oj.

Como abc = 0, temos que a = 0oub = 0ouc = 0e como ab + ac + bc = -1


=> o2 = -1, apenas um deles é nulo.
Para a = 0, temos que bc = -1.
o, = 2 => 0 + b + c = 2 => b + c = 2
S2 = 6 => O2 + b2 + c2 = 6 => |b2 +c2 =~ê].

Sp=|s2-Sn_1.Sn+1| => Sp=|(bn+c")2-(bn-1 :n-1).(bn+1+Cn+1)|

=> Sp = jjX + 2bncn + "c2^ - jX - bn-1cn+1 - c':n-1bn+1


- c

=> Sp =|bn-1cn-1(2bc-c2-b2)| =. Sp =|(bc)n-1[2bc-


(b2+C2)]|
- Sp=|(-1)n-1[2(-1)-(6)]| => Sp=|(-1)n-1[-8]| => Sp=|(-1)n.8|

|sF4
431

Bibliografia

[1] ANDREESCU, Titu; Enesc, Bogdan. Mathematical Olympiad Treasures.


Birkhauser. 2004

[2] INSTITUTO DE CIÊNCIAS Y HUMANIDADES. Álgebra, Tomo I.


Lumbreras. 2011

[3] LIMA NETO. Sérgio. A Matemática do Vestibular do IME. Vestseller.


2011

[4] LIMA NETO, Sérgio. A Matemática do Vestibular do ITA. Vestseller.


2013

[5] LITIVINENKO, V; MORDOKVITCH, A. Solving Problems in Álgebra and


Trigonometry. Mir Moscou 1987
Io século XVIII, o matemático francês Jean Le Rond D'Alcmbert
afirmou "A álgebra é generosa: frequentemente ela dá mais do que
se lhe pediu". D'Alembert, que possui importantes descobertas na
álgebra, como o Teorema Fundamental da Álgebra, não poderia ter
sido mais feliz em sua célebre frase. A álgebra é o ferramenta
mestra para o desenvolvimento de todas as ciências naturais.
O que seria da física sem a álgebra? O que seria das outras áreas da
matemática sem a álgebra?
De modo a auxiliar nesse aprofundamento, a obra "Os Segredos
da Álgebra Para IME/ITA/OLIMPÍADAS", do eminente autor Miller
Dias De Araújo, é perfeita. Todos os tópicos da álgebra fundamental
estão presentes, incluindo potenciação, radiciação, produtos
notáveis e fatoração. Em cada capítulo, a teoria algébrica é
apresentada de forma detalhada, seguida de muitos problemas
resolvidos. Várias identidades algébricas são demonstradas,
algumas pouco conhecidas. Certamente, a cereja do bolo são os
exercícios propostos, em grande quantidade e de excelente
qualidade, a maioria retirada de concursos militares e olimpíadas de
matemática de todo o mundo. Você vai se deparar com exercícios do
Colégio Naval, ITA, IME e também de olimpíadas de diversos países.
Deve-se destacar a organização dos capítulos, nos quais os
conceitos mais fundamentais são apresentados primeiro, para
somente depois constarem os tópicos mais avançados, facilitando o
entendimento do leitor. O último capítulo do livro é reservado para
í r
as soluções e dicas dos exercícios propostos.
A presente obra é um verdadeiro tesouro, indispensável para
qualquer pessoa que deseja formar uma base sólida em álgebra.
Recomendo ao leitor que resolva esse livro de capa a capa, na
íntegra, sem saltar nada, prestando atenção nas nuances de cada
passagem algébrica. Existe muita poesia, muita arte e magia nessa
obra prima do Miller Dias. Parabéns ao grande mestre!

Marcelo Rufino de Oliveira

/r» ^teditora

Você também pode gostar